You are on page 1of 600

LAND & BUILDING

This material has been prepared in accordance with the syllabus prescribed by IBBI and for the sole
purpose of being the referral study material to the 50 hours Educational Courses Under rule 12(2)(a),
for registration as Valuers (Land & Building / Plant & Machinery / Securities or Financial Asset).

The views and opinions expressed in these materials are those of the authors and not of AaRVF,
Haryana. Though prepared and edited with due care, AaRVF, Haryana do not undertake any liability
for the views expressed herein.

No part of this publication may be reproduced, transmitted, or utilized or stored in any form or by
any means now known or hereinafter invented, electronic, digital or mechanical, including
photocopying, scanning, recording or by any information storage or retrieval system, without prior
permission of AaRVF, Haryana.

Assessors & Registered Valuers Foundation


CONTENT – Common Topics
MICROECONOMICS
1. CONSUMPTION
2. PRICE MECHANISM
3. TYPE OF MARKET STRUCTURES AND PRICING IN DIFFERENT
MARKET STRUCTURES
CHAPTER-1 4. Factors of Production
5. Theory of Rent
6. Capital and Interest
7. Organisation and Profit

MACROECONOMICS
1. Introduction
2. Functions of Money
3. Inflation
CHAPTER-2 4. Deflation
5. Savings
6. Investment
7. Component of Economy
8. Concepts of GDP and GNP
BOOK KEEPING & ACCOUNTANCY
1. BOOK KEEPING
2. Books
CHAPTER-3 3. Type of Accounts
4. Preparation of balance sheet
5. Multiple Choice questions

LAW-GENERAL
1. Indian Legal System
2. Government: Executive, Legislature &Judiciary
3. Laws of Contract
4. Tort and its General Principals
5. Law of Arbitration and Conciliation: Salient Features
6. Auction
CHAPTER-4
7. Laws of Evidence: Burden of Proof, Presumptions, Conclusive
Proof
8. Salient Features of the Insolvency and Bankruptcy Code 2016
9. SALIENT FEATURES OF THE COMPANIES (REGISTERED
VALUERS AND VALUATION) RULES, 2017
10. Salient Features of SARFAESI Act 2002 Concerning
Valuation
11. Section 5(n) of the Banking Regulation Act 1946
12. The Companies Act 2013: Section 192(2), 230 (1), 230(2),
230 (3),231,232, 247 and 281 (1)
13. Multiple Choice Questions

INTRODUCTION TO STATISTICS
1. Data Classification and Processing
2. Univariate data Analysis
3. Theory of Probability & Probability Distribution
CHAPTER-5
4. Test of Significance & Bi-Variate Analysis
5. Introduction to Time Series
6. Index Numbers

ENVIRONMENTAL ISSUES IN VALUATION


1. Environment and Valuation
2. Environmental Issues of Air Pollution, Water Pollution
CHAPTER-6 3. Environmental Legislations
4. Laws Related to Industrial Health and Safety
5. Multiple Choice Question

PROFESSIONAL ETHICS & STANDARDS


1. Model Code of Conduct as notified by Ministry of
Corporate Affairs Under the Companies (Registered Valuers
CHAPTER-7
and Valuation) Rules, 2017
2. Ethical Considerations under terms of Engagements
CONTENT – L&B
LAW REAL ESTATE
1. Land Acquisition
2. General Building rules and regulations
Chapter - 1
3. Rent control laws
4. Right of way and section 52
5. Salient features of the real estate
VALUATION OF REAL ESTATE
1. Cost, Price and Value
2. Types of Value
3. Basic elements of value
4. Factors affecting value
5. Highest and best use
6. Real Property
7. Annuities, Capitalization, Rate of Capitalization
Chapter - 2 8. Construction and use of tables
9. Simple Interest amount working, Compound Interest amount
working, Present Value of Rupee working, Amount of Rest. 1 /
year working, Annual Sinking fund working, Present Value of
Future Income of Rs. 1 / year. (Single rate basis), Present Value
of Future Income of Rs. 1 / year (Duel rate basis)
10. Urban Infrastructure and its influence
11. Real Estate market and its characteristics
12. 11. Concepts of Green building

INCOME APPROACH TO VALUE

1. Relation between Income and Value


2. Valuation of Property affected by the Rent Control Act,
Licensed property under the Easement Act, 1882 and
Leasehold properties under the Transfer of Property
Act,1882
3. Derivation of Yield Rate from Market Derived Data
Chapter - 3 4. Remunerative Rate of Interest and Accumulative Rate of
Interest
5. Types of rent: Outgoings, Income, Yield, Years‘ Purchase
6. Determination of Market Rent and Standard Rent
7. Lease: lessor and lessee: Types of Lease, Lease provisions
and Covenants
8. Valuation of Lessor‘s Interest, Lessee‘s Interest including
Sub-Lease in Leased Property. Premature Termination of
Lease or Surrender of Lease
9. Real Estate as an Investment, Yield from Real Estate vis-à-
vis other forms of Investments- Sound Investment
Comparison
10. Investment Decisions: Discounted Cash Flow Techniques,
Internal Rate of Return (IRR), Net Present Value (NPV) and
CAPM Technique
11. Profit Method: Valuation of Special Properties: Hotels,
Cinema, Mall, Petrol Pump, Hill resorts

MARKET APPROACH TO VALUATION

1. Types of Market, Market Survey & Data Collection, Sources


of SaleTransactions
2. Comparison of Sale Instances – Factors of comparison and
weightages for adjustment in value
3. Hedonic Model and Adjustment Grid Model under Sales
Chapter – 4
comparisonMethod
4. Land characteristics and its effect on Land Values
5. Hypothetical Plotting Scheme for value of large sizeland
6. Residue Technique and other development methods
7. Valuation for Joint Venture Development of property

COST APPROACH TO VALUE

1. Methods of Cost Estimates forBuildings


2. Life of Building:Economic/Physical/Legal; Factors affecting
life of thebuilding
3. Total Life, Age, Estimating FutureLife
4. Various methods of Computation of Depreciation,
Chapter – 5 Functional, Technological and Economic Obsolescence
5. Reproduction Cost/Replacementcost,
6. Depreciated Replacement Cost (DRC) working, adopting
DRC as Value subject to Demand and Supply aspect
7. Land Value by Market Approach and Building Value by Cost
Estimation Method for Owner Occupied Bungalows,
Factories, Public Buildings

VARIOUS PURPOSE OF VALUATION


1. Valuation of properties for purposes such as: Bank Finance,
Auction Reserve, Building Insurance, Sale, Purchase,
Valuation Disputes in Court, Probate,Partition
Chapter - 6
2. Study of Valuation Standards as per the provisions of the
Companies Act2013
3. StudyofIndianAccountingStandards IndAS 16 as applicable
to Valuation, IndAS 36, IndAS 40, IndAS 113
4. Valuer as an Expert witness in Court
5. Valuers‘ Functions & Responsibilities, Error of Judgement
and ProfessionalNegligence

IMPORTANT CASE LAWS ON PRINCIPLES OF VALUATION OF REAL ESTATE


1. R.C. Cooper Vs. Union of India, (1970) AIR SC 564
2. Sorab D. Talati Vs. Joseph Michem, AppealNo.
3. 101 0f 1949 in R.A. Application No. 805 of 1948
4. CWT Vs. P.N. Sikand (1977) 107 ITR 922(SC)
5. Wenger & Co. Vs. DVO (1978) 115 ITR 648 Delhi HC
(Combination of Methods)
6. JawajeeNagnathan Vs. Revenue Divisional Officer (1994)
Chapter - 7 SCC (4) 595(SC)
7. ChimanlalHargovinddas Vs. SLAO, AIR SC 1652
8. CED Vs. Radhadevi Jalan (1968) 67 ITR 761 (Cal)
9. CIT Vs. Ashima Sinha (1979) 116 ITR 26(Cal)
10. CIT Vs. Anupkumar Kapoor & others (1980) 125 ITR 684
(Cal)
11. CIT Vs. Smt. VimlabenBhagwandas Patel (1979)118 ITR
134(Guj)

PRINCIPLES OF INSURANCE & LOSS ASSESSMENT


1. Insurance
2. FirePolicies
Chapter - 8 3. Elements ofInsurance
4. Preparation ofclaim
5. Obligations and Rights of Insurer andInsured

REPORT WRITING
1. Reports-Quality, Structure,Style
2. Report writing for various purposes of valuation- Sale,
Purchase, Purchase, Mortgage, Taxation, Insurance,
Liquidation etc
Chapter - 9 3. Contents of the report: Instruction of Clients, Date as on
which valuation is made, date of Report and Site
Inspection, Location, Ownership History, Data Collection
and Analysis, Type of Construction, Valuation Method,
Value Estimation, Assumptions and Limiting Conditions
including Caveats and Conclusion

CASE STUDIES
Chapter - 10
1. This section will have a case study to test the ability to
apply valuation techniques. There will be a comprehension
narrating a transaction based on which questions will be
asked. 4 questions for 8marks
2. This section will have a case study to test the ability to
apply valuation techniques. There will be a comprehension
narrating a transaction based on which questions will be
asked. 6 questions for 12 marks
MICROECONOMICS
INTRODUCTION

Economics can be divided into three parts, namely, descriptive economics, economic
theory, and applied economics. In descriptive economics one collects together all the
relevant facts about a particular phenomenon. While economic theory or analysis gives
a simplified version of the way in which an economic system functions. Applied
economics takes the framework of analysis provided by economic theory.

There are three broad assumptions namely assumptions regarding behaviour of


individual. Economists are concerned with people as consumers and as businessmen.
When economists discuss the actions of consumers, they assume that they behave
rationally. It means that they try to maximize their satisfaction with minimum possible
expenditure. In the same manner, economic theories assume that the businessmen try
to maximize their profits. It is their economic rationality.

The second group of assumptions is about the physical structure of the world i.e.
natural conditions. They always remain to be given. It is these conditions give rise to
economic problem because resources are limited in relation to their demand.
Therefore, goods and services are scarce in supply. The scarcity of resources leads to
economic system and economic problem. What is worse is that the scarce resources
have alternative uses. This makes all the more difficult for human being to solve economic
problems.

The third group of assumptions relates to social and economic institutions. Under this
group of assumptions, comes political stability. Without which neither consumers nor
producers attain their goals. For economic prosperity, political stability is a must.
Applied economists are often concerned with ‘test’ theory studying statistical and
other evidence to discover if it appears to support particular economic theory. Hence,
economics is concerned with a study of one of the aspects of human beings. It
enquires into how a human being gets his income to satisfy his unlimited wants with
limited means. It deals with day-to-day activities of human being relating to his efforts
of maximizing his satisfaction. Therefore scope of economics centers around wants –
efforts – satisfaction.
MICRO ECONOMICS

1. CONSUMPTION

INDIFFERENCE CURVES ANALYSIS

In microeconomics, indifference curve is an important tool of analysis in the study of consumer


behavior.
INDIFFERENCE CURVE
• An indifference curve is a locus of all combinations of two goods which yield the same
level of satisfaction (utility) to the consumers.
• Since any combination of the two goods on an indifference curve gives equal level of
satisfaction, the consumer is indifferent to any combination he consumes. Thus, an
indifference curve is also known as ‘equal satisfaction curve’ or ‘iso-utility curve’.
• On a graph, an indifference curve is a link between the combinations of quantities which
the consumer regards to yield equal utility. Simply, an indifference curve is a graphical
representation of indifference schedule.

Figure: Graphical representation of indifference curve


ASSUMPTIONS OF INDIFFERENCE CURVE
The indifference curve theory is based on few assumptions. These assumptions are:
• Two commodities
• Non satiety
• Ordinal utility
• Diminishing marginal rate of substitution
• As indifference curve theory is based on the concept of diminishing marginal rate of
substitution, an indifference curve is convex to the origin.
• Rational consumers

PROPERTIES OF INDIFFERENCE CURVE


There are four basic properties of an indifference curve. These properties are:
• Indifference curve slope downwards to right
• In the above diagram, IC is an indifference curve, and A and B are two points which
represent combination of goods yielding same level of satisfaction.
• Indifference curve is convex to the origin
• According to diminishing marginal rate of substitution, the rate of substitution of
commodity X for Y decreases more and more with each successive substitution of X for
Y.
• Also, two goods can never perfectly substitute each other. Therefore, the rate of
decrease in a commodity cannot be equal to the rate of increase in another commodity.
• Thus, indifference curve is always convex (neither concave nor straight).
• Indifference curve cannot intersect each other
• Higher indifference curve represents higher level of satisfaction
• The indifference curve analysis measures utility ordinal. It explains consumer behavior in
terms of his preferences or rankings for different combinations of two goods, say X and
Y. An indifferent curve is drawn from the indifference schedule of the consumer.
• The latter shows the various combinations of the two commodities such that the
consumer is indifferent to those combinations. According to Watson, “An indifference
schedule is a list of combinations of two commodities the list being so arranged that a
consumer is indifferent to the combinations, preferring none of any other.”

CONSUMER SURPLUS
• Consumer surplus, also called social surplus and consumer’s surplus, in economics,
the difference between the price a consumer pays for an item and the price he would be
willing to pay rather than do without it.
• The concept is still retained by economists, in spite of the difficulties of measurement, to
describe the benefits of purchasing mass-produced goods at low prices. It is used in the
fields of welfare economics and taxation

ELASTICITY
• In economics, a measure of the responsiveness of one economic variable to another. A
variable y (e.g., the demand for a particular good) is elastic with respect to another
variable x (e.g., the price of the good) if y is very responsive to changes in x; in
contrast, y is inelastic with respect to x if yresponds very little (or not at all) to changes
in x.
• Technically, the elasticity of y with respect to x is calculated as the ratio of the
percentage change in the quantity of y to the percentage change in the quantity of x. In
algebraic form, elasticity (E) is defined as E = %Δy/%Δx.

Determinants of Elasticity of Demand


Definition: The Elasticity of Demand is a measure of sensitiveness of demand to the
change in the price of the commodity.

Apart from the price, there are several other factors that influence the elasticity of demand.
These are:

2. PRICE MECHANISM

DEMAND AND SUPPLY

2.1 DEMAND

In economics, demand has a distinct meaning. Supposing, you desire to have a car, but
you do not have enough money to buy it. Then desire will remain just a wishful
thinking; it will not be called demand. If you have enough money, you do not want to
spend it on car, demand does not emerge. The desire becomes demand only when you
are ready to spend money to buy the car. Thus, Demand for a commodity refers to the
desire to buy a commodity backed with sufficient purchasing power and willingness to
spend. Hence demand is equal to desire plus purchasing power plus willingness to pay.
Demand for a commodity is always refers to price. At higher price quantity demanded
will be low, and at lower price quantity demanded will be high.

Demand schedule:
It is a numerical tabulation, showing the quantity that is demanded at different prices.
It expresses the relation between price and demand of a commodity. A demand
schedule can be of two types –
▪ Individual Demand Schedule
▪ MarketDemand Schedule

Individual Demand Schedule:


Individual demand schedule is defined as the quantity of a given commodity which a
consumer will buy at all, possible price, at a particular period of time.

Table -Individual demand schedule for Apples

In the above table we can see that as the price of apples increases, quantity demanded
is decreases.

Market demand Schedule


In every market, there are several consumer of a commodity. Market demand schedule
is one that shows total demand of all the consumers in the market at different price of
the commodity.

Table - Market demand schedule for Apples


Above Table shows that with the rise in the price of apples the market demand for apples
is decreasing.

Demand Curve
Demand Curve is the graphical representation of demand schedule expressing the
relation between different quantities demanded at different possible prices of the
commodity. There are two type of demand curve:
▪ Individual Demand Curve: It is a curve showing different quantities of a commodity
that one particular buyer is ready to buy at different possible price of the
commodity at a point of time. Individual demand curve is shown in figure:

▪ Market demand Curve: The market demand consists of the total quantity
demanded by each individual in the market. The market demand curve is formed
by computing the horizontal summation of the individual demand curves for all
consumers. This process is illustrated in Figure

▪ We take a hypothetical case in which there are only two consumers in the market
namely, Mr Ravi and Mr Sahil. The total quantity demanded in the market is just
the sum of the quantities demanded by each individual. The market demand curve
is derived by adding together the quantities demanded by all consumers at each
and every possible price.

Both individual and market demand curves slope downward from left to right indicating
an inverse relationship between price and quantity demanded of goods.

Determinants of demand:
Demand function is show the relation between demand for a commodity and its
various determinants. The determinants are also known as the factors which affect
demand of a commodity. It shows how demand is related to different factors like price,
income etc. the demand function can be expressed as follows-

D = f(P, Pr, Y, T, FE,S,W, ……..)

where,

P – Price of the commodity


Pr – Price of related goods
Y – Income of the consumer
T – Taste and preference
FE – Future expectations of the consumers
S – Size and composition of the population
W – Weather condition

The determinants are explain below -


i) Price of the Commodity: Quantity demanded and the price of the commodity is
inversely related. It means that with the rise in the price of commodity, quantity
demanded decreases and with the fall in the prices there is a rise in the quantity
demanded.
ii) Price of the related goods: Demand for a commodity is also influenced by change in the
price of related goods. There are two types of related goods - Substitutes and Complements.
Substitute Goods are those goods which can be the goods which can be used in place of each
other, such as tea and coffee. If an increase in the price of one causes a rise in the demand for
the other then the two goods are substitutes. On the other hand the complementary goods are
those goods which are consumed together.

If an increase in the price of one goods causes the reduction in the demand for the
otherthen the two goods are complementary goods. Car and petrol are
complimentary goods.

iii) Income of the consumers: Normally there is a direct relationship between the
income of the consumer and his demand for the commodity. For a normal good with
the rise in consumer’s income demand will rise and vice versa. Goods like television
sets, cars, clothes etc are considered normal goods. If the demand for a goods
decreases with the rise in consumers income then that goods are known as Inferior
Goods. For e.g. coarse grain like Jowar, Bajra, Maize, etc. If the income of the
consumer rises she will reduce the consumption of these goods.

If the demand increases with an increase in income and thereafter it remains


constant irrespective of the level of income then the goods in question are known as
necessities for example salt, match box, etc.

iv) Consumer’s Taste and Preference: Consumer’s demand for the goods is greatly
influenced by the taste and preferences which in turn depends on social
customs,habits, fashion, etc.

Consumer’s Expectation: If a consumer expects a fall in the price of a commodity in a


Near future, then he will postpone his present demand and if he anticipates a rise in
price then he will increase his current demand. For instant if you are thinking about
purchasing a computer and you obtain information that may lead to rise in the future
price then you will buy the computer today itself. However, a reduction in the expected
future price will result in a reduction in current demand.
If expected future income rises, demand for many goods today is likely to rise. On the
other hand, if expected future income falls, individuals may reduce their current
demand for goods so that they can save more today in anticipation of the lower future
income.

v) Size and Composition of Population: Larger the population, larger is likely to be the
numbe rof consumers thus greater will be the demand. The composition of
population refers to number of children, adults, males, females, etc. in the
population. If the number of children are more in the population then more of baby
products will be demanded whereas in an education township like Vallabh Vidya
nagar in Anand district of Gujarat where 50 to 60 per cent of the population is of
students (between the age group of 18 to 24 years) more of stationary, hostels, fast
foods etc will be demanded.The type of people inhabiting the country will also
influence the consumer demand. Since the market demand curve consists of the
horizontal summation of the demand curves of all buyers in the market, an increase
in the number of buyers would cause demand to increase. As the population
increases, the demand for food, houses, cars and virtually all other commodities, is
expected to increase. A decline in population will result in a reduction in demand.

vi) Weather condition: Another factor which affects demand is the weather
conditions. For example during summer there will be greater demand for sun
glasses, cotton wears, ice creams etc, whereas during rainy season the demand for
umbrellas and raincoats will increase.

Law of Demand: Law of demand expresses the functional relationship between the
price of commodity and its quantity demanded. It states that the demand for a
commodity is inversely related to its price, other things remaining constant. In other
words a fall in price of a commodity will lead to a rise in demand of that commodity
and a rise in price will lead to fall in demand. Thus there is an inverse relationship
between the price of a good and the quantity demanded in a given time period, ceteris
paribus.

Assumption:
The law of demand is based on certain assumptions. These are as follows -
a. There is no change in the Income of the people.
b. Taste, preference and habits of consumers unchanged.
c. Prices of related goods i.e., substitute and complementary goods
remaining unchanged
d. There is no expectation of future change in price of the commodity.
e. The commodity in question is not consumed for its prestige value.
As shown in figure 3.3 the relationship between price and quantity demanded is
represented by a demand curve. At price OP the quantity demanded is OQ when the
price increase from OP to OP1 quantity demanded decreases from OQ to OQ1. Thus
when the price increases, demand decreases and vice versa. Demand curve slopes
down ward from left to right showing inverse relationship between price and quantity
demanded. This downward slope of demand curve is expression of law of demand.

Reasons for downward slope of demand curve:


Downward slope of demand curve indicates that consumers buy more of a commodity
at lower prices and vice versa. Thus, there is Negative relationship between price and
quantity demanded. The reasons for downward slope of demand curve are –

(i) Law of Diminishing Marginal Utility: This law states that when a consumer buyers
More units of same commodity, the marginal utility of that commodity continues to
decline. The consumer will buy more of that commodity when price falls. When
lessunits are available the utility will be high and consumer will prefer to pay more for
that commodity. Thus the demand would be more at lower prices and less at a higher
price and so the demand curve is downward sloping.

(ii) Income effect: As the price of the commodity falls the real income of the consumer
will increase and consumer can increase his consumption. He will spend less to buy the
same quantity of goods. On the other hand, with a rise in price of the commodities the
real income ofthe consumer will reduce and consumer will buy less of that good.

(iii) Substitution Effect: When the price of a commodity falls, the price of its
Substitutes remaining the same, the consumer will buy more of that commodity and
this is called the substitution effect. The consumer will like to substitute cheaper good
for the relatively expensive good. On the other hand, with a rise in price the demand
falls due to unfavorable substitution effect. It is because the commodity has now
become relatively expensive which forces the consumer’s to buy less.

iv) Number of uses of a Good: Goods which can be put to a number of uses like milk
which can be used for making tea, curd, cold drinks, paneer etc. When the price of milk
commodity is higher, it will sparingly used. On the other hand, if the price of milk
decreases consumer will use it for a variety of purposes leading to a rise in demand.
Thus the demand for the product with the change in price is determined by the number
of uses of a commodity.

v) Change in number of buyers: Lower price will attract new buyers and higher
price reduces the buyers. Such buyers are known as marginal buyers.
Owing to the above mentioned reasons the demand falls when price rises and so the
demand curve is downward sloping.

Exceptions to the law of demand:


Law of demand has some exceptions as well. There are some goods whose demand
increases when price rises and decrease when price falls. They are –

i) Conspicuous Goods These are the goods which are purchases by the consumers to
project their status and prestige .Expensive cars, diamond jewellery, etc. are such
goods. The conspicuous goods are purchased more at a higher price and less at a
lower price.

ii) Giffen Goods : Giffen goods named after Sir Robert Giffen. These are inferior
goods whose demand increases even if there is a rise in price. For e.g.: - coarse
grain, clothes, etc.

iii) Share’s speculative Market : It is often found that people buy shares of those
companies whose price is rising in anticipation of further rise in price. Whereas,
they buy less shares in case the prices are falling as they expect a further fall in
price of such shares. Here the law of demand fails to apply.

iv) Bandwagon effect: Here the consumer demand of a commodity is affected by the
taste and preference of the social class to which he belongs to. If sports car
fashionable among business community, then as the price of sports cars rises,
these consumers may increase the demand for such goods to project their position
in the society.

v) Veblen Effect: Many a times consumer judge the quality of a product by its price.
Consumer feels that a higher price means better quality and lower price means
Poor quality. So the demand goes up with the rise in price for example branded
consumer goods.

Change in Quantity Demanded and Change in Demand


A change in quantity demanded refers to increase or decreases in quantity purchased of a
commodity in response to decrease or increase in its price, other things remain constant. It is
expressed through movement along the demand curve.On the other hand a change in demand,
refers to increase or decrease in quantity demanded of a commodity in response to change in
factors other than price. It is expressed through shift in demand curve-forward shift or backward
shift.

(a) Movement of Demand curve or Extension and Contraction of Demand or


change in quantitydemanded.
With the change in the price of a commodity the quantity demanded will increases or decreases
depending upon the fall or rise in the price ofa commodity alone, ceteris paribus.This is called
movement along the demand curve or extension or contraction of Demand. As shown in figure 3.4,
when the price increases, other factors affecting demand remain constant, the quantity demand will
decreases and vice versa

The figure 3.4 show that when price increases from OP to OP1 the demand decreases from OQ to
OQ1. Thus with the fall in price there is a movement on the demand curve from point A to point B.
Similarly with the rise in price from OP to OP2 the quantity demanded decreases from OQ to OQ2
causing a shift from point A to point C on the demand curve. The increase in demand due to fall in
price is also called extension of demand. The reduction in quantity demanded due to increase in
price is known as contraction.

In Figure 3.5 DD is the original demand curve and the consumer in buying OQ units of the
commodity at price OP. For example with an increase in the income of the consumer , price of the
product remains constant, the demand increases the new demand curve is D1D1. This new curve is
an outward shift in the demand curve and shows an increase in the demand for the product from
OQ to OQ1. Similarly due to the fall in the income of the consumer, the demand curve will shift
inward from DD to D2D2. Quantity of good purchased will reduce from OQ to OQ2. This is called
decrease in demand.

Thus in the above figure quantity demanded has increased from OQ to OQ1, the price of
commodityremaining constant at OP. This is shown by a right ward shift of the Original demand
curve toform new demand curve D1D1. This is called increase in demand in demand.The left ward
shift from the original demand curve DD to D2D2 is known as decrease in demand, price of the
product remains same at OP.

THEORY OF SUPPLY

Supply is defined as a quantity of a commodity offered by the produces to besupplied at a particular


price and at a certain time. Same as demand supply has three elements namely quantity of
commodity, particular price and particular time.

The term ‘supply’ is different from ‘stock’ of a commodity. The total amount of the commodity
which a seller can bring out for sale in the market is his stock. However, producer often does not
offer his entire stock for sale in the market.Supply has been defined as that part of the stock of a
commodity which is offered for sale at a particular price during a period of time. For example a
farmer produces 500 tons of potatoes during a given period. He may offer only 300 tones for sale at
Rs 1000 per ton. In this case the stock of potatoes is 500 tons but supply is only 300 tones at a given
price.
Individual Supply and Market Supply

Individual supply refers to the quantity of a commodity which a producer is willing to produce
andoffer for sale. On the other hand, the quantity which all producers are willing to produce and sell
isknown as market supply. If, at a given price, producer A is willing to sell 200 units of a commodity
and producer B is willing to sell 500 units, and then if there are only two firms producing this
particular commodity, market supply will be 700 units.

Law of Supply

Law of supply states that, other thing remaining constant, as the price increase quantity supplied
will increase and with the decrease in price the supply will reduce. Thus there is a positive
relationship between price of a commodity and its quantity supplied. More is supplied at higher
price and less at the lower price. The law of supply is based on following assumptions -

Assumption of the law of supply


(1) Prices of the factors of production are constant.
(2) Price of the related goods remain constant
(3) Technique of production is constant.
(4) No change in the Objectives of the firm
(5) Producers do not expect any change in the future price of the product.

The law can be explained with the help of following supply schedule and supply curve.

Supply Schedule
Supply schedule is a table which shows various quantities of a commodity offered for sale at
different possible prices of that commodity. There are two types of supply schedule –
(i) Individual supply schedule, and
(ii) Market supply schedule.

(iii) An individual supply schedule shows the different quantities of a commodity that a
producer would offer for sale at different prices.

Table 3.3 shows a hypothetical individual supply schedule of apples. When the price of
apples is Rs 10 per Kg the producer is interested in selling only 1 kg of apples. As the price
rises, supply increases. Thus higher the price higher is the supply.
Table 3.3.Individual supply schedule of Apples

Price of Apples (Rs.) Quantity supplied (Kg)

10 1
20 2
30 3
40 4

Market supply schedule: Market supply refers to supply of all the producers in the market
producing a particularcommodity.Firm is an individual unit producing a commodity. A group
of firms producing a similar good is called an Industry. Thus, market supply schedule is also
referred to supply of the industry as whole.

Table: Market Supply schedule.


Price of Apples Supply by Producer Supply by Producer Market supply
(Rs.) ‘A’ (Units) ‘B’ (Units)
10 1 0 1+0=1
20 2 5 2+5=7
30 3 10 3+10=13
40 4 15 4+15=19

From the above table we see that when price of apples is Rs.10 per Kg, then the producer A
will supply only 1 kg of apples whereas producer B is not interested any quantity. When price
increase to Rs.20, producer ‘A’ supplies 2 kg and producer ’B’ supplies 5 units.

Thus the market supply is 2 + 5 = 7Kg of apples.When the price rise to Rs 30 per kg of apples
market supply increase to 13 kg. Thus at higher price the market supply will increases.

Supply curve: Supply curve is a graphic presentation of supply schedule. Supply curve has
positive slope which indicates positive relationship between price of a commodity and its
quantity supplied.
Same as the supply schedule supply curve can be divided into
(i) Individual supply schedule and
(ii) Market supply schedule.

An individual supply curve is a graphical representation of supply schedule of an individual


producer in the market. It slopes upwards indicating positive relationship between price of a
product and its quantity supplied. The individual supply curve is given in figure 3.6

In the above figure SS is the supply curve which has positive slope. It shows that more of a
commodity is supplied at a higher price.

Market supply curve as shown in figure 3.7 is the horizontal summation of all individual
supply curves. This is also known as the supply curve of the industry as a whole. Supply curve
SS is of producer A and S1S1 is the supply curve of producer B.
For deriving the market supply curve, same as the market demand curve explained earlier in
this chapter we add the individual supply curves horizontally.

Factor Determining supply or supply function


Supply function represents the functional relationship between supply of a commodity and
its various determinants. The supply of a commodity mainly depend on the objective of the
firm, price of the commodity, price of related goods, price of factors of production and the
state of technology.

Supply function can be written as –


S =f (P, O, Pr, F, T, G,…….)
Where
P- Price of the commodityO – Objectives of the firm Pr – Price of related goodsI – Input Prices
T – State of Technology
G – Government Policies
E- Future expectation of the prices F – Number of sellers in the marketN – Natural Factors

The above mentioned determinants of supply are explained below –

(i) Price of the commodity (P): With change in the price of the product the supply
changes. When the price increases, producer increase the supply and vice versa.With
no change in cost of production, higher the price, higher will be the profit margin. This
will encourage the producers to supply larger quantity at higher prices. When the
price decline the supply will also decline.

(ii) Objectives of the firm (O):Firms have several objectives such


asprofitmaximization, sales maximization, employee satisfaction maximization etc. If
the objective is tomaximize profit, then higher the profit from the sale of a
commodity, the higherwill be the quantity supplied by the firm and vice-versa. Thus,
the supply of goods willalso depend upon the priority of the firm regarding these
goals and the extent towhich it is prepared to sacrifice one goal to the other.

(iii) Expectation about future prices (E): If the produces expect an increase in the
future price of acommodity, then the present supply will reduce as producer will
stock the goods to sell in future at higher prices. On the contrary if he expects a fall in
future prices then he will increase the present supply.

(iv) Input Prices (I):Supply depends upon the prices of inputs like raw materials,labour
and other inputs. Any rise in the input cost will reduce the profitmargin and
ultimately lead to a lower supply. However, with the fall in inputs prices , profit
margin will increase and the supply will also increase.
(v) State of Technology (T):An improved and advanced technology is used for
the productionof a commodity will reduce its cost of production and increases

the supply. On the contrary, outdated and old technology will increase the cost
of production and reduced supply.

(vi) Government policies (G):Policies of Government such as fiscal policy which

leads to imposition of taxes,excise duty, sales tax etcwill affect the production
of commodities and supply adversely. Any reduction in the taxes will increase

the supply. Subsidy policy also influences the supply of a commodity. When
government increase the subsidy the profit margin will increase and supply will

increase.

(vii) Prices of the related goods (Pr):An increase in the prices of related goods

othercommodities makes the production of that commodity whose price


has not risenrelatively less attractive we thus, expect that other things

remaining the same, thesupply of one good falls as the price of other goods
rises. For instance a farmer produces bananas as well as potatoes his farm. If
the price of potatoes increases hewill grow more of potatoes and less of
bananas. Hence the supply of bananas will reduce.

(viii) Number of Sellers in the market (F):Market supply is the sum total of the
supply by number of individual suppliers. Larger the number of the firmsin the

market the greater will be the supply. Adecrease in the number of firms

reduces the supply and vice versa.

(ix) Natural factor (N):Natural factors too affect the supply. In case of natural
calamities like flood, drought, earthquake etc. the supply of acommodity
especiallyof agricultural products is adversely affected.
Exceptions to the Law of Supply
(i) Agricultural Goods: For agricultural goods it is not possible for the supply to be
adjusted to market conditions. As the production and supply of agricultural

goods is largely dependent on natural factors like rainfall, temperature etc. and it
is mostly seasonal in nature it cannot be increased with a rise in price.

(ii) Rare Objects: The supply of certain commodities like rare coins, classical

paintings old manuscripts, etc. cannot be increased or decreased with the change
in price. Therefore, such goods have inelastic supply.

(iii) Labour Market: with a rise in wages workers will work for less number of
hours,and will prefer leisure over work. Thus the labour market, the behavior of

the supply of labour goesagainst the law of supply.

Change in Quantity Supplied and Change in Supply


A change in quantity supplied refers to change in quantity purchased of a commodity in
response to change in price, other things remain constant. It is expressed through movement
along the Supply curve. On the other hand a change in Supply refers to change in quantity
Supplied of a commodity in response to change in factors other than price of the commodity.
It is expressed through shift in Supply curve-forward shift or backward shift.

(a) Movement of Supply curve or Extension and Contraction of Supply or


change in quantity Supplied.
With the change in the price of a commodity the quantity Supplied will increases or decreases
depending upon the rise or fall in the price ofa commodity alone, ceteris paribus.This is called
movement along the Supply curve or extension or contraction of Supply. As shown in figure
3.4, when the price increases, other factors affecting Supply remain constant, the quantity
Supply will decreases and vice versa.
The figure 3.8 show that when price increases from OP to OP1 the Supply increases from
OQ to OQ1. Thus with the rise in price there is a movement on the Supply curve from point A
to point B. Similarly with the fall in price from OP to OP2 the quantity Supplied decreases from
OQ to OQ2 causing a shift from point A to point C on the Supply curve. The increase in Supply
due to rise in price is also called extension of Supply. The reduction in quantity supplied due
to fall in price is known as contraction.

Change in Supply or shift of Supply or Increase and Decrease in Supply:


When the quantity supplied of a commodity increases or decreases due to change in factors
other than price of the product like price of related goods, prices of inputs etc. it is known as
change in supply or shift in supply.

In Figure 3.9 SS is the original Supply curve and the consumer in buying OQ units of the
commodity at price OP. If the input cost reduces, price of the product remains constant, the
supply will increase, and the new supply curve is S1S1. This new curve is an outward shift in the
supply and it shows an increase in the supply for the product from OQ to OQ1. Similarly due
to the rise in input cost, price of the product remaining same, the supply curve will shift
inward from SS to S2S2. Quantity of good purchased will reduce from OQ to OQ2. This is called
decrease in Supply.
Thus in the above figure quantity supplied has increased from OQ to OQ1, the price of
commodity remaining constant at price OP. This is shown by a right ward shift of the original
supply curve to form new supply curve S1S1. This is called increase in supply.
The left ward shift from the original supply curve SS to S2S2 is known as decrease in Supply,
price of the product remains same at OP.

EQUILIBRIUM OF DEMAND SUPPLY AND PRICE DERTEMINTION


Equilibrium means a state of balance. The term equilibrium in Economics means the state in
which there is no tendency on the part of consumers and producers to change. Market
equilibrium is a situation of the market in which demand for a commodity is equal to supply
of the commodity at a particular price. Hence when there is equilibrium between demand and
supply of a commodity at a particular price, there is neither excess demand nor excess supply.
At this position the prevailing price is called the equilibrium price and the corresponding
quantity supplied/demand is called equilibrium quantity

Determination of equilibrium price:


According to Alfred Marshall demand and supply are the two blades of pair of scissors.
Through intersection of demand and supply the equilibrium price and equilibrium quantity of
a commodity is determined.

The force of demand and supply determinethe price of a commodity. There is a conflict in the
aim of producers and consumers. Consumers are interested in buying the goods at the
lowest price to maximize satisfaction and producer aim at selling the goods at the highest
price to maximize profit.Equilibrium price will be determined where quantity demanded is
equal to the quantity supplied. This called market price. The determination of equilibrium
price is explained with the help of a schedule given in table 3.5 and figure 3.10.
Table 3.5 – Equilibrium Price
Price of Quantity Quantity
Apples dema sup
(Rs.) nded plied
(Kilogram) (Kilogram)
10 4 0
20 3 1
30 2 2
40 1 3

Table 3.5 gives a hypothetical schedule which depicts different price and the respective
quantity demanded and supplied. When the prices increases from Rs 10 to Rs 40, the quantity
demanded decreases from 4 Kg to 1 Kg and the quantity supplied increases from nothing to 4
Kg respectively. At price Rs 10 the quantity demanded is 4 Kg and suppliers are not interested
in supplying at all. Thus at lower price consumers will demand more and suppliers will supply
less. At price Rs 40 per Kg demand is 1 Kg and supply is 4 Kg. With an increase in price the
demand decrease and supply increases. We can observe in table 3.5 at price at price Rs 30 the
quantity demanded is equal to quantity supplied, and that is the equilibrium price and
equilibrium quantity is 2 Kg. At prices less than Rs 30 there is an excess of demand over
supply and at price higher than Rs 30 per Kg the supply is more than demand.
In figure 3.10 price is measured on Y axis and quantity demanded and supplied taken on X-
axisprice per unit. DD is the demand curve and SS is the supply curve. The demand curve and
supply curve intersect each other at point E. At the equilibrium point E the quantity
demanded is equal to the quantity supplied i.e. PE and therefore the equilibrium price is OP
the equilibrium quantity is OQ.
Above this equilibrium price OP, at OP1 the quantity demanded decrease to P1Gand quantity
supplied increase to P1H. At price higher than equilibrium price there is an excess of supply
over demand GH. At price OP2, which is lower than the equilibrium price quantity supplied
decreases to P2I and quantity demanded increases to P2K. Hence at price lower than the
equilibrium price there is an excess of demand over supply.

Effect of change in supply and demand:


The equilibrium price and quantity changes with the shift in supply curve demand remaining
same or shift in demand curve supply remaining same or shift in both. The change in
equilibrium due to change in demand with no change in supply can be seen in figure 3.11,
which shows price on Y axis and quantity demanded and supplied on x axis.

71
Demand curve DD intersects supply curve SS at point E, which determines the equilibrium
price OP and equilibrium quantity OQ. With an increase in demand the demand curve shifts
from DD to D1D1. And the new equilibrium is at E1. Thus with the increase in demand supply
remaining same there is an increase in the price to OP1. When the demand decreases from
DD to D2D2 an inward shift in the demand curve the equilibrium shifts to E2 leading to a
reduction in the equilibrium price. With the increase in demand the equilibrium price increase
and vice versa.
The effect of change in the supply is shown in figure 3.12, where price on Y axis and quantity

demanded and supplied on x axis.

Demand curve DD intersectssupply curve SS at point E, which determines the equilibrium


price OP and equilibrium quantity OQ. With an increase in supply the supply curve shifts
from SS to S1S1. And the new equilibrium is at E1. Thus with the increase in supply demand
remaining same there is a reduction in the price to OP1. When the supply decreases from SS
to S2S2 an inward shift in the supply curve the equilibrium shifts to E2 leading to an increase
in the equilibrium price to OP2.Thus with the increase in supply the equilibrium price decrease
and vice versa.

Thus we learnt that

i) When the demand increases equilibrium price will increase,


ii) When the demand decreases equilibrium price will decrease,
iii) When the supply increase equilibrium price will decrease, and
iv) When the supply decreases equilibrium price will increase.
Importance of Time Element

Marshall, who propounded the theory that price is determined by both demand and supply,
also gave a great importance to the time element in the determination of price. Time elements
is of great relevance in the theory of value, since one of the two determinants of price, namely
supply, and depends on the time allowed to it for adjustment. It is worth mentioning that
Marshall divided time into different periods from the viewpoint of supply and not from the
viewpoint of demand.

Time is short or long according to the extent to which supply can adjust itself. Marshall felt it
necessary to divide time into different periods on the basis of response of supply because it
always takes time for the supply to adjust fully to the changed conditions of demand.

The reason why supply takes time to adjust itself to a change in the demand conditions is that
nature of technical conditions of production is such as to prohibit instantaneous adjustment
of supply to changed demand conditions. A period of time is required for changes to be
made in the size, scale and organisation of firms as well as of the industry.

Another point is worth noting. When Marshall distinguished short and long periods he was
not using clock or calendar time as his criterion, but ‘operational’ time in terms of economic
forces at work. In this regard, as said above, supply forces were given the major attention and
a time was short or long according to the extent of adjustment in the forces of supply. The
greater the adjustability of the supply forces, the greater the length of the time irrespective of
the length in clock-time.
Time can be divided into following three periods on the basis of
response of supply to a given and permanent change in demand:

1. Market Period:
The market period is a very short period in which the supply is fixed, that is, no
adjustment can take place in supply conditions. In other words, supply in the market
period is limited by the existing stock of the good. The maximum that can be supplied in
the market period is the stock of the good which has already been produced.

In this period more good cannot be produced in response to an increase in demand. This
market period may be a day or a few days or even a few weeks depending upon the
nature of the good. For instance, in case of perishable goods, like fish, the market period
may be a day and for a cotton cloth, it may be a few weeks.

2. Short Run:

Short run is a period in which supply can be adjusted to a limited extent. During the short

period the firms can expand output with given equipment by changing the amounts of

variable factors employed. Short periods is not long enough to allow the firm to change

the plant or given capital equipment. The plant or capital equipment remains fixed or

unaltered in the short run. Output can be expanded by making intensive use of the given

plant or capital equipment by varying the amounts of variable factors.

3. Long Run:

The long run is a period long enough to permit the firms to build new plants or abandon

old ones. Further, in the long run, new firms can enter the industry and old ones can

leave it. Since in the long run all factors are subject to variation, none is a fixed factor.

During the long period forces of supply fully adjust them to a given change in demand;

the size of individual firms as well as the size of the whole industry expands or contracts

according to the requirements of demand.


From above, it is clear that because of the varying response of supply over a
period of time to a sudden and once-for-all increase in demand Marshall found,
it necessary and useful to study the pricing process in:
a. The market period,

b. The short-run and

c. The long-run depending respectively upon whether the supply conditions have

time to make (i) no adjustment, (ii) some adjustment of labour and other

variable factors, and (iii) full adjustment of all factors and all costs. Therefore,

Marshall explained how the equilibrium between demand and supply was

established in three time periods and determined market price, short-run price

and long-run price.

We thus see that the price that will prevail depends upon the period under consideration. If a
sudden and a once-and-for all increase in demand take place, the market price will register a
sharp increase, since supply cannot increase in the market period. In this market period, firms
can sell only the output that has already been produced. However, in the short run some
limited adjustment in supply will take place as a result of the firms moving along their short
run marginal cost curves by expanding output with the increase in the amount of variable
factors. Consequently, the short run price will come down from the new high level of the
market price.

But this short-run price will stand above the level of original market price which prevailed
before the increase in demand occurred. In the long run the firms would expand by building
new plants, that is, by increasing the size of their capital equipment.

In other words, firms would expand along the long-run marginal cost curves. Besides, the new
firms will enter the industry in the long run and will add to the supply of output. As a result of
these long-run adjustments in supply, the price will decline.

Thus the long run price will be lower than the short-run price. But this long-run price will be
higher than the original price which ruled before the increase in demand took place, if the
industry happens to be increasing-cost industry.
The adjustment of supply over a period of time and consequent changes in price is illustrated
in figure above where long-run supply curve LRS of an increasing-cost indus- try along with
the market-period supply curve MPS and the short-run supply curve SRS have been drawn.
Originally, demand curve DD and market-period supply curve MPS intersect at point E and
price OP is determined. Suppose that there is a once- for-all increase in demand from DD to
D’D’.

Supply cannot increase in the market period and remains the same at OM. Market- period
supply curve MPS intersects the new demand curve D’D’ at point Q. Thus, the market price
sharply rises to OP”. Short-run supply curve SRS intersects the new demand curve D’D’ at
point R.

The short-run price will therefore be OP” which is lower than the new market price OP’. As a
result of the long-run adjustment the price will fall to OP’” at which the long-run supply curve
LRS intersects the demand curve D’D’.

The new long-run price OP'” is lower than the new market price OP’ and the short-run price
OP”, but will be higher than the original price OP which prevailed before the increase in
demand took place. This is so because we are assuming an increasing-cost industry. If the
industry is subject to constant costs, the long-ran price will be equal to the original price.
Further, if the industry is subject to decreasing costs, the long-run price will be lower than the
original price.

It follows from above that the price which prevails in the market depends upon the period
under consideration. It is thus clear that the time plays an important role in the determination
of price. Another significance of the time-period analysis of pricing is that it enabled
Marshall to resolve the controversy current among economists whether it is demand or
supply which determines price.

Marshall propounded the view that both demand and supply took part in the determination
of price. But, “as a general rate”, said Marshall, “the shorter the period which one considers
the greater must be the share of our attention which is given to the influence of demand on
value, and the longer the period more important will be the influence of cost of production
on value.

Actual value at any time—the market value as it is often called—is often influenced by
passing events and causes whose action is fitful and short-lived than by those which work
persistently. But in the long run these fitful and irregular causes in a larger measure efface one
another’s influence so that in the long run persistent causes dominate value completely”.

From the above quotation from Marshall it follows that in the market period, demand
exercises a predominant influence over price but in the long run it is the supply which is of
overwhelming importance as a determinant of price. Roughly speaking, we can say that in the
market period it is the force of demand which determines price and in the long period it is the
force of supply which governs price.

Thus those economists who held that value was governed by demand were in a way right and
so were those who contended that cost of production (i.e., force working on the supply side)
determines price. The difference in the two views was due to the fact that one group of
economists was emphasising the determination of the market price over which demand has
determining influence and over which cost of production does not exercise much influence,
while the other group was stressing on the determination of long-run price over which cost of
production has got paramount influence. It is thus clear that Marshall by putting forth the
view that both demand and supply determine price by their interaction brought about
synthesis between the views of earlier econo- mists.

Both the two opposite views of earlier economists were in a way right but each was one-
sided. Each view provided us with a force which governed price. The two forces of supply and
demand furnished by the two opposing views were sufficient determining factors.

Therefore, Marshall gave equal importance to both demand and supply as determinants of
price, though the influence of the two varied in different time periods. Marshall introduced
time period analysis into pricing process to bring out the varying influence of each of two
forces over price of the product in different time periods.

It follows from what has been said above that Marshall and modern economists following him
study the effect of the varying response in supply in different time periods on price to a
sudden and permanent change in demand conditions.

On the contrary, economists do not study the effect on price of the adjustment in demand
over time in response to a change in supply conditions. The reason why we do not study
adjustment in demand to a change in supply and consequent effect on price is better
brought out in the worlds of Professors Stonier and Hague. “There is no reason why, if
supply conditions change, demand conditions should change as well, or if they do, why they
should change differently in the short run and the long run.

Changes in consumer’s tastes are not dependent on technology in the way that supply
conditions are. Admittedly, consumers’ tastes may and probably will change as time goes on.
But this will be a change of data and not a change induced by changed supply conditions.

There is no necessary reason why the long-run demand curve should differ from the short-run
demand curve, however odd the behaviour of supply has been-we must expect that the
longer is the period during which demand and supply are coming into equilibrium, the more
changes will have time to take place. If we were to study the changes in demand and supply
which would take place in respect to any change of data during many successive very short
periods of time, we should find that we had introduced unnecessary and intolerable detail
into the analysis.”

We shall explain below in detail the market-period equilibrium, short-run equilibrium, long-
run equilibrium between demand and supply and thus the determination of market price,
short-run price and long-run price under conditions of perfect competition.

Elasticity of Demand

The law of demand fails to tell us as to what extent demand for a commodity vary when there
is a change in price. In other words, the law of demand merely indicates the direction to
which demand moves when there is a change in price. But concept of elasticity explains the
exact change in demand when there is a change in price. The price elasticity of demand is
defined as “The degree of responsiveness or sensitiveness of demand to a change in price of
a commodity or service.”
Algebraically, it is stated as
e(p) = Q  P

Q P
Where  = means a change
Q = Quantity
P = Price
e(p) = Price elasticity of
demand

There are five cases of price elasticity of demand.

1. Unitary elastic demand (e = 1)


Demand is said to be unitary elastic when proportionate change in price is equal to
proportionate change in quantity demanded of any commodity. The value of the
elasticity is equal to one (e=1) in such cases demand curve is convex to the origin as
shown in the diagram.
2. Relatively elastic demand (e > 1)
Demand for a commodity is said to be elastic when proportionate change in quantity
demanded is greater than proportionate change in price. In such cases value of the
elasticity is greater than one and shape of the demand curve is flatter as shown in the
following diagram.

 P
e(p) = Q 1
 P
Q P P'O P
= LM

OM
L OP
= 1
M P P'

OM
3. Relatively inelastic demand (e<1)
Demand for a commodity is said to be relatively inelastic when proportionate change in
demand is smaller than proportionate change in price of the commodity. In such cases,
value of the elasticity is less than one (e<1) and the demand curve is steeper. The
following diagram exhibits the said demand curve.

 P
e(p) = Q 1
 P
Q P P'
L
M
= 
O OP
M
OP
L 1
= P P'
M

O
M

4. Perfectly elastic demand (e=  )


Demand for a commodity is said to be perfectly elastic when a small change (rise or fall)
in price brings about either complete contraction or infinite expansion in demand. In such
cases value of the elasticity is infinity (e=  ) and demand curve is horizontal to ‘x’ axis.
The following figure depicts the same.
5. Perfectly inelastic demand (e=0)
Demand for any commodity is said to be perfectly inelastic when there is no change in
demand at a high or low price. The value of the elasticity is zero in such cases and
demand curve is vertical to ‘x’ axis. The following figure depicts the perfectly inelastic
demand.
Determinants of Elasticity
1. Nature of commodity
In case of necessaries of life demand is inelastic while luxuries relatively elastic.

2. Number of uses
In case of large number of uses, demand is relatively elastic and in case a few uses, it
is relatively inelastic.

3. Number of substitutes
If the substitutes are more demand is relatively elastic while less number of
substitutes, demand is relatively inelastic.

4. Durability of goods
Durable goods have relatively elastic demand while perishable goods haverelatively
inelastic demand.

5. Low priced commodities


Low priced commodities like salt, newspapers, matchboxes etc. have relatively
inelastic demand.

6. Proportion of income spent


Commodities needing less expenditure generally have relatively inelastic demand.

Importance of Elasticity

The concept of elasticity is very much useful in day-to-day life. Firstly, it deeply analyses
price-demand relationship. Secondly, it helps producers in fixing prices of their product.
Thirdly it is helpful to government to declare certain industries as public utility services.
Fourthly it also helps the government to frame economic policies. Fifthly it helps finance
minister in matter of taxation. The concept elasticity explains why there exists poverty in the
midst of plenty. It is also helpful in international trade to determine terms of trade between
the two countries.

Income Elasticity of Demand


Pr oportionatechange in quantity demanded
e(i) =
Pr oportionatechange in income
It is defined as “the degree of responsiveness or sensitiveness of demand to a change in
income.” In other words, it shows a degree of responsiveness of demand to a change in
income.
e(i) = Q I

Q I

Cross Elasticity

It measures elasticity of demand of related goods. It means that when price of say ‘x’ good
changes, the demand for related good say ‘y’ changes. Thus, the cross elasticity of demand
measures the response of the quantity demanded of a particular commodity to the change in
price of some other related commodity. Generally it takes place in complementary goods and
substitutes
Pr oportionatechange in quantity demanded of say ' x' good
Cross =
Elasticity Pr oportionatechange in price or related good say ' y' good

The cross elasticity in case of substitutes is always positive but it is negative


in case of complementary goods.
4. FACTORS OF PRODUCTION

LAND, LABOUR, CAPITAL AND ENTREPRENEUR | NATIONAL INCOME


Some of the important factors of production are: (i) Land (ii) Labour (iii) Capital (iv) Entrepreneur.
Whatever is used in producing a commodity is called its inputs. For example, for producing wheat, a farmer
uses inputs like soil, tractor, tools, seeds, manure, water and his own services.
All the inputs are classified into two groups—primary inputs and secondary inputs. Primary inputs render
services only whereas secondary inputs get merged in the commodity for which they are used.
In the above example, soil, tractor, tools and farmer’s services are primary inputs because they render
services only whereas seeds, manure, water and insecticides are secondary inputs because they get merged
in the commodity for which they are used. It is primary inputs which are called factors of production.
Primary inputs are also called factor inputs and secondary inputs are known as non-factor inputs.
Alternatively, production is undertaken with the help of resources which can be categorized into natural
resources (land), human resources (labour and entrepreneur) and manufactured resources (capital).
All factors of production are traditionally classified in the following four groups:

-Land
It refers to all natural resources which are free gifts of nature. Land, therefore, includes all gifts of nature
available to mankind—both on the surface and under the surface, e.g., soil, rivers, waters, forests,
mountains, mines, deserts, seas, climate, rains, air, sun, etc.

-Labour
Human efforts done mentally or physically with the aim of earning income is known as labour. Thus, labour
is a physical or mental effort of human being in the process of production. The compensation given to
labourers in return for their productive work is called wages (or compensation of employees).
Land is a passive factor whereas labour is an active factor of production. Actually, it is labour which in
cooperation with land makes production possible. Land and labour are also known as primary factors of
production as their supplies are determined more or less outside the economic system itself.
-Capital
All man-made goods which are used for further production of wealth are included in capital. Thus, it is man-
made material source of production. Alternatively, all man-made aids to production, which are not
consumed/or their own sake, are termed as capital.
It is the produced means of production. Examples are—machines, tools, buildings, roads, bridges, raw
material, trucks, factories, etc. An increase in the capital of an economy means an increase in the productive
capacity of the economy. Logically and chronologically, capital is derived from land and labour and has
therefore, been named as Stored-Up labour.
-Entrepreneur
An entrepreneur is a person who organizes the other factors and undertakes the risks and uncertainties
involved in the production. He hires the other three factors, brings them together, organizes and
coordinates them so as to earn maximum profit. For example, Mr. X who takes the risk of manufacturing
television sets will be called an entrepreneur.
An entrepreneur acts as a boss and decides how the business shall run. He decides in what proportion
factors should be combined. What and where he will produce and by what method. He is loosely identified
with the owner, speculator, innovator or inventor and organizer of the business. Thus, entrepreneur ship is a
trait or quality owned by the entrepreneur.
Some economists are of the opinion that basically there are only two factors of production—land and
labour. Land they say is appropriated from gifts of nature by human labour and entrepreneur is only a
special variety of labour. Land and labour are, therefore, primary factors whereas capital and entrepreneur
are secondary factors.

5. THEORY OF RENT

➢ Ricardian Theory of Rent

The classical theory of rent is associated with the name of David Ricardo. He begins with a group of new
settlers in a new country.

Let us suppose ourselves to be the settlers in a hitherto unknown island which we shall call jawahar Island
after our late beloved leader. Let Tarapur in 'A' Part of the Island.
This is the most fertile land and gives us the largest produce per acre. Enough land is available of this
quality to satisfy all our needs at the moment. Therefore, it is u free good and will not command any price,
i.e., rent. But as time passes, the mouths to be fed increase in number. This may be due to more
immigrants, who have heard of our good luck, or due to an increase in population.
• Rent in Extensive Cultivation: A time comes when all land of the best quality has been taken up.
But some demand still remains unsatisfied. We have then to resort to ‘B’ quality land. It is inferior to
‘A’ and yields only 30 quintals of wheat per plot as compared with 35 quintals of ‘A’ with the
same expenditure of labour and capital. Naturally plots in ‘A’ now acquire a greater value as
compared with ‘B’. A tenant will be prepared to pay up to 5 quintals of wheat in order to get a plot
in the ‘A’ zone, or take ‘B’ quality land free of charge.

This difference, paid to the owner (if the cultivator is a tenant) or kept to himself (if he is the owner), is
economic rent. In the first case (i.e., when the cultivator is a tenant) it is contractual rent; and in the latter
(i.e., when the cultivator is the owner) it is known as implicit rent. ‘B’ plots do not pay any rent. To go a step
further, we see that after all land of ‘B’ quality has also been
taken up, we begin cultivating ‘C’ plots. Now even ‘B’ quality land comes to have differential surplus over ‘C’.
Rent of ‘A’ increases still further.

When the demand increases still more, we are pushed to the use of the worst land, which is of
‘D’ quality yielding 25 quintals per plot. ‘D’ quality land is now no-rent land or marginal land
while ‘A’, ‘B’, ‘C all earn rent. This growing demand shows itself in rising prices. They raise high
enough to cover the expenses of cultivation on the lowest grade land, i.e., ‘D’ quality.

Let us suppose that one unit of productive effort is equal to Rs. 3,500. When only A’ quality land, where a
plot produces 35 quintals is under the plough, the price of wheat will be Rs. 100 per quintal. When owing
to increased demand, the price of wheat rises to Rs.-110 then and only
then will ‘B’ quality land be cultivated which produces 30 quintals of wheat. And when that
happens ‘A’ land will have a surplus of 5 quintals X Rs. 110 = Rs. 550 per plot. This becomes rent.

The difference, in other words, between the return from a plot of land above the margin and the
marginal plot (i.e., the one just paying its way) is called rent or economic rent.

• Rent in Intensive Cultivation: The settlers in Jawahar Island realize that there is another way too
of increasing the produce. Why not apply more labour and capital to superior lands, and resort to
intensive cultivation? This is done but it is seen that the law of diminishing returns sets in. Now
consider that A, B, C and D are the different doses of labour and capital (instead of different grades
of land) applied to the same grade of land. The first dose yields 35 quintals.

The second unit of labour and capital used on ‘A’ plot will almost definitely give us less than the first. We
suppose it gives us only 30 quintals. So we have the choice of either taking new plots in ‘B’ land, or
cultivating ‘A’ lands more intensively. If we adopt the latter course, the first unit of labour and capital will
be yielding a surplus over the second unit—which unit produces just enough to cover the expenses. This
surplus, again, is rent. As more and more units of labour andcapital are applied, the return per unit will go
on falling.

• Rent Due to Differential Advantages: Transport charges are a part of the cost of production,
because production is complete only when the commodity reaches the hands of
consumers. The better-situated plots, which have to bear less transport charges, will enjoy a
surplus over the distant ones. This surplus will be another cause of rent. Hence, economic
rent is a surplus which arises on account of natural differential advantages, whether of
fertility or of situation, possessed by the land in question over the marginal land.

• No-rent or Marginal Land: The cases described above show that rent is earned due to a certain
paces being better suited for cultivation or being better situated in regard to markets. But better
than what? Of course better than some other plot of land. This ‘some other’ plot is marginal land
which just covers its expenses and no more. This land is called ‘no-rent land’. All rents are measured
from it upwards.
➢ Quality of Land of Does of Capital and Labour

It is quite possible that we may not be able to spot the ‘no-rent land’ because:
(a) It may be paying scarcity rent, or
(b) The owner might have invested some capital in it and the interest thereon might be mistaken for
rent, or
(c) The no-rent land may be in some other country or
(d) The no-rent tracts may form part of a rent-paying area and be concealed in it.

• Scarcity Rent: In our new home-country, Jawahar Island, we at last come to a situation when all
the lands have been brought under the plough, and are being cultivated intensively too. But the
price rises still further under the pressure of demand. Population has been increasing fast. Our
country has become old and no more land is available as we are an island country. Prices of
agricultural produce go up and, therefore, incomes from land go up.

Hence, all land begins to get surplus above expenses. This surplus above costs in the quality land, our
previous no-rent land, is scarcity rent. Superior lands will be paying this surplus over and above differential
gain.

Conclusion: Summing up, we can say that, according to the Ricardian theory, rent is a differential
surplus and arises from the fact that land possesses certain peculiarities as a factor of production. It
is limited in area and its fertility varies. Besides, its situation is fixed. Thus rent results because:

(a) Fertility is more or less fixed by nature;

(b) The total stock of land is fixed and cannot be increased. On this basis, Ricardo defines rent as
“that portion of the produce of the earth which is paid to the landlord for the original and
indestructible powers of the soil.” According to him fertility, situation and limited total stock—
these qualities, which are original as well as permanent, give, rise to rent.

• Criticism of Ricardian Theory:

The Ricardian theory of rent has been widely criticized as under:

(i) It is pointed out that fertility of land is not original: Much of the present productive capacity of
land is the result of human efforts, use of manures and other improvements. Thus, it is not
possible to say which qualities of land are original and which of them are man’s creation. Situation
is something which man cannot change. Obviously it is not possible to move a plot of land to
another place. But man can improve the means of transport so much that the distance between
two places matters little Thus he can manage to change the character of a place. The planned
cities and factory towns of today are the product of man’s brain. Although this criticism has a leg
to stand upon, it cannot be denied that certain original qualities do matter. No human effort will
change Rajasthan into Kashmir.

(ii) The idea of indestructibility is objected to: Area, it is said, is everlasting but not fertility.
Continued cultivation exhausts fertility. We observe this in the case of land in India. Lands are
reported to be less fertile and, therefore, less productive per hectare today than they were in the
past.

Ricardo’s doctrine, however, cannot be wholly rejected. Land which is naturally fertile regains its
fertile qualities more easily, if it is manures or left fallow. Creation of fertility in a barren land is
more difficult. Besides no amount of use will entirely kill the fertility of land.

(iii) Certain American economists like Carey have criticized the classical theory of rent on historical
grounds. They say that cultivation did not begin with the most fertile lands when the first settlers
arrived in America, nor did it pass on to the less fertile lands in that order. The reason was that
some of the most fertile lands were covered with thick forests while others were open to enemy
attack. The settlers naturally preferred less fertile areas which were open and could be defended.
This criticism answers itself. Not necessarily the most fertile, but the land offering the best reward
for a definite effort is occupied first. Moreover, the order of cultivation is not so important. Even if
the order is changed, when two types of land are being cultivated, the more fertile or better
situated plot will produce a surplus above the cost. The surplus will arise whichever land is
cultivated before the other. Rent will still arise even if all the lands were of uniform quality. It will
arise in the intensive form.

(iv) It is said that rent is not due to differential advantages only. Even if all lands were of uniform
quality, rent would still arise. Rent arises from scarcity.

(v) Ricardian theory does not say why rent is paid; it only tells us that superior lands command
higher rent.

(vi) The concept of marginal land is said to be imaginary, theoretical and not realistic.

(vii) It is also urged that no special theory of rent is necessary. Demand and supply theory, which
explains all values, can explain rent also.

(viii) Modern economists think that it is only from the point of view of economy as a whole that
land has perfectly inelastic supply and earns a surplus or rent. This surplus is not included in cost
and hence does not enter into price. But from the point of view of individual farmer or industry, a
payment has to be made to prevent land from being transferred to some other use.

The payment, called transfer earnings, is an element of cost and hence enters into price. For the
individual farmer the whole of rent is cost. “This concept of transfer earnings helps to bring the
simple Ricardian Theory—where transfer earnings are zero because it is the whole economy which is
being studied—into a closer relation with reality.”— (Stonier and Hague).

• Rent as Payment for the Use of Land: Modern View:

So far as the use of land is concerned, the modern economists have offered a better explanation of rent.
This payment is obviously determined by the demand for and the supply of land.

➢ Demand Side: The demand for land is a derived demand. It is derived from the demand for the
products of land. If the demand for these products rises or falls, the demand for the use of land will
correspondingly rise or fall leading to increase or decrease of rents. For instance, if the population
of a country increases, the demand for food will increase, resulting in increased demand for land
and rise m its rent, and vice versa.
➢ Supply Side: The
supply of land is fixed so far as the community is concerned, although individuals can increase their
own supply by acquiring more land from others or decrease its supply by parting with land. In spite
of reclamation projects, the effect of which on the total supply is negligible, the supply of land
remains practically fixed.

It is a case of perfectly inelastic supply, which means that whatever the rent (the rent may rise or fall), the
supply remains the same. That is why it is said that land has no supply price. In other words, the supply of
land in general is absolutely inelastic and as such its supply is independent of what it earns.

➢ Interaction of Demand and Supply: If


the land is of different qualities, then each quality will have a separate demand curve and they will
command different rents. Hence the theory explains differential rent too. Thus, the rent of land, like
the remuneration of other factors, is determined by the equilibrium between demand for and
supply of land.

In other words, it is scarcity in relation to demand that determines rent. Fundamentally speaking, rent is
paid for land because the produce of land is scarce in relation to its demand. The scarcity of land is in fact
derived from the scarcity of its products. It is this scarcity which explains all values and rent is no exception.
➢ Land for a Particular Use:
We have analyzed above total demand and total supply of land for the community as a whole. Let
us now consider it from the point of view of a particular industry or use. For a particular use or
industry, the supply of land cannot be regarded as fixed. By offering more rent, it can be increased;
the supply will decrease if the rent in this particular case goes down.

❖ Modern Theory of Rent

Modern theory of rent is an amplified and modified version of Ricardian theory of Rent. It was first of
all discussed by J.S. Mill and after that developed by economists like Jevons, Pareto, Marshall, Joan
Robinson etc.

According to modern theory, economic rent is a surplus which is not peculiar to land alone. It can be a part of
income of labour, capital, entrepreneur.
According to modern version rent is a surplus which arises due to difference between actual earning and
transfer earning.

i.e. Rent = Actual Earning-Transfer Earning.

➢ What is Transfer Earning?


In this universe, each factor of production has varied uses. When we transfer one factor from one
use to another, we have to sacrifice the income earned by it from its earlier use.
Sacrifice of earning is called transfer earning.

Basically, the concept of transfer earning in economics is introduced by Prof. Benham. According
to him, “The amount of money which any particular unit could earn in its best paid alternative use is
sometimes called its transfer earnings.” A similar idea was developed by Pigou. Different economists
consider transfer earnings as that amount of money which any particular unit could cam in its best paid
alternative use.

Thus, what a person, piece of land or capital can earn in the next best alternative use is known as transfer
earnings. Thus, according to Mrs. Robinson, “The price which is necessary to retain a given unit of three
factors in a certain industry may be called its transfer earning. “Suppose a piece of land can cam Rs. 100/-
when it is used for producing wheat and the same amount if it is used for cotton. There is no extra earning
because there are no transfer earnings. If, however, the same piece of land could cam Rs. 60 when put to
the use of cotton.
Its transfer earning would be Rs. 40 and the extra gain of Rs. 40 which is surplus could be called Rent. So,
according to this theory, we can define rent as a payment of excess of the transfer
earnings. In the words of Benham, “In general the excess of what any unit gets over its transfer earnings is
of the nature of rent.” In the above example, true rent is Rs. 10 and transfer earningRs. 40.

➢ Modern Definitions of Rent:

“Rent is a payment in excess of transfer earning.” Stonier and Hague

“The essence of the conception of rent is the conception of a surplus earned by a particular part of a
factor of production over and above the minimum sum necessary to induce it to do its work”. Mrs.
Joan Robinson

❖ Features of Modern Theory of Rent:

The major features of the modern theory of rent are as under: 1. Rent can be a part of the income of

all factors of production.

2. Amount of rent depends upon the difference between actual earning and transfer earning. 3. Rent

arises when supply of the factor is either perfectly inelastic or less elastic.

➢ Why Rent Arises:


➢ According to modern theory, rent arises due to scarcity of land. Supply of other factors like
labour, capital etc. can also be scare in relation to demand. Therefore, income earned by
these factors in excess of their minimum income is called economic rent.

Prof. Wieser divided factors of production into two parts viz.; specific factors and non•specific factors.

• Specific Factors: These factors refer to those factors which have only one use. For example, a
farm used for growing wheat alone. Such factors have no mobility.

• Non-Specific Factors: These factors are those which have mobility and can be put to
different uses. It is only due to the reason that specific factors cannot be put to another use.
Specificity of factors is the main cause of the emergence of rent. It is so because
specific factors cannot be put to any other use. So, its opportunity cost is zero. In other words, its
transfer earning is zero. So its entire actual earning in the existing use is rent.
❖ Determination of Rent:

Modern economists studied the determination of rent in two forms as:

1. Rent of Land

2. General concept of Rent.

• Determination of Rent of Land or Scarcity Theory of Rent: Modern


economists opined that rent arises due to scarcity of land. Scarcity of land means that
demand for land exceeds its supply. Rent will be determined at a point where demand for
land is equal to its supply.

• Demand for Land: Land has derived demand. It means that demand for land depends on the
demand for agricultural products. If demand for food grains increases, demands for land
will also increase and vice-versa. Moreover, demand for land is influenced by its marginal
productivity. It means as more and more land is used its MP1 goes on diminishing.

• Supply of Land: Supply of land is fixed. Its supply is perfectly inelastic. It means, increase in
the price of land will not evoke any increase in its supply.

Supply of Land Rent as the Difference between Actual Earnings and Transfer Earnings:

According to modern economists rent is the difference between actual earning and transfer earning.
Rent can be a part of income of factors of production. But, these factors will earn rent only when
their supply is less than perfectly elastic. Thus, from elasticity point of view, there are three
possibilities, i.e.:

1. Supply of factors of production is perfectly elastic.

2. Supply of factors of production is perfectly inelastic.

3. Supply of factors of production is less than perfectly elastic.

➢ When Supply is Perfectly Elastic: When change in demand at existing rate is followed by
corresponding change in supply, then the supply is said to be perfectly elastic i.e. such a
factor is not scare. At the existing rate, any amount of that factor is available. Therefore, its
actual earning and transfer earning will be equal.
Actual Earning = Transfer Earning Rent
➢ Actual Earning – Transfer Earning = Zero

• Perfectly Elastic Supply Curve of Land


Since, transfer earnings are equal to actual earnings i.e. OSEN, there is no surplus and, thus, no rent. If this
firm does not pay the price, the factor units will be shifted to other uses and earn there as much, because
present earnings equates the transfer earnings. In this way, we may conclude that if the supply is perfectly
elastic, then there exists no surplus and hence no economic rent.

➢ When the Supply is Inelastic: Inelastic supply of a factor indicates that any increase or
decrease in demand is not followed by the supply. In such a case, transfer earnings will be
zero and the difference between actual earning and transfer earning will be equal to actual
earning. Therefore, all the actual earnings will be called rent.

Rent = Actual Earning (Since Transfer Earning is zero)

• Perfectly Inelastic Supply Curve of Land


➢ When the Supply is less than Perfectly Elastic: Less than perfectly elastic supply means that
the transfer earnings of all the factor units are not equal. Mrs. Joan Robinson used
the concept of ‘Transfer Earnings’ to explain the amount of rent earned by a factor unit in a
particular use. She defines transfer earnings as the price which is necessary to retain a given unit of
a factor in a certain industry.

❖ Theories of Wages (With Criticisms)

The following points highlight the top six theories of wages. The theories are:

1. The Subsistence Theory of Wages


2. Standard of Living Theory
3. Wage Fund Theory
4. Residual Claimant Theory
5. Marginal Productivity Theory
6. Discounted Marginal Productivity Theory.
Theory # 1. The Subsistence Theory of Wages:

The theory was formulated by physiocrats. According to them wages would be equal to theamount just
sufficient for subsistence. Lassale, a German economist developed this theory. According to this theory,
wages are determined by the cost of production of labour or subsistence level. The wages so determined
will remain fixed.

It actual wages are higher than the subsistence level, then population will increase leading to an increase in
labour supply and lower wages. If on the other hand, the actual wages fall below the subsistence level,
population will decrease resulting in a decline in labour supply and rise in wages. Since there is a tendency
for the wages to remain fixed at the subsistence level, it is called as Iron Law of Wages or Brazen Law of
Wages. This theory is based on two assumptions:

a. Food production is subject to the law of diminishing returns, i.e., there is a limit to
expansion of food production.
b. Population increases at an increasing rate.

Criticisms: 1. the
subsistence theory of wages explains wages from the supply side and ignores the
demand side. 2. If all laborers must get the bare necessaries of life, all must get equal wages. But
there are
many differences in wages. Thus this theory ignores wage
differences. 3. This theory asserts that wages are fixed at the subsistence level. Therefore, it assumes
that the
trade unions are powerless in increasing the wages. This is a
wrong notion. 4. This theory is based on the Malthusian theory of population according to which
a rise in wages
above the subsistence level will lead to rapid increase in population. But experience shows that a
rise in wages leads to higher standard of living and not increase
in population. 5. This theory is pessimistic because it excludes all possibility of improvement in the
conditions of
labour either through increased efficiency or due to general economic progress.

Theory # 2. Standard of Living Theory:

This theory is an improved and refined version of subsistence theory. According to this theory, wage is
determined by the standard of living of the workers. Standard of living refers to the bare necessaries of life
and also education, and recreation to which the worker is habituated.
Merits: This theory has two merits:
a. This theory gives importance to the efficiency and productivity of the worker.
b. When workers are paid a high wage rate for a considerable period of time, they
become accustomed to a high standard of living and they will try to maintain the
same high standard of living.
• Criticisms:

In spite of its merits, the theory has been subjected to many criticisms:

1. Individuals do not have any fixed standard of living. Critics point out that there is no such thing as
a standard of living to which a worker is accustomed.

2. When wages depend on standard of living, the latter should not change. But workers’ standard
of living remains fixed for sometimes but wages change frequently.

3. No doubt, wages are determined by standard of living. It is also true that standard of living is
determined by wages.

Theory # 3. Wage Fund Theory:

This theory was developed by J.S.Mill. According to him, the employers set apart a certain amount of
capital to pay wages for labourers. This is fixed and constant. This is called as wages fund. Wage is
determined by the amount of wages fund and the total number of labourers.

According to J.S.Mill, “wages depend upon the demand and supply of labour or as it is often expressed as
proportion between population and capital. By population is here meant the number only of the laboring
classes or rather of those who work for hire and by capital, only circulating
capital ........... “.

Wage rate=Wage fund / Number of labourers

An increase in wage rate is possible only by an increase in wage fund or by a reduction in the number of
labourers. Thus there exists a direct relation between wage rate and wages fund and inverse relation
between wage rate and number of labourers. This theory also states that trade unions are powerless in
rising the general wage rate.
• Criticisms:

1. Wage fund theory states that the wage rate is found by dividing the wage fund by the number of
workers. But it does not tell us about the sources of wages fund and the method of estimating it.

2. Wage fund theory is unscientific and illogical because it first decides the wages fund and then
determines wages. But in reality, wages should be found first and from that wage fund should be
calculated. This theory neglects the quality and efficiency of the workers in determining the wage
rate. This is considered to be a basic weakness of the theory.

3. This theory neglects the quality and efficiency of the workers in determining the wage rate. This is
considered to be a basic weakness of the theory.

4. This theory assumes that wages can increase only at the expense of profit. This is not correct. The
operation of the law of increasing returns will lead to a great increase in total output which may be
sufficient to raise both wages and profits.

5. The wages fund theory has been criticised by the trade unions for its assumption that wages
cannot be increased through bargaining.

6. Wages fund theory has failed to explain the differences in wage rate.

7. This theory believes that wages are paid out of circulating capital. But when the process of
production is short, wages are paid out of current production. When the process of production is
long, wages are paid out of capital.

Theory # 4. Residual Claimant Theory:

This theory was propounded by Walker. According to this theory, rent and interest are contractual
payments. After deducting rent and interest from total product, the employer will deduct his profits. What
remains after deducting rent, interest and profits is wages. It is possible to increase wages by increasing the
total product by improving the efficiency of the workers.

This theory has several defects:

1. This theory assumes that the share of landlords, capitalists and entrepreneurs are fixed and it is
absolutely wrong.

2. It is not the worker who is the residual claimant but the entrepreneur. 3. It does not explain the

influence of trade union in wage determination.


4. The supply side of labour has been totally ignored by the theory.

Theory # 5. Marginal Productivity Theory:

Marginal productivity theory of wages is an extension of marginal productivity theory of distribution.


According to this theory, wage for labour should be equal to the value of the marginal product under
conditions of perfect competition. Marginal product is the addition made to total product by the
employment of one unit of labour. The value of the marginal product of labour is equal to the price at
which the marginal product can be sold.

Under conditions of perfect competition, an employer will continue to employ more and more of labourers
till the value of the marginal product is equal to marginal factor cost(MFC). Marginal factor cost is the cost
of employing an additional worker. In order to find out the marginal productivity of labour we have to
keep the quantity of other factors constant while employing one more unit of labour.

The difference in total production is the marginal productivity. The employment of an additional unit of
labour will result in increase in output and cost. As long as MPP is greater than MFC, the employer will
employ additional units of labour. But he will stop employing additional units of labour when MPP=MC.

➢ Assumptions: This theory is based upon the following assumptions:

1. There is perfect competition in factor market and in product market. 2. Labour is homogeneous.

3. The law of diminishing returns operates in production. 4. There is free entry and exit of the firms.

5. There is perfect knowledge about the market conditions. 6. All factors of production can be

substituted for each other. 7. There is free mobility of factors of production.

8. Factors of production are divisible.


• Criticism: The theory is found to be unsatisfactory and various criticisms have been leveled
against this theory.

1. The theory deals with the demand side only. The supply side is totally ignored.

2. This theory is unjust because wages are determined by the marginal productivity. But justice
demand that workers should be paid on the basis of average productivity.

3. Further, marginal productivity of the worker cannot be calculated as factors are not divisible into
small units.

4. Factors of production are neither mobile nor perfect substitutes. Their Knowledge is also imperfect.

5. This theory assumes perfect competition in the product market. But the market for goods is
characterised by imperfect competition.

6. Marginal product of labour depends not only on its support but also on the supply of other factors.
If other factors are plentiful and labour is scarce, marginal product of labour will be high and vice
versa.

7. This theory fails to explain the differences in wages.

Rejecting the marginal productivity theory Marshall states, “This doctrine has been put forward as a
theory of wages. But there is no valid ground for any such pretension… Demand and supply exert
equally important influences on wages; neither has a claim to predominance; any more than has
either blade of scissors, or either pier of an arch… The doctrine throws into clear light, one of the
causes that governs wages”.

Theory # 6. Discounted Marginal Productivity Theory:

Taussig has given a modified version of the Marginal Productivity theory of wages. According to this
theory, the wage for labour is determined not by its marginal product but by the discounted marginal
product. Labourers cannot get the entire amount of the marginal product because production is a long
drawn out process.

In the same way, sales also take time. As the labourers are poor and cannot wait till the product is sold,
they have to be supported by the employers. The employer does not pay the full amount of the marginal
product of labour. In order to compensate the risk involved in giving advance to the workers, the employer
deducts a certain percentage from the final output. This deduction is
made at the current rate of interest. It is the discounted marginal product that determines thewage of the
labourers.

• Criticisms.......................................................................................................................................... 1.
this theory is abstract. It is “a dim and abstract one remote from the problem of real life”. 2. It is
very difficult to determine the discounted marginal product of labour.
3. This theory fails to take into account other factors which determine the wage rate.
4. This theory has failed to explain differences in wage rate.
Taussing’s theory is another version of the Residual Claimant Theory of wages. Therefore, it is
subject to all criticisms put forward against the Residual Claimant Theory.

6. CAPITAL & INTEREST

CAPITAL
• Wealth, especially in the form of financial or physical assets, used in the production or accumulation of mor
ewealth.
• Accumulated assets or advantages used for economic or political gain: "The president lacks the politicalcapit
al to override their objections" (The Economist).
• The money invested in a corporation, including debt and equity.
• Net worth.
• Capital stock.

Capital in economics is a word of many meanings. They all imply that capital is a “stock” by contrast with
income, which is a “flow.” In its broadest possible sense, capital includes the human population; nonmaterial
elements such as skills, abilities, and education; land, buildings, machines, equipment of all kinds; and all
stocks of goods—finished or unfinished—in the hands of both firms and households.
In the business world the word capital usually refers to an item in the sheet representing that part of the net
worth of an enterprise that has not been produced through the operations of the enterprise. In economics
the word capital is generally confined to “real” as opposed to merely “financial” assets. Different as the two
concepts may seem, they are not unrelated. If all balance sheets were consolidated in a closed economic
system, all debts would be cancelled out because every debt is an asset in one balance sheet and a liability in
another. What is left in the consolidated balance sheet, therefore, is a value of all the real assets of a society
on one side and its total net worth on the other. This is the economist’s concept of capital.
A distinction may be made between goods in the hands of firms and goods in the hands of households, and
attempts have been made to confine the term capital structure to the former. There is also a distinction
between goods that have been produced and goods that are gifts of nature; attempts have been made to
confine the term capital to the former, though the distinction is hard to maintain in practice. Another
important distinction is between the stock of human beings (and their abilities) and the stock of nonhuman
elements. In a slave society human beings are counted as capital in the same way as livestock or machines.
In a free society each man is his own slave—the value of his body and mind is not, therefore, an article of
commerce and does not get into the accounting system. In strict logic persons should continue to be
regarded as part of the capital of a society; but in practice the distinction between the part of the total stock
that enters into the accounting system, and the part that does not, is so important that it is not surprising
that many writers have excluded persons from the capital stock.
Another distinction that has some historical importance is that between circulating and fixed capital. Fixed
capital is usually defined as that which does not change its form in the course of the process of production,
such as land, buildings, and machines. Circulating capital consists of goods in process, raw materials, and
stocks of finished goods waiting to be sold; these goods must either be transformed, as when wheat is
ground into flour, or they must change ownership, as when a stock of goods is sold. This distinction, like
many others, is not always easy to maintain. Nevertheless, it represents a rough approach to an important
problem of the relative structure of capital; that is, of the proportions in which goods of various kinds are
found. The stock of real capital exhibits strong complementarities. A machine is of no use without a skilled
operator and without raw materials for it to work on.

TYPES OF CAPITAL

Working Capital
Working Capital (WC) is calculated by subtracting a corporation’s current liabilities from its current assets.
This financial metric determines a corporation’s operating liquidity and is necessary in understanding how
readily available funds are for necessary financial transactions related to daily operations. A corporation
whose current liabilities are more than its current assets is considered to have a working capital deficiency.
Working Capital (WC) is calculated as follows:
Working Capital = Current Assets - Current Liabilities
The danger of having negative working capital is that a corporation may be unable to meet its short-term
liabilities. Examples of current assets are cash, accounts receivables and inventory. These assets are easily
liquidated for the purpose of raising funds. If a corporation is unable to meet its short-term obligations,
then in a worst case scenario, bankruptcy may be a threat. In addition, negative working capital may also be
a sign of trouble ahead. If negative working capital remains over a long period of time, this may be an
indication that sales volumes are on the decline.
Equity
Equity describes the value of an ownership stake or interest in a property. When a corporation is initially
established, owners contribute funds to help finance assets. In exchange for those funds, a liability is created
on the corporation to its owners in the form of share capital. The capital that is formed is equity and it
represents the sum of the investments made in a corporation - typically in exchange for shares. Equity is
also known as risk capital or liable capital.
Shareholders’ equity is the amount of funds that have been contributed by the owners of the corporation,
plus any retained earnings that have accumulated. Losses incurred by the corporation will negatively affect
shareholders equity since the losses are ultimately the responsibility of the owners. Equity is found on the
corporations balance sheet and is an integral element of how a business’s finances are managed and
accounted for.
Because equity is inherently risky, equity investors are generally looking for at least a 25% return if not more
which is fundamentally the reason why equity capital is considered the most expensive form of capital
compared to mezzanine debt.
Senior Debt
Senior debt is the first level of a corporation’s liabilities which means it is paid out first, ahead of all other
creditors. Senior debt, as opposed to junior debt, is first in seniority and is often secured by collateral in the
form of a lien.
Senior debt is among the safest form of financing for the party providing the funds. Due to its inherent low
risk, it also provides the least amount of return. However, in exchange for this low return, significant
protection is provided even in the event of bankruptcy. Should a corporation go bankrupt, any remaining
funds, dissolved assets or other available sources of value must first repay senior debt before other creditors
are able to collect.
Senior debt is financing that has been lent to a corporation for a pre-negotiated period of time with interest
paid on the principal. The lender profits from this arrangement due to the scheduled period of borrowing
on which the interest applies. The risk is low, since the borrower is contractually obligated to make
payments on a pre-determined schedule. The lender does not gain the benefit of a higher potential return
since the financing and its recoupment is not based on the borrower’s financial performance. For this
reason, senior debt is prioritized over other investments and creditors.
Mezanine Debt
Mezzanine debt or mezzanine capital is a form of hybrid capital that has been around for 30 years and
which can be structured as either preferred equity or unsecured debt. It is generally referred to the layer of
debt that sits between senior debt and equity. Mezzanine debt lays claim to a corporation’s assets, yet also
incorporates equity-based security options in its structure. It is senior only to common shares and is often a
more expensive form of financing because of its positioning and inherently higher level of risk to the lender.
And unlike Venture Capital, Mezzanine debt is used for adolescent and mature companies who are cash
flow positive that need capital for a number of growth-related uses.

GROSS INTEREST AND NET INTEREST


In common language interest is a payment made by a borrower to the lender for the use of money usually
stated as per. cent per year. But economics interest is defined differently by different economists. According
to wick sell interests is "a payment made by the borrower of capital by virtue of its productivity as a reward
for his abstinence." According to Keynes interest is a monetary phenomenon.
It is the reward for parting with liquidity. The whole amount paid by a borrower to the lender for the use of
borrowed fund is known as interest. A lender who receives interest not only receives rewards on capitalize
but also rewards for other factors. The payment which is paid exclusively for the use of capital is known as
net interest.

Net interest is otherwise known as pure interest. Thus Gross interest is a wider concept in which net interest
is a part. Besides net interest Gross interest includes many elements. There elements are reward for risk
taking wages of management and payment for inconveniences.Net interest is a payment for the loan of
capital, when no risk, no inconveniences and no work is entailed on the lender.

NET INTEREST=PAYMENT FOR THE USE OF CAPITAL


Gross interest=Net interest + reward for risk + reward for inconvenience + reward for management.

7. ORGANIZATION & PROFIT

FUNCTIONS OF ENTREPRENEUR
The following points highlight the top five functions of an entrepreneur. The functions are: 1. Decision
Making 2. Management Control 3. Division of Income 4. Risk-Taking and Uncertainty-Bearing 5. Innovation.

Function # 1. Decision Making:


The primary task of an entrepreneur is to decide the policy of production. An entrepreneur is to determine
what to produce, how much to produce, how to produce, where to produce, how to sell and’ so forth.
Moreover, he is to decide the scale of production and the proportion in which he combines the different
factors he employs. In brief, he is to make vital business decisions relating to the purchase of productive
factors and to the sale of the finished goods or services.
Function # 2. Management Control:
Earlier writers used to consider the management control one of the chief functions of the entrepreneur.
Management and control of the business are conducted by the entrepreneur himself. So, the latter must
possess a high degree of management ability to select the right type of persons to work with him. But, the
importance of this function has declined, as business nowadays is managed more and more by paid man-
agers.

Function # 3. Division of Income:


The next major function of the entrepreneur is to make necessary arrangement for the division of total
income among the different factors of production employed by him. Even if there is a loss in the business,
he is to pay rent, interest, wages and other contractual incomes out of the realized sale proceeds.

Function # 4. Risk-Taking and Uncertainty-Bearing:


Risk-taking is perhaps the most important function of an entrepreneur. Modern production is very risky as
an entrepreneur is required to produce goods or services in anticipation of their future demand. Broadly,
there are two kinds of risk which he has to face. Firstly, there are some risks, such as risks of fire, loss of
goods in transit, theft, etc., which can be insured against. These are known as measurable and insurable
risks. Secondly, some risks, however, cannot be insured against because their probability cannot be
calculated accurately. These constitute what is called uncertainty (e.g., competitive risk, technical risk, etc.).
The entrepreneur undertakes both these risks in production.

Function # 5. Innovation:
Another distinguishing function of the entrepreneur, as emphasized by Schumpeter, is to make frequent
inventions — invention of new products, new techniques and discovering new markets — to improve his
competitive position, and to increase earnings.

Conclusion:
The above description indicates the supreme position of the entrepreneur in an organization. This is
particularly true in the capitalistic or even mixed economy which is based on the price-profit system. In the
socialistic economy, the state becomes the entrepreneur; the scope of private entrepreneur is extremely
limited in such an economy.

PROFIT
Meaning
We all are familiar with the term ‘Profit’. It is quite a common-place word, but different people use it in
different senses.
In Economics, however, the term has a precise meaning. Profit may be defined as the net income of a
business after all the other costs—rent, wages and interest etc., have been deducted from the total income.
Profits are, therefore, uncertain and vary from person to person and from firm to firm. They may become
zero, when costs are equal to income, and if the costs are higher, profits may actually be converted into loss.
There are various theories of profit which have been advanced from time to time regarding the nature of
profit. Almost all of them differ basically from one another and are inadequate to explain the actual role of
profit in the operation of free economy.
The most important theories are:
1. Hawley's Risk Bearing Theory of Profit.
2. Uncertainty Theory of Profit.
3. Rent Theory of Profit.
4. Marginal Productivity Theory of Profit.
5. Dynamic Theory of Profit.
6. Monopoly Theory of Profit.

(1) HAWLEY'S RISK BEARING THEORY OF PROFIT


Definition and Explanation:
This risk bearing theory of profit is associated with the name of F.B. Hawley. According to him:
"Profit is the reward of risk taking in a business. During the conduct of any business activity, all other factors
of production, i.e., land, labor and capital have their guaranteed incomes from the entrepreneur. They are
least concerned whether the entrepreneur makes profit or undergoes tosses".
In a business activity, as we know, there are every chance at any moment in the variation of demand for the
commodity produced, The demand may change due to changes in fashion, tastes, condition of trade, prices
of substitutes, distribution of wealth, etc., or the project undertaken may prove to be a complete failure.
In all such cases, if the entrepreneur is not able to cover his total costs from the sale of the commodities,
then it is he who ultimately bears the loss. So he must be compensated for undertaking such risks.
Thus, according to Hawley, profit is a payment or a reward for the assumption of risks by the
entrepreneur. The 'greater the risk, the higher must be the profits. It is because if the return on risky
enterprise is at the same level as that obtained from the safe investment, then not a single entrepreneur will
invest his capital in a risky enterprise.

Criticism:
Hawley's risk theory of profit is criticized on the following grounds:
1. According to Hawley, profit is a reward for bearing risks in a business. The modern economists believe that
there is no doubt that profits contain some remuneration for risk-taking in a business but it is wrong to
assume that profits are in their entirely due to the element of risk- The profits can I arise on account of
better management, better supervision or they may I be due to the monopolistic position of the
entrepreneur or they may be I due to sheer chance, etc., etc.
2. Another criticism levied by Carver is that profits arise not because risks I are borne but because the superior
entrepreneurs are able to reduce the risks.
3. It is also pointed out that profits are never in proportion to the risk undertaken, it can. happen that in a
more risky enterprise, the profits may be low and high in a less risky enterprise,
4. There are certain businesses where risks can be more or less accurately foreseen by statistical evidence, e.g.
in insurance, the entrepreneurs who I run these businesses earn profits. This theory fails to explain as to how
I the profits are earned in such business where the risks can be insured.

(2) UNCERTAINTY THEORY OF PROFIT


Definition and Explanation:
According to Professor Knight:
"Profit is the reward for uncertainly-bearing and not of risk-taking in a business".
According to him there are two kinds of risks which entrepreneur has to bear? Some risks are of such a
nature that they can be anticipated to a fair degree of accuracy, e.g., the risk of death, accident, etc., and so
can be insured in return for premium. The entrepreneur can include the payment made in the form of
premium in the total cost of production, So such risks which can be calculated and insured should not
entitle the entrepreneur to a profit. On the other hand, there are some risks which are unpredictable and
unforeseen and so they are non-insurable.
For instance, if the demand for the product of at entrepreneur suddenly down due to changes in fashions,
tastes, etc., then he may not be able to; cover his total costs of production. Such risks which are unforeseen
and cannot be statistically measured are called by Knight, as uncertainty-bearing risks.
"Profits, according to him are the reward of uncertainty-bearing j rather than risk-taking which is insurable".
Criticism:
1. The total profits which an entrepreneur receives cannot be attributed solely to the element of uncertainty in
a business. He performs other functions also such as coordinating, bargaining, and innovation in the
business. So he must be paid for these services also.
2. It is not simply due to uncertainty-bearing that the supply of entrepreneur is restricted. There are other
factors also which influence the supply the entrepreneur. For instance, lack of knowledge, lack of capital,
opportunity, etc., do restrict the supply of an entrepreneur in a business.

(3) Rent Theory of Profit:


Definition and Explanation:
The Rent Theory of Profit is associated with the name of American economist, Francis A Walker. According
to him:
"Profits are of the same genius as rent".
The main points of Walker's Theory of Profit can be summed up as such:
1. Profit is rental in character. Just as superior grades of land earn more rent than the inferior grades of land,
similarly superior entrepreneurs due to their exceptional ability or opportunity earn more profits than the
inferior entrepreneurs.
2. As in the case of land, there is a no-rent or marginal land, so in the business also is a no-profit or marginal
entrepreneur. The marginal entrepreneur is one whose ultimate receipts from the sale of the commodities
just cover his total costs.
3. Just as rent is measured from the non-rent land, in the same way profits of the superior businessmen are
calculated from the marginal entrepreneur.(iv) The rent does not enter into price of agricultural production
of the manufactured goods.
4. From all that we have said above, it can be concluded that profits are the reward of differential business
ability.

Criticism:
The modern economists have discarded the Walker's rent theory of profit on the following grounds:
1. It simply provides a measure of profit. It does not throw light on the nature of profit which is of more
importance.
2. Marshall is of the opinion that there is much difference between the rent of land and the entrepreneur's
profit. The rent of land can either be positive or zero, but in case of business, the total receipts from the sale
of the product can fall short of total costs. So the entrepreneur may suffer losses and thus his profit may be
in the negative. In the opinion of Marshall, the price of the commodity in the market is determined not by
the cost of production of marginal firm but by the representative firm. Representative firm is that "which has
a fairly long lease of life and has a fair degree of success, which is managed with normal ability and which
has access to the normal economies of production".
3. It is also pointed out that profit may not form a part of the cost of production of a commodity in the short
period but in the long period if the business is to be continued, it must enter in the price of the product.
4. Profits do not arise simply because of the superior or exceptional ability of the entrepreneur, but they can
also result due to chance gains or monopolistic position of the entrepreneur or they may be of the nature of
the windfall income.

(4) Marginal Productivity Theory of Profit:


Definition and Explanation:
According to this theory:
"The earning of entrepreneur like the reward of other factors of production can be explained by the
marginal productivity analysis".
In the words of Champmon:
"The profit tends to be equal to the marginal social worth of the employers in exactly the same sense in
which the labor gets his marginal net product from the employers. The marginal net product of an
entrepreneur is the amount which the community is able to produce with his help over and above what it
could produce without his help".
Thus, we conclude that under conditions of perfect competition, the reward of the entrepreneur tends to be
equal to the .marginal social worth of the employer. If the marginal productivity of the employer is high, the
profit will also be high and the marginal net productivity is low, then profit will also be low.
Criticism:
One very important criticism levied on this theory is that the unit of factor, i.e., the enterprise is very large, if
for finding out the marginal net productivity of the entrepreneur, we withdraw it from the business, and
then it will disorganize the entire productive organization. It, thus, becomes very difficult to ascertain the
marginal net productivity of the labor.

(5) Dynamic Theory of Profit:


Definition and Explanation:
In the world of reality, according to J.B. Clark:
"Profit arises only in a dynamic economy. An economy is said to be dynamic when there is a change in the
population growth or a change in the method of production or a change in the consumers wants, etc., A
society which is without these changes is called a static society. In a static society only, monopoly profits
continue to exist. All other economic profits are gradually eliminated by competition".
In a dynamic society, an entrepreneur is always confronted with continuous unpredictable changes in
demand for his product. The variation in demand may take place due to change in fashions, tastes, standard
of living, distribution of income, population, new inventions, international repercussion and technological
advances, etc. A prudent entrepreneur will always keep an eye on the future demand for his products. If he
succeeds in increasing his sale by lowering the cost of production or by adoption of an innovation, then he
can secure profits. Thus, we find, that profits are a reward, of progress, Schumpeter calls it the reward of
innovation.
In a dynamic economy, if an entrepreneur produces a new thing and creates demand for his products, then
he is likely to obtain big profits. But the profits of the entrepreneur cannot continue to exist for long period.
The other entrepreneurs also adopt the innovation and produce similar products. As total output increases,
the profits, gradually come down. Thus, we find that perpetual profits are the result of perpetual new
successful innovations.
Criticism:
Prof, Knight has criticized the Clarkian Theory of profit on the ground that it is wrong to attribute all profits
to dynamic changes. According to him, there are certain changes which are of a recurring and calculable
nature. They can be anticipated and the output can be adjusted according to that. The profits do not arise
on those regular changes but on those which are unforeseen or unpredictable. He thus observes that:
"It is not dynamic changes nor any changes as such which cause profits but the divergence of actual
conditions from those which have been expected and on the basis of which business arrangements have
been made".

(6) Monopoly Theory of Profit:


Definition and Explanation:
There is no doubt that profits arise from dynamic changes, innovations and from making a correct estimate
of future economic conditions. Another view point of profit is that monopolistic and monopolistic
competition in the market also gives rise to profits. The firms under monopoly or monopolistic competition
have greater control over the price of the product. They are the price makers rather than the price takers. As
such they raise prices by restricting the level of output and thus keep profit at higher level. Monopoly
power, thus, is the basic sources of business profits.
Criticism:
This Kalocki's theory of monopoly profits has also been criticized. It is said that monopoly is no doubt an
important cause and source of monopoly profits but it does not replace other theories. Monopoly power
only supplements other theories.
Conclusion:
After discussing various theories of determination of profits, we come to this conclusion that all these
theories are defective in one way or the other. The basic defect with these theories is that they particularize
certain aspects of the function of an entrepreneur to the neglect of others. While the fact is that his
functions are many aid varied. The profit can arise due to monopolistic position of the entrepreneur or
adoption of innovation or sheer chance or some of the factors stated above.
Thus, we conclude that there is not a single theory of profit which gives as correct and comprehensive
explanation of the nature and determination of the profits. Such a theory is yet to be propounded.
MACROECONOMICSI02
1. INTRODUCTION

In economics money is defined as an asset (a store of value) which functions as a generally accepted medium of
exchange, i.e., it can in principle be used directly to buy any good. A note of IOU (a bill of exchange) may also be
a medium of exchange, but it is not generally accepted and is therefore not money. Generally accepted mediums
of exchange are also called means of payment. So money is characterized by being a fully liquid asset. An asset is
fully liquid if it can be used directly, instantly, and without any costs or restrictions to make payments. Generally,
liquidity should be conceived as a matter of degree so that an asset has a higher or lower degree of liquidity
depending on the extent to which it can easily be exchanged for money or be used directly for making payments.
By “easily” we mean “immediately, conveniently, and cheaply”. So an asset’s liquidity is the ease with which the
asset can be converted into money or be used directly for making payments. Where to draw the line between
“money” and “non-money assets” depends on what is appropriate for the problem at hand. In the list below of
different monetary aggregates 1 corresponds most closely to the traditional definition of, money. Defined as
currency in circulation plus demand deposits held by the 1 embraces all in practice fully liquidnon-bank public
in commercial banks, assets in the hands of the non-bank public. The reason that a market economy uses money
is that money facilitates trade enormously, thereby reducing transaction costs. Money helps an economy to avoid
the need for a “double coincidence of wants”. The classical way of illustrating this is by the exchange triangle from
the Fig. (No direct exchange possible. A medium of exchange, here good 2, solves the problem) The individuals A,
B, and C are endowed with one unit of the goods 1, 3, and 2, respectively. But A, B, and C want to consume 3, 2,
and 1, respectively. Thus, no direct exchange is possible between two individuals each wanting to consume the
other’s good. There is a lack of double coincidence of wants. The problem can be solved by indirect exchange
where A exchanges good 1 for good 2 with C and then, in the next step, uses good 2 in an exchange for good 3
with B. Here good 2 serves as a medium of exchange. If good 2 becomes widely used and accepted as a medium
of exchange, it is money. Extending the example to a situation with n goods, we have that exchange without
money (i.e., barter) requires n(n-1)/2 markets (“trading spots”). Exchange with money, in the form of modern
“paper money”, requires only n markets.

Figure: No direct exchange possible. A medium of exchange, here good 2, solves the problem.

2. FUNCTIONS OF MONEY
1. Primary functions
2. Secondary functions
3. Contingent functions

1. Primary functions- Medium of exchange-the fundamental role of money is to serve as


medium of exchange this is the most important function of money by working as a
medium of exchange money divides the exchange transactions into two parts namely
sale and purchase this function of money has solved one of the biggest problems of the
barter system that is lack of double coincidence of wants any commodity or service can
be bought with and sold for money money represents general purchasing power a
person can sell any commodity today for money and can use the money in future to
purchase any commodity or service some unique characteristics of money like general
acceptability probability durability etc have helped money to work as medium of
exchange

Measure of value or unit of account-the value of all goods and services is expressed in terms of money
it is a unit of account when the value of commodity is expressed in terms of money it is called price it
helps us to compare the value of on commodities by comparing the prices of different commodities
relative values of this commodities can be calculated example price of a table is 2000 and price of a
share is 500 it indicates that the value of a table is equivalent to the value of purchase goods and
services are quantified in different units it would have been difficult to express the value of 1 kilogram
of sugar in terms of certain litre of milk or certain metre of cloth in the absence of money is difficulty is
overcome when the prices of all these goods are expressed in terms of money which is unit of account
every country has a standard money or monetary unit in terms of which values are expressed and
measured example dollars pounds nctc in India rupee is the unit of account incomes and expenditures
of all kinds assets and liabilities of all kinds budgets of the government etc are stated in terms of money
as a unit of account

2. Secondary functions:-Standard of deferred payments-in the modern economy many


transaction take place without instant payments the debtors make a promise to make
payments on some future date such future payments are possible because of money
underwater system taking loan was easy but its requirement was difficult because loans
were in the form of grains or cattle money facilitates lending and borrowing because
the borrowings are in the form of money and the repayment are also in the form of
money due to general acceptability stability of value compared to other goods
durability etc money acts as standard of deferred payments
➢ Store of value-money works as a store of value along with the satisfaction of present
wants provision for satisfaction of future worms is equally important it requires savings
from the current earnings money is a convenient means through which things can be
done easily according to Lord J M Keynes, money is a link between present and the
future Manisha Asus store of value because money has purchasing power.
It can be used to purchase real assets like land house etc. and financial assets like shares debentures
bonds etc.

➢ Transfer of value-Today the extension of trade among various countries and


organizations it becomes necessary to transfer purchasing power from one place to
another this is easily done by money. money helps to shift the purchasing power from
one place to another example real acids like building on agricultural land from one
place can be sold and with the help of that one building or land can be purchased at
some other place

3. Contingent/incidental functions of money:-According to professor kinley, money in


modern times also perform certain contingent functions.

Following are some contingent or incidental functions of money

Measurement and division of national income: Money facilitates estimation and distribution of national
income numerous goods and services are produced in a country during period of time when these
goods and services are converted in terms of money calculation of national income becomes possible
factors of production like land labour capital and organization contribute to national income all these
factors get their respective rewards like rent wages interest and profit in terms of money the total
production and factor prices are easily expressed in terms of money

➢ Basis of credit- Modern economy is based on credit commercial banks create credit on
the basis of their cash holdings without the use of money credit instruments cannot
operate one cannot issue cheque without having a bank balance money provides the
liquid base of the banking system.
➢ Imparts liquidity to wealth- Money is called the most liquid asset money can be easily
converted into any asset and any asset can be converted into money example of person
can purchase gold and if you want to cancel it and can purchase government bonds
securities etc liquidity of money has improved the mobility of capital from a business in
laws to a profit making business it also facilitates transfer of capital from less productive
use of two or more productive use.

Equalization of marginal utilities and marginal productivity is with price-prices of goods services and
prices of all factors of production are expressed in terms of money it helps the consumers to compare
marginal utilities of goods with their prices based on this comparison consumers can allocate their
income various goods in such a way that the price of each commodity is equal to its marginal utility.

Producers compare factor prices like current fall and wages for labour interest for capital with marginal
productivity (contribution made by additional factor* to total productivity) factors of production. The
producers strike maximize their profits by equalizing marginal productivity of a vector with its price.

➢ Estimation of macroeconomics variables-

Macroeconomics variables gross national production, total savings, total investment etc can be easily
estimated in monetary terms. It also facilitates government for collection budget etc
Thus, in modern monetized economy money can facilitate production distribution saving etc. increases
international trade transport organization of capital market and other financial institutions.

Along with these functions money also perform some other functions like

It helps in the maintenance of repayment capacity which is called guarantee of solvency

Money can be used for any purpose according to the priority of an individual or an organization.
Money saved by person to purchase of house in future can be used for higher education of children. P.
Graham has called this function of money as bearer option.

It is the base of price mechanism prices of all goods and all factors of production are expressed in
terms of money price mechanism guides important decisions like what to produce how much to
produce how to distribute etc. when money performs various functions different qualities or
characteristics of money help it to perform this functions.

➢ Measures of credit control by Central Bank

The central bank of country has the responsibility of controlling the volume and direction of credit in the
economy in order to achieve the objective of growth with stability.

The central bank adopts two types of measures for controlling credit activities of the banks in the
economy these are quantitative measures of credit control and qualitative or selective measures of
credit control. Let us study these measures.

Let us study them in detail

➢ Quantitative measures of credit control

These measures are macroeconomic in effect and are used to control the volume or quantity of credit so
as to control the inflationary and deflationary pressures caused by expansion or contraction of credit.
These measures also am at controlling the cost of credit these include

➢ Bank rate
➢ Open market operations
Variable reserve ratio or cash reserve ratio CRR

➢ Bank rate-Bank rate is the minimum rate of interest charged by the central bank to
commercial banks while giving loans to them against eligible securities or by
rediscounting bills of exchange it is also called as it is also rediscount rate. The bank rate
of its both the cost and availability of credit the bank rate is important because it is the
best sector to other market rates of interest the money market rates adjust
automatically to the changes in bank rate the central bank can arrange for reduce the
bank rate to increase or decrease the money supply in the economy.
When the bank rate is increased, the cost of borrowing from Central Bank was of which in turn process
the commercial banks to search higher lending rate to cover up their increased cost businessman and
industrialist or entrepreneurs feel discouraged to borrow more money when the lending rate of interest
charged by commercial banks is hi this would lead to contraction in bank credit and decrease in money
supply in the economy the reduction in money supply will reduce aggregate demand or money
expenditure as a result prices will fall and inflation will be checked Central Bank follows this dear money
policy during the period of inflation to control general rise in prices of all goods and services.

The reverse will happen when the bank rate is reduced in order to overcome the problem of recession
or depression in the economy this is called cheap money policy of the central bank

Derby started with a chip manipulating with bank rate of 3% in 1951 this rate has undergone a lot of
changes from time to time as per the requirements of the economy

➢ Open market operations- Open market operations refer to deliberate buying and selling
of government securities and treasury bills by the central bank in the open market by
doing so the central bank can increase or decrease cash reserves of the bank open
market operations is an important instrument of stabilizing the general price level IT
affects the money supply directly and rate of interest indirectly.

when the central bank sells securities to the public commercial banks and other financial institutions in
open market money flows from them to Central Bank this reduces the demand deposits held by the
public and the cash reserve of banks, which in turn reduces the ability to create credit does the sale of
securities by the central bank leads to a decrease in quantity of money supply in the economy and the
rise in market rate of interest. Listen to reduce the demand for money due to higher cost and those
helps to check inflationary tendencies in the economy.

Windows Central Bank purchases securities money flows from it to commercial banks are public this
increases cash reserves with the bank and helps to expand credit and money supply as a result
interested declines and demand for money increases in this way Central Bank by selling government
securities during the period of prosperity and purchasing government securities during the period of
precession or depression controls the credit activities of the commercial banks

❖ Variable cash reserve ratio

Lord JM Keynes has popularized this as a method of credit control by the central banks.

❖ Cash reserve ratio CRR

Bad banking at commercial banks have to maintain a certain amount of cash with Central Bank for
example RBI as reserves against their demand and time deposits under the RBI act of 1935 every
commercial bank has to keep certain minimum cash reserves with RPF at conveying CRR between 3%
and 15% of total time and demand deposits

This amount cannot be used by banks for lending activities there for the amount available for lending
rates reduced to the extent of cash reserve ratio this reserve ratio is changed to regulate credit it
directly affects the lending capacity of banks and the rate of interest charged by banks.
An increase in CRR leads to contraction of credit increase in lending interest rates and reduction
in money supply in the economy this can reduce inflationary
pressure. Decrease in CRR leads to an expansion of credit or decrease in lending interest rates and
an
increase in money supply in the economy.

When Central Bank wants to reduce money supply the CRR will be raised and it will be reduced
to expand the quantity of money it is very effective instrument as it affects the base of credit
creation.

❖ Statutory liquidity ratio SLR In addition


to CRR the commercial banks have to maintain a certain percentage 25% of their total demand
and time deposits with the RBI in the form of liquid assets that is in the form of cash gold and in
approved securities this is known as statutory liquidity requirement. This amount is kept in
reserve to invest in government securities as every commercial bank has to make this
investments hence it is not available to banks for lending purposes

Central Bank most to decrease money supply in the economy it will increase the seller which will reduce
commercial banks’ ability to create credit a higher salary increase the lending rates of bank in the
central bank forms to increase money supply in the economy it will reduce SLR which will increase
banks’ ability to create credit.

➢ Repo rate or purchase rate that is liquidity adjustment facility LAF

Repo rate or repurchase rate is that rate at which commercial Banks borrow money from the central
bank for short period by selling the securities to the central bank with an agreement to repurchase
them at a future date at pre-determined price

➢ Concept of reverse repo rate

Reverse repo rate is the rate of interest at which the central bank borrow funds from other commercial
banks for short duration the commercial banks deposit their short term excess funds with the central
bank and earn interest on it reverse repo rate is used by the central bank to absorb liquidity from the
economic when it feels that there is too much money floating in the market it increases the reverse
repo rate meaning that Central Bank will pay higher rate of interest to the banks for depositing money
with it.

An increase in the reverse repo rate causes the banks to transfer more funds to the central bank
because banks are attractive interest rates and also their money is in safe hands this results in the
money be drawn out of the banking system does banks are left with lesser funds.

Thus, by reverse repo rate Central Bank injects equality in the banking system and by increasing the
reverse repo rate it absorbs liquidity from the banking system.

➢ Qualitative or selective credit control measures

these measures adopted by the central bank to control the use of credit they do not affect the inter
economy they help to divert the flow of funds to their desirable and product uses these methods are
known as selective credit control measures
1. Regulation of margin requirements
2. Regulation of consumers credit
3. Issue of directives
4. Credit rationing
5. Publicity
6. Moral suasion
7. Direct action
Let us study these measures in detail
1. Regulation of margin requirement- Margin requirements to trimming the loan value of
collateral security offered by borrower the loan value of a security is the difference
between the market value and the machine requirement for example in the market
value of 10 grams of gold is 30000 and the margin requirement is 25% then the loan
value of 10 grams of gold as collateral security will be 22500 that is 30000 - 7500. Equity
shares debentures of the stock companies precious metals Central assets are accepted
by banks as collateral security for granting loans. Increase or decrease in watching
requirements changes the loan value of security demand for loans can be discouraged
by increasing the margin and vice versa. Central Bank has the power to determine the
margin requirements it is a very effective instrument used to control speculative
activities both in commodity market as well as money and capital market.
2. Regulation of consumer credit- This technique of selective or qualitative credit control is
used to regulate the terms and conditions under which consumer installment credit and
hire purchase finance is provided by the banks in developed and developing countries
substantial part of the credit is used for consumer durable goods make cars motorcycle
TV sets refrigerators computers microwave ovens domestic furniture etc. when their
purchased on hire purchase or installment credit system Axis as well as insufficient
demand for these items disturb the production towards this problem consumer credit is
required to be regulated. This method or technique is implemented by determining the
minimum down payment and maximum period of the payment that is maximum
equated monthly installments EMI in order to check consumer credit the central bank
may increase the minimum down payment and to reduce the maximum period of
payment by reducing the number of equated monthly installment this week discuss the
prospective buyers on the other hand or lower down payment and more number of
installments will encourage them.
3. Issue of directives- Recently the central banks have started issuing directives to
commercial banks in this method the central bank issues directives in the form of oral or
return statements or declarations in the newspaper sepals and warnings in order to
follow a particular course of action it may ask commercial banks to be lived in granting
loans so that the credit policy followed by them is in harmony with the overall monetary
policy followed by the commercial banks
4. Credit rationing-In this method the central bank main poses ceiling on the loans and
advances to regulate and control the purpose for which credit is offered by commercial
banks this is done so that industries to not indulge in speculation and shortages and
black marketing can be avoided under this method the creditors rationed by limiting
the amount available to each applicant credit rationing is done during times of
quantitative shortage and declining gold reserves.
5. Publicity-Central Bank may employ publicity as an instrument of credit control not only
for influencing the credit policies who commercial banks but also to educate
influence public opinion in the country in the words of professor de kock, includes publishing
regularly the weekly statements of their assets and liabilities reviews of credit and business
conditions and comprehensive annual reports on their operations and activities money market
and banking conditions generally public finance trade industry agriculture etc the published
literature makes it easy for the commercial bank and public at large to anticipate future changes
in the monetary and credit policies of Central Bank. The central bank provides a guideline to
commercial banks regarding their credit creation activities.
Quantitative and qualitative methods of credit control are not travels but on the contrary they
supplement each other.

6. Moral suasion-Moral suasion implies persuading the commercial banks by the central
bank to corporate with it in following a proper credit policy more rigorously. This
method of credit control is used by Central Bank in many countries that is a
psychological instrument of monetary policy the executive head or governor of the
central bank calls meeting of the heads of commercial banks and during the meeting
explains the need of adopting a particular monetary policy under prevailing economic
conditions and appeals them to operate with the bank in recent years this method has
become more effective has the commercial banks are apprehensive of stronger
measures that may follow in case they fail to adhere to the central banks appeal.

7. Direct action-Central Bank main technique of direct action against defaulting


commercial banks for which has to follow the directions given by it this method can be
used to enforce both quantitative and selective or qualitative credit control and is
generally used not in isolation but as a supplement to other methods the following
modes of direct action may be taken by the central bank. The central bank may refuse
her discounting facilities to those commercial banks which may be granted too much
credit for speculative and productive purposes in excess of their capital and reserves.
The central bank near restraint those from granting advances against the collateral
securities of certain commodities. Picnic charge of penal rate of interest on those banks
which want to borrow from it beyond prescribed limit. The central bank may threaten a
commercial bank to be taken over by debit fails to follow its policies and instructions.

3. INFLATION
Inflation is a key concept in macroeconomics, and a major concern for government policymakers, companies,
workers and investors. Inflation refers to a broad increase in prices across many goods and services in an
economy over a sustained period of time. Conversely, inflation can also be thought of as the erosion in value of
an economy's currency (a unit of currency buys fewer goods and services than in prior periods).
The Consumer Price Index (CPI) is among the most commonly-used measures of inflation. The CPI uses a so-
called "market basket" of goods to measure the changes in prices experienced by average consumers in the
economy. Economists and central bankers will often subdivide the CPI into so-called "core inflation," a measure
that excludes the price of food and energy.
The Producer Price Index (PPI) is a measure of inflation that tracks the prices that producers obtain for their
goods. Though a long-followed economic statistic, the change in composition of some economies away from
manufacturing and towards services is eroding the value of this statistic.
The GDP deflator is another option for measuring prices and inflation. As the name suggests, the GDP deflator is a
price measurement tool that is used to convert nominal GDP to real GDP. The GDP deflator is a broader measure
than the CPI, as it includes goods and services bought by businesses and governments.
While there is little consensus on the "right" rate of inflation for an economy (or even if inflation is necessary at
all), there is little disagreement in the differing impacts of expected and unexpected inflation. When inflation is
expected, agents in the economy can plan for it and act accordingly – businesses raise prices, workers demand
higher wages, lenders raise interest rates and so on.
Unexpected inflation is considerably more problematic. When inflation is higher than expected, it tends to hurt
workers, recipients of fixed incomes, and savers. In contrast, unexpected inflation often benefits companies (who
can raise prices quickly without needing to raise wages in tandem) and borrowers (who can repay their debts with
money that is now worth less than when they borrowed it).
Over the long term, unanticipated inflation can cause a number of problems for an economy. Businesses will
invest less in long-term projects because of the uncertainty of returns, price information becomes distorted, and
consumers will spend more time trying to protect themselves from inflation and less time engaging in productive
activities. Periods of inflation also tend to redirect investment from businesses and toward hard assets, thus
depriving companies of the capital they need to grow and expand.
What causes inflation is also a key argument in economic theory. Some economists believe that there are
different types of inflation – cost-push and demand-pull inflation. Cost-push inflation is supposed to be a type of
inflation caused by rising prices in goods or services with no suitable alternatives. An oft-cited example of this
inflation is the oil crisis of the 1970s. Cost-push inflation is largely a Keynesian argument, as monetarists do not
believe that increased prices for goods and services lead to inflation absent an increase in the money supply.
Demand-pull inflation is a rise in the price of goods and services created by aggregate demand in excess of
aggregate supply, sometimes referred to as "too much money chasing too few goods." As with cost-push
inflation, monetarists argue against the existence of demand-pull inflation absent changes in the money supply.

INFLATIONARY GAP
We have so far used the theory of aggregate demand to explain the emergence of DPI in an economy. This theory
can now be used to analyze the concept of ‘inflationary gap’—a concept introduced first by Keynes. This concept
may be used to measure the pressure of inflation.
If aggregate demand exceeds the aggregate value of output at the full employment level, there will exist an
inflationary gap in the economy. Aggregate demand or aggregate expenditure is composed of consumption
expenditure (C), investment expenditure (I), government expenditure (G) and the trade balance or the value of
exports minus the value of imports (X – M).
Let us denote aggregate value of output at the full employment by Yf. This inflationary gap is given by C + I + G +
(X – M) >Yf. The consequence of such gap is price rise. Prices continue to rise so long as this gap persists.
Inflationary gap thus describes disequilibrium situation.
Inflationary gap is thus the result of excess demand. It may be defined as the excess of planned levels of
expenditure over the available output at base prices. An example will help us to clear the meaning of the concept
of inflationary gap.
Suppose, the aggregate value of output at current price is Rs. 600 crore. The government now takes away output
worth Rs. 100 crore for its own requirements, leaving thus Rs. 500 crore for civilian consumption. National income
analysis says that the value of aggregate money income equals the net value of aggregate output.
Here also the total money income of the people (Rs. 500 crore) is equal to the net value of aggregate output (i.e.,
Rs. 600 crore – Rs. 100 crore = Rs. 500 crore). Thus, prices will remain stable since aggregate expenditure is equal
to aggregate output. Let us further assume that the money income of the community is increased to Rs. 800 crore
by creating additional purchasing power.
Fig: Inflationary Gap
Let us assume that Yf is the full employment level of national income. If C + I + G + (X – M) is the aggregate
demand (AD) curve that cuts the 45° line at point A then an equilibrium income is determined at Y f. There will not
be any price rise since aggregate demand equals aggregate supply. Now if the AD curve shifts up to AD’,
equilibrium output will not increase since output cannot be increased beyond the full employment level.
In other words, because of full employment, output cannot increase to Y*. Thus at Yf level of full employment
output, there occurs an inflationary gap to the extent of AB. The vertical distance between the aggregate demand
and the 45° line at the full employment level of national income is termed the inflationary gap. Or at full
employment, there is an excess demand of AB that pulls up prices.
To describe inflationary gap in a simple way, we use Fig: Inflationary Gap (a). In this figure, we weigh aggregate
demand (i.e., C + I + G + X-M) and aggregate supply. Since the former exceeds the latter, an inflationary gap
emerges.

Fig: Inflationary Gap (a)

Inflationary gap can be eliminated/ minimized by using monetary policy and or fiscal policy instruments. Under
the monetary policy, money supply is reduced and/or interest rates are increased. This gap, however, can be
reduced either by reducing money income through reduction in government expenditure, or by increasing output
of goods and services, or by increasing taxes.

CONTROL OF INFLATION
a) Monetary Measure
b) Fiscal Measure
c) Direct Measure
MONETARY MEASURES:
The government of a country takes several measures and formulates policies to control economic activities.
Monetary policy is one of the most commonly used measures taken by the government to control inflation.
In monetary policy, the central bank increases rate of interest on borrowings for commercial banks. As a result,
commercial banks increase their rate of interests on credit for the public. In such a situation, individuals prefer to
save money instead of investing in new ventures.
This would reduce money supply in the market, which, in turn, controls inflation. Apart from this, the central bank
reduces the credit creation capacity of commercial banks to control inflation.
The monetary policy of a country involves the following:
(a) Rise in Bank Rate:
Refers to one of the most widely used measure taken by the central bank to control inflation.
The bank rate is the rate at which the commercial bank gets a rediscount on loans and advances by the central
bank. The increase in the bank rate results in the rise of rate of interest on loans for the public. This leads to the
reduction in total spending of individuals.
The main reasons for reduction in total expenditure of individuals are as follows;
• Making the borrowing of money costlier:
• Refers to the fact that with the rise in the bank rate by the central bank increases the interest rate on loans
and advances by commercial banks. This makes the borrowing of money expensive for general public.
• Consequently, individuals postpone their investment plans and wait for fall in interest rates in future. The
reduction in investments results in the decreases in the total spending and helps in controlling inflation.
• Creating adverse situations for businesses:
• Implies that increase in bank rate has a psychological impact on some of the businesspersons. They
consider this situation adverse for carrying out their business activities. Therefore, they reduce their
spending and investment.
• Increasing the propensity to save:
• Refers to one of the most important reason for reduction in total expenditure of individuals. It is a well-
known fact that individuals generally prefer to save money in inflationary conditions. As a result, the total
expenditure of individuals on consumption and investment decreases.

(b) Direct Control on Credit Creation:


Constitutes the major part of monetary policy.
The central bank directly reduces the credit control capacity of commercial banks by using the following
methods:

• Performing Open Market Operations (OMO):


• Refers to one of the important method used by the central bank to reduce the credit creation capacity of
commercial banks. The central bank issues government securities to commercial banks and certain private
businesses.
• In this way, the cash with commercial banks would be spent on purchasing government securities. As a
result, commercial bank would reduce credit supply for the general public.
• Changing Reserve Ratios:
• Involves increase or decrease in reserve ratios by the central bank to reduce the credit creation capacity of
commercial banks. For example, when the central bank needs to reduce the credit creation capacity of
commercial banks, it increases Cash Reserve Ratio (CRR). As a result, commercial banks need to keep a
large amount of cash as reserve from their total deposits with the central bank. This would further reduce
the lending capacity of commercial banks. Consequently, the investment by individuals in an economy
would also reduce.
FISCAL MEASURES
Apart from monetary policy, the government also uses fiscal measures to control inflation. The two main
components of fiscal policy are government revenue and government expenditure. In fiscal policy, the
government controls inflation either by reducing private spending or by decreasing government expenditure, or
by using both.
It reduces private spending by increasing taxes on private businesses. When private spending is more, the
government reduces its expenditure to control inflation. However, in present scenario, reducing government
expenditure is not possible because there may be certain on-going projects for social welfare that cannot be
postponed.
Besides this, the government expenditures are essential for other areas, such as defense, health, education, and
law and order. In such a case, reducing private spending is more preferable rather than decreasing government
expenditure. When the government reduces private spending by increasing taxes, individuals decrease their total
expenditure.
For example, if direct taxes on profits increase, the total disposable income would reduce. As a result, the total
spending of individuals decreases, which, in turn, reduces money supply in the market. Therefore, at the time of
inflation, the government reduces its expenditure and increases taxes for dropping private spending.
PRICE CONTROL
Another method for ceasing inflation is preventing any further rise in the prices of goods and services. In this
method, inflation is suppressed by price control, but cannot be controlled for the long term. In such a case, the
basic inflationary pressure in the economy is not exhibited in the form of rise in prices for a short time. Such
inflation is termed as suppressed inflation.
The historical evidences have shown that price control alone cannot control inflation, but only reduces the extent
of inflation. For example, at the time of wars, the government of different countries imposed price controls to
prevent any further rise in the prices. However, prices remain at peak in different economies. This was because of
the reason that inflation was persistent in different economies, which caused sharp rise in prices. Therefore, it can
be said inflation cannot be ceased unless its cause is determined.

4. DEFLATION
Deflation is the reduction of prices of goods, and although deflation may seem like a good thing when you’re
standing at the checkout counter, it’s not. Rather, deflation is an indication that economic conditions are
deteriorating. Deflation is usually associated with significant unemployment, which is only corrected after wages
drop considerably. Furthermore, businesses’ profits drop significantly during periods of deflation, making it more
difficult to raise additional capital to expand and develop new technologies.
“Deflation” is often confused with “disinflation.” While deflation represents a decrease in the prices of goods and
services throughout the economy, disinflation represents a situation where inflation increases at a slower rate.
However, disinflation does not usually precede a period of deflation. In fact, deflation is a rare phenomenon that
does not occur in the course of a normal economic cycle, and therefore, investors must recognize it as a sign that
something is severely wrong with the state of the economy.

WHAT CAUSES DEFLATION?


Deflation can be caused by a number of factors, all of which stem from a shift in the supply-demand curve.
Remember, the prices of all goods and services are heavily affected by a change in the supply and demand, which
means that if demand drops in relation to supply, prices will have to drop accordingly. Also, a change in the
supply and demand of a nation’s currency plays an instrumental role in setting the prices of the country’s goods
and services.
Although there are many reasons why deflation may take place, the following causes seem to play the largest
roles:
1. Change in Structure of Capital Markets: When many different companies are selling the same goods or
services, they will typically lower their prices as a means to compete. Often, the capital structure of the
economy will change and companies will have easier access to debt and equity markets, which they can use
to fund new businesses or improve productivity.
2. There are multiple reasons why companies will have an easier time raising capital, such as declining interest
rates, changing banking policies, or a change in investors’ aversion to risk. However, after they have utilized
this new capital to increase productivity, they are going to have to reduce their prices to reflect the
increased supply of products, which can result in deflation.
3. Increased Productivity: Innovative solutions and new processes help increase efficiency, which ultimately
leads to lower prices. Although some innovations only affect the productivity of certain industries, others
may have a profound effect on the entire economy.
4. For example, after the Soviet Union collapsed in 1991, many of the countries that formed as a result
struggled to get back on track. In order to make a living, many citizens were willing to work for very low
prices, and as companies in the United States outsourced work to these countries, they were able to
significantly reduce their operating expenses and bolster productivity. Inevitably, this increased the supply
of goods and decreased their cost, which led to a period of deflation near the end of the 20th century.
5. Decrease in Currency Supply: As the currency supply decreases, prices will decrease so that people can
afford goods. How can currency supplies decrease? One common reason is through central banking
systems.
6. For instance, when the Federal Reserve was first created, it considerably contracted the money supply of the
United States. In the process, this led to a severe case of deflation in 1913. Also, in many economies,
spending is often completed on credit. Clearly, when creditors pull the plug on lending money, customers
will spend less, forcing sellers to lower their prices to regain sales.
7. Austerity Measures: Deflation can be the result of decreased governmental, business, or consumer
spending, which means government spending cuts can lead to periods of significant deflation. For example,
when Spain initiated austerity measures in 2010, preexisting deflation began to spiral out of control.
8. Deflationary Spiral: Once deflation has shown its ugly head, it can be very difficult to get the economy
under control for a number of reasons. First of all, when consumers start cutting spending, business profits
decrease. Unfortunately, this means that businesses have to reduce wages and cut their own purchases. In
turn, this short-circuits spending in other sectors, as other businesses and wage-earners have less money to
spend. As horrible as this sounds, it continues to get worse and the cycle can be very difficult to break.

EFFECTS OF DEFLATION

Deflation can be compared to a terrible winter: The damage can be intense and be experienced for many seasons
afterwards. Unfortunately, some nations never fully recover from the damage caused by deflation. Deflation may
have any of the following impacts on an economy:
1. Reduced Business Revenues: Businesses must significantly reduce the prices of their products in order to
stay competitive. Obviously, as they reduce their prices, their revenues start to drop. Business revenues
frequently fall and recover, but deflationary cycles tend to repeat themselves multiple times.Unfortunately,
this means businesses will need to increasingly cut their prices as the period of deflation continues.
Although these businesses operate with improved production efficiency, their profit margins will
eventually drop, as savings from material costs are offset by reduced revenues.
2. Wage Cutbacks and Layoffs: When revenues start to drop, companies need to find ways to reduce their
expenses to meet their bottom line. They can make these cuts by reducing wages and cutting positions.
Understandably, this exacerbates the cycle of inflation, as more would-be consumers have less to spend.
3. Changes in Customer Spending: The relationship between deflation and consumer spending is complex and
often difficult to predict. When the economy undergoes a period of deflation, customers often take
advantage of the substantially lower prices. Initially, consumer spending may increase greatly; however,
once businesses start looking for ways to bolster their bottom line, consumers who have lost their jobs or
taken pay cuts must start reducing their spending as well. Of course, when they reduce their spending, the
cycle of deflation worsens.
4. Reduced Stake in Investments: When the economy goes through a series of deflation, investors tend to
view cash as one of their best possible investments. Investors will watch their money grow simply by
holding onto it. Additionally, the interest rates investors earn often decrease significantly as central banks
attempt to fight deflation by reducing interest rates, which in turn reduces the amount of money they
have available for spending.
5. In the meantime, many other investments may yield a negative return or are highly volatile, since investors
are scared and companies aren’t posting profits. As investors pull out of stocks, the stock market inevitably
drops.
6. Reduced Credit: When deflation rears its head, financial lenders quickly start to pull the plugs on many of
their lending operations for a variety of reasons. First of all, as assets such as houses decline in value,
customers cannot back their debt with the same collateral. In the event a borrower is unable to make their
debt obligations, the lenders will be unable to recover their full investment through foreclosures or
property seizures.
7. Also, lenders realize the financial position of borrowers is more likely to change as employers start cutting
their workforce. Central banks will try to reduce interest rates to encourage customers to borrow and
spend more, but many of them will still not be eligible for loans.

DEFLATIONARY GAP
If the equilibrium level of income is estimated to be below the full employment level of income then emerges
deflationary gap. If in the economy there arises insufficient aggregate demand, equilibrium in the economy will
occur to the left of the full employment income (Yf).
In other words, a deflationary gap shows the amount by which aggregate demand must be increased so that
equilibrium level of income is increased to the full employment level. Fig Deflationary Gap shows that
equilibrium level of income is OY* while full employment output is Yf.

Fig. Deflationary Gap


Thus, the economy faces unemployment situation. The distance between the 45° line and the AD line at the full
employment output situation is referred as the deflationary gap. It is AB in
Fig. DeflationaryGap. Since aggregate demand is less than the country’s potential output, the economy suffers
from unemployment of labour and other resources.

MEASURES TO CONTROL DEFLATION


Deflation can be controlled by adopting monetary and fiscal measures in just the opposite manner to control
inflation.

1.1 Monetary Policy:


To control deflation, the central bank can increase the reserves of commercial banks through a cheap money
policy. They can do so by buying securities and reducing the interest rate. As a result, their ability to extend credit
facilities to borrowers increases. But the experience of the Great Depression tells us that in a serious depression
when there is pessimism among businessmen, the success of such a policy is practically nil.
In such a situation, banks are helpless in bringing about a revival. Since business activity is almost at a standstill,
businessmen do not have any inclination to borrow to build up inventories even when the rate of interest is very
low. Rather, they want to reduce their inventories by repaying loans already drawn from the banks.
Moreover, the question of borrowing for long-term capital needs does not arise during deflation when the
business activity is already at a very low level. The same is the case with consumers who faced with
unemployment and reduced incomes do not like to purchase any durable goods through bank loans.
Thus all that the banks can do is to make credit available but they cannot force businessmen and consumers to
accept it. In the 1930s, very low interest rates and the piling up of unused reserves with the banks did not have
any significant impact on the depressed economies of the world. Thus the success of monetary policy in
controlling deflation is severely limited.

2.1 Fiscal Policy:


Fiscal policy through increase in public expenditure and reduction in taxes tends to raise national income,
employment, output, and prices. An increase in public expenditure during deflation increases the aggregate
demand for goods and services and leads to a large increase in income via the multiplier process, while a
reduction in taxes has the effect of raising disposable income thereby increasing consumption and investment
expenditures of the people.
The government should increase its expenditure through deficit budgeting and reduction in taxes. The public
expenditure includes expenditure on such public works as roads, canals, dams, parks, schools, hospitals and other
buildings, etc. and on such relief measures as unemployment insurance, pensions, etc.
Expenditure on public works creates demand for the products of private construction industries and helps in
reviving them while expenditure on relief measures stimulates the demand for consumer goods industries.
Reduction in such taxes as corporate profits tax, income tax, and excise taxes tends to leave more income for
spending and investment.
Borrowing by the government to finance budget deficits utilizes idle money lying with banks and financial
institutions for investment purposes. But the effectiveness of public expenditure primarily depends upon the
public works programme, its importance in the economic system, the volume and nature of public works and their
planning and timing.

DEFICIT FINANCING
Deficit financing, practice in which a government spends more money than it receives as revenue, the difference
being made up by borrowing or minting new funds. Although budget deficits may occur for numerous reasons,
the term usually refers to a conscious attempt to stimulate the economy by lowering tax rates or increasing
government expenditures. The influence of government deficits upon a national economy may be very great. It is
widely believed that a budget balanced over the span of a business cycle should replace the old ideal of an
annually balanced budget. Some economists have abandoned the balanced budget concept entirely, considering
it inadequate as a criterion of public policy.
Deficit financing, however, may also result from government inefficiency, reflecting widespread tax evasion or
wasteful spending rather than the operation of a planned countercyclical policy.
Where capital markets are undeveloped, deficit financing may place the government in debt to foreign creditors.
In addition, in many less-developed countries, budget surpluses may be desirable in themselves as a way of
encouraging private saving.
5. SAVINGS & INVESTMENT

Saving is income not spent, or deferred consumption. Methods of saving include putting money
aside in, for example, a deposit account, a pension account, an investment fund, or as cash.[1]
Saving also involves reducing expenditures, such as recurring costs. In terms of personal finance,
saving generally specifies low-risk preservation of money, as in a deposit account, versus
investment, wherein risk is a lot higher; in economics m Determinants of Saving

There are a number of determinants of saving. These determinants are the major forces that shape the
economic scenario of a country. At the same time, determinants of saving are also responsible for the
development or downfall of the investment sector of a country.
There are a number of factors that are termed as the determinants of savings. Some of these factors are
the demographics of the particular region or country. At the same time, there are the income ratios that
play an important role for the development or downfall of savings rate.
Along with the income level, the production level or the productivity is also related to the savings rate
of a country. Along with these, the factor of consumerism is also among the important determinants of
saving rates.

Consumerism is dominating the modern society and because of the availability of numerous opportunities of
borrowing, individuals are now able to spend as much as they like. Now, the increased availability of
borrowing opportunities is stimulating the process of debt creation and because of this, individuals are
saving less.
This in turn is creating problems for the individuals as well as for the economy.

There are some other factors also that are related to the savings rate and can be termed as determinants of
saving. Among these, there are the uncertainties that are related to the economies of almost every country
and this is particularly true for the developing countries all over the world. Again, the price difference
between the domestic goods and the foreign goods also influences the savings rate. Public Finance is another
important factor for savings.

Some financial decisions of the public sector also play an important role as the determinants of savings. The
percentages of children and old people are also among the determinants of
savings. This section of a country’s population is not expected to generate income. Because of this, the
portion is dependent on the remaining part of the population for maintaining their livelihood. All these
factors cause the saving capacity of the workforce to come down to a certain level.

At the same time, there are the factors like real interest rates and the inflation rates that are also among the
important determinants of saving.

OR broadly, it refers to any income not used for immediate consumption.

➢ Saving Factors, Factors of Saving


The various factors affecting savings decision play a major role in influencing the savings plans of
individuals. There are a number of reasons that influence the savings decisions both at the
microeconomic and macroeconomic level. The various measures that work on the savings decisions try
to explain the tendency of people to consume or save.
Opportunity cost is one of the major factors affecting savings decision. Opportunity cost refers to the
alternative cost that should be waived while consuming certain product or service. In other words it
can also be said that the opportunity cost is actually a benefit that is enjoyed by the purchaser over
certain alternative action.

Generally the opportunity cost is expressed as the relative price of one choice to another. The concept of
opportunity cost affects the decision of savings of an individual gravely.
Before going for purchasing a good or service the individual thinks about the opportunity cost of that
particular purchase and hence decides on saving.
Another important factor that affects the savings decision is the theory of diminishing returns. Also known as
the diminishing marginal returns, the diminishing return says that after a certain point, the additional variable
input unit does not yield the same but rather yields less additional output.
In other way it can also be said that creating a single unit actually costs more variable inputs. This theory
affects the decision making concept of individual and says that after a certain point the consumer prefers to
save rather than going for more consumption.
Economies of scale refers to a process of production where, if there is an increase in the scale of the firm, the
long run average cost is decreased for each unit. The two of the most
popular ways of achieving the economies of scale are – low fixed cost and decreasing
marginal cost and constant marginal cost. This ‘economies of scale’ theory influences a firm’s
decision to save or invest.

❖ Investment Demand: Types, Meaning and Determinants! Levels of national


income and employment in the short run depend upon the level of
aggregate demand. In the Keynes’s two sector model aggregate demand consists of two constituents-
consumption demand and investment demand. Since consumption function is more or less stable in the short
run, investment demand is of crucial importance in the determination of income and employment.

Types: Investment is of three types:


(1) Business Fixed Investment (i.e. investment in fixed capital, such as machines, tools) (2) Residential
Investment i.e., investment in building of houses) &
(3) Inventory Investment (i.e. investment in building stocks of goods and raw materials).
The greater the level of investment, the greater the level of income and employment. According to Keynes,
the equilibrium of a capitalist economy is generally not established at the level of full employment, primarily
because investment demand is insufficient to fill up the saving gap corresponding to the level of full
employment. Therefore, underemployment equilibrium, on which Keynes laid a great stress, is due to the lack
of investment demand ascompared to the amount of saving at full-employment level of income.

On the other hand, as Keynes later showed, when investment demand is more than the magnitude of saving
at full-employment level of income, there emerges an inflationary gap in the economy which brings about the
rise in general price level. We thus see that investment demand plays a vital role in the determination of
national income, employment and prices in the country.

❖ Meaning of Investment:
It is useful to make the meaning of investment clear. When a person buys shares, bonds or debentures of a
public limited company from the market, it is generally said that he has made investment. But this is not the
real investment which determines income and employment in the country and with which we are here
concerned. Buying of existing shares and bonds by an individual is merely a financial investment.

When one individual purchases the shares or bonds, some other one would sell them. Thus, the purchase and
sale of the shares merely represents the change in the ownership of assets which already exists rather the
creation of new capital assets. It is the new addition to the stock of physical capital such as plant, machines,
trucks, new factories and so on that creates income and employment. Therefore, by real investment we mean
the addition to the stock of physical capital.

Thus, in economics, investment means the new expenditure incurred on addition of capital goods such as
machines, buildings, equipment, tools, etc. The addition to the stock of physical capital i.e., net investment
raises the level of aggregate demand which brings about addition to the level of income and employment in
the economy. Keynes and many other economists also include the increase in the inventories of consumer
goods in the capital of the country and therefore in the investment.

❖ Autonomous Investment and Induced Investment:


After Keynes two types of investments have been distinguished. One is autonomous investment and the
second is induced investment. By autonomous investment we mean the investment which does not change
with the changes in the income level and is therefore independent of income. Keynes thought that the level
of investment depends upon marginal efficiency of capital and the rate of interest. He thought changes in
income level will not affect investment. This view of Keynes is based upon his preoccupation with short-run
problem.

He was of the opinion that changes in income level will affect investment only in the long ran. Therefore,
considering as he was the short-ran problem he treated investment as independent of the changes in the
income level. In fact the distinction between autonomous investment and induced investment has been made
by post-Keynesian economists.
Autonomous investment refers to the investment which does not depend upon changes in the income level.
This autonomous investment generally takes place in houses, roads, public undertakings and in other types of
economic infrastructure such as power, transport and communication.

This autonomous investment depends more on population growth and technical progress than on the level
of income. Most of the investment undertaken by Government is of the autonomous nature. The investment
undertaken by Government in various development projects to accelerate economic growth of the country is
of autonomous type. The autonomous investment is depicted in Fig. 8.1 where it will be seen that whatever
the level of national income, investment remains the same at Ia. Therefore the autonomous investment curve
is a horizontal straight line.

On the other hand, induced investment is that investment which is affected by the changes in the level of
income. The greater the level of income, the larger will be the consumption of the community. In order to
produce more consumer goods, more investment has to be made in capital goods so that greater output of
consumer goods becomes possible. Keynes regarded rate of interest as a factor determining induced
investment but the empirical evidence gathered so far suggests that induced investment depends more on
income than on the rate of interest.

Induced investment is shown in Fig. 8.2 where it will be seen that with the increase in national income,
induced investment is increasing. Increase in national income implies that demand for output of goods and
services increases. To produce greater output, more capital goods are required to produce them. To have
more capital goods more investment has to be undertaken. This induced investment is undertaken both in
fixed capital assets and in inventories.

The essence of induced investment is that greater income and therefore greater aggregate demand affects
the level of investment in the economy. The induced investment underlines the concept of the principle of
accelerator, which is highly useful in explaining the occurrence of trade cycles.

❖ Determinants of Investment:
Inducement to invest or investment demand depends upon two factors:

(1) Expected rate of profits to which Keynes gives the name Marginal Efficiency of Capital, and
(2) The rate of interest. It can be easily shown that investment is determined by expected rate of
profit and the rate of interest.

A person having an amount of savings has two alternatives before him. Either he should invest his savings in
machines, factories, etc., or he can lend his savings to others on a certain rate of interest. If investment
undertaken in machines or factories is expected to fetch 25% rate of profit, while the current rate of interest is
only 15%, then it is obvious that the person would invest his savings in machinery or factory.

It follows from above, if investment is to be profitable then the expected rate of profit must not be less than
the current market rate of interest. If the expected rate of profit is greater than the market rate of interest,
new investment will take place.

If an entrepreneur does not invest his own savings but has to borrow from others, then it becomes much
clear that the expected rate of profit from investment in any capital asset must not be less than the rate of
interest he has to pay. For instance, if an entrepreneur borrows funds from others at 15% rate of interest, then
the investment proposed to be undertaken will actually be undertaken only if the expected rate of profit from
it is more than 15 per cent.

We thus see that investment depends upon marginal efficiency of capital on the one hand and the rate of
interest on the other. Investment will be undertaken in any given form of capital asset so long as expected
rate of profit or marginal efficiency of capital falls to the level of the current market rate of interest. The
equilibrium of the entrepreneur is established at the level of investment where expected rate of profit or
marginal efficiency of capital is equal to the current rate of interest. Therefore, the theory of investment is also
based upon the assumption that the entrepreneur tries to maximize his profits.

Of the two determinants of inducement to invest, marginal efficiency of capital or expected rate of profit is of
comparatively greater importance than the rate of interest. This is because rate of interest does not change
much in the short run; it is more or less sticky.

But changes in the expectations of profits greatly affect the marginal efficiency of capital and make it very
much unstable and volatile. As a result of changes in marginal efficiency of capital, investment demand is
greatly affected which causes aggregate demand to fluctuate very much. The changes in aggregate demand
bring about economic fluctuations which are generally known as trade cycles.
When profit expectations of businessmen are good, large investment is undertaken which causes aggregate
demand to rise and bring about conditions of boom and prosperity in the economy. On the other hand,
when expectations regarding profits are adverse, the rate of investment falls which causes decline in
aggregate demand and brings about depression, unemployment and excess productive capacity in the
economy. Thus, the changes in the marginal efficiency of capital play a crucial role in causing changes in the
investment level and economic activity.

The theory of interest that, according to Keynes, the rate of interest is determined by liquidity
preference and the supply of money. The greater the liquidity preference, the greater the rate of
interest. Given the liquidity preference curve, the greater the supply of money, the lower will be the
rate of interest. We have already studied how the rate of interest is determined. We explain below in
detail the concept of marginal efficiency of capital and describe the factors on which it depends.

➢ Marginal Efficiency of Capital:

As has been pointed out above, the concept of marginal efficiency of capital refers to the rate of profit
expected to be made from investment in certain capital assets. The rate of profit expected from an extra unit
of a capital asset is known as marginal efficiency of capital.

Now, the question is how the marginal efficiency of capital is measured. Suppose an entrepreneur invests
money in a certain machinery, how will he estimate the expected rate of profit from it. To estimate the
marginal efficiency of capital, the entrepreneur will first take into consideration how much price he has to pay
for the particular capital asset.

The price which he has to pay for the particular capital asset is called its supply price or cost of capital. The
second thing which he will consider is that how much yield he expects to obtain from investment from that
particular capital asset. A capital asset continues to produce goods and yield income over a long period of
time. Therefore, an entrepreneur has to estimate the prospective yield from a capital asset over his whole life
period. Thus, the supply price and the prospective yields of a capital asset determine the marginal efficiency
of capital.

By deducting the supply price from the prospective yield during whole life of a capital asset the entrepreneur
can estimate the expected rate of profit or marginal efficiency of capital. Keynes has defined the marginal
efficiency of capital in the following words: “I define the marginal efficiency of capital as being equal to that
rate of discount which would make the present value of the series of annuities given by the returns expected
from the capital asset during its life just equal to its supply price.”

Thus, according to Keynes, marginal efficiency of capital is the rate of discount which renders the prospective
yields from a capital asset over its whole life period equal to the supply price of that asset.

Therefore, we can obtain the marginal efficiency of capital in the following way: Supply Price = Discounted

Prospective Yields

C = R1/1+r + R2/(1+r)2+ R3/(1+r)3 ...................... + Rn/(1+r)n

In the above formula, C stands for Supply Price or Replacement Cost and R1, R2, R3, .. Rn etc., represent the
annual prospective yields from the capital asset, r is that rate of discount which renders the annual
prospective yields equal to the supply price of the capital asset. Thus, r represents the expected rate of profit
or marginal efficiency of capital.

The measurement of marginal efficiency of capital can be illustrated by an arithmetical example. Suppose it
costs 3000 rupees to invest in a certain machinery and the life of the machinery is two years, that is, after two
years it becomes quite useless, having no value. Suppose further that in the first year the machinery is
expected to yield income of Rs. 1100 and in the second year Rs. 2420/-. By substituting these values in the
above formula, we can calculate the value of r, that is, the marginal efficiency of capital.

2
Supply Price = Discounted Prospective Yields C = R1/1+r + R2/(1+r)

3000 = 1,100/1+r + 2,420/(1+r)2

On calculating the value of r in the above equation it is found to be equal to 10. In other words, marginal
efficiency of capital is here equal to 10 per cent. If we put the value of r, that is, 10 in the above equation, we
obtain the following:

3,000 = 1100/1.10 + 2,420/ (1.10)2

= 1,000 + 2,000 = 3,000

❖ Marginal Efficiency of Capital in General:

We have explained above the marginal efficiency of a particular type of capital asset. But we also require to
know the marginal efficiency of capital in general. It is the marginal efficiency of capital in general that will
indicate the scope for investment opportunities at a particular time in any economy.

At a particular time in an economy the marginal efficiency of that particular capital asset which yields the
greatest rate of profit, is called the marginal efficiency of capital in general. In other words, marginal
efficiency of capital in general is the greatest of all the marginal efficiencies of various types of capital assets
which could be produced but have not yet been produced. Thus, marginal efficiency of capital in general
represents the highest expected rate of return to the community from an extra unit of a capital asset which
yields the maximum profit which could be produced.

Keeping in view the existing stock of a capital asset, we can always calculate the marginal efficiency of any
particular capital asset. The marginal efficiency of capital will vary when more is invested in a given particular
capital asset. In any given period of time marginal efficiency of capital from every type of capital asset will
decline as more investment is undertaken in it. In other words, marginal efficiency of a particular type of
capital asset will be sloping downward as the stock of capital increases.

The main reason for the decline in marginal efficiency of capital with the increase in investment in it is that
the prospective yields from capital asset fall as more units are installedand used for production of a good.
Prospective yields decline because when more quantity of a good is produced with the greater amount of
capital asset prices of goods decline. The second reason for the decline in the marginal efficiency of capital is
that the supply price of the capital asset may rise because the increase in demand for it will bring about
increase in its cost of production.

❖ Rate of interest and Investment Demand Curve:


We have explained above how the marginal efficiency of capital is estimated. We can represent diminishing
marginal efficiency of capital in general by a curve which will slope downward. This has been done in Fig. 8.3
in which along the X-axis investment in capital assets is measured and along the X-axis marginal efficiency of
capital in general is shown. It will be seen from the figure that when investment in capital asset is equal to 07,,
marginal efficiency of capital is i,. When the investment is increased to OI2, marginal efficiency of capital falls
to i2. Likewise, when investment rises to OI3, marginal efficiency of capital further diminishes to i3.

We have seen above that inducement to invest depends upon the marginal efficiency of capital and the rate
of interest. With the given particular rate of interest and given the curve of marginal efficiency of capital in
general we can show what will be the equilibrium level of investment in the economy. If in Fig. 8.3 along the
Y-axis, rate of interest is also shown, then level of investment can be easily determined.

The equilibrium level of investment will be established at the point where marginal efficiency of capital
becomes equal to the given current rate of interest. Thus, if the rate of interest is i1 then I1 investment will be
undertaken. Since at OI1 level of investment marginal efficiency of capital is equal to the rate of interest i1. If
the rate of interest falls to i2, investment in the capital assets will rise to OI2, since at OI2 level of investment
the new rate of interest i2 is equal to the marginal efficiency of capital. Thus, we see that the curve of marginal
efficiency of capital in general shows the demand for investment or inducement to invest at various rates of
interest.

Hence marginal efficiency of capital curve represents the investment demand curve. This investment demand
curve shows how much investment will be undertaken by the entrepreneurs at various rates of interest. If the
investment demand curve is less elastic, then investment demand will not increase much with the fall in the
rate of interest. But if the investment demand curve or marginal efficiency of capital curve is very much
elastic, then the changes in the rate of interest will bring about large changes in investment demand.

❖ Business Expectations and Investment Demand Curve:


We have studied above that the marginal efficiency of capital depends upon the supply price of capital on
the one hand and prospective yields on the other. It is important to note here that the prospective yields are
greatly affected by the expectations of the entrepreneurial class regarding profit making. These expectations
generally change very frequently. Indeed, it is the profit expectations of the entrepreneur which determine the
level of investment. When the expectations of the entrepreneur regarding profit making become dim, the
marginal efficiency of capital declines and as a result demand for investment falls.
The occurrence of depression is mainly due to the pessimistic expectations of the entrepreneurial class
regarding profit making. On the contrary, when the expectations of the entrepreneurs regarding profit
opportunities increase, their inducement to invest rises. As a result of the increase in investment, aggregate
demand in the economy increases and levels of income and employment increase. We thus see that profit
expectations of entrepreneurs greatly affect investment demand and consequently the level of income and
employment.

We have explained above that, according to Keynes, marginal efficiency of capital along with rate of interest
determines the level of investment and therefore of the levels of income and employment. A significant
contribution of Keynes is that he emphasized the important role that business expectations play in
determining the prospective yield from assets on which marginal efficiency of capital and therefore the level
of investment depends. He drew distinctions between short-term expectations and long-term expectations.

Keynes thought that prospective yields of capital assets depended on partly existing facts which we can
assume to be known with a fair degree of certainty and partly on future events which can be predicted with a
degree of uncertainty and involves greater risk. It is the existing facts which are known almost with certainty
that determine short-term expectations.

The short-term business expectations about prospective yields from investment depend on (i) the
existing stock of capital assets & (ii) the strength of consumer demand for goods which require a good
deal of those assets for their production. The investors think that the current consumption demand
will continue in immediate future too and base their expectations of prospective yield from
investment in capital assets on it.

As regards the state of long term expectations, Keynes emphasized future changes in the stock of capital
assets and changes in the level of aggregate demand during the entire future life of the assets whose
prospective yields are being estimated.

Business men acquiring new assets which have long life for production of goods are more concerned with
long-run forces on which future earnings from capital assets depend. It is these long-run forces which make
long-run expectations about future yields that make them quite uncertain. Changes in these long term
expectations make investment quite volatile.
Therefore, Keynes gave great importance to the state of confidence for determining investment.

It should be carefully noted that when the expectations regarding prospective yields change, the whole curve
of the marginal efficiency of capital will shift. If profit expectations fall, the marginal efficiency of capital curve,
that is, investment demand curve will shift downward to left, as shown by the shift of the curve from II to I”I”
in Fig. 8.4. On the other hand, when the profit expectations of the entrepreneurial class become better than
before, the marginal efficiency of capital curve will shift upward to the right, as shown by the marginal
efficiency of capital curve I’I’. Downward shift in the marginal efficiency of capital curve indicates that at the
given rate of interest, less investment will be undertaken than before.

And upward shift in the marginal efficiency of capital curve indicates that more will be invested at the given
rate of interest than before. In Fig. 8.4 when in the beginning, investment demand curve, that is, marginal
efficiency of capital curve is represented by II, at the rate of interest r, demand for investment is OI, As a result
of downward shift in the marginal efficiency of capital curve to I”I”, investment demand at the rate of interest
r falls to OI”. When the marginal efficiency of capital shifts upward to I’I’, investment demand at the
given rate of interest r rises to OI’.
It follows, therefore, that the rate of investment depends on marginal efficiency of capital and the rate of
interest. If investment demand is interest-elastic, a reduction in rate of interest tends to stimulate investment.
But it may fail to do so, if marginal efficiency of investment is already lower than the rate of interest (as may
well happen during a depression). Of the two determinants of the rate of investment, marginal efficiency of
investment is more volatile than the rate of interest.

The rate of interest is usually ‘sticky’ in the short run, while marginal efficiency of investment can fluctuate
from one extreme to another. If there is a divergence between the two, usually the marginal efficiency of
investment will adjust to the rate of interest. If, for example, the marginal efficiency of investment is 20 per
cent, while the current rate of interest is 12 per cent, forces will be set in motion so as to bring the former to
the level of the latter.

In such a situation, there will be more investment because marginal efficiency of investment is greater than
the rate of interest, and with the increase in investment, marginal efficiency of investment will fall. At the
point where it is just reduced to the level of the current rate of interest, further investment will cease.

❖ The Relationship between Saving and Investment!

An important controversy in macroeconomics relates to the relationship between saving and investment.
Many economists before J.M. Keynes were generally of the view that saving and investment are generally not
equal; they are equal only under condition of equilibrium.
Besides, they thought that equality between saving and investment is brought about by
changes in the rate of interest. Keynes in his famous work “General Theory of Employment, Interest and
Money” put forward the view that saving and investment are always equal.

This gave rise to a severe controversy in economics as to whether saving and investment are always equal or
they are generally unequal. This controversy has now been resolved, and there is general agreement among
the economists about the correct relationship between saving and investment.

Modern economists use the concepts of saving and investment in two different senses. In one sense, saving
and investment are always equal, equilibrium or no equilibrium. In the second sense, saving and investment
are equal only in equilibrium; they are unequal under conditions of disequilibrium. We shall explain below in
detail the relationship between savingand investment in these two different senses.

When in a certain year there is net addition to the stock of capital, investment is said to have taken place. It is
worth mentioning here that by investment we do not mean the stock of capital but the net addition to the
stock of capital i.e., investment is a flow concept. Of course, addition to the stock of capital is made through
the flow of investment. In every year stock of capital expands through net investment.

On the other hand, by saving we mean the part of the income which has not been spent on consumer goods
and services. In other words, saving is the difference between income and consumption expenditure. It is
worth noting that in consumption expenditure all types of expenditure are not included. If an individual
spends a part of his income on providing irrigation facilities, on buying tools and machinery, then that
expenditure is not the consumption expenditure, it is in fact an investment expenditure.

In order to obtain the saving, we have only to deduct the consumption expenditure from income and not the
investment expenditure. When an individual makes investment expenditure he is deemed to spend his saved
income on investment. For instance, if a
farmer’s annual income is Rs. 10,000 and he spends Rs. 6,000 on consumer goods and services and spends
Rs. 1,000 on the construction of a well for his fields, and another Rs. 1,000 on building a drainage system for
his fields and providing fencing, then his saving would be 10 – 6 = Rs. 4 thousands.
The expenditure of Rs. 2,000 on well, drainage and fencing will be included in the saving and will not
constitute the consumption expenditure. If Y represents the national income of a country and C the total
consumption, then the saving of the country will be equal to Y – C. Thus, S = Y – C

Ex-post Savings and Ex-post Investment are always equal:


Pre-Keynesian economists were of the view that savings and investment are generally not equal. This is firstly
because saving and investment are made by two different classes of people. While investment is undertaken
by entrepreneurial class of the society, saving is done by the general public. Secondly, saving and investment
depend upon different factors and are made for different purposes and motives.

Therefore, it is not inevitable that savings and investment of a society must always be equal. Besides, some
pre-Keynesian economists pointed out that investment expenditure is also undertaken by borrowing money
from the banks which create new credit for this purpose.

It was thus pointed out that more amount of investment than savings is possible because excess of
investment over savings is financed by new bank credit. But Keynes expressed a totally opposite view that
saving and investment are always equal. The sense in which savings and investment are always equal refers to
the actual savings and actual investment made in the economy during a year.

They are also called ex-post saving and ex-post investment. If we have to calculate that during the year 2002-
03, how much actual savings and investment have been made in India, we will have to deduct the total
consumption expenditure made by the citizens of India during that year from the national income.

Likewise, the real investment during the year 2002-03 of the Indian economy will be obtained by summing up
the investments actually made by the Indian people during that year. In fact, national income estimates of
savings and investment are made in this actual or ex-post sense.

The second sense in which saving and investment words are used is that in a certain year how much saving or
how much investment people of the country desire or intend to do. Therefore, saving and investment in this
sense are known as desired, intended or planned savings and investment. They are also called ex-ante saving
and ex-ante investment.

Keynes in his book, “General Theory of Employment, Interest and Money” showed that in spite of the fact that
saving and investment are done by two different classes of people and also for different purposes and
motives, actual saving and actual investment are always equal.

Thus, he used the word saving and investment in the ex-post or actual sense and proved the equality between
saving and investment in the following way:

Income of a country is earned in two ways:


(1) By producing and selling consumer goods and services, and (2) By producing and selling
capital goods.
i.e. national income of a country is composed of the value of consumer goods and services and the value of
capital goods.
This can be expressed in the form of the following equation:
National Income = Consumption + Investment or Y = C + I ….(i)
where Y stands for national income, C for consumption and I for investment.

The above equation represents the production or earning side of the national income. The second aspect of
national income is the expenditure side. The total national income can be fully consumed but generally it does
not happen so. In actual practice, a part of the total income is spent on consumption and the remaining part
is saved.
From this we get the following equation: National Income
= Consumption + Saving Or Y = C + S .......................................(ii)
where Y stands for national income, C for consumption and S for saving.

In the above two equations (i) and (ii) it is clear that national income is equal to the sum of consumption and
investment and also equal to the sum of consumption and saving.
From this it follows that:
Consumption + Saving = Consumption + Investment C + S = C + I ….(iii)
In equation (iii) above, since C occurs on both sides of the equation, we get:
Saving = Investment or S = I
From the foregoing analysis, it follows that saving and investment are defined in such a Way that
they are necessarily equal to each other. In equation (i) investment is that part of national income
which is obtained from the production of goods other than those consumed and equation (ii) saving is
that part of national income which is not spent on consumption.

Hence the actual or ex-post sense, saving and investment by definition are equal. It is worth mentioning that
in macroeconomics, saving and investment do not refer to the saving and investment by an individual; they
refer to the saving and investment of the whole community or economy. Saving and investment by an
individual can differ but in the ex-post sense, the saving of the whole country must always be equal to the
investment.

Now the question arises, why ex-post saving and ex-post investment are always equal. For instance, when
more investment is undertaken by the entrepreneurs how actual saving becomes equal to this larger
investment and if the saving falls how investment will become equal to smaller savings. In this connection it is
worth mentioning that modern economists, as did Keynes, include the addition to the inventories of consumer
goods in investment.

Now, when saving increases, it implies that consumption will be less. The decline in consumption would result
in the addition to the inventories of consumer goods with the shopkeepers and manufacturers, which were
not planned or intended by them. This addition to inventories, though unintended, will raise the level of actual
investment.

Thus unintended increase in inventories will raise the level of investment and in this way investment will
increase to become equal to the greater saving. On the other hand, if in any year saving declines, it will result
in the unplanned decline in the inventories of consumer goods with the traders and manufacturers. This
unintended decline in inventories will mean
the fall in actual investment. In this way, investment will decline to become equal to the lower savings.

❖ Ex-ante saving and Ex-ante Investment are Equal only in Equilibrium:


As said above, in the desired, planned or ex-ante sense, saving and investment can differ. In fact planned or
ex-ante saving and investment are generally not equal to each other. This is due to the fact that the persons
or classes who save are different from those who invest.

Savings are done by general public for various objectives and purposes. On the other hand, investment is
made by the entrepreneurial class in the community and is generally governed by marginal efficiency of
capital on the one hand and rate of interest on the other hand.

Therefore, savings and investment in planned or ex-ante sense generally differ from each other. But through
the mechanism of change in the income level, there is tendency for ex- ante saving and ex-ante investment to
become equal.

When in a year planned investment is larger than planned saving, the level of income rises. At a higher level of
income, more is saved and therefore intended saving becomes equal to intended investment. On the other
hand, when planned saving is greater than planned investment in a period, the level of income will fall.

At a lower level of income, less will be saved and therefore planned saving will become equal to planned
investment. We thus see that planned or ex-ante saving and planned or ex-ante investment are brought to
equality through changes in the level of income. When ex-ante saving and ex-ante investment are equal, level
of income is in equilibrium i.e., it has no tendency to rise or fall.

It is thus clear that whereas realised or ex-post saving is equal to realised or ex-post investment, intended,
planned or ex-ante saving and investment may differ; intended or ex- ante saving and investment have only a
tendency to be equal and are equal only at the equilibrium level of income.

That the planned or intended saving is equal to intended investment only at the equilibrium level of income
can be easily understood from Fig. 8.3. In this figure, national income is measured along the X-axis while
saving and investment are measured along the Y-axis.
SS is the saving curve which slopes upward indicating thereby that with the rise in income, saving also
increases. II is the investment curve. Investment curve II is drawn as horizontal straight line because, following
Keynes, it has been assumed that investment is independent of the level of income i.e., it depends upon
factors other than the current level of income.

It will be seen from the Fig. 8.5 that saving and investment curves intersect at point E. Therefore, OY is the
equilibrium level of income. If the level of income is OY1, the intended investment is Y1H whereas the
intended saving is Y1L. It is thus clear that at OY1 level of income, intended investment is greater than
intended saving.

As a result of this, level of income will rise and at higher levels of income more will be saved. It will be seen
that with the rise in income to OY2, saving rises and becomes equal to investment. On the other hand, if in
any period, level of income is OY3 intended investment is Y3K and intended saving is Y3J. As a result of this,
level of national income will fall to OY2 at which ex-ante saving and ex-ante investment are once again equal
and thus level of national income is in equilibrium.

To sum up, whereas ex-post savings and ex-post investment are always equal, ex-ante saving and ex-
ante investment are equal only in equilibrium.

6. COMPONENTS OF ECONOMY
An industry involved in the extraction and collection of natural resources, such as copper and timber, as well
as by activities such as farming and fishing. A company in a primary industry can also be involved in turning
natural resources into products. Primary industry tends to make up a larger portion of the economy of
developing countries than they do for developed countries. See also service industry, secondary industry. The
primary sector is concerned with the extraction of raw materials. It includes fishing, farming and mining.
Primary industry is a larger sector in developing countries; for instance, animal husbandry is more common in
countries in Africa than in Japan. Mining in 19th-century South Wales provides a case study of how an
economy can come to rely on one form of activity.
In developed countries the primary industry has become more technologically advanced, for instance the
mechanization of farming as opposed to hand picking and planting. In more developed countries, additional
capital is invested in primary means of production. As an example, in the United States' corn belt, combine
harvesters pick the corn, and sprayers spray large amounts of insecticides, herbicides and fungicides,
producing a higher yield than is possible using less capital-intensive techniques. These technological
advances and investment allow the primary sector to require less workforce and, this way, developed
countries tend to have a smaller percentage of their workforce involved in primary activities, instead having a
higher percentage involved in the secondary and tertiary sectors.
Developed countries are allowed to maintain and develop their primary industries even further due to the
excess wealth. For instance, European Union agricultural subsidies provide buffers for the fluctuating inflation
rates and prices of agricultural produce. This allows developed countries to be able to export their
agricultural products at extraordinarily low prices. This makes them extremely competitive against those of
poor or underdeveloped countries that maintain free market policies and low or non-existent tariffs to
counter them. Such differences also come about due to more efficient production in developed economies,
given farm machinery, better information available to farmers, and often larger scale.

The secondary sector of the economy includes industries that produce a finished, usable product or are
involved in construction. This sector generally takes the output of the primary sector and manufactures finished
goods or where they are suitable for use by other businesses, for export, or sale to domestic consumers. This
sector is often divided into light industry and heavy industry. Many of these industries consume large
quantities of energy and require factories and machinery to convert raw materials into goods and products.
They also produce waste materials and waste heat that may cause environmental problems or cause
pollution. The secondary sector supports both the primary and tertiary sector.
Some economists contrast wealth-producing sectors in an economy such as manufacturing with the service
sector which tends to be wealth-consuming. Examples of service may include retail, insurance, and
government. These economists contend that an economy begins to decline as its wealth-producing sector
shrinks. Manufacturing is an important activity to promote economic growth and development. Nations that
export manufactured products tend to generate higher marginal GDP growth which supports higher incomes
and marginal tax revenue needed to fund the quality of life initiatives such as health care and infrastructure
in the economy. The field is an important source for engineering job opportunities. Among developed
countries, it is an important source of well-paying jobs for the middle class to facilitate greater social mobility
for successive generations on the economy.

The tertiary sector or service sector is the third of the three economic sectors of the three- sector theory. The
others are the secondary sector (approximately the same as manufacturing), and the primary sector (raw
materials).
The service sector consists of the production of services instead of end products. Services (also known as
"intangible goods") include attention, advice, access, experience, and affective labor. The production of
information has long been regarded as a service, but some economists now attribute it to a fourth sector, the
quaternary sector.
The tertiary sector of industry involves the provision of services to other businesses as well as final consumers.
Services may involve the transport, distribution and sale of goods from producer to a consumer, as may
happen in wholesaling and retailing, pest control or entertainment. The goods may be transformed in the
process of providing the service, as happens in the restaurant industry. However, the focus is on people
interacting with people and serving the customer rather than transforming physical goods.

❖ Difficulty of definition
It is sometimes hard to define whether a given company is part and parcel of the secondary or tertiary sector.
And it is not only companies that have been classified as part of that sector in some schemes; government
and its services such as police or military, and non-profit organizations such as charities or research
associations can also be seen as part of that sector.
In order to classify a business as a service, one can use classification systems such as the United Nations'
International Standard Industrial Classification standard, the United StatesStandard Industrial Classification
(SIC) code system and its new replacement, the North American Industrial Classification System (NAICS), the
Statistical Classification of Economic Activities in the European Community (NACE) in the EU and similar
systems elsewhere. These governmental classification systems have a first-level hierarchy that reflects whether
the economic goods are tangible or intangible.
For purposes of finance and market research, market-based classification systems such as the Global Industry
Classification Standard and the Industry Classification Benchmark are used to classify businesses that
participate in the service sector. Unlike governmental classification systems, the first level of market-based
classification systems divides the economy into functionally related markets or industries. The second or third
level of these hierarchies then reflects whether goods or services are produced.
❖ Theory of progression
For the last 100 years, there has been a substantial shift from the primary and secondary sectors to the
tertiary sector in industrialized countries. This shift is called tertiarisation. The tertiary sector is now the largest
sector of the economy in the Western world, and is also the fastest-growing sector. In examining the growth
of the service sector in the early Nineties, the globalistKenichi Ohmae noted that:
"In the United States 70 percent of the workforce works in the service sector; in Japan, 60 percent, and in
Taiwan, 50 percent. These are not necessarily busboys and live-in maids. Many of them are in the professional
category. They are earning as much as manufacturing workers, and often more.”
Economies tend to follow a developmental progression that takes them from a heavy reliance on agriculture
and mining, toward the development of manufacturing (e.g. automobiles, textiles, shipbuilding, steel) and
finally toward a more service-based structure. The first economy to follow this path in the modern world was
the United Kingdom. The speed at which other economies have made the transition to service-based (or
"post- industrial") economies has increased over time.
Historically, manufacturing tended to be more open to international trade and competition than services.
However, with dramatic cost reduction and speed and reliability improvements in the transportation of people
and the communication of information, the service sector
now includes some of the most intensive international competition, despite residual protectionism.
Issues for service providers
Service providers face obstacles selling services that goods-sellers rarely face. Services are intangible, making
it difficult for potential customers to understand what they will receive and what value it will hold for them.
Indeed, some, such as consultants and providers of investment services, offer no guarantees of the value for
price paid.
Since the quality of most services depends largely on the quality of the individuals providing the services,
"people costs" are usually a high fraction of service costs. Whereas a manufacturer may use technology,
simplification, and other techniques to lower the cost of goods sold, the service provider often faces an
unrelenting pattern of increasing costs.
Product differentiation is often difficult. For example, how does one choose one investment adviser over
another, since they are often seen to provide identical services? Charging a premium for services is usually an
option only for the most established firms, who charge extra based upon brand recognition.
Examples of tertiary sector industries Examples of tertiary industries may include:
• Telecommunication
• Hospitality industry/tourism
• Mass media
• Healthcare/hospitals
• Public health
• Information technology
• Waste disposal
• Consulting
• Gambling
• Retail sales
o Fast-moving consumer goods (FMCG)
• Franchising
• Real estate
• Education
• Financial services
o Banking
o Insurance
o Investment management
• Professional services
o Accounting
o Legal services
o Management consulting
The informal sector, informal economy, or grey economyis the part of an economy that is neither taxed nor
monitored by any form of government. Unlike the formal economy, activities of the informal economy are not
included in a country's gross national product (GNP) or gross domestic product (GDP). The informal sector
can be described as a grey market in labour.
Other concepts that can be characterized as informal sector can include the black market (shadow economy,
underground economy), agorism, and System D. Associated idioms include "under the table", "off the books",
and "working for cash".
Although the informal sector makes up a significant portion of the economies in developing countries, it is
often stigmatized as troublesome and unmanageable. However, the informal sector provides critical economic
opportunities for the poor and has been expanding rapidly since the 1960s.Integrating the informal economy
into the formal sector is an important policy challenge.
➢ Definition
The original use of the term 'informal sector' is attributed to the economic development model put forward
by W. Arthur Lewis, used to describe employment or livelihood generation primarily within the developing
world. It was used to describe a type of employment that was viewed as falling outside of the modern
industrial sector. An alternative definition uses job security as the measure of formality, defining participants
in the informal economy as those "who do not have employment security, work security and social security".
While both of these definitions imply a lack of choice or agency in involvement with the informal economy,
participation may also be driven by a wish to avoid regulation or taxation. This may manifest as unreported
employment, hidden from the state for tax, social security or labour law purposes, but legal in all other
aspects. Edgar L. Feige has proposed a taxonomy for describing unobserved economies including the informal
economy as being characterized by some form of non-compliant behavior with an institutional set of rules.
Feige argues that circumvention of labor market regulations specifying minimum wages, working conditions,
social security, and unemployment and disability benefits gives rise to an informal economy that deprives
some workers of deserved benefits while conveying undeserved benefits to others.
The term is also useful in describing and accounting for forms of shelter or living arrangements that are
similarly unlawful, unregulated, or not afforded protection of thestate. 'Informal economy' is increasingly
replacing 'informal sector ‘as the preferred descriptor for this activity.
Informality, both in housing and livelihood generation has often been seen as a social ill, and described either
in terms of what participant's lack, or wish to avoid. A countervailing view, put forward by prominent Dutch
sociologist Saskia Sassen is that the modern or new 'informal' sector is the product and driver of advanced
capitalism and the site of the most entrepreneurial aspects of the urban economy, led by creative
professionals such as artists, architects, designers and software developers. While this manifestation of the
informal sector remains largely a feature of developed countries, increasingly systems are emerging to
facilitate similarly qualified people in developing countries to participate.

➢ Characteristics
The informal sector is largely characterized by several qualities: easy entry, meaning anyone who wishes to
join the sector can find some sort of work which will result in cash earnings, a lack of stable employer-
employee relationships, a small scale of operations, and skills gained outside of a formal education. Workers
who participate in the informal economy are typically classified as employed. The type of work that makes up
the informal economy is diverse, particularly in terms of capital invested, technology used, and income
generated. The spectrum ranges from self-employment or unpaid family labor to street vendors, shoe shiners,
and junk collectors. On the higher end of the spectrum are upper-tier informal activities such as small-scale
service or manufacturing businesses, which have more limited entry. The upper-tier informal activities have
higher set-up costs, which might include complicated licensing regulations, and irregular hours of operation.
However, most workers in the informal sector, even those are self-employed or wage workers, do not have
access to secure work, benefits, welfare protection, or representation. These features differ from businesses
and employees in the formal sector which have regular hours of operation, a regular location and other
structured benefits.
The most prevalent types of work in the informal economy are home-based workers and street vendors.
Home-based workers are more numerous while street vendors are more visible. Combined, the two fields
make up about 10–15% of the non-agricultural workforce in developing countries and over 5% of the
workforce in developed countries.
While participation in the informal sector can be stigmatized, many workers engage in informal ventures by
choice, for either economic or non-economic reasons. Economic motivations include the ability to evade
taxes, the freedom to circumvent regulations and licensing requirements, and the capacity to maintain certain
government benefits. A study of informal workers in Costa Rica illustrated other economic reasons for staying
in the informal sector, as well as non-economic factors. First, they felt they would earn more money through
their informal sector work than at a job in the formal economy. Second, even if workers made less money,
working in the informal sector offered them more independence, the chance to select their own hours, the
opportunity to work outside and near friends, etc.
While jobs in the formal economy might bring more security and regularity, or even pay better, the
combination of monetary and psychological rewards from working in the informal sector proves appealing for
many workers.
The informal sector was historically recognized as an opposition to formal economy, meaningit included all
income earning activities beyond legally regulated enterprises. However, this understanding is too inclusive
and vague, and certain activities that could be included by that definition are not considered part of the
informal economy. As the International Labour Organization defined the informal sector in 2002, the informal
sector does not include the criminal economy. While production or employment arrangements in the
informal economy may not be strictly legal, the sector produces and distributes legal goods and services. The
criminal economy produces illegal goods and services. The informal economy also does not include the
reproductive or care economy, which is made up of unpaid domestic work and care activities. The informal
economy is part of the market economy, meaning it produces goods and services for sale and profit. Unpaid
domestic work and care activities do not contribute to that, and as a result, are not a part of the informal
economy.

➢ Social and political implications and issues


According to development and transition theories, workers in the informal sector typically earn less income,
have unstable income, and do not have access to basic protections and services. The informal economy is
also much larger than most people realize, with women playing a huge role. The working poor, particularly
women, are concentrated in the informal economy, and most low-income households rely on the sector to
provide for them. However, informal businesses can also lack the potential for growth, trapping employees in
menial jobs indefinitely. On the other hand, the informal sector can allow a large proportion of the population
to escape extreme poverty and earn an income that is satisfactory for survival.
Also, in developed countries, some people who are formally employed may choose to perform part of their
work outside of the formal economy, exactly because it delivers them more advantages. This is called
'moonlighting'. They derive social protection, pension and child benefits and the like, from their formal
employment, and at the same time have tax and other advantages from working on the side.
From the viewpoint of governments, the informal sector can create a vicious cycle. Being unable to collect
taxes from the informal sector, the government may be hindered in financing public services, which in turn
makes the sector more attractive. Conversely, some governments view informality as a benefit, enabling
excess labor to be absorbed, and mitigating unemployment issues. Recognizing that the informal economy
can produce significant goods and services, create necessary jobs, and contribute to imports and exports is
critical for governments.
As the work in informal sector is not monitored or registered with the state, its workers are not entitled to
social security, nor can they form trade unions.

Women tend to make up the greatest portion of the informal sector, often ending up in the most erratic and
corrupt segments of the sector. In developing countries, most of the female non-agricultural labor force is in
the informal sector. Major occupations in the informal sector include home-based workers (such as
dependent subcontract workers, independent own account producers, and unpaid workers in family
businesses) and street vendors, which
both are classified in the informal sector. In India, women working in the informal sector often work as
ragpickers, domestic workers, coolies, vendors, beauticians, construction laborers, and garment workers.
Female representation in the informal sector is attributed to a variety of factors. One such factor is that
employment in the informal sector is the source of employment that is most readily available to women. A
2011 study of poverty in Bangladesh noted that cultural norms, religious seclusion, and illiteracy among
women in many developing countries, along with a greater commitment to family responsibilities, prevent
women from entering the formal sector.
According to a 2002 study commissioned by the ILO, the connection between employment in the informal
economy and being poor is stronger for women than men. While men tend to be over-represented in the top
segment of the informal sector, women overpopulate the bottom segment. Men are more likely to have
larger-scale operations and deal in non- perishable items while few women are employers who hire others.
Instead, women are more likely to be involved in smaller-scale operations and trade food items. Women are
under- represented in higher-income employment positions in the informal economy and over- represented
in lower-income statuses. As a result, the gender gap in terms of wage is higher in the informal sector than
the formal sector. Labor markets, household decisions, and states all propagate this gender inequality.

➢ Political power of agents

Workers in the informal economy lack a significant voice in government policy.[13] Not only is the political
power of informal workers limited, but the existence of the informal economy creates challenges for other
politically influential actors. For example, the informal workforce is not a part of any trade union, nor does
there seem a push or inclination to change that status. Yet the informal economy negatively affects
membership and investment in the trade unions. Laborers who might be formally employed and join a union
for protection may choose to branch out on their own instead. As a result, trade unions are inclined to
oppose the informal sector, highlighting the costs and disadvantages of the system. Producers in the formal
sector can similarly feel threatened by the informal economy. The flexibility of production, low labor and
production costs, and bureaucratic freedom of the informal economy can be seen as consequential
competition for formal producers, leading them to challenge and object to that sector. Last, the nature of the
informal economy is largely anti- regulation and free of standard taxes, which diminishes the material and
political power of government agents. Whatever the significance of these concerns are, the informal sector
can shift political power and energies.
The relationship between the informal sectors and poverty certainly is not simple nor does a clear, causal
relationship exist. An inverse relationship between an increased informal sector and slower economic growth
has been observed though. Average incomes are substantially lower in the informal economy and there is a
higher preponderance of impoverished employees working in the informal sector. In addition, workers in the
informal economy are less likely to benefit from employment benefits and social protection programs. For
instance,a survey in Europe shows that the respondents who have difficulties to pay their household bills
have worked informally more often in the past year than those that do not (10% versus 3% of the
respondents).
Children work in the informal economy in many parts of the world. They often work as scavengers (collecting
recyclables from the streets and dump sites), day laborers, cleaners, construction workers, vendors, in
seasonal activities, domestic workers, and in small workshops; and often work under hazardous and
exploitative conditions. It is common for children to work as domestic servants across Latin America and parts
of Asia. Such children are very vulnerable to exploitation: often they are not allowed to take breaks or are
required
to work long hours; many suffer from a lack of access to education, which can contribute to social isolation
and a lack of future opportunity. UNICEF considers domestic work to be among the lowest status, and reports
that most child domestic workers are live-in workers and are under the round-the-clock control of their
employers. Some estimates suggest that among girls, domestic work is the most common form of
employment.
During times of economic crisis many families experience unemployment and job loss, thus compelling
adolescents to supplement their parents’ income by selling goods or services to contribute to the family
economy. At the core, youth must compromise their social activities with other youth, and instead prioritize
their participation in the informal economy, thus manufacturing a labor class of adolescents who must take
on an adult role within the family. Although it revolves around a negative stigma of deviance, for a majority of
individuals, mostly people of color, and the informal economy is not an ideal choice but a necessity for
survival. Participating in the informal economy is becoming normalized due to the lack of resources available
in low-income and marginalized communities, and no matter how hard they have to work, will not advance in
the economic hierarchy. When a parent is either unemployed or their job is on low demand, they are
compelled to find other methods to provide for themselves but most importantly their children. Yet, due to
all the limitations and the lack of jobs, children eventually cooperate with their parent/s and also work for
their family's economic well-being. By having to assist in providing for the family, children miss out on their
childhood because instead of engaging in activities other youth their age participate in, they are obligated to
take on an adult role, put the family first and contribute to the family's well-being.
The participation of adolescents in the informal economy, is a contentious issue due to the restrictions and
laws in place for youth have to work. One of the main dilemmas that arise when children engage in this type
of work, is that privileged adults, denounce children participation as forced labor. Due to the participant
being young, the adults are viewed as “bad” parents because first they cannot provide for their children,
second they are stripping the child from a “normal” childhood, and third, child labor is frowned upon.
Furthermore, certain people believe that children should not be working because children do not know the
risks and the pressure of working and having so much responsibility, but the reality is that for most families,
the children are not being forced to work, rather they choose to help sustain
their family’s income. The youth become forced by their circumstances, meaning that because of their
conditions, they do not have much of a choice. Youth have the capability to acknowledge their family’s
financial limitations and many feel that it is their moral obligation to contribute to the family income. Thus,
they end up working without asking for an allowance or wage, because kids recognize that their parents
cannot bring home enough income alone, thus their contribution is necessary and their involvement
becomes instrumental for their family's economic survival.
Emir Estrada and Pierrette Hondagneu-Sotelo have gone to predominantly Latinx communities of Los
Angeles, CA. to observe the daily actions of street vendors. They analyze why adults participate in the informal
economy. Although it revolves around a negative stigma of deviance, for a majority of individuals, the
informal economy is not an ideal choice but an action necessary for survival. While witnessing the constant
struggle of Latinx individuals to make ends meet and trying to earn money to put food on the table, they
witnessed how the participation of children either benefits the family or even hurt it.
Through field notes derived from their participation, Estrada states, “children are not the
‘baggage’ that adult immigrants simply bring along. In the case of street vendors, we see that they are also
contributors to family processes”. Estrada's findings demonstrate that children are working in order to help
contribute to their household income, but most importantly, they play a vital role when it comes to language
barriers. The kids are not simply workers, they achieve an understanding of how to manage a business and
commerce.
➢ Expansion and growth
The division of the economy into formal and informal sectors has a long heritage. Arthur Lewis in his seminal
work Economic Development with Unlimited Supply of Labour, published in the 1950s, was the celebrated
paradigm of development for the newly independent countries in the 1950s and 1960s. The model assumed
that the unorganized sector with the surplus labour will gradually disappear as the surplus labour gets
absorbed in the organized sector. The Lewis model is drawn from the experience of capitalist countries in
which the share of agriculture and unorganized sector showed a spectacular decline, but it didn't prove to be
true in many developing countries, including India. On the other hand, probabilistic migration models
developed by Harris and Todaro in the 1970s envisaged the phenomenon of the informal sector as a
transitional phase through which migrants move to the urban centers before shifting to formal sector
employment. Hence it is not a surprise to see policy invisibility in the informal sector. Curiously, the informal
sector does not find a permanent place in the Marxian theory since they anticipate the destruction of the pre-
capitalist structure as a result of the aggressive growth of capitalism. To them, in the course of development,
'the small fish is being eaten by the big fish'. Therefore, neither in the Marxian theory nor in the classical
economic theory, the unorganized sector holds a permanent place in the economic literature. The informal
sector has been expanding as more economies have started to liberalize. This pattern of expansion began in
the 1960s when a lot of developing countries didn't create enough formal jobs in their economic
development plans, which led to the formation of an informal sector that didn't solely include marginal work
and actually contained profitable opportunities. In the 1980s, the sector grew alongside formal industrial
sectors. In the 1990s, an increase in global communication and competition led to a restructuring of
production and distribution, often relying more heavily on the informal sector.
Over the past decade, the informal economy is said to account for more than half of the newly created jobs in
Latin America. In Africa it accounts for around eighty percent. Many explanations exist as to why the informal
sector has been expanding in the developing world throughout the past few decades. It is possible that the
kind of development that has been occurring has failed to support the increased labor force in a formal
manner. Expansion can also be explained by the increased subcontracting due to globalization and economic
liberalization. Finally, employers could be turning toward the informal sector to lower costs and cope with
increased competition.
Such extreme competition between industrial countries occurred after the expansion of the EC to markets of
the then new member countries Greece, Spain and Portugal, and particularly after the establishment of the
Single European Market (1993, Treaty of Maastricht). Mainly for French and German corporations it led to
systematic increase of their informal sectors under liberalized tax laws, thus fostering their mutual
competitiveness and against small local competitors. The continuous systematic increase of the German
informal sector was stopped only after the establishment of the EURO and the execution of the Summer
Olympic Games 2004, which has been the first and (up to now) only in the Single Market. Since then the
German informal sector stabilized on the achieved 350 bn € level which signifies an extremely high tax
evasion for a country with 90% salary-employment.
According to the Swedish International Development Cooperation Agency (SIDA), the key drivers for the
growth of the informal economy in the twenty-first century include: limited absorption of labour, particularly
in countries with high rates of population or urbanisation
• excessive cost and regulatory barriers of entry into the formal economy, often motivated by
corruption
• weak institutions, limiting education and training opportunities as well as infrastructure development
• increasing demand for low-cost goods and services
• migration motivated by economic hardship and poverty
• difficulties faced by women in gaining formal employment
Historically, development theories have asserted that as economies mature and develop, economic activity
will shift from the informal to the formal sphere. In fact, much of the economic development discourse is
centered around the notion that formalization indicates how developed a country's economy is; for more on
this discussion see the page on fiscal capacity. However, evidence suggests that the progression from
informal to formal sectors is not universally applicable. While the characteristics of a formalized economy –
full employment and an extensive welfare system – have served as effective methods of organizing work and
welfare for some nations, such a structure is not necessarily inevitable or ideal. Indeed, development appears
to be heterogeneous in different localities, regions, and nations, as well as the type of work practiced. For
example, at one end of the spectrum of the type of work practiced in the informal economy are small-scale
businesses and manufacturing; on the other "street vendors, shoe shiners, junk collectors and domestic
servants." Regardless of how the informal economy develops, its continued growth that it cannot be
considered a temporary phenomenon.

➢ Policy suggestions
As it has been historically stigmatized, policy perspectives viewed the informal sector as disruptive to the
national economy and a hindrance to development. The justifications for such criticisms include viewing the
informal economy as a fraudulent activity that results in a loss of revenue from taxes, weakens unions, creates
unfair competition, leads to a loss of regulatory control on the government's part, reduces observance of
health and safety standards, and reduces the availability of employment benefits and rights. These
characteristics have led to many nations pursuing a policy of deterrence with strict regulation and punitive
procedures.
In a 2004 report, the Department for Infrastructure and Economic Cooperation under SIDA explained three
perspectives on the role of government and policy in relation to the informal economy.
• Markets function efficiently on their own; government interference would only lead to inefficiency
and dysfunction.
• The informal economy functions outside of government control, largely because those who
participate wish to avoid regulation and taxation.
• The informal economy is enduring; suitable regulation and policies are required.
As informal economy has significant job creation and income generation potential, as well as the capacity to
meet the needs of poor consumers by providing cheaper and more accessible goods and services, many
stakeholders subscribe to the third perspective and support government intervention and accommodation.
Embedded in the third perspective is the significant expectation that governments will revise policies that
have favored the formal sphere at the expense of the informal sector.
Theories of how to accommodate the informal economy argue for government policies that, recognizing the
value and importance of the informal sector, regulate and restrict when necessary but generally work to
improve working conditions and increase efficiency and production.
The challenge for policy interventions is that so many different types of informal work exist; a solution would
have to provide for a diverse range of circumstances. A possible strategy would be to provide better
protections and benefits to informal sector players. However, such programs could lead to a disconnect
between the labor market and protections, which would not actually improve informal employment
conditions. In a 2014 report monitoring street vending, WIEGO suggested urban planners and local economic
development strategists study the carrying capacity of areas regularly used by informal workers and deliver
the urban infrastructure necessary to support the informal economy, including running water and toilets,
street lights and regular electricity, and adequate shelter and storage facilities.
That study also called for basic legal rights and protections for informal workers, such as appropriate licensing
and permit practices.
An ongoing policy debate considers the value of government tax breaks for household services such
cleaning, babysitting and home maintenance, with an aim to reduce the shadow economy's impact. There are
currently systems in place in Sweden and France which offer 50 percent tax breaks for home cleaning services.
There has also been debate in the UKabout introducing a similar scheme, with potentially large savings for
middle-class families and greater incentive for women to return to work after having children. The European
Union has used political measures to try and curb the shadow economy. Although no definitive solution has
been established to date, the EU council has led dialogue on a platform that would combat undeclared work.
The World Bank's 2019 World Development Report on The Changing Nature of Workdiscusses the extension
of social assistance and insurance schemes to informal workers given that, in 2018, 8 in 10 people in
developing countries still receive no social assistance and 6 in 10 work informally.

➢ Asia-Pacific
The International Labour Organization mentioned that in most developing nations located in the Asia-Pacific,
the informal sector comprises a significant and vital percentage of the labor force. This sector constitutes
around 60 percent of the labor force. Informal economy includes economic activities of laborers (legally and
in practice) which are not or inadequately covered by official employment contracts or agreements. Informal
employment means payment of wagers may not be guaranteed and retrenchment can be implemented
without prior notice or compensation from employers. There are generally substandard health and safety
conditions as well as nonexistence of social benefits which include sick pay, pension, and health coverage. The
informal economy absorbs a larger part of the ever- growing workforce in urban hubs. In 2015, urban
populations of Asian countries started to grow while the service sector also continued to increase. These
developments contributed to the extensive expansion of urban informal economy in practically all of Asia. In
India, the country’s informal sector accounted for over 80 percent of the non-agricultural industry
during the last 20 years. Inadequate employment denotes the option for majority of India’s citizens is to find
work in the informal sector which continues to grow because of the contract system and outsourcing of
production. An article in First Post (June 2018) said approximately 1.3 billion people or more than 68 percent
of employed persons in the Asia- Pacific earn through the informal economy. It is prevalent in the countryside
(around 85 percent) and almost 48 percent in urban locations. 2 billion of the global population (61 percent)
works in the informal sector. According to an article published in Eco-Business in June 2018, the informal
sector has emerged as an essential component of the economic environment of cities in this region.
Henceforth, the importance of contribution of informal workers deserves recognition.

7. CONCEPT OF GDP & GNP


GROSS DOMESTIC PRODUCT (GDP)
The most important concept of national income is Gross Domestic Product. Gross domestic product is the
money value of all final goods and services produced within the domestic territory of a country during a year.
Algebraic expression under product method is,
GDP=(P*Q)where,
GDP=Gross Domestic Product
P=Price of goods and service
Q=Quantity of goods and service denotes the summation of all values.

According to expenditure approach, GDP is the sum of consumption, investment, government expenditure,
net foreign exports of a country during a year.Algebraic expression under expenditure approach is,
GDP=C+I+G+(X-M)
Where,
C=Consumption
I=Investment
G=Government expenditure
(X-M)=Export minus import

GDP includes the following types of final goods and services. They are:
1. Consumer goods and services.
2. Gross private domestic investment in capital goods.
3. Government expenditure.
4. Exports and imports.

GROSS NATIONAL PRODUCT (GNP)


Gross National Product is the total market value of all final goods and services produced annually in a country
plus net factor income from abroad. Thus, GNP is the total measure of the flow of goods and services at
market value resulting from current production during a year in a country including net factor income from
abroad. The GNP can be expressed as the following equation:

GNP=GDP+NFIA (Net Factor Income from Abroad)


or, GNP=C+I+G+(X-M)+NFIA
Hence, GNP includes the following:
1. Consumer goods and services.
2. Gross private domestic investment in capital goods.
3. Government expenditure.
4. Net exports (exports-imports).
5. Net factor income from abroad.
CAPITAL FORMATION

➢ Meaning of Capital Formation:


Capital formation means increasing the stock of real capital in a country.
In other words, capital formation involves making of more capital goods such as machines, tools, factories,
transport equipment, materials, electricity, etc., which are all used for future production of goods.
For making additions to the stock of Capital, saving and investment are essential.

Process of Capital Formation:


In order to accumulate capital goods some current consumption has to be sacrificed. The greater the extent
to which the people are willing to abstain from present consumption, the greater the extent that society will
devote resources to new capital formation. If society consumes all that it produces and saves nothing, future
productive capacity of the economy will fall as the present capital equipment wears out.
In other words, if whole of the current productive activity is used to produce consumer goods and no new
capital goods are made, production of consumer goods in the future will greatly decline. To cut down some
of the present consumption and wait for more consumption in the future require far-sightedness on the part
of the people. There is an old Chinese proverb, “He who cannot see beyond the dawn will have much good
wine to drink at noon, much
green wine to cure his headache at dark, and only rain water to drink for the rest of his days.”

• Three Stages in Capital Formation:


Although saving is essential for capital formation, but in a monetized economy, saving may not directly and
automatically result in the production of capital goods. Savings must be invested in order to have capital
goods. In a modern economy, where saving and investment are done mainly by two different classes of
people, there must be certain means or mechanism whereby the savings of the people are obtained and
mobilized in order to give them to the businessmen or entrepreneurs to invest in capital.
Therefore, in a modern free enterprise economy, the process of capital formation consists of the following three
stages:
(a) Creation of Savings: An increase in the volume of real savings so that resources, that would
have been devoted to the production of consumption goods, should be released for purposes of
capital formation.
(b) Mobilization of Savings: A finance and credit mechanism, so that the available resources are
obtained by private investors or government for capital formation.
(c) Investment of Savings: The act of investment itself so that resources are actually used for the
production of capital goods.
We shall now explain these three stages:
(a) Creation of Savings: Savings are done by individuals or households. They save by not
spending all their incomes on consumer goods. When individuals or households save, they release
resources from the production of consumer goods. Workers, natural resources, materials, etc., thus
released are made available for the production of capital goods.
The level of savings in a country depends upon the power to save and the will to save. The power to save or
saving capacity of an economy mainly depends upon the average level of income and the distribution of
national income. The higher the level of income, the greater will be the amount of savings.
The countries having higher levels of income are able to save more. That is why the rate of savings in the
U.S.A. and Western European countries is much higher than that in the under- developed and poor countries
like India. Further, the greater the inequalities of income, the greater will be the amount of savings in the
economy. Apart from the power to save, the total amount of savings depends upon the will to save. Various
personal, family, and national considerations induce the people to save.
People save in order to provide against old age and unforeseen emergencies. Some people desire to save a
large sum to start new business or to expand the existing business.
Moreover, people want to make provision for education, marriage and to give a good start in business for
their children.
Further, it may be noted that savings may be either voluntary or forced. Voluntary savings are those savings
which people do of their own free will. As explained above, voluntary savings depend upon the power to save
and the will to save of the people. On the other hand, taxes by the Government represent forced savings.
Moreover, savings may be done not only by households but also by business enterprises” and
government. Business enterprises save when they do not distribute the whole of their profits,
but retain a part of them in the form of undistributed profits. They then use these undistributed profits for
investment in real capital.
The third source of savings is government. The government savings constitute the money collected as taxes
and the profits of public undertakings. The greater the amount of taxes collected and profits made, the
greater will be the government savings. The savings so made can be used by the government for building up
new capital goods like factories, machines, roads, etc., or it can lend them to private enterprise to invest in
capital goods.
(b) Mobilization of Savings: The next step in the process of capital formation is that the savings
of the households must be mobilized and transferred to businessmen or entrepreneurs who require
them for investment. In the capital market, funds are supplied by the individual investors (who may
buy securities or shares issued by companies), banks, investment trusts, insurance companies,
finance corporations, governments, etc.
If the rate of capital formation is to be stepped up, the development of capital market is very necessary. A
well- developed capital market will ensure that the savings of the society-will be mobilized and transferred to
the entrepreneurs or businessmen who require them. (C)Investment of Savings in Real Capital: For savings to
result in capital formation, they must be invested. In order that the investment of savings should take place,
there must be a good number of honest and dynamic entrepreneurs in the country who are able to take risks
and bear uncertainty of production.
Given that a country has got a good number of venturesome entrepreneurs, investment will be made by them
only if there is sufficient inducement to invest. Inducement to invest depends on the marginal efficiency of
capital (i.e., the prospective rate of profit) on the one hand and the rate of interest, on the other.
But of the two determinants of inducement to invest-the marginal efficiency of capital and the rate of
interest—it is the former which is of greater importance. Marginal efficiency of capital depends upon the cost
or supply prices of capital as well as the expectations of profits.
Fluctuations in investment are mainly due to changes in expectations regarding profits. But it is the size of the
market which provides scope for profitable investment. Thus, the primary factor which determines the level of
investment or capital formation, in any economy, is the size of the market for goods.

➢ Foreign Capital: Capital formation in a country can also take place with the help of foreign capital, i.e.,
foreign savings.
Foreign capital can take the form of:
(a) Direct private investment by foreigners,
(b) Loans or grants by foreign governments,
(c) Loans by international agencies like the World Bank.
There are very few countries which have successfully marched on the road to economic development without
making use of foreign capital in one form or the other. India is receiving a good amount of foreign capital
from abroad for investment and capital formationunder the Five-Year Plans.
• Deficit Financing: Deficit financing, i.e., newly-created money is another source of capital formation
in a developing economy. Owing to very low standard of living of the people, the extent to which
voluntary savings can be mobilised is very much limited. Also, taxation beyond limit becomes
oppressive and, therefore, politically inexpedient. Deficit financing is, therefore, the method on
which the government can fall back to obtain funds.
However, the danger inherent in this source of development financing is that it may lead to inflationary
pressures in the economy. But a certain measure of deficit financing can be had without creating such
pressures.
There is specially a good case for using deficit financing to utilize the existing under- employed labour in
schemes which yield quick returns. In this way, the inflationary potential of deficit financing can be neutralized
by an increase in the supply of output in the short-run.
• Disguised Unemployment: Another source of capital formation is to mobilize the saving potential that
exists in the form of disguised unemployment. Surplus agricultural workers can be transferred from
the agricultural sector to the non- agricultural sector without diminishing agricultural output.
The objective is to mobilize these unproductive workers and employ them on various capital creating projects,
such as roads, canals, building of schools, health centers and bunds for floods, in which they do not require
much more capital to work with. In this way’, the hitherto unemployed, labour can be utilized productively
and turned into capital, as it were.
• Capital Formation in the Public Sector: In these days, the role of government has greatly increased.
In an under-developed country like India, government is very much concerned with the
development of the economy. Government is building dams, steel plants, roads, machine-making
factories and other forms of real capital in the country. Thus, capital formation takes place not only in
the private sector by individual entrepreneurs but also in the public sector by government.
There are various ways in which a government can get resources for investment purposes or for capital
formation. The government can increase the level of direct and indirect taxation and then can finance its
various projects. Another way of obtaining the necessary resources is the borrowing by the Government from
the public.
The government can also finance its development plans by deficit financing. Deficit financing means the
creation of new money. By issuing more notes and exchanging them with the productive resources the
government can build real capital. But the method of deficit financing, as a source of development finance, is
dangerous because it often leads to inflationary pressures in the economy. A certain measure of deficit
financing, however, can be had without creating such pressures.
Another source of capital formation in the public sector is the profits of public undertakings which can be
used by the government for further investment. As stated above, government can also get loans from foreign
countries and international agencies like World Bank. India is getting a substantial amount of foreign
assistance for investment purposes under the Five- Year Plans.

PARALLEL ECONOMY
Parallel economy, based on the black money or unaccounted money, is a big menace to the Indian economy.
It is also a cause of big loss in the tax-revenues for the government. As such, it needs to be curbed. Its
elimination will benefit the economy in more than one way.

In a general way, we can define black economy as the money that is generated by activities that are kept
secret, in the sense that these are not reported to the authorities. As such, this money is also not accounted
to the fiscal authorities i.e., taxes are not paid on this money.

• Harmful Effects of Parallel Economy:


The circulation of black money has adversely affected the economy in several ways. First, is the misdirection
of precious national resources? A part of black money is kept in a form that
contributes nothing/little to productive activities. Again, much around half to two third is squandered away on
ostentatious consumption of goods and services.
Second, it has enormously worsened the income distribution, and has thereby undermined the fabric of the
society.
Third, the existence of a big-sized unreported segment of the economy is a big handicap in making a correct
analysis and formulation of right policies for it. Nor. it is possible to monitor the development in the economy
with precision.
Fourth, the black money has eroded the social values of the society. The undeclared income
is ‘earned’ by illegitimate ways. This is spent in undesirable and vulgar manner.

1. Dual Economy
The increase in the amount of black money in India over a period of time lead to the perpetual growth of
economic dualism which consists of Parallel economy (black money economy) operating side by side with the
Official or Reported economy on the country.
The black economy represents not less than one fifth of the aggregate economic transactions. There is also
interaction between the reported and unreported activities such that it is difficult to identify black money from
the white money economy. Such a Parallel Economy will ruin theentire economic development of the country.

2. Under-estimation
A large underground economy and growth of black income lead to under-estimation of the true size and
incorrect picture of the economy by the officially complied national income data. Since unreported economy is
apparently excluded from the official records of the Gross National Product, the estimates of savings and
consumption of nations to the national income and measurement of other macroeconomic variables would be
biased and misleading for accurate policy making and planning considerations.

3. Loss of Revenue to the Government


Black money is largely attributed to tax evasion. Its direct impact is the loss of the Government revenue. Since
the Government fails to get sufficient tax revenue due to large-scale tax evasion, it is forced to resort to high
taxation and deficit financing which again carry their ill- economic effects.

4. Undermining the Equity


When the Government resorts to progressive direct taxation to maintain equity in the distribution of the tax
burden, the tax evasion and growth of black money affect the very concept of social justice by not allowing
the desirable reduction in inequalities of incomes. Again, when underground activities like smuggling etc.
could not be taxed, the Government will impose higher taxes on officially sanctioned activities.
Further, the tax evasion will also equally enjoy the public services without paying the due contribution; to that
extent also social enquiry is undermined. The honest have to bear high tax burden to make up for the deficit
in revenue caused by the tax evasion of black money makers.

5. Widening the Gap between the Rich and the Poor


Growth of the black economy causes regressive distribution of income in the society. When the black money
grows faster, rich becomes richer and the poor become poorer. By way of concentration of income and
wealth in few hands, the black money widens the gap between the rich and the poor.

6. Lavish Consumption Spending


Black money is disposed off by lavish spending on travels and tours, entertainment, ostentatious articles,
financing of extravagant elections etc. This has also lead to many social evils and deteriorated the values of
life of the common people.

7. Distortion of Production Pattern


The black money has altered the choice coefficients in the market in favor of luxuries, which lead to the
diversification of productive resources from essential goods to the non-essential goods.

8. Distribution of Scarce Resources


Black money holders are always in a position to put their prior claim over the scarce goods in the market due
to their readiness and ability to pay more, thereby depriving the honest and poor people from their
legitimate share. This obviously reduces the net economic welfare of the Indian society at large.

9. Deteriorate the General Moral Standards of the Society


Black money is largely responsible for the deterioration of general moral standards of the society. Black
income generation implies a deviation from the accepted norms in society and from the point of view of the
society is unethical.
Socially, we can say that the structure and ethos of a society undergoes a massive change. Social values of
honesty, hard work, thrift and simplicity get eroded. Even the political institutions and organizations lose their
credibility as they also gradually become a part of the entire system of black income generation.

10. Average Effect on Production


As a consequence, the consumption pattern is titled in favor of the rich and elite, at the cost of encouraging
production of articles of mass consumption. A rise in overall consumption leaves fewer resources for
investment in priority areas, having an adverse effect on production.

➢ Conclusion
From the above discussion, the exact effect of black money on any economy shall be summarized as follows.
1. Growth of economic dualism,
2. Under-estimation of the true size of the economy,
3. Tax evasion, thereby loss of revenue to Government,
4. Undermining equity,
5. Widening Gap between the Rich and Poor,
6. Lavish Consumption Spending,
7. Distortion of Production Pattern,
8. Distribution of Scarce Resources,
9. Deterioration of General Moral Standards of the Society,
10. Effects on Production.
BOOK- KEEPING & ACCOUNTANCY

1. BOOKKEEPING

Bookkeeping is the recording systematically of financial transactions, and is part and parcel of process
of accounting in business. Transactions include receipts, purchases, payments, and sales by an individual person
or an organization/corporation.
There are various methods of bookkeeping, vis-a-vis the single-entry bookkeeping system and the double-entry
system of bookkeeping. These are not just two methods of keeping books, infact any method involved in
recording financial transactions can be termed as process of bookkeeping. Bookkeeping is normally performed by
a bookkeeper.

Bookkeeper is normally responsible for writing the daybooks, which contain transactions of sales, payments,
purchases and receipts. It is the responsibility of the bookkeeper to record all transactions related to cash or
credit in a daybook, customer ledger, general ledger and supplier’s ledger. Reports can then be created from the
information hence generated by the bookkeeper’s action. So, the aspects of accounting are mainly because of
bookkeeper’s record keeping.

Preparing documents of source for all operations, transactions and other events of the business are part of
bookkeeping. The bookkeeper gets the books to a trial balance stage. Using this trial balance and ledgers hence
prepared by the bookkeeper the balance sheet and income statement may be prepared by the accountant.

OBJECTIVES OF BOOK KEEPING

1. The objectives of book keeping are as follows,


2. To have a record of every business transaction.
3. To show the effect financially of each and every transaction hence recorded.
4. To gauge the effect of each and every transaction (happened within an accounting period) on a financial
position at a particular date.
5. To expose the factors which are responsible for profit making or loss suffering at a given time?
6. The recoverable amount by a business from debtors and payable to creditors.
7. Determining the tax-liability of a business.
8. Preventing any frauds or errors.
9. Asset Protection
10. System of control to be effectively exercised in a measured environment.

DOUBLE-ENTRY BOOKKEEPING

In accounting, double entry is a system of bookkeeping. Why is it called double entry? Because each entry to a
particular account is required to correspond to a counter entry to another account. The two different yet equal
sides are named as debit and credit. It is standard to have debit on the left hand side and credit on the right hand
side. For instance accounting for a sale of Rs. 100 requires two entries: a debit of Rs. 100 to the cash account and
a credit of Rs. 100 to the revenue account.

The accounting equation, is an error detection tool; if at any point the sum of debits for all accounts does not
equal the corresponding sum of credits for all accounts, an error has occurred. However, it is no guarantee that
even after the equation is settled there will be no errors. The ledger will still be balanced because wrong accounts
of the ledgers may have been credited or debited.

2. BOOKS

Transactions get recorded first in the prime entry books or the original entry books. This might not form a part of
double entry process.
The primary Prime Entry Books are:
1. Purchase day book
2. Sales day book
3. Sales returns day book
4. Purchases returns day book
5. Cash Receipts Book
6. Bank Book
7. Cash Payments Book
8. Petty Cash Payments Book
9. Petty Cash Receipts Book
10. Journal

CASH BOOK
Transactions held in cash or by cheque are recorded in the book. There are two sides in a cash book. Cash
receipts get recorded on the left hand side and the cash payments on the right hand side. The five types of Cash
Book are: single column cash book, double column cash book, triple column cash book, bank cash book and petty
cash book. The receipt and payment of cash are recorded in the single column cash book.
In the double column cash book, receipt of cash, receipt of cash discount, payment of cash and cash discount
allowed are recorded. In the triple column cash book along with the transactions which are recorded in double
column cash book, cheque received and cheque paid are recorded. In the bank cash book the receipt of cheque,
payment of cheque, cash discount allowed and cash discount received are recorded. In the petty cash book only
small payments of cash are recorded by the petty cashier.
PURCHASE BOOK
All credit purchase of goods is written in this book. Cash purchase of goods and credit purchase of assets are not
recorded in this book. Other names of purchase book are purchase day book, purchase journal, bought journal,
inward invoice book etc.
SALES BOOK
All sales of goods are written in this book. Cash sale of goods and credit sale of assets are not recorded in this
book. Other names of Sales Book are Sales Day Book, Sales Journal, Sold book, Outward Invoice Book etc.

PURCHASE RETURN BOOK


It may be necessary to return some goods that the firm has bought on credit for a variety of reasons. All returns
of such goods are recorded primarily in Return Outward Book. This book is also known as Purchase Return Book.
SALES RETURN BOOK
Goods may be returned by the customers for a variety of reasons. All goods returned from customers are
recorded in Sales Return Book. This book is also known as Return Inward Book.

BILLS RECEIVABLE BOOK


When credit sales of goods are made the purchaser gives his guarantee to make payment in future in the form of
bill. When the seller receives such bill, it is Bill Receivable for him as he will receive payment in future against such
bill. In case a business house receives a number of bills, a Bills Receivable Book is maintained to record all such
bills.

BILLS PAYABLE BOOK


When credit purchases are made by a firm it gives a guarantee to the seller to make payment in future in the form
of a bill. This bill is said to be Bills Payable for the firm as he will pay for the bill in future. A Bills Payable Book is
opened to record all such bills.

JOURNAL PROPER
It is a subsidiary book maintained to record the transaction which cannot be recorded in other special subsidiary
books. Usually the transactions of infrequent character are recorded in the journal proper. The entries like
adjustment entries, opening entries, closing entries, transfer entries, purchase and sale of assets on credit, interest
on capital, interest of drawing etc. are recorded in journal proper.

Another description for books of prime entry, are a manual accounting system of special journals and subsidiary
ledgers. In a basic accounting system, or one that follows the original Venetian method, is to write an English
statement of a transaction at time of occurrence in a diary. Then, a this entry, which has at least a detail of a date,
an amount, a sentence describing what happened, is then transferred to the general journal as an entry as soon as
possible, the entry will be a first step in accounting classification, naming accounts to debit and amounts to debit,
and accounts to credit and amounts to credit, where sum of debits equals sum of credits, and the entry is dated.
There may be a further description written in brackets under the debits and credits, which may describe more
information, such as quantities sold, and to whom, or quantities bought, and from whom, and receipt numbers).

The general journal entry is then latter entered in the appropriate general ledger accounts, with a date and
possibly a back reference to the journal page, and forward references in the journal entry to the ledger account ID
numbers to indicate posting from the general journal to the general ledger of accounts. In a special journal /
subsidiary ledger system, or books of prime entry system, several constraints are added. Instead of making an
entry for every transaction in the general journal, there are several journals as named above.

The posting rules are as follows,

1. Credit sales are entered in the day sales journals. Likewise, sales returns are entered in the day sales returns
journals.
2. Purchases on credit are entered in the day purchases journal. Likewise, purchase returns are entered in the
purchase returns journal.
3. Cash payments i.e. payments by cheque, or via direct bank transactions such as bank fees and interest paid,
are made to the cash payments journal.
4. Cash receipts i.e. payments by cheque, direct deposits, visa receipts with reference numbers, are made in the
cash receipts journal.

At the end of each day, any entries into credit sales, sales returns, credit purchases, purchase returns, cash
receipts, and cash payments, are posted to the relevant subsidiary ledger accounts. There will be subsidiary
accounts receivables for each regular customer, and subsidiary accounts payable for each regular supplier.

In general, credit sales are posted as debits to the relevant subsidiary account receivables, and cash receipts from
that customer are posted as credits that account. Credit purchases are credits in the subsidiary accounts payables,
and cash payments to the same supplier are debits to that same subsidiary accounts payable.

At the end of each month, each column of the credit sales journal, credit purchases journal, cash receipts journal,
and cash payments journal, as well as the sales returns and purchase returns journal, is totaled at the bottom of
each column, and then posted to the relevant control ledger account. For instance, a more elaborate credit sales
journal for GST collections, will have a debit column for accounts receivables, whose end of month total is posted
to accounts receivables control, as well as a credit column for sales ledger account, which is not posted daily but
totaled for monthly posting, and a credit column for GST collections, which is also posted as a monthly credit to
the GST collections account (liability). Likewise, credit purchase journal will have a debit column for purchases, a
debit column for GST paid, and a credit column for accounts payable. The credits for accounts payable are posted
daily to subsidiary accounts payable (creditors), and the monthly total of accounts payable as a credit to accounts
payable control. The GST paid debit column is posted as a monthly total to GST paid (a contra-liability account).
The purchases are totaled monthly, and posted to the purchases account, which in the periodic inventory system,
is, at period's end, added to the beginning inventory in the income statement to give cost of goods available for
sale, and the stock take determined ending inventory subtracted to find the cost of goods sold, which can be
subtracted from gross revenue, to get gross profit.

the reason for the daily posting to subsidiary ledger accounts , and the monthly posting to the control ledger
accounts, is that a monthly cross check can be made by doing a monthly schedule of accounts receivables, and
a schedule of accounts payable : the sum of the ending balances of one type of subsidiary ledger accounts should
equal the balance of the control account after the monthly control posting e.g. subsidiary accounts receivable
balance sum equals accounts receivable control balance.

The general journal is for other entries that are not related to cash or credit, such as period-opening, reversing
entries; period-ending, adjusting entries, and closing entries transferring income and expense account balances to
the summary profit and loss temporary account.

Closing of temporary income and expense accounts can be to a temporary profit and loss summary account,
made initially in the general journal, and then posted to the income accounts, expense accounts, and profit and
loss summary account.

Petty cash payments may operate with the interest system, where there is a fixed interest amount for which a
petty cash fund is reimbursed to, and on reimbursement, an entry is made in the cash payments journal, with
debits for the previously petty cash expenses recorded in the petty cash voucher book, and a credit to cash in
bank for the cheque that is used to reimburse the petty cash. (If there are also receipts of petty cash, then
reimbursement might consist of the sum of unaccounted petty cash payments vouchers, less the sum of
unaccounted petty cash receipt vouchers, with possibly separate voucher books for payments and receipts).

Any sales returns journal entries are also recorded as credits daily in the relevant subsidiary account receivables
ledgers.

Any cash receipts journal entries related to trade debtor settlements are recorded as credits daily in the relevant
subsidiary accounts receivables ledgers.

At the end of the month, a schedule of subsidiary accounts receivable balances is made, and the sum of the
balances noted.

At the end of the month, the total of entries in the sales journal, less the total in the sales returns journal, less the
total in the accounts receivable credit column of the cash receipts journal, should equal the total of the schedule
of subsidiary accounts receivable.

Credit purchase journal entries are similarly posted in the subsidiary accounts payable daily, as well as cash
payment journal entries related to settlement of trade creditor accounts; similarly monthly total of trade creditor
account ending balances should equal totals of credit purchase journal, less credit purchase return journal, less
cash payment journal purchase column total.

Any credit purchase or sale of non-current assets such as office furniture and equipment should not be entered in
the credit purchases or credit sales journal, which is reserved for recording sale of trade inventory. Transfer of
account balances from subsidiary ledgers (containing nominal or temporary accounts) to income summary
account at the end of an accounting period. Also called closing the accounts or closing the books.

3. TYPES OF ACCOUNTS
TRADING ACCOUNTS
It is the first stage in the preparation of final accounts. Trading Account shows Gross Profit or Gross Loss during
an accounting year. Its main components are sales, services rendered on credit side and expenses directly
attributed for scoring the same on debit side. The account is based on matching the selling price of goods and
services with the cost of goods sold and services rendered. Expenses include Purchase, Electricity, Direct Wages,
Factory Overheads, and Transport etc. Income includes Sales, Services, and Discounts etc. Stock in hand as of end
of year is shown on credit side and Opening Stock is shown debit side of the account.

PROFIT AND LOSS ACCOUNT


The next step after preparing the Trading Account, is preparing the Profit and Loss Account. Profit and Loss
Account is prepared to calculate net profit or net loss of a business for a given accounting period.
A Profit and Loss Account is an account into which all gains and losses are collected in order to ascertain the
excess of gains over the losses or vice versa.
The Profit and Loss Account starts with the credit from Trading Account in respect of Gross Profit (or debit if there
is Gross Loss). Thereafter all indirect expenses and losses are debited to Profit and Loss Account. It means all
those indirect expenses or income which is not debited or credited to Trading Account are debited or credited to
Profit and Loss Account. Indirect expenses include Administration Expenses, Selling and Distribution Expenses,
Bad Debts, Depreciation etc. Indirect Income includes interest, rent etc.

Trading Account Profit and Loss Account


1 It is the first stage of 1 It is the second stage of the final accounts.
final accounts.
2 It shows the gross result 2 It shows the net results (net profit or net loss) of the
(gross profit or gross business.
loss) of the business.
3 All direct expenses 3 All expenses connected with sales and administration
(expenses connected (indirect expenses) of business are considered.
with purchase or
production of goods)
are considered in it.
4 It does not start with 4 It always starts with the balance of a trading account
the balance of any (gross profit or gross loss).
account.
5 Its balance (G.P or G.L) 5 Its balance (N.P or N.L) is transferred to capital account in
is transferred to profit balance sheet.
and loss account.

INCOME AND EXPENDITURE ACCOUNT


All transactions relating to non-profit-seeking concerns like Club, Library etc. are recorded in the books of account
strictly according to Double Entry System. At the year-end result is determined through Final Accounts. Final
Accounts consist of two stages:
1. Income and Expenditure Account
2. Balance Sheet
Here we are going to discuss income and expenditure account.
Definition and Explanation:
The account through which surplus or deficit of a non-profit-seeking concern is ascertained, is called Income and
Expenditure Account.
All the information necessary for preparation of this account will be available from ledger accounts. Its left-hand
(i.e. Debit) side records all revenue expenditure, while the right-hand (i.e. Credit) side records all revenues relating
to the current year. The balance of the account, if credit, indicates surplus, i.e. excess of income over expenditure.
Conversely, the balance of the account, if debit, indicates deficit, i.e. excess of expenditure over income.
Characteristics:
The following are the characteristics of Income and Expenditure Account:
1. It is in fact like a Profit and Loss Account of a profit-seeking concern.
2. All expenses are recorded on Debit side and all revenues on Credit side.
3. Only revenue transactions are included in it. No capital items are taken into account.
4. All the items of income/revenue concerning current year — whether received in cash or not—and all items of
expense —whether paid in cash or not—are taken into account. But no item of income or expense
concerning last year or next year is included in it.
5. Surplus or deficit of a concern is ascertained through this account. Credit balance "indicates surplus, while
debit balance indicates deficit.
6. Its balance is transferred to Capital Fund Account.
7. It is prepared on the last day of an accounting year.
8. It does not start with any opening balance.

3. PREPARATION OF BALANCE SHEET


In business, balance sheets are one of the two most important financial statements for anyone interested in the
financial health of the company. These parties may include managers, shareholders, and anyone else interested in
how the company is doing. A balance sheet is a financial statement that takes a point-in-time picture of the
financial state of the company, by listing all assets owned and all liabilities owed.

CURRENT ASSETS
Current assets include cash and other assets that in the normal course of events are converted into cash within
the operating cycle. For example, a manufacturing enterprise will use cash to acquire inventories of materials.
These inventories of materials are converted into finished products and then sold to customers. Cash is collected
from the customers. This circle from cash back to cash is called an operating cycle. In a merchandising business
one part of the cycle is eliminated. Materials are not purchased for conversion into finished products. Instead, the
finished products are purchased and are sold directly to the customers. Several operating cycles may be
completed in a year, or it may take more than a year to complete one operating cycle. The time required to
complete an operating cycle depends upon the nature of the business. It is conceivable that almost all of the
assets that are used to conduct your business, such as buildings, machinery, and equipment, can be converted
into cash within the time required to complete an operating cycle. However, your current assets are only those
that will be converted into cash within the normal course of your business. If you happen to hold these assets in
the regular course of business, you can include them in the inventory under the classification of current assets.
Current assets are usually listed in the order of their liquidity and frequently consist of cash, temporary
investments, accounts receivable, inventories and prepaid expenses.
CASH
Cash is simply the money on hand and/or on deposit that is available for general business purposes. It is always
listed first on a balance sheet. Cash held for some designated purpose, such as the cash held in a fund for
eventual retirement of a bond issue, is excluded from current assets.
MARKETABLE SECURITIES
These investments are temporary and are made from excess funds that you do not immediately need to conduct
operations. Until you need these funds, they are invested to earn a return. You should make these investments in
securities that can be converted into cash easily; usually short-term government obligations.

ACCOUNTS RECEIVABLE
Simply stated, accounts receivables are the amounts owed to you and are evidenced on your balance sheet by
promissory notes. Accounts receivable are the amounts billed to your customers and owed to you on the balance
sheet's date. You should label all other accounts receivable appropriately and show them apart from the accounts
receivable arising in the course of trade. If these other amounts are currently collectible, they may be classified as
current assets.

INVENTORIES
Your inventories are your goods that are available for sale, products that you have in a partial stage of
completion, and the materials that you will use to create your products. The costs of purchasing merchandise and
materials and the costs of manufacturing your various product lines are accumulated in the accounting records
and are identified with either the cost of the goods sold during the fiscal period or as the cost of the inventories
remaining at the end of the period.

PREPAID EXPENSES
These expenses are payments made for services that will be received in the near future. Strictly speaking, your
prepaid expenses will not be converted to current assets in order to avoid penalizing companies that choose to
pay current operating costs in advance rather than to hold cash. Often your insurance premiums or rentals are
paid in advance.

INVESTMENTS
Investments are cash funds or securities that you hold for a designated purpose for an indefinite period of time.
Investments include stocks or the bonds you may hold for another company, real estate or mortgages that you
are holding for income-producing purposes. Your investments also include money that you may be holding for a
pension fund.
PLANT ASSETS
Often classified as fixed assets, or as plant and equipment, your plant assets include land, buildings, machinery,
and equipment that are to be used in business operations over a relatively long period of time. It is not expected
that you will sell these assets and convert them into cash. Plant assets simply produce income indirectly through
their use in operations.

INTANGIBLE ASSETS
Your other fixed assets that lack physical substance are referred to as intangible assets and consist of valuable
rights, privileges or advantages. Although your intangibles lack physical substance, they still hold value for your
company. Sometimes the rights, privileges and advantages of your business are worth more than all other assets
combined. These valuable assets include items such as patents, franchises, organization expenses and goodwill
expenses. For example, in order to become incorporated you must incur legal costs. You can designate these legal
costs as organizing expenses.
OTHER ASSETS
During the course of preparing your balance sheet you will notice other assets that cannot be classified as current
assets, investments, plant assets, or intangible assets. These assets are listed on your balance sheet as other
assets. Frequently, your other assets consist of advances made to company officers, the cash surrender value of
life insurance on officers, the cost of buildings in the process of construction, and the miscellaneous funds held
for special purposes.

CURRENT LIABILITIES
On the equity side of the balance sheet, as on the asset side, you need to make a distinction between current and
long-term items. Your current liabilities are obligations that you will discharge within the normal operating cycle
of your business. In most circumstances your current liabilities will be paid within the next year by using the assets
you classified as current. The amount you owe under current liabilities often arises as a result of acquiring current
assets such as inventory or services that will be used in current operations. You show the amounts owed to trade
creditors that arise from the purchase of materials or merchandise as accounts payable. Other current liabilities
may include the estimated amount payable for income taxes and the various amounts owed for wages and
salaries of employees, utility bills, payroll taxes, local property taxes and other services.

LONG-TERM LIABILITIES
Your debts that are not due until more than a year from the balance sheet date are generally classified as long-
term liabilities. Notes, bonds and mortgages are often listed under this heading. If a portion of your long-term
debt is due within the next year, it should be removed from the long-term debt classification and shown under
current liabilities.

DEFERRED REVENUES
Your customers may make advance payments for merchandise or services. The obligation to the customer will, as
a general rule, be settled by delivery of the products or services and not by cash payment. Advance collections
received from customers are classified as deferred revenues, pending delivery of the products or services.

OWNER'S EQUITY
Your owner's equity must be subdivided on your balance sheet: One portion represents the amount invested
directly by you, plus any portion of retained earnings converted into paid-in capital. The other portion represents
your net earnings that are retained. This rigid distinction is necessary because of the nature of any corporation.
Ordinarily, stockholders, or owners, are not personally liable for the debts contracted by a company. A
stockholder may lose his investment, but creditors usually cannot look to his personal assets for satisfaction of
their claims. Under normal circumstances, the stockholders may withdraw as cash dividends an amount measured
by the corporate earnings. The distinction in this rule gives the creditors some assurance that a certain portion of
the assets equivalent to the owner's investment cannot be arbitrarily withdrawn. Of course, this portion could be
depleted from your balance sheet because of operating losses. The owner's equity in an unincorporated business
is shown more simply. The interest of each owner is given in total, usually with no distinction being made
between the portion invested and the accumulated net earnings. The creditors are not concerned about the
amount invested. If necessary, creditors can attach the personal assets of the owners.

COST
Cost is conventionally used as the basis for accountability. Assets, when acquired under normal circumstances, are
recorded at the price negotiated between two independent parties dealing at arm's length. Simply stated, the cost
of an asset to the purchaser is the price that he or she must pay now or later in order to obtain it. The fair value of
the asset is not relevant in recording the transaction on your balance sheet. A purchaser may acquire an asset at a
cost that is greater or less than the fair value determined in the marketplace. If the asset is acquired, the purchaser
accounts for the assets at his cost, value notwithstanding. A simple formula to remember in determining cost
is: Assets = Liability + Equity or Equity = Assets - Liability

Balance Sheet
Assets
Current Assets:
Cash 0

Accounts 0

Receivable
Inventory 0

Prepaid Expenses 0

Fixed Assets:
Equipment 0

Equipment 0

Depreciation
Computers 0

Computer 0

Depreciation
Other Assets:
Goodwill 0

Total Assets xxxxxxxx


Liabilities
Current Liabilities:
Accounts Payable 0

Expenses Payable 0

Payroll Taxes 0

Withheld
Loans Payable 0

(short term)
Long-term Liabilities:
Loans Payable 0

(long term)
Shareholders' Equity
Beginning Retained 0

Earnings
Net Income 0

Capital Stock 0

Paid in Capital 0

Total Liabilities xxxxxxxxxx

ACCOUNTING STANDARD 1- DISCLOSURE OF ACCOUNTING POLICIES


Disclosure of Accounting Policies
The following is the text of Accounting Standard I (AS-1) issued by the Accounting Standards Board of the
Institute of Chartered Accountants of India, on “Disclosure of Accounting Policies”. The Standard deals with the
disclosure of significant accounting policies followed in preparing and presenting financial statements.
Introduction
1. This statement deals with the disclosure of significant accounting policies followed in preparing and
presenting financial statements.
2. The view presented in the financial statements of an enterprise of its state of affairs and of the profit or loss
can be significantly affected by the accounting policies followed in the preparation and presentation of the
financial statements. The accounting policies followed vary from enterprise to enterprise. Disclosure of
significant accounting policies followed is necessary if the view presented is to be properly appreciated.
3. The disclosure of some of the accounting policies followed in the preparation and presentation of the
financial statements is required by law in some cases.
4. The Institute of Chartered Accountants of India has, in Statements issued by it, recommended the disclosure
of certain accounting policies, e.g. translation policies in respect of foreign currency items.
5. In recent years, a few enterprises in India have adopted the practice of including in their annual reports to
shareholders a separate statement of accounting policies followed in preparing and presenting the financial
statements.
6. In general, however, accounting policies are not at present regularly and fully disclosed in all financial
statements. Many enterprises include in the Notes on the Accounts, descriptions of some of the significant
accounting policies. But the nature and degree of disclosure vary considerably between the corporate and
the non-corporate sectors and between units in the same sector.
7. Even among the few enterprises that presently include in their annual reports a separate statement of
accounting policies, considerable variation exists. The statement of accounting policies forms part of
accounts in some cases while in others it is given as supplementary information.
8. The purpose of this statement is to promote better understanding of financial statements by establishing
through an accounting standard the disclosure of significant accounting policies and the manner in which
accounting policies are disclosed in the financial statements. Such disclosure would also facilitate a more
meaningful comparison between financial statements of different enterprises.
9. Fundamental Accounting Assumptions:
10. Certain fundamental accounting assumptions underlie the preparation and presentation of financial
statements. They are usually not specifically stated because their acceptance and use are assumed. Disclosure
is necessary if they are not followed.
11. The following have been generally accepted as fundamental accounting assumptions:—
• Going Concern:
• The enterprise is normally viewed as a going concern, that is, as continuing in operation for the
foreseeable future. It is assumed that the enterprise has neither the intention nor the necessity of
liquidation or of curtailing materially scale of the operations.
• Consistency:
• It is assumed that accounting policies are consistent from one period to another.
• Accrual:
• Revenues and costs are accrued, that is, recognized as they are earned or incurred (and not as money
is received or paid) and recorded in the financial statements of the periods to which they relate. (The
considerations affecting the process of matching costs with revenues under the accrual assumption are
not dealt with in this statement).

Nature of Accounting Policies


The accounting policies refer to the specific accounting principles and the methods of applying those principles
adopted by the enterprise in the preparation and presentation of financial statements.

There is no single list of accounting policies which are applicable to all circumstances. The differing circumstances
in which enterprises operate in a situation of diverse and complex economic activity make alternative accounting
principles and methods of applying those principles acceptable. The choice of the appropriate accounting
principles and the methods of applying those principles in specific circumstances of each enterprise calls for
considerable judgment by the management of the enterprise.
The various statements of the Institute of Chartered Accountants of India combined with the efforts of
government and other regulatory agencies and progressive managements have reduced in recent years the
number of acceptable alternatives particularly in the case of corporate enterprises.

While continuing efforts in this regard in future are likely to reduce the number still further, the availability of
alternative accounting principles and methods of applying those principles is not likely to be eliminated
altogether in view of the differing circumstances faced by the enterprises.

Areas in which differing accounting policies are encountered


The following are examples of the areas in which different accounting policies may be adopted by different
enterprises:

1. Methods of depreciation, depletion and amortization


2. Treatment of expenditure during construction
3. Conversion or translation of foreign currency items
4. Valuation of inventories
5. Treatment of goodwill
6. Valuation of investments
7. Treatment of retirement benefits
8. Recognition of profit on long-term contracts
9. Valuation of fixed assets
10. Treatment of contingent liabilities

The above list of examples is not intended to the exhaustive.

Considerations in the Selection of Accounting Policies


The primary consideration in the selection of Accounting Policies by an enterprise is that the financial statements
prepared and presented on the basis of such accounting policies should represent a true and fair view of the state
of affairs of the enterprise as at the Balance Sheet date and of the profit or loss for the period ended on that date.
For this purpose, the major considerations governing the selection and application of accounting policies are:
5. Prudence:
2. In view of the uncertainty attached to future events; profits are not anticipated but recognized only when
realized though not necessarily in cash. Provision is made for all known liabilities and losses even though the
amount cannot be determined with certainty and represents only a best estimate in the light of available
information.
3. Substance over Form:
4. The accounting treatment and presentation in financial statements of transactions and events should be
governed by their substance and not merely by the legal form.
5. Materiality:
6. Financial statements should disclose all “material” items, i.e. items the knowledge of which might influence the
decisions of the user of the financial statements.

Accounting Standard
All significant accounting policies adopted in the preparation and presentation of financial statements should be
disclosed.

The disclosure of the significant accounting policies as such should form part of the financial statements and the
significant accounting policies should normally be disclosed in one place.
Any change in the accounting policies which has material effect in the current period or which is reasonably
expected to have a material effect in later periods should be disclosed. In the case of a change in accounting
policies which has a material effect in the current period, the amount by which any item in the financial
statements is affected by such change should also be disclosed to the extent ascertainable. Where such amount is
not ascertainable, wholly or in part, the fact should be indicated.

If the fundamental accounting assumptions, viz. Going Concern, Consistency and Accrual are followed in financial
statements, specific disclosure is not required. If a fundamental accounting assumption is not followed, the fact
should be disclosed.

5. COST
In business and accounting, cost is the monetary value that a company has spent in order to produce something.
Cost is defined as the cash amount (or the cash equivalent) given up for an asset. Cost includes all costs necessary
to get an asset in place and ready for use. For example, the cost of an item in inventory also includes the item's
freight-in cost. The cost of land includes all costs to get the land ready for its use.

What are the Elements of Cost in Cost Accounting?

The elements that constitute the cost of manufacture are known as the elements of cost. Such element of cost is
divided into three categories. In a manufacturing concern, raw materials are converted into a finished product
with the help of labor and other service units. They are Material, Labor and Expenses.
Again, these elements of cost are divided into two categories such as Direct Material and Indirect Material, Direct
Labor and Indirect Labor, Direct Expenses and Indirect Expenses. All direct material, direct labor and direct
expenses are added to get prime cost. Likewise all indirect material, indirect labor and indirect expenses are
added to get overhead. Again, overhead is divided into four categories. They are factory overhead, administration
overhead, selling overhead and distribution overhead.

1. Direct Material:
It refers to material out of which a product is to be produced or manufactured. The cost of direct material is
varying according to the level of output. For example: Milk is the direct material of butter.

2. Indirect Material:
It refers to material required to produce a product but not directly and does not form a part of a finished
product. For example: Nails are used in furniture. The cost of indirect material is not varying in direct
proportion of product.

3. Direct Labor:
It refers to the amount paid to the workers who are directly engaged in the production of goods. It varies
directly with the output.

4. Indirect Labor:
It refers to the amount paid to the workers who are indirectly engaged in the production of goods. It does not
vary directly with the output.

5. Direct Expenses:
It refers to the expenses that are specifically incurred by the company to produce a product. A product cannot
be produced without incurring such expenses. It varies directly with the level of output.

6. Indirect Expenses:
It refers to the expenses that are incurred by the organization to produce a product. But, these expenses
cannot be easily found out accurately. For example: Power used for production.

7. Overhead:
It is the combination of all indirect materials, indirect labour and indirect expenses.

8. Factory Overhead:
It is otherwise called Production Overhead or Works Overhead. It refers to the expenses that are incurred in the
production place or within factory premises. For example: Indirect material, rent, rates and taxes of factory,
canteen expenses etc.

9. Administration Overhead:
It is otherwise called Office Overhead. It refers to the expenses that are incurred in connection with the general
administration of the company. For example: Salary of administrative staff, postage, telegram and telephone,
stationery etc.
10. Selling Overhead:
It refers to all expenses incurred in connection with sales. For example: Salary of sales department staff,
travelers’ commission, advertisement etc.
11. Distribution Overhead:
It refers to all expenses incurred in connection with the delivery or distribution of goods and services from the
producer to the consumer. For example: Delivery van expenses. Loading and unloading, customs duty, salary of
deliverymen etc.

Fixed Cost and Variable Cost


Based on variability, the costs has been classified into three categories, they are fixed, variable and semi
variable. Fixed costs, as its name suggests, is fixed in total i.e. irrespective of the number of output
produced. Variable costs vary with the number of output produced. Semi-variable is the type of costs, which have
the characteristics of both fixed costs and variable costs. Fixed costs are one that does not change with the
change in activity level in the short run. Conversely, Variable cost refers to the cost of elements, which tends to
change with the change in level of activity. While working on costs of production, one should know the difference
between fixed cost and variable cost. So, take a read of the given article in which we have compiled all the
important points of distinction in tabular form along with examples.

Difference between Fixed & Variable cost:

BASIS FOR FIXED COST VARIABLE COST


COMPARISON
Meaning The cost which remains same, The cost which changes with
regardless of the volume the change in output is
produced, is known as fixed considered as a variable cost.
cost.
Nature Time Related Volume Related

Incurred when Fixed costs are definite, they Variable costs are incurred
are incurred whether the only when the units are
units are produced or not. produced.

Unit Cost Fixed cost changes in unit, i.e. Variable cost remains same,
as the units produced per unit.
increases, fixed cost per unit
decreases and vice versa, so
the fixed cost per unit is
inversely proportional to the
number of output produced.
Behavior It remains constant for a It changes with the change in
given period of time. the output level.

Combination of Fixed Production Overhead, Direct Material, Direct Labor,


Fixed Administration Direct Expenses, Variable
Overhead and Fixed Selling Production Overhead,
and Distribution Overhead. Variable Selling and
Distribution Overhead.

Examples Depreciation, Rent, Salary, Material Consumed, Wages,


Insurance, Tax etc. Commission on Sales, Packing
Expenses, etc.
Fixed Cost
Fixed Cost remains constant does not mean that they are not going to change in future, but they tend to be fixed
in the short run. This can be explained with an example, If your company is operating the business in a rented
building, so whether you produce tons of output, or you produce nothing, you have to pay the rent of the
building, so this is a fixed expense which is constant over a period until the rent of the building increases or
decreases.

Fixed cost will be same in total but changes in per unit. To explain this, we have an example If the fixed cost is Rs.
10000 and the output produced in the first, second and third quarter are 4000, 5000 and 3000 units. Now, in this
situation, what you can see is, the total fixed cost is unchanged in all the three-quarters, but the unit fixed cost in
the first quarter is Rs. 10000/4000 units, i.e. Rs. 2.5, in the second quarter it is Rs. 10000/5000 units, i.e. Rs. 2 and in
the third quarter it is Rs. 10000/3000 units, i.e. Rs. 3.33.
There are two types of Fixed Cost:

1. Committed Fixed Cost

2. Discretionary Fixed Cost

Definition of Variable Cost


The cost which changes with the changes in the quantity of output produced is known as Variable Cost. They are
directly affected by the fluctuations in the activity levels of the enterprise.

Variable Cost
Variable cost varies with the variations in the volume, i.e. when there is an increase in the production, variable cost
will also increase proportionately with the same percentage and when there is no production there will be no
variable cost. The Variable cost is directly proportional to the units produced by the enterprise.

Now, variable cost remains same in per unit, but changes in total. You can understand this with an example, i.e. if
the variable cost is Rs. 6 per unit and output produced in the first, second and third quarter is 5000, 6000 and
4000 units. You might wonder that the output level is changed in all the three-quarters, so the variable cost will
also change, but only in the total amount but not in the unit price. So the variable cost in the first quarter is
5000*6 = Rs. 30000, in the second quarter it will be 6000*6 = Rs. 36000 while in the third quarter, it is 4000*6 =
Rs. 24000.

The Variable cost is divided into two categories, they are:

1. Direct Variable Cost

2. Indirect Variable Cost

BREAK-EVEN ANALYSIS, AND HOW IT WORKS


Break-even analysis is a business tool widely used across all industries to evaluate business performance in terms
of costs, since this is a supply-side analysis. Break-even analysis is an important aspect of a good business plan,
since it helps the business determine the cost structures, and the number of units that need to be sold in order to
cover the cost or make a profit. Break-even analysis is usually done as part of a business plan to see the how
practical the business idea is, and whether or not it is worth pursuing. Even after a business has been set-up,
break-even analysis can be immensely helpful in the pricing and promotion process, along with cost control.
Simply put, break-even point can be determined by calculating the point at which revenue received equals the
total costs associated with the production of the goods or services.

Break-even Point = Fixed Costs/ (Unit Selling Price – Variable Costs)

Using the above formula, the business can determine how many units it needs to produce in order to break-even.
Once the business has reached this point, in sales or units sold, all costs (Fixed and Variable) have been recovered.
Beyond this point, every additional unit sold will result in increasing profit for the business. The increase in profit
will be by the amount of unit contribution margin, which is the amount of additional revenues that goes towards
covering the fixed costs and profit. It can be calculated as follow:

Unit Contribution Margin = Sales Price – Variable Costs

Calculation of Break-even point in units


Break-even point is usually calculated in units, which gives the company the number of units it must produce in
order to break-even. It can be calculated by dividing contribution margin by total fixed costs:

Break-even point (Units) = Fixed Costs/Contribution margin per unit

Calculation of Break-even point in sales value


In the previous example, the break-even point was calculated in terms of number of units. Break-even point can
also be calculated in sales value. This can be done by dividing company’s total fixed costs by contribution margin
ratio.

Contribution Margin = Contribution Margin per Unit/Sales Price per Unit

OR

Contribution Margin = (Sales Price – Variable Costs) /Sales Price per Unit
APPLICATION OF BREAK EVEN ANALYSIS

COST CALCULATION
Break-even analysis is widely used to determine the number of units the business needs to sell in order to avoid
losses. This calculation requires the business to determine selling price, variable costs and fixed costs. Once these
numbers are determined, it is fairly easy to calculate break-even point in units or sales value.

BUDGETING AND SETTING TARGETS


Break-even charts and calculation be used for budgeting process, since the business know exactly how many units
need to be sold in order to break-even. Moreover, the company is also aware of the profits the company will be
able to earn at various points, which can be easily illustrated on a simple break-even chart. This can help business
set realistic, achievable targets for itself.

MOTIVATIONAL TOOL
Break-even analysis also helps to motivate the employees, especially the sales staff, since it clearly shows the
profits at various points of sales. The chart clearly shows the impact extra sales would have on the profitability of
the company.

MARGIN OF SAFETY
Margin of safety is a tool which complements break-even analysis, since these two tool are interrelated. This
concept is used when a major proportion of sales are likely to decline or in period of recession or economic turn
down. Managers can better make better production and sales decision if they know the margin of safety for a
particular product or service. When the margin of safety is large, the business would want to try new pricing,
marketing and take risks hoping to further increase sales and revenues. On the other hand, if the margin of safety
is meager, managers are likely not to change anything, since any small change could trigger losses. In such a
situation managers would want to reduce costs, so that margin of safety can be increased.

The concept of margin of safety might not be useful for businesses with seasonal demand for their products or
services, since there will be a lot of variations on monthly basis. The result could be complied for an entire year, so
that seasonal fluctuations are removed.

Margin of safety can be calculated by subtracting the current break-even point from current sales, and dividing by
current level of sales.

Margin of safety = (Current Sales Level – Break-even Point)/Current Sales Level

There are two ways to calculate margin of safety as below,

1. If the company wishes to calculate margin of safety for a budgeted, future period, it can replace the current
sales level with budgeted sales level.

2. If a business wants to calculate margin of safety (Version #2) for number of units sold, then instead of current
sales level, selling price per unit in the denominator.

Margin of safety = (Current Sales Level – Break-even Point)/Selling Price per Unit.
COST CONTROL AND MONITORING
Since costs (Fixed and Variable) affect the profitability of the business directly, the managers can easily see these
changes through break-even analysis. This would help them control costs, and make sure that they remain within
a given range.

Helps devise a pricing strategy

Selling price is an important determinant of break-even analysis. If managers have access to break-even charts,
they will be able to see the impact, changes in selling price has on the overall profitability. Hence, this tool
provides more information for the mangers to make better pricing decision, considering the supply-side of the
production process.

6. Multiple choice questions


1. Mr. Customer purchased goods from Mr. Seller on credit. This is a/an
a) Event
b) Transaction
c) Both (a) and (b)
d) None of the above

2. The first step of accountancy is,


a) Journal Entry
b) Ledger Posting
c) Balancing of Accounts
d) Trial Balance

3. Cash is an example of
a) Personal Account
b) Real Account
c) Nominal Account
d) Both (a) and (b)

4. Net Realizable Value of an asset means


a) The historical cost of the asset
b) Its sale value as on a particular date
c) Its written down value as on a particular date
d) None of the above

5. Transfer to General Reserve is a charge against


a) Trading Account
b) Profit and Loss Account
c) Profit and Loss Appropriation Account
d) Balance Sheet

6. All indirect expenses are charged against


a) Trading Account
b) Profit and Loss Account
c) Manufacturing Account
d) None of the above
7. Basic objective of costing is,
a) Tax compliance
b) Financial audit
c) Cost ascertainment.
d) Profit analysis

8. The long term assets that have no physical existence but are rights that have value is known as,
a) Current assets
b) Fixed assets
c) Intangible assets
d) Investments

9. Cost classification can be done in,


a) Two ways
b) Three ways
c) Four ways
d) Several ways.

10. Costing process is suitable for,


a) Oil refineries
b) Transport firms
c) Hospitals
d) Brick laying firms

11. An additional cost, incurred for some specific activity to bring processed product on to next production stage
is,
a) Partial cost
b) Relevant cost
c) Incremental cost
d) Irrelevant cost

12. A joint cost allocation method is based on relative value of total sales, at point of split off is classified as,
a) Sales value at split off method
b) Joint costs at split off point method
c) Joint products value at split off method
d) Main product cost at split off method

13. Difference between final sales value and separable costs is equal to,
a) Net Income
b) Net realizable value
c) Gross margin
d) Gross realizable value

14. Relationship based on unrelated level of activity and past data of cost is measured with help of,
a) Cost estimation
b) Price estimation
c) Unit estimation
d) Production estimation
LAW–GENERAL
➢ Salient Features of the Indian Constitution

Indian constitution being the most unique and the longest written constitution in the world, originally
had 395 articles which are further divided into 22 parts and 8 schedules. As a result of various
amendments over time, at present, our constitution contains 448 articles, with 12 schedules. The
constitution of India was put together by a Constituent Assembly. After several meetings and
discussions, finally, on 26th January 1950, our constitution came into effect.

Article 368 of the constitution laid down that the Parliament can keep amending the constitution
according to the need of the time in future. As held by the Apex Court, every part of the constitution can
be modified, except the “basic structure” of the Constitution. Any old or new law which violates the
basic structure of the constitution is declared unconstitutional and invalid by the Courts.

1. Longest Written Constitution:

Indian Constitution is a fully written document which incorporates various laws for proper management
of the country. Indian constitution contains separate provisions for states and centre and their inter-
relationship. The constituent assembly had borrowed various provisions from several other constitutions
of the world which made it very detailed. Moreover, there are separate provisions for scheduled castes,
scheduled tribes, other backward classes, children and women.

2. Single Citizenship:

Presently there are 28 states and 7 Union Territories in India and all citizens enjoy a common uniform
citizenship. All those citizens are further entitled to equal freedom, rights, and protection.

3. Fundamental Rights:

The fundamental rights of an Indian citizen are guaranteed under Part IIIC (Articles 12-35). It is also
called the ‘Indian Bill of Rights’. After the 44th Amendment Act 1979, there are presently six
fundamental rights in our constitution. It is to be noted that these fundamental rights are not absolute
and are subject to certain limitations that are expressly mentioned in theconstitution itself.

The six fundamental rights are as follows:

(i) Right to Equality

This fundamental right provides for equality before law and equality of opportunity. It also ends
discrimination and abolishes untouchability and titles.

(ii) Right to Freedom

This fundamental right further includes six kinds of freedoms.

These are:- Freedom of speech and expression, Freedom to assemble peacefully without arms, Freedom
to form associations, Freedom to move freely in India, Freedom to reside in any part of India, Freedom
of adopting any profession, trade, or occupation.
(iii) Right to freedom of Religion

This fundamental rights, grants an individual, a right to believe, profess, or worship any religion. The State
does not have a common religion. It also guarantees the right to establish and maintain any religious
institution. No person can be compelled to follow a particular religion.

(iv) Cultural and Educational Rights

This fundamental right guarantees all the minorities a right to maintain, develop, and further teach their
language and culture. It also provides them a right to establish, maintain and administer their own
educational institutions.

(v) Right against Exploitation

This fundamental right prohibits forced labour, child labour, and human trafficking. It ensures protection
to individuals against various human atrocities.

(vi) Right to Constitutional Remedies (Article 32)

This fundamental right is the heart and soul of all of the fundamental rights. It ensures effective
enforcement of all the other fundamental rights and protection from violation of these rights. Under this
article, the Supreme Court and Hight Court have to power to issue writs for enforcement of these
rights.

4. Fundamental Duties:

Part IVA (Article 51 A) of the Constitution describes various Fundamental Duties of each citizen. Those
fundamental duties are as follows:

i. To abide by the Constitution and respect its ideals and institutions; as well as, respect the
National Flag and the National Anthem
ii. To cherish the noble ideals of the freedom struggle
iii. To uphold and protect the unity, sovereignty, and integrity of India
iv. To defend the country and render national service when called upto to do so.
v. To promote harmony and the spirit of common brotherhood amongst all the people of
India transcending religious, linguistic, and regional, or sectional diversities; to renounce
practices derogatory to the dignity of women
vi. To value and preserve the rich heritage of our exquisite culture
vii. To protect and improve the natural environment including forests, lakes, rivers, and wild
life, and to have compassion for living creatures
viii. To develop the scientific temper, humanism, and the spirit of inquiry and reform
ix. To safeguard public property and to abjure violence
x. To strive towards excellence in all spheres of individual and collective activity so that the
nation constantly rises to higher levels of endeavour and achievement.
xi. To provide opportunities for education by the parent or the guardian, to his child, or a
ward between the age of 6-14 years as the case may be.

5. Directive Principles of State Policy:

Part IV of the Indian Constitution deals with the ‘Directive Principles of State Policy’. This part of the
Constitution mentions certain guidelines for the State with the objective of socio- economic
development. All three governments of the country, Central, State, and Local are expected to frame
welfare policies in accordance with this part of the Constitution. However, these policies are not
enforceable in a Court of Law.

For example, some of the policies that the State is directed to make are to ensure fairer distribution of
wealth, protection of women and children, old age pensions, local governments, adequate means of
livelihood to people, etc.

6. Mixture of Rigidity and Flexibility:

The Constitution of India is said to be the perfect mixture of rigidity and flexibility. It is neither wholly
rigid nor wholly flexible. It is because, some provisions of the Constitution can be amended very easily
while the others can only be amended by complex processes.

7. Adult Suffrage: (Universal Adult Franchise)

This part of the Constitution promotes the policy of ‘one person one vote’. According to this part of the
Constitution, every individual of 18 years and above have a right to vote irrespective of their age, gender,
race, colour, religion, etc.

8. India as a Democratic, Socialist, Sovereign, Secular, and Republic State:

• India is a Democratic state means that the citizens of India have the power to elect their
representatives in the government and thus, the government is responsible to the
people.
• India is a Socialist state because the 42nd Amendment of the Constitution states the
achievement of socialist goals through democratic and non-violent means.
• Indian Constitution states that India is a Sovereign state because it is absolutely
independent and it is not under the control of any other state. It manages all of its
internal and external affairs freely without any external forces.
• India is a Secular state as the state has no specific religion of its own. Every citizen is free
to follow, practice, and profess the religion of their choice. Moreover, the state cannot
discriminate amongst its citizens on the basis of religion.
• India is a Republic because it is not ruled by a heredity monarch, whereas, the head of
the state (the President) is elected by the people indirectly for a fixed period of 5 years.

9. Parliamentary system of government:

Indian Constitution provides for a parliamentary form of government. President is the nominal or
Constitutional head of the state. He is indirectly elected by the citizens of India for a fixed period of 5
years. While, the Prime Minister is the real or executive head of the state and is collectively responsible
for the management of the Council of Ministers.

10. Independent Judiciary:

The Indian judiciary is free from interference from the other organs of the government (namely, the
executive and the legislature). Some facts mentioned in the Constitution that further prove the
independence of the judiciary are:-

• Judges are appointed by the President,


• Judges of Supreme Court cannot be removed from office except through an extremely
complex process,
• The salaries of judges are very high,
• The Supreme Court has its own staff.

11. Judicial Activism:

Through the Public Interest Litigation system (PIL) and many other ways, the judiciary has been keeping
an eye on the functioning of the legislature and the executive and if the judiciary feels that there is need
of some more effective policies or laws on a certain topic of public or national importance, the Judiciary
can suggest or make temporary laws in the favour of those topics.

12. Judicial Review:

This part of the Constitution gives the judiciary, a right to review each law and further declare them as
unconstitutional or invalid according to the decisions

➢ Fundamental Rights and Duties

As a citizen of India, we are entitled to certain rights as well as obliged to certain duties. It is our duty as
responsible citizens that we abide by these laws and carry out our duties.
Similarly, knowledge of our fundamental rights is important so as to prevent injustice. Let us update
ourselves about the Fundamental Rights and Duties laid down by the constitution of India

❖ Introduction to Fundamental Rights and Duties

During the period of 1947 to 1949, Constitution of India developed and prescribed the fundamental
obligations of the State to its citizens and the duties and the rights of the citizens towards the State
under the following sections which constitute the vital elements of the constitution.

• Fundamental Rights
• Directive Principles of State Policy
• Fundamental Duties

These sections consist of a constitutional bill of rights for government policy-makingand lay down a
foundation for the appropriate the behavior and conduct of citizens.

➢ The Fundamental Rights

These are the basic human rights of all citizens, defined in Part III of the Constitution. These are
applicable irrespective of race, place of birth, religion, caste, creed, or gender. They are enforceable by
the courts, subject to specific restrictions. Following are some of the important rights of the citizens of
India in accordance with the Constitution.

• Right to Equality
• Right to freedom
• Right against exploitation
• Right to freedom of religion
• Cultural and Educational Rights
• Right to Constitutional Remedies

➢ Directive Principles of State Policy


These are included in Part IV of the Constitution. For the framing of certain Laws, the Government
requires certain guidelines. These are included in the Directive Principles of State Policy. According to
Article 37, they are not enforceable by the courts. It just lays down the fundamental principles and
guidelines on which they are based are fundamental guidelines for governance that the State need to
follow while designing the laws. Emphasis is on the Welfare of State Model.

The establishment of Directive Principles of State Policy is in accordance with certain articles of the
Constitution of India. Let us explore more about the importance of these articles.

Article Significance
37- Non-enforceability in court
39A- Securing participation of workers in management of industries
Mandate the state to endeavor to secure to all citizens right
41- to work, living wage, security, maternity relief and a decent
standard of living
Promotion of industries, Establishment of Several
43-
Boards for the promotion of Khadi and other handlooms
Free and compulsory education to children between age
group of 6-14
45-
After 2002 amendment childhood care age limit
was shifted to below 6 years
Raise the standard of living and improve public health
47- Prohibition of consumption of intoxicating
drinks and drugs injurious to health
Organize agriculture and animal
48-
husbandry on modern and scientific lines
Protection of the environment and
48A-
safeguard the forests and wildlife of the country
Preservation of monuments and objects
49-
of national importance
Separation of judiciary from the executive
50-
in public services
International peace and security
51- Implementation of Laws giving effect to
International Treaties

➢ The Fundamental Duties

These are defined as the moral obligations of all citizens to help promote a spirit of patriotism and to
uphold the unity of India and concern the individuals and the nation. Included in Part IVA of the
Constitution, like the Directive Principles, they are not enforceable by the law. According to the
constitution, following are the duties to be followed by every citizen of India

• To abide by the Constitution and respect its ideals and institutions, the National Flag
and the National Anthem.
• To cherish and follow the noble ideals which inspired our national struggle for freedom.
• To uphold and protect the sovereignty, unity, and integrity of India.
• To defend the country and render national service when called upon to do so.
• To promote harmony and the spirit of common brotherhood amongst all the people of
India transcending religious, linguistic and regional or sectional diversities; to renounce
practices derogatory to the dignity of women.
• To value and preserve the rich heritage of our composite culture.
• To protect and improve the natural environment including forests, lakes, rivers, wildlife
and to have compassion for living creatures.
• To develop the scientific temper, humanism and the spirit of inquiry and reform.
• To safeguard public property and to abjure violence.
• To strive towards excellence in all spheres of individual and collective activity, so that the
nation constantly rises to higher levels of endeavor and achievement.
• Who is a parent or guardian, to provide opportunities for education to his child, or as the
case may be, ward between the ages of six to fourteen years.
• According to the 86th constitutional amendment in 2002, it is the duty of the people of
India to adapt to make India a safer place to live, to be clean and make the surrounding
clean and not to hurt anybody physically and mentally.

➢ The Relationship Between the Fundamental Rights, Directive Principles and Fundamental
Duties

Directive Principles have been used to uphold the Constitutional validity of legislation in case of conflict
with Fundamental Rights. According to the amendment of 1971, any law that even though it deviates
from the Fundamental Rights, but has been made to give effect to the Directive Principles in Article
39(b)(c) would not be deemed invalid. The Fundamental Duties will be held obligatory for all citizens
subject to the State enforcing the same by means of a valid law.

• Solved Question For You

Que: Which of these is not a Fundamental Right according to The Constitution of India?
a. Right to Constitutional Remedies
b. Cultural and Educational Rights
c. Right to Equality
d. Right to a luxurious life

Ans: The correct answer is Option d. Right to a luxurious life. It is not a fundamental right according to
the Constitution of India.

➢ Directive Principles of Our State Policy: Part IV (Articles 36-51)

Part IV of Indian Constitution deals with Directive Principles of our State Policy (DPSP). The provisions
contained in this Part cannot be enforced by any court, but these principles are fundamental in the
governance of the country and it shall be the duty of the State to apply these principles in making laws.

The concept of Directive Principles of State Policy was borrowed from the Irish Constitution. While most
of the Fundamental Rights are negative obligations on the state, DPSPs are positive obligations on the
state, though not enforceable in a court of law.

Article 36: Definition In this Part, unless the context otherwise requires, “the State” has the same
meaning as in Part III.

Article 37: Application of the principles contained in this Part

The provisions contained in this Part shall not be enforced by any court, but the principles therein laid
down are nevertheless fundamental in the governance of the country and it shall be the duty of the
State to apply these principles in making laws.

Article 38: State to secure a social order for the promotion of the welfare of the people

(1) The State shall strive to promote the welfare of the people by securing and protecting as
effectively as it may a social order in which justice, social, economic and political, shall inform
all the institutions of the national life.
(2) The State shall, in particular, strive to minimize the inequalities in income, and endeavor to
eliminate inequalities in status, facilities and opportunities, not only amongst individuals but
also amongst groups of people residing in different areas or engaged in different vocations.

Article 39: Certain principles of policy to be followed by the State

The State shall, in particular, direct its policy towards securing –


(a) that the citizen, men and women equally, have the right to an adequate means of livelihood;
(b) that the ownership and control of the material resources of the community are so distributed
as best to sub serve the common good;
(c) that the operation of the economic system does not result in the concentration of wealth and
means of production to the common detriment;
(d) that there is equal pay for equal work for both men and women;
(e) that the health and strength of workers, men and women, and the tender age of children are
not abused and that citizens are not forced by economic necessity to enter avocations
unsuited to their age or strength;
(f) that children are given opportunities and facilities to develop in a healthy manner and in
conditions of freedom and dignity and that childhood and youth are protected against
exploitation and against moral and material abandonment.

Article 39A: Equal justice and free legal aid

The State shall secure that the operation of the legal system promotes justice, on a basis of equal
opportunity, and shall, in particular, provide free legal aid, by suitable legislation or schemes or in any
other way, to ensure that opportunities for securing justice are not deniedto any citizen by reason of
economic or other disabilities.

Article 40: Organization of village panchayats

The State shall take steps to organize village panchayats and endow them with such powers and
authority as may be necessary to enable them to function as units of self-government.

Article 41: Right to work, to education and to public assistance in certain cases

The State shall, within the limits of its economic capacity and development, make effective provision for
securing the right to work, to education and to public assistance in cases of unemployment, old age,
sickness and disablement, and in other cases of undeserved want.

Article 42: Provision for just and humane conditions of work and maternity relief

The State shall make provision for securing just and humane conditions of work and for maternity relief.

Article 43: Living wage, etc., for workers

The State shall endeavor to secure, by suitable legislation or economic organization or in any other way,
to all workers agricultural, industrial or otherwise, work, a living wage, conditions of work ensuring a
decent standard of life and full enjoyment of leisure and social and cultural opportunities and, in
particular, the State shall endeavor to promote cottage industries on an individual or co-operative basis
in rural areas.

Article 43A: Participation of workers in management of industries

The State shall take steps, by suitable legislation or in any other way, to secure the participation of
workers in the management of undertakings, establishments or other organization engaged in any
industry.

Article 44: Uniform civil code for the citizen

The State shall endeavour to secure for the citizens a uniform civil code throughout the territory of
India.

Article 45: Provision for free and compulsory education for children

The State shall endeavor to provide, within a period of ten years from the commencement of this
Constitution, for free and compulsory education for all children until they complete the age of fourteen
years.

Article 46: Promotion of educational and economic interests of Scheduled Castes, Scheduled Tribes
and other weaker sections

The State shall promote with special care the educational and economic interests of the weaker
sections of the people, and in particular, of the Scheduled Castes and the Scheduled Tribes, and shall
protect them from social injustice and all forms of exploitation.

Article 47: Duty of the State to raise the level of nutrition and the standard of living and to improve
public health

The State shall regard the raising of the level of nutrition and the standard of living of its people and
the improvement of public health as among its primary duties and, in particular, the State shall
endeavor to bring about prohibition of the consumption except for medicinal purpose of intoxicating
drinks and of drugs which are injurious to health.

Article 48: Organization of agriculture and animal husbandry

The State shall endeavor to organize agriculture and animal husbandry on modern and scientific lines
and shall, in particular, take steps for preserving and improving the breeds, and prohibiting the
slaughter, of cows and calves and other mulch and draught cattle.

Article 48A: Protection and improvement of environment and safeguarding of forests and wildlife

The State shall endeavor to protect and improve the environment and to safeguard the forests and
wildlife of the country.

Article 49: Protection of monuments and places and objects of national importance

It shall be the obligation of the State to protect every monument or place or object of artistic or historic
interest, declared by or under law made by Parliament to be of national importance, from spoliation,
disfigurement, destruction, removal, disposal or export, as the case may be.

Article 50: Separation of judiciary from the executive

The State shall take steps to separate the judiciary from the executive in the public services of the State.

Article 51: Promotion of international peace and security The State shall endeavor to –
(a) promote international peace and security;
(b) maintain just and honorable relations between nations;
(c) foster respect for international law and treaty obligations in the dealings of organized people
with one another; and
(d) encourage settlement of international disputes by arbitration.

 What is the difference between legislature, executive and judiciary?

➢ The modern democratic setup is nowadays is generally based on the concept of the
“Doctrine of Separation of Powers”. Earlier all the powers were vested in only one
authority, i.e the King. Concentration of all the powers in one particular authority led to
abuse of power as the decision taken by the authority was whimsical and arbitrary on
several occasions. A French lawyer and Political thinker Montesquieu proposed the
“Doctrine of Separation of Powers” in his book “De l'esprit des Lois” also known as the
“Spirit of Laws”. This theory suggested formation of three separate authorities for
separate functions.
While framing the Indian constitution the Constituent Assembly was working meticulously to frame a
system of Governance in which the powers conferred by the people won't be vested on a single
person/institution. Hence they devised the principle of 'Separation of powers' among the three pillars of
democracy!
• Legislature to Legislate,
• Executive to implement,
• Judiciary to adjudicate,

Therefore, every ideal democratic state has three basic organs. The Legislature, the Executive and the
Judiciary.

➢ The Legislature: It is the law and policy making body. Generally, new laws or
policies are introduced in the Parliament/ State Legislature in the form of Bills. These Bills
once passed by the Legislature are sent to the President for assent. Once the passed bill
gets the assent of the President, it becomes the law or the policy comes
into effect. Parliament or the State Legislature are the forms of “Legislature”. It is the
policy making body of India. Each and every bill proposed by the executive has to be
initiated, discussed, reviewed, amended and voted upon in the legislature. So ultimately it
is the legislature that decides which bills should be passed. The Executive can bypass the
legislature through Ordinance. But the validity of this ordinance is six months only and it
has to be ratified by the legislature.

➢ The Executive: The executive is responsible for the effective implementation of the laws/
policies passed by the Legislature. The President, Vice- President along with the Council
of Ministers and the Attorney General form the Executive branch of the Center, whereas
Governor is the Executive at the State level along with the council of Ministers.

Prime Minister along with his council of ministers is called the Temporary Executives.
They implement the policies framed by the legislature. They are elected for every five years and
hence called as Temporary Executives. These executives are drawn from the legislature.

Civil servants and other officers, staffs working under the government of India is called the
Permanent Executives. They are assigned the task of policy implementation.

The Bureaucrats and civil servants are also parts of the executive as they are responsible for effective
implementation of the law and policies at local level.

➢ The Judiciary: It is the most important feature of democracy. It is responsible for


safeguarding the interests and the fundamental rights of the people. Judiciary consists
of the Hon’ble Supreme Court, Hon’ble High Court and other lower courts.
Judiciary keeps a tab on the activities of the government and plays an important role in the
event of violation of Fundamental Rights of the people of the country. Judiciary also has the
authority to examine the validity of the Laws enacted by the Parliament on the constitutional
parameters. Judiciary is the adjudicating body.
Indian judiciary is an integrated and independent judiciary meaning Supreme court is at
the helm of the judiciary and all other courts comes below the Supreme court and the
Judiciary is independent of the executive and the legislature.

Judiciary has the power to review any law passed by the Parliament and can declare a law
null and void if it violates the constitution.
LAW OF CONTRACT

 Who Performs the Contract?


➢ There are at least two parties to a contract, a promisor, and a promisee. A
promisee is a party to which a promise is made and a promisor is a party which
performs the promise. Three sections of the Indian Contract Act, 1872 define
who performs a contract – Section 40, 41, and 42. In this article, we will look at
these sections to understand the concept of the performance of a contract
better.

❖ Section 40
Section 40 of the Indian Contract Act, 1872 states
If the nature of a contract indicates that either of the parties intended that the promise
contained in the contract must be performed by the promisor himself
• Then the promisor is obligated to perform the promise
• Else the promise can be performed by the promisor or his representatives or an
employed agent.
Let’s look at some examples: Peter promises to pay Rs 50 to John. In this case, Peter can
perform the promise himself by paying the money to John or can ask someone else to pay him.
Also, if Peter dies before fulfilling his promise, then his representatives are required to perform
the promise or employ someone to do the same.
We will take a look at another example. Peter is a singer and he promises to sing a song at
John’swedding reception. In this case, the nature of the contract requires Peter to perform
the promise himself. He cannot delegate it to someone. So, there are three possibilities for the
performance of the promise. It can be done by the promisor, his representatives or his agent,
depending on the nature of the contract.

• Promisor Performs the Promise


If a contract indicates that the parties intended for the promisor to fulfill the promise himself,
then the promisor is obligated to perform the promise. Usually, these include promises which
involve personal skills, experience, or expertise and are usually based on trust between the
promisor and the promisee.
The example 2 cited above about Peter singing at John’s wedding reception is a good example
of
a personal skill being required to perform the promise.

• Agent Performs the Promise


If the contract does not require the personal consideration of the promisor, then the promisor can
employ a competent person to perform the promise. The example 1 cited above is a good example
of Peter employing an agent to pay Rs 50 to John.

• Legal Representatives Perform the Promise


If the promisor dies before performing the promise, then the legal representatives become
responsible for the same. If the promise involves utilization of personal skills or expertise, then the
consideration ceases with the death of the promisor. However, in all other scenarios, the legal
representatives are obligated to perform the promise unless the contract has a contrary intention
specified. Also, the liability of the legal representatives is limited to the value of the property
inherited by them.
Peter promises to pay John an amount of Rs 10,000 within one month of delivery of certain
goods. John delivers the goods. However, Peter dies before he can pay the money to John. Now, it
is his legal representative’s responsibility to ensure that John receives the payment. The
representative can pay himself or employ someone for the same.
❖ Section 41
If the promisee accepts the performance of a promise from a third person, then he cannot
enforce it against the promisor at a later date. Hence, the performance of the promise by a third-
party discharges the promisor of his obligations even if he has not authorized the third-party to
perform the promise.
Peter promises to pay John and an amount of Rs 10,000 for painting his house. John finishes the
job but Peter is unable to pay him. Oliver, a common friend of Peter and John, offers Rs 6,000 to
John on behalf of Peter, which he accepts. Eventually, John files a suit for recovery against Peter. The
Court holds that:
• John accepted Rs 6,000 from a third-person.
• Peter has not authorized the third-person.
• Hence, John’s act has discharged Peter of his liability to pay the entire amount.
• John can only claim Rs 4,000 from Peter now.

❖ Section 42
If the promisors agree to perform a promise together – joint promise – then they are jointly
obligated to fulfil the promise, unless the contract specifies a contrary intention. Also, if any of the
promisors die, then their legal representatives must fulfil the promise jointly with the surviving
promisors. If all the promisors die, then the legal representatives of each of them must perform the
promise jointly.
Peter, John, and Oliver jointly promise to pay Rs 900,000 to Rita. However, Peter dies before paying
the money. In this case, Peter’s representative – Jack, is now jointly responsible along with John and
Oliver to pay Rita the amount.
Unfortunately, before Rita receives her money, all the three promisors, i.e. Peter, John, and Oliver
die. In this case, Jack – Peter’s representative, Tony – John’s representative, and Sam – Oliver’s
representative must jointly pay the amount to Rita.

✓ Solved Example for You


Q: Peter and John are childhood friends. Peter is a painter and agrees to paint John’s portrait
for a payment of Rs 20,000. However, he is in urgent need and requests John to pay him in
advance. John obliges and they enter into a contract for the same. Peter starts making
John’s portrait. However, before he can finish, he dies in a car accident. Jack inherits Peter’s
property. Can John file a suit for recovery since he had already made the payment but did
not get his portrait in return?

Ans: Since the contract was based on personal consideration, that is Peter’s painting skills
and there was no clause in the contract regarding a refund if Peter fails to deliver the
portrait, John cannot file a suit for recovery of his money.

❖ Types of Contracts – Based on Validity


Now that you know what a contract is, can you identify the various Types of Contracts? A proper
knowledge of the types of contracts is essential as it will allow you to decide the legal
ramifications of an agreement. Here we will see the different Types of Contracts classified as per
their validity.

➢ Types of Contracts - On The Basis Of Validity

Chapter 2 of the Indian Contract Act, 1872 discusses the voidable contracts and void agreements.
On the basis of validity or enforceability, we have five different types of contracts as given below.

➢ Valid Contracts

The Valid Contract as discussed in the topic on “Essentials of a Contract” is an agreement that is
legally binding and enforceable. It must qualify all the essentials of a contract.

➢ Void Contract or Agreement

The section 2(j) of the Act defines a void contract as “A contract which ceases to be enforceable by
law becomes void when it ceases to be enforceable”. This makes all those contracts that are not
enforceable by a court of law as void. We have already stated examples of these kinds of contracts
in the “Essentials of a Contract”.

Example: A agrees to pay B a sum of Rs 10,000 after 5 years against a loan of Rs. 8,000. A
dies of natural causes in 4 years. The contract is no longer valid and becomes void due to
the non- enforceability of the agreed terms.

➢ Voidable Contract

These types of Contracts are defined in section 2(i) of the Act: “An agreement which is
enforceable by law at the option of one or more of the parties thereto, but not at the option of the
other or others, is a voidable contract.” This may seem difficult to wrap your head aroundbut
consider the following example:

Suppose a person A agrees to pay a sum of Rs. 10,0000 to a person B for an antique chair. This
contract would be valid, the only problem is that person B is a minor and can’t legally enter a
contract. So this contract is a valid contract from the point of view of A and a “voidable” contract
from the point of view of B. As and when B becomes a major, he may or may not agree to the
terms. Thus this is a voidable contract.

A voidable contract is a Valid Contract. In a voidable contract, at least one of the parties has to be
bound to the terms of the contract. For example, the person A in the above example. The other
party is not bound and may choose to repudiate or accept the terms of the contract. If they so
choose to repudiate the contract, the contract becomes void. Otherwise, a voidable contract is a
valid contract.

➢ Illegal Contract

An agreement that leads to one or all the parties breaking a law or not conforming to the norms of
the society is deemed to be illegal by the court. A contract opposed to public policy is also illegal.

Several examples may be cited to illustrate an illegal contract. For example, A agrees to sell
narcotics to B. Although this contract has all the essential elements of a valid contract, it is still
illegal. The illegal contracts are deemed as void and not enforceable by law. As section 2(g) ofthe
Act states: “An agreement not enforceable by law is said to be void.”

Thus we can say that all illegal contracts are void but the reverse is not true. Both the void
contracts and the illegal contracts can’t be enforceable by law. Illegal contracts are actually void ab
initio (from the start or the beginning).

Also because of the criminal aspects of the illegal contracts, they are punishable under law. All the
parties that are found to have agreed on an illegal promise are prosecuted in a court of law.

➢ Unenforceable Contracts

Unenforceable contracts are rendered unenforceable by law due to some technical. The
contract can’t be enforced against any of the two parties.

For example, A agrees to sell to B 100kgs of rice for 10,000/-. But there was a huge flood in the
states and all the rice crops were destroyed. Now this contract is unenforceable and cannot be
enforced against either party.

Solved Examples For You

Q1: List the main differences between a void and a voidable contract?
Answer: The following table will illustrate the major differences between a void and a
voidable contract.
Void Contract Voidable Contract

“An agreement which is enforceable by law at


“A contract which ceases to be enforceable by law the option of one or more of the parties thereto,
becomes void when it ceases to be enforceable”. but no at the option of the other or others, is a
voidable contract.”

A contract becomes void if either it lacks the


A contract becomes a voidable contract when
essential elements, the law changes drastically
at least one of the parties reserves its consent or
or the terms of the contract change such that it
the consent of one of the parties was not free
is no longer possible to enforce the contract in a
at the time of the formation of the contract.
court of law.

The validity and enforceability of the voidable


contract depend on the choice of the unbound
Void contracts can’t be fulfilled.
part If the unbound party decides to repudiate
the contract it becomes void.

This type of contract can’t grant any rights or The right to rescind a voidable contract is
considerations to any of the involved parties. retained by the unbound party.
➢ FREE CONSENT

➢ Introduction to Free Consent

According to Section 13,” two or more persons are said to be consented when they agree upon the same
thing in the same sense (Consensus-ad-idem). According to Section 14, consent is said to be free if it is
not caused by coercion, undue influence, fraud, misrepresentation and mistake.

➢ Elements vitiating free Consent

• Coercion (Section 15) : “Coercion” is the committing, or threatening to commit, any act
forbidden by the Indian Penal Code under(45,1860), or the unlawful detaining, or
threatening to detain, any property, to the prejudice of any person whatever, with the
intention of causing any person to enter into an agreement. For example, “A” threatens to
shoot “B” if he doesn’t release him from a debt which he owes to “B”. “B” releases “A”
under threat. Since the release has been brought about by coercion, such release is not
valid.

• Undue influence (Section 16): “Where a person who is in a position to dominate the will of
another enters into a contract with him and the transaction appears on the face of it, or
on the evidence, to be unconscionable, the burden of proving that such contract was not
induced by undue influence shall lie upon the person in the position to dominate the
will of the other.”(Section 16(2)) States that “A person is deemed to be in a position to
dominate the will of another;
✓ Where he holds a real or apparent authority over the other. For example, an employer
may be deemed to be having authority over his employee. An income tax authority over to
the assesse.
✓ Where he stands in a fiduciary relationship to other, For example, the relationship of
Solicitor with his client, spiritual advisor and devotee.
✓ Where he makes a contract with a person whose mental capacity is temporarily or
permanently affected by the reason of age, illness or mental or bodily distress”

• Fraud (Section 17) : “Fraud” means and includes any act or concealment of material fact or
misrepresentation made knowingly by a party to a contract, or with his connivance, or by
his agent, with intent to deceive another party thereto of his agent, or to induce him to
enter into the contract. Mere silence is not fraud. a contracting party is not obliged to
disclose each and everything to the other party. There are two exceptions where even
mere silence may be fraud, one is where there is a duty to speak, and then keeping
silence is fraud. Or when silence is in itself equivalent to speech, such silence is fraud.

• Misrepresentation (Section 18) : “causing, however innocently, a party to an agreement to


make a mistake as to the substance of the thing which is the subject of the agreement”.
• Mistake of fact (Section 20) : “Where both the parties to an agreement are under a
mistake as to a matter of fact essential to the agreement, the agreement is void”. A
party cannot be allowed to get any relief on the ground that he had done some
particular act in ignorance of law. Mistake may be bilateral mistake where both parties to
an agreement are under mistake as to the matter of fact. The mistake must relate to a
matter of fact essential to the agreement.
❖ Discharge of a Contract
A contract creates certain obligations on one or all parties involved. The discharge of a contracthappens
when these obligations come to an end. There are many ways in which a contract is discharged. In this
article, we will look at various such scenarios.

1] Discharge by Performance

When the parties to a contract fulfill the obligations arising under the contract within the time and manner
prescribed, then the contract is discharged by performance.
Example: Peter agrees to sell his cycle to John for an amount of Rs 10,000 to be paid by John on the
delivery of the cycle. As soon as it is delivered, John pays the promised amount.

Since both the parties to the contract fulfill their obligation arising under the contract, then it is discharged
by performance. Now, discharge by the performance of a contract can be by:
1. Actual performance
2. Attempted performance

As shown in the example above, actual performance is when all the parties to a contract do what they
had agreed for under the contract. On the other hand, it is possible that when the promisor attempts to
perform his promise, the promise refuses to accept it. In such cases, it is called attempted performance or
tender.

2] Discharge by Mutual Agreement

If all parties to a contract mutually agree to replace the contract with a new one or annul or remit
or alter it, then it leads to a discharge of the original contract due to a mutual agreement.
Example: Peter owes Rs 100,000 to John and agrees to repay it within one year. They document the
debt under a contract. Subsequently, he loses his job and requests John to accept Rs 75,000 as a
final settlement of the loan. John agrees and they make a contract to that effect. This discharges
the original contract due to mutual consent.

3] Discharge by the Impossibility of Performance

If it is impossible for any of the parties to the contract to perform their obligations, then the impossibility
of performance leads to a discharge of the contract. If the impossibility exists from the start, then it is
impossibility ab-initio. However, the impossibility might also arise later due to:

• An unforeseen change in the law


• Destruction of the subject-matter essential to the performance
• The non-existence or non-occurrence of a particular state of things which was considered a
given for the performance of the contract.
• A declaration of war
Example: Peter enters into a contract with John to marry his sister Olivia within one year. However,
Peter meets with an accident and becomes insane. The impossibility of performance leads to a
discharge of the contract.

4] Discharge of a Contract by Lapse of Time

The Limitation Act, 1963 prescribes a specified period for performance of a contract. If the promisor fails
to perform and the promise fails to take action within this specified period, then the latter cannot seek
remedy through law. It discharges the contract due to the lapse of time.

Example: Peter takes a loan from John and agrees to pay installments every month for the next five
years. However, he does not pay even a single installment. John calls him a few times but then
gets busy and takes no action. Three years later, he approaches the court to help him recover his
money. However, the court rejects his suit since he has crossed the time-limit of three years to
recover his debts.

5] Discharge of a Contract by Operation of Law

A contract can be discharged by operation of law which includes insolvency or death of thepromisor.

6] Discharge by Breach of Contract

If a party to a contract fails to perform his obligation according to the time and place specified, then he is
said to have committed a breach of contract. Also, if a party
repudiates a contract before the agreed time of performance of a contract, then he is said to have
committed an anticipatory breach of contract.

In both the cases, the breach discharges the contract. In case of:

• an actual breach, the promisee retains his right of action for damages.

• an anticipatory breach of contract, the promisee cannot file a suit for damages. It also
discharges the promisor from performing his part of the contract.

7] Discharge of a Contract by Remission

A promisee can waive or remit the performance of promise of a contract, wholly or in part. He can also
extend the time agreed for the performance of the same.

In example 3 above, Peter only repays a part of the money he owes to John. However, John agrees
to accept it as a final settlement of the debt. John’s act of remission discharges the contract.
8] Discharge by Non-Provisioning of Facilities

In many contracts, the promisee agrees to offer reasonable facilities to the promisor for the performance
of the contract. If the promisee fails to do so, then the promisor is discharged of all liabilities arising due to
non-performance of the contract.

Example: Peter agrees to fix John’s garage floor provided he keeps his car out for at least 6 hours.
Peter approaches him a few times but John is reluctant to get his car out. John fails to provide
reasonable facilities to Peter (an empty floor). This discharges him of all obligations arising under
the contract.

9] Discharge of a Contract due to the Merger of Rights

In some situations, it is possible that inferior and superior right coincides in the same person. Insuch cases,
both the rights combine leading to a discharge of the contract governing the inferior rights.

Example: Peter rents John’s apartment for two years. One year into the contract, he offers to buy the
property from John, who agrees. The enter a sale contract and Peter becomes the owner of the
apartment. Here Peter has two rights; one accorded by the lease agreement making him the renter
and second by the sale agreement making him the owner. The former being an inferior right
merges with the superior one and discharges the lease contract.

✓ Solved Question for You

Q: Peter agrees to sell his laptop to John for an amount of Rs 15,000. He also promises to deliver it within 2
days. The next day, when Peter approached John with his laptop, John refuses to accept it without any
valid reason. Is the contract discharged?

Ans: Yes, in the above case the contract is discharged. The contract is discharged since Peter
attempted the performance of his promise.

❖ Breach Of Contract: Remedies

The five basic remedies for breach of contract include the following: money damages, restitution,
rescission, reformation, and specific performance. A money damage award
includes a sum of money that is given as compensation for financial losses caused by a breach
of contract. Parties injured by a breach are entitled to the benefit of the bargain they entered,
or the net gain that would have accrued but for the breach. The type of breach governs the
extent of damages that may be recovered.

If the breach is a total breach, a plaintiff can recover damages in an amount equal to the sum or value
the plaintiff would have received had the contract been fully performed by the defendant, including lost
profits. If the breach is only partial, the plaintiff may normally seek damages in an amount equal to the
cost of hiring someone else to complete the performance contemplated by the contract. However, if
the cost of completion is prohibitive and the portion of the unperformed contract is small, many courts
will only award damages in an amount equal to the difference between the diminished value of the
contract as performed and the full value contemplated by the contract.

For example, if the plaintiff agreed to pay the defendant $200,000 to build a house, but the
defendant only completed 90 percent of the work contemplated by the contract, a court might be
inclined to award $20,000 in damages if it would cost the plaintiff twice as much to hire someone
else to finish the last 10 percent. The same principles apply to damages sought for contracts that
are fully performed, but in a defective manner. If the defect is significant, the plaintiff can recover
the cost of repair. But if the defect is minor, the plaintiff may be limited to recovering the difference
between the value of the good or service actually received and the value of the good or service
contemplated by the contract.

Restitution is a remedy designed to restore the injured party to the position occupied prior to the
formation of the contract. Parties seeking restitution may not request to be compensated for lost profits
or other earnings caused by a breach. Instead, restitution aims at returning to the plaintiff any money
or property given to the defendant under the contract. Plaintiffs typically seek restitution when
contracts they have entered are voided by courts due to a
defendant’s incompetence or incapacity. The law allows incompetent and incapacitated persons to
disavow their contractual duties but generally only if the plaintiff is not made worse off by their
disavowal.

Parties that are induced to enter into contracts by mistake, fraud, undue influence, or duress may seek to
have the contract set aside or have the terms of the contract rewritten to do justice in the case.
Rescission is the name for the remedy that terminates the contractual duties of both parties, while
reformation is the name for the remedy that allows courts to change the substance of a contract to
correct inequities that were suffered. Like contracts
implied in law, however, courts are reluctant to rewrite contracts to reflect the parties’ actual agreement,
especially when the contract as written contains a mistake that could have been rectified through pre-
contract investigation. Thus, one court would not reform a contract that stipulated an incorrect amount
of acreage being purchased, since the buyer could have ascertained the correct amount by obtaining a
land survey before entering the contract.

Specific performance is an equitable remedy that compels one party to perform, as nearly as
practicable, his or her duties specified by the contract. Specific performance is available only when
money damages are inadequate to compensate the plaintiff for the breach. This ruling often happens
when the subject matter of a contract is in dispute.

Every parcel of land by definition is unique, if for no other reason than its location. However, rare
articles that are not necessarily one of a kind are still treated by the law as unique if it would be
impossible for a judge or jury to accurately calculate the appropriate amount of damages to award the
plaintiff in lieu of awarding him or her the unique article contemplated by the contract. Heirlooms and
antiques are examples of such rare items for which specific performance is usually available as a
remedy. However, specific performance may never be invoked to compel the performance of a personal
service, since doing so would constitute slavery in violation of the Thirteenth Amendment to the U.S.
Constitution.
❖ Contract of Indemnity and Law Of Guarantee
The term Indemnity literally means “Security against loss". In a contract of indemnity one party – i.e. the
indemnifier promise to compensate the other party i.e. the indemnified against the loss suffered by the
other.

The English law definition of a contract of indemnity is – “it is a promise to save a person
harmless from the consequences of an act". Thus it includes within its ambit losses caused not
merely by human agency but also those caused by accident or fire or other natural calamities.

The definition of a contract of indemnity as laid down in Section 124 – “A contract by which one
party promises to save the other from loss caused to him by the conduct of the promisor himself,
or by the conduct of any other person”, is called a contract of indemnity.

The definition provided by the Indian Contract Act confines itself to the losses occasioned due to the act
of the promisor or due to the act of any other person.

Under a contract of indemnity, liability of the promisor arises from loss caused to the promisee
by the conduct of the promisor himself or by the conduct of other person. [Punjab National
Bank v Vikram Cotton Mills].

Every contract of insurance, other than life insurance, is a contract of indemnity. The definition is
restricted to cases where loss has been caused by some human agency. [Gajanan Moreshwar v
Moreshwar Madan]

Section 124 deals with one particular kind of indemnity which arises from a promise made by an
indemnifier to save the indemnified from the loss caused to him by the conduct of the indemnifier
himself or by the conduct of any other person, but does not deal with those classes of cases where the
indemnity arises from loss caused by events or accidents which do not depend upon the conduct of
indemnifier or any other person. [Moreshwar v Moreshwar]

"Contract of indemnity" defined.-A contract by which one party promises to save the other from
loss caused to him by the conduct of the promisor himself, or by the conduct of any other person,
is called a "contract of indemnity".

Illustration

A contracts to indemnify B against the consequences of any proceedings which C may take against
B in respect of a certain sum of 200 rupees. This is a contract of indemnity.

➢ Nature of Contract of Indemnity –

A contract of indemnity may be express or implied depending upon the circumstances of the case,
though Section 124 of the Indian Contract Act does not seem to cover the case of implied indemnity.

A broker in possession of a government promissory note endorsed it to a bank with forged


endorsement. The bank acting in good faith applied for and got a renewed promissory note from the
Public Debt Office. Meanwhile the true owner sued the Secretary of State for conversion who in turn
sued the bank on an implied indemnity. It was held that – it is general principle of law when an act is
done by one person at the request of another which act is not
in itself manifestly tortious to the knowledge of the person doing it, and such act turns to be injurious to
the rights of a third person, the person doing it is entitled to an indemnity from him who requested
that it should be done. [Secretary of State v Bank of India].

The Indian Contract Act also deals with special cases of implied indemnity –

1. U/s 69 if a person who is interested in payment of money which another is bound by law to pay
and therefore pays it, he is entitled to be indemnified. For instance – if a tenant pays certain
electricity bill to be paid by the owner, he is entitled to be indemnified by the owner.

2. Section 145 provides for right of a surety to claim indemnity from the principal debtor for all
sums which he has rightfully paid towards the guarantee.

3. Section 222 provides for liability of the principal to indemnify the agent in respect of all amounts
paid by him during the lawful exercise of his authority.

The plaintiff, an auctioneer, acting on the instruction of the defendant sold certain cattle which
subsequently turned out to belong to someone else other than the defendant. When the true
owner sued the auctioneer for conversion, the auctioneer in turn sued the defendant for
indemnity. The Court held that the plaintiff having acted on the request of the defendant was
entitled to assume that, if it would turned out to be wrongful, he would be indemnified by the
defendant. [Adamson v Jarvis].

➢ Validity of Indemnity Agreement

A contract of indemnity is one of the species of contracts. The principles applicable to contracts in
general are also applicable to such contracts so much so that the rules such as free consent, legality of
object, etc., are equally applicable.

Where the consent to an agreement is caused by coercion, fraud, misrepresentation, the agreement is
voidable at the option of the party whose consent was so caused. As per the requirement of the
Contract Act, the object of the agreement must be lawful. An agreement, the object of which is opposed
to the law or against the public policy, is either unlawful or void depending upon the provision of the
law to which it is subject.

➢ Contract of indemnity when enforceable –

The question whether the liability of indemnifier commences only when the indemnified has actually
suffered loss or when there is an apprehension that the indemnified by all chances is likely to suffer it.

The former view was held in cases like – Shankar Nimbaji v Laxman Sapdu / Chand Bibi v
Santosh Kumar Pal.

The plaintiff filed a suit to recover Rs. 5,000/- and interest from defendant by the sale of a mortgaged
property and, in case of deficit, for a decree against the estate of defendant 2 which was in the hands of
his sons, the defendant 2 died during the pendency of the suit. It was held that plaintiff cannot sue the
defendant in anticipation that the proceeds realized by the sale of the mortgaged property would be
insufficient and there would be some deficit. [Shankar Nimbaji v Laxman Sapdu]
The defendant’s father while purchasing certain property covenanted to pay off mortgage debt
incurred by the plaintiff and also promised to indemnify him if they were made liable for the mortgage
debt. The defendant’s father failed to pay off the mortgage debt and plaintiff filed an action to enforce
the covenant. It was held as the plaintiff had not yet suffered any damage, the suit was premature so far
as the cause of action on indemnity was concerned. [Chand Bibi v Santosh Kumar Pal]

A different point of view was held by the Courts in the following cases –

Plaintiff company agreed to act as commission agent for the defendant firm for purchase and sale of
“Hessian" and “Gunnies" and charge commission on all such purchases and the defendant firm agreed
to indemnify the plaintiff against all losses in respect of such transactions. The plaintiff company
purchased certain Hessian from one Maliram Ramjidas. The defendant firm failed to pay for or take
delivery of the Hessian. Then Maliram Ramjidas resoled it at lesser price and claimed the difference as
damages from the plaintiff company. The plaintiff company went into liquidation and the liquidator
filed a suit to recover the amount claimed by Maliram from the defendant firm under the indemnity.
The defendant argued that in as much as the plaintiff had not yet paid any amount to Maliram in
respect of their liability they were not entitled to maintain the suit under indemnity. It was held
negative and decided in plaintiff’s favour with a direction that the amount when recovered
from the defendant firm should be paid to Maliram Ramjidas. [Osmal Jamal & Sons Ltd. v
Gopal Purushotham]

After the landmark decision in the case of Gajanan Moreshwar v Moreshwar Madan Mantri it has been
well established that the liability of the indemnifier commences as soon as the loss of the indemnified
becomes absolute, certain or imminent. It is not necessary that the promisee should pay for the loss.

❖ Right of the indemnity holder – (Section 125)

An indemnity holder (i.e. indemnified) acting within the scope of his authority is entitled to the following
rights –

1. Right to recover damages – He is entitled to recover all damages which he might have been
compelled to pay in any suit in respect of any matter covered by the contract.

2. Right to recover costs – He is entitled to recover all costs incidental to the institution and
defending of the suit.

3. Right to recover sums paid under compromise – He is entitled to recover all amounts which he
had paid under the terms of the compromise of such suit. However, the compensation must not
be against the directions of the indemnifier. It must be prudent and authorized by the indemnifier.

4. Right to sue for specific performance – He is entitled to sue for specific performance if he has
incurred absolute liability and the contract covers such liability. The promisee in a contract of
indemnity, acting within the scope of his authority, is entitled to recover from the promisor-

(1) All damages which he may be compelled to pay in any suit in respect of any matter to which
the promise to indemnify applies
(2) All costs which he may be compelled to pay in any such suit if, in bringing or defending it, he
did not contravene the orders of the promisor, and acted as it would have been prudent for him
to act in the absence of any contract of indemnity, or if the promisor authorized him to bring or
defend the suit ;

(3) All sums which he may have paid under the terms of any compromise of any such suit, if the
compromise was not.

It is important to note here that the right to indemnity cannot be claimed of dishonesty, lack of
good faith and contravention of the promisor’s request. However, the right cannot be negatived in
case of oversight. [Yeung v HSBC]

❖ Right of Indemnifier –

Section 125 of the Act only lays down the rights of the indemnified and is quite silent of the rights of
indemnifier as if the indemnifier has no rights but only liability towards the indemnified.

In the logical state of things if we read Section 141 which deals with the rights of surety, we
can easily conclude that the indemnifier’s right would also be same as that of surety.

Where one person has agreed to indemnify the other, he will, on making good the indemnity, be
entitled to succeed to all the ways and means by which the person indemnified might have protected
himself against or reimbursed himself for the loss. [Simpson v Thomson]

Principle of Subrogation is applicable because it is an essential part of law of indemnity and is


based on equity and the Contract Act contains no provision in contravention with [Maharaja
Shri Jarvat Singhji v Secretary of State for India]

➢ Contract of guarantee, surety, principal debtor and creditor:-

A "contract of guarantee " is a contract to perform the promise, or discharge the liability, of a third
person in case of his default. The person who gives the guarantee is called the " surety";

The person in respect of whose default the guarantee is given is called the " principal debtor ",
and the person to whom the guarantee is given is called the " creditor ". A guarantee may be
either oral or written.

Consideration for guarantee.-Anything done, or any promise made, for the benefit of the principal
debtor, may be a sufficient consideration to the surety for giving the guarantee.

Illustrations

(a) B requests A to sell and deliver to him goods on credit. A agrees to do so, provided C will
guarantee the payment of the price of the goods. C promises to guarantee the payment in
consideration of

As promise to deliver the goods. This is a sufficient consideration for C’s promise.
(b) A sells and delivers goods to B. C afterwards requests A to forbear to sue B for the debt for a
year, and promises that, if he does so, C will pay for them in default of payment by B. A agrees to
forbear as requested. This is a sufficient consideration for C’s promise.

(c) A sells and delivers goods to B. C afterwards, without consideration, agrees to pay for them in
default of B. The agreement is void.

➢ Suretys liability:-

The liability of the surety is coextensive with that of the principal debtor, unless it is otherwise provided
by the contract.

Illustration

A guarantees to B the payment of a bill of exchange by C, the acceptor. The bill is dishonored by
C. A is liable not only for the amount of the bill but also for any interest and charges which may
have become due on it.

Continuing guarantee.-A guarantee which extends to a series. Series of transactions is called a


"continuing guarantee".

Illustrations

(a) A, in consideration that B will employ C in collecting the rent of Bs zamindari, promises B to be
responsible, to the amount of 5,000 rupees, for the due collection and payment by C of those
rents.

This is a continuing guarantee.

(b) A guarantees payment to B of the price of five sacks of flour to be delivered by B to C and to be
paid for in a month. B delivers five sacks to C. C pays for them. Afterwards B delivers four sacks to
C, which C does riot pay for. The guarantee given by A was not a continuing guarantee, and
accordingly he is not liable for the price of the four sacks.

Revocation of continuing guarantee.-A continuing guarantee may at any time be revoked by the
surety,as to future transactions, by notice to the creditor.

Illustrations

(a) A, in consideration of Bs discounting, at As request, bills of exchange for C, guarantees to B, for


twelve months, the due payment of all such bills to the extent of 5,000 rupees. B discounts bills
for C to the extent of 2,000 rupees. Afterwards, at the end of three months, A revokes the
guarantee. This revocation discharges A from all liability to B for any subsequent discount. But A
is liable to B for the 2,000 rupees, on default of C.

• Revocation of continuing guarantee by surety’s death-The death of the surety operates, in


the absence of any contract to the contrary, as a revocation of a continuing guarantee, so
far as regards future transactions. Discharge of surety by variance in terms of contract.-
Any variance, made without the surety’s consent, in the terms of the contract between the
principal 1[debtor] and the creditor, discharges the surety as to transactions subsequent to
the variance.

Illustrations

(a) A becomes surety to C for Bs conduct as a manager in Cs bank. Afterwards B and C contract,
without As consent, that Bs salary shall be raised, and that he shall become liable for one-fourth
of the losses on overdrafts. B allows a customer to overdraw, and the bank loses a sum of money.
A is discharged from his surety ship by the variance made without his consent, and is not liable
to make good this loss.

(b) A guarantees C against the misconduct of B in an office to which B is appointed by C, and of


which the duties are defined by an Act of the Legislature. By a subsequent Act, the nature of the
office is materially altered. Afterwards, B misconducts himself. A is discharged by the change
from future liability under his guarantee, though the misconduct of B is in respect, of a duty not
affected by the later Act.

(c) C contracts to lend B 5,000 rupees on the 1st March. A guarantees repayment. C pays the
5,000 rupees to B on the 1st January. A is discharged from his liability, as the contract has been
varied, inasmuch as C might sue B for the money before the 1st of March.

• Discharge of surety by release or discharge of principal debtor: - The surety is


discharged by any contract between the creditor and the principal debtor, by which the
principal debtor is released or by any act or omission of the creditor, the legal
consequence of which is the discharge of the principal debtor.

Illustrations

(a) A contracts with B to grow a crop of indigo an As land and to deliver it to B at a fixed rate, and
C guarantees As performance of this contract. B diverts a stream of water which is necessary for
irrigation of As land and thereby prevents him from raising the indigo. C is no longer liable on his
guarantee.

Discharge of surety when creditor compounds with, gives time to, or agrees not to sue, principal
debtor- A contract between
the creditor and the principal debtor, by which the creditor makes a composition with, or
promises to give time to, or not to sue, the principal debtor, discharges the surety, unless the
surety assents to such contract.

Surety not discharged when agreement made with third person to give time to principal debtor.
Where a contract to give time to the principal debtor is made by the creditor with a third person,
and not with the principal debtor, the surety is not discharged.

Illustration

(a) C, the holder of an overdue bill of exchange drawn by A as surety for B, and accepted by B,
contracts with M to give time to B. A is not discharged.
• Release of one co-surety does not discharge others-
Where there are co-sureties, a release by the creditor of one of them does not
discharge the others; neither does it free the surety so released from his responsibility
to the other sureties. Discharge of surety by creditors act or omission impairing
surety’s eventual remedy.

• Guarantee obtained by misrepresentation invalid-


Any guarantee which has been obtained by means of misrepresentation made by the
creditor, or with his knowledge and assent, concerning a material part of the
transaction, is invalid.

• Guarantee on contract that creditor shall not act on it until co-surety joins- Where a
person gives a guarantee upon a contract that the creditor shall not act upon it until
another person has joined in it as co-surety, the guarantee is not valid if that other
person does not join.

• Co-sureties liable to contribute equally-Where two or more persons are CO-sureties


for the same debt or duty, either jointly or severally, and whether under the same or
different contracts, and whether with or without the knowledge of each other, the co-
sureties, in the absence of any contract to the contrary, are liable, as between
themselves, to pay each an equal share of the whole debt, or of that part of it which
remains unpaid by the principal debtor.

Illustrations

(a) A, B and C are sureties to D for the sum of 3,000 rupees lent to E. E makes default in
payment. A, la and C are liable, as between themselves, to pay 1,000 rupees each.

(b) A, B and C are sureties to D for the sum of 1,000 rupees lent to E, and there is a contract
between A, B and C that A is to be responsible to the extent of one-quarter, B to the extent of
one-quarter, and C to the extent of one-half. E makes default in payment. As between the
sureties, A is liable to pay 250 rupees, B 250 rupees, and C 500 rupees.

• Liability of co-sureties bound in different sums- Co-sureties who


are bound in different sums are liable to pay equally as far as the limits of their
respective obligations permit.

Illustrations
(a) A, B and C, as sureties for D, enter into three several bonds, each in a different penalty, namely,
A in the penalty of 10,000 rupees, B in that of 20,000 rupees, C in that of 40,000 rupees,
conditioned for Ds duly accounting to E. D makes default to the extent of 30,000 rupees. A, B and
C are liable to pay 10,000 rupees.

(b) A, B and C, as sureties for D, enter into three several bonds, each in a different penalty,
namely, A in the penalty of 10,000 rupees, B in that of 20,000 rupees, C in that of 40,000 rupees,
conditioned for Ds duly accounting to E. D makes default to the extent of 70,000 rupees. A, B and
C have to pay each the full penalty of his bond.

➢ Difference between Indemnity and Guarantee:-

1. In a contract of indemnity there are two parties i.e. indemnifier and indemnified. A
contract of guarantee involves three parties i.e. creditor, principal debtor and surety.
2. An indemnity is for reimbursement of a loss, while a guarantee is for security of the
creditor.
3. In a contract of indemnity the liability of the indemnifier is primary and arises when the
contingent event occurs. In case of contract of guarantee the liability of surety is
secondary and arises when the principal debtor defaults.
4. The indemnifier after performing his part of the promise has no rights against the third
party and he can sue the third party only if there is an assignment in his favour. Whereas
in a contract of guarantee, the surety steps into the shoes of the creditor on discharge of
his liability, and may sue the principal debtor.

TORT & ITS GENERAL PRINCIPLES

DEFINITION OF TORTS
The term “Tort” is the French equivalent to the English word “Wrong” and Roman Law term “Delict”. The word
“Tort” is derived in turn from Latin term “tortum” i.e. to twist, and implies the conduct which is twisted or
tortuous.
It means breach of some duty independent of contract giving rise to a civil cause of action and for which
compensation if recoverable.
The term Tort may also be defined as a civil wrong independent of contract for which appropriate remedy is an
action for unliquidated damages. The principal aim of the law of torts is compensation of victims or their
dependents.
Person committing a tort is called a tort-feasor or wrong doer and his misdoing is a tortuous act.

CONTRACT AND TORT


While considering the definition of tort as defined above the words prominently appearing in the definition are
“independent of contract” meaning thereby that the tort is not something which form part of contract and in turn
implying that there is a marked and a clear distinction between a Contract and a Tort. To understand the
distinction between the two and distinctions may be tabulated as under.
CONTRACT TORT
A Contract is founded upon consent. A Tort is inflicted against or without consent.
Contract necessitates a privity between the In Tort there is no such privity between the
parties to it. parties.
Breach of Contract is an infringement of a A Tort is a right in rem i.e. of a right vested in
right in personam, i.e. of a right available some determined person, either personally or
only against some determinate person or as a member of a community, and such right is
body, and which the community at large has available against the whole world large.
no concern.
In case of Contract the duty is fixed by the In case of the Tort, the duty is one imposed by
will and consent of the parties, and it is owed the law and is owed to the community at large.
to a definite person or persons. E.g. If A E.g. if A assaults B or damages B’s property
agrees to sell goods to B for a price, and without lawful cause or excuse, it is Tort. The
either party fails to perform the Contract, the duty violated here is a duty imposed by the
case is one of breach of Contract. In this case law, and that is the duty not to do unlawful
there is no duty owed by A except to B and harm to the person or property of another.
none owned in turn by B except to A. The
duty that is violated is a specific duty owed
by either party to the other alone, as
distinguished to the general duty owed to
the community at large.
In case of breach of Contract, the motive In case of Tort, it is often taken into
(intention) for the breach is immaterial. consideration.
In case of breach of Contract, damages are In an action for Tort however when the injury is
only compensation. to a person, character or feelings and the facts
disclose improper motive or conduct such as
fraud, malice, violence, cruelty, which
aggravate the injury, aggravated and
exemplary damages are awarded, so as to
deter the wrong doer to prevent such breach
in future.
Intention of imposing damages under the Intention of Torts is aimed at allocation or
law relating to Contract is with an aim to see prevention of losses.
that the promises as are made under the
Contract are performed.

General Principles of Torts: -


Law imposes a duty on every individual to respect the legal right bestowed on others and any person interfering
with someone else’s enjoyment of their legal rights said to have committed a tort. The basic principle of the law
of tort is that every person has certain rights/ interests which are protected by law. Any act of omission or
commission which causes damage to the legally protected interest of an individual shall be considered to be a
tort, the remedy for which is an action for unliquidated damages. Tort is generally a breach of duty. In India, the
law of tort is uncodified and is still in the process of development.

However, following are the 10 most important principles of tort:


1. PRINCIPLE OF DAMNUM SINE INJURIA AND INJURIA SINE DAMNUM
• Damnum sine injuriais a Latin maxim which means damage without legal injury. When there is an
actual damage caused to the plaintiff without an infringement of his legal right, no action lies against
the defendant. In order to make someone liable in tort, plaintiff must prove that he has sustained legal
injury. Damage without injury is not actionable in the law of torts.
Example: A sets up a rival school opposite to B’s school with a low fee structure as a result of which students
from B’s school flocked to A’s school thereby causing a huge financial loss to A. This act of A is not actionable
in law of torts since it did not lead to the violation of any legal right of the plaintiff although he has sustained
financial loss.
Injuria sine damnum is a Latin term which means legal injury without any damage. This implies an
infringement of the legal rights of a person without any actual loss. Loss in this sense could mean loss of
health, monetary loss etc. Since there is an infringement of legal right of a person, right to sue for a remedy is
available against the wrongdoer regardless of the fact whether any actual loss is sustained or not.

2. PRINCIPLE OF VICARIOUS LIABILITY


It is a general rule that a person is responsible for his own act of omission and commission but in certain
cases a person is liable for the act of others. This is known as vicarious liability.The essential elements of
vicarious liability are as follows:
• There must be a relationship of a certain kind.
• The wrongful act must be related to the relationship in a certain way.
• The wrongful act must be done within the course of employment.
Most common example of vicarious liability include:
Employers liability for the act of his servant during the course of employment: This liability is based on
the principle of “respondent superior” whereby a person is responsible for the act of his subordinate
andquifacit per alium per se which means he who does an act through another is deemed in law to do it
himself.
The essential elements amounting to vicarious liability of a master for the tort of his servant are as follows:
• There should be a master-servant relation.
• The act of omission or commission should be done within the course of employment.
Example: If A, driver of B in his course of employment negligently knocks down C while driving a car, B will be
responsible for the negligence of his driver A.
• Principal’s liability for the act of his agent: When an agent performs an act which is authorised by
the principle, the latter becomes liable for such an act of the agent provided the act is done within the
course of employment.
• Liability of partners for each other’s torts: When a partner in the normal course of business of a
partnership firm commits a tort, all the other partners are equally responsible for the tort as the guilty
partner.

3. PRINCIPLE OF VOLENTI NON FIT INJURIA


The Latin maxim volenti non fit injurialiterally means “to one who volunteers, no harm is done”. A person who
after knowing the risks and circumstances willingly and voluntarily consents to take the risk cannot ask for
compensation for the injury resulting from it. A person who voluntarily abandons his rights cannot sue for
any damage caused to him. It is used as a complete defence in the law of torts liberating the defendant from
all kinds of liability.Essential elements constituting volenti non fit injuria are as follows:
• Voluntary
• Agreement (express or implied)
• Knowledge of the risk
Example: By participating in a football match, the player willingly consents to bear the risk that may arise in
the normal course of the game.

4. PRINCIPLE OF NEGLIGENCE
Negligence is said to have been committed when a person owes a duty of care towards someone and
commits a breach of duty by failing to perform it resulting in a legal damage caused to the complainant. In
other words, a tort of negligence is committed when a person is injured due to the irresponsibility of another.
The damage so caused must be an immediate cause of the act of negligence and not a remote cause.
Essential elements of negligence are as follows:
• Duty to take care
• Beach of such a duty
• Legal damage caused to the complainant due to a breach of duty
Reasonable foreseeability is the basic principle on which the tort of negligence is based. When a person
before or at the time of committing an act can reasonably foresee that his act is likely to cause a damage to
the other person and he still continues to do it, he is said to have committed a tort of negligence.
COMPOSITE NEGLIGENCE: When the negligent act of two or more person results in the same damage, it is
called composite negligence. The liability in such a case is joint and several.
The burden of proof falls on the plaintiff that he has sustained legal damage due to a breach of duty on the
part of the defendant.However, in certain cases the plaintiff doesn’t have to prove negligence on the
defendant’s part. Such cases fall under the principle of re ipsa loquitur which means “things speak for itself”
where it is evident from the facts of the case that there has been negligence on the side of the defendant.
Example: A doctor while performing an operation leaves a pair of scissors inside the stomach of the patient.

5. PRINCIPLES OF PERSONAL SECURITY


Principles of personal security are as follows:
• ASSAULT
Assault is an act which creates in the mind of a person reasonable apprehension of a physical threat or a
harm accompanied by a capacity to carry out such a threat. It is important to note that there is an absence
of physical contact in assault. Essential elements of assault are as follows:
o Apprehension of harm
o Intention to use force
o Capacity to use force
• BATTERY
Battery refers to a harmful, offensive and unlawful touching of a person against his will. It is an application
of force to the body of another in an offensive manner. Battery is an accomplished assault.
Essential elements:
o Intention to use physical force
o Actual physical contact

• FALSE IMPRISONMENT
Unlawfully restraining a person without his will by someone who does not have any legal authority to do
so amounts to false imprisonment. A person may also be made liable for false imprisonment if he
intentionally restricts another person’s freedom of movement without any lawful justification. Arrest of a
person without any legal warrant and authority also amounts to false imprisonment.
Essential elements:
o Wilful detention
o Detention without consent
o Detention is unlawful
Example: A person locking another person in a room without the consent of the person being locked.

6. PRINCIPLE OF NUISANCE
The word nuisance is derived from the French word ‘nurie’ which means ‘to hurt’ or ‘to annoy’. Nuisance is an
unlawful interference with a person’s enjoyment of land or some rights over or in connection with it.
There are two types of nuisance:
• PUBLIC NUISANCE: It is an interference with the right to enjoyment of land of a large number of people
thereby causing inconvenience and annoyance. It is committed against the community at large and not
any particular individual. It covers a wide variety of minor crimes that harms or threatens the safety,
comfort and welfare of people at large. The extent to which the inconvenience has been caused may differ
from person to person.
Examples: Fireworks in the street, construction of a structure in the middle of a public way obstructing the
passage of people, etc.
• PRIVATE NUISANCE: It refers to an unlawful interference with a person’s use or enjoyment of his land
causing inconvenience and annoyance to the person. It should be noted that while public nuisance affects
the community at large, private nuisance affects an individual.
Example: Destruction of crops of an individual, a poisonous dog of a person enters into the neighbour’s
premises and causes destruction.
REMEDIES
o Damages
o Injunction
o Abatement

7. PRINCIPLE OF TRESPASS TO PROPERTY


Trespass to property refers to an unjustifiable physical encroachment of land of one person by another. If a
person directly enters upon another person’s land without permission or remains upon the land or places any
object upon the land, he is said to have committed the tort of trespass to land.
For an act of trespass to be actionable, it is necessary that the land in which the trespass has been committed
must be in direct possession of the plaintiff. For example, use of camera in order to view activities on the land of
another. The encroachment on plaintiff’s land should arise out of the direct consequence of the act of the
defendant and not any remote or indirect cause.Also, one of the most important elements of trespass to land is
the intention in the mind of the defendant not to commit trespass but to commit the act that amounts to
trespass. Trespass to land is actionable per se.
However, it should be noted that there is a difference between trespass to land and nuisance. Trespass is an
encroachment or interference on the property of a person whereas nuisance is an interference with the right to
enjoy his property.
CONTINUING TRESPASS: Continuing trespass occurs when there is a continuation of the presence after the
permission has been withdrawn or when the offending object remains on the property of the person entitled to
possession. For example, continuing to keep an object on someone’s land even after the permission has been
withdrawn.
Ways in which trespass to land can occur:
• Entry upon land
• Trespass to airspace (limited)
• Trespass to the ground beneath the surface

REMEDIES
Damages: the plaintiff is entitled to full compensation of the loss incurred by him.
Injunction: order by the court directing the defendant from doing or restraining from doing an act.

8. PRINCIPLES OF REPUTATION AND PRIVACY


The principles of reputation and privacy are as follows:

DEFAMATION
Defamation means publishing false and defamatory statement about someone without any lawful justification
which lowers his reputation in the eyes of the right thinking members of the society. In other words, defamation
means intentional false communication either written or spoken which harms a person’s reputation.

Two types of Defamation are:


LIBEL: This is a written form of defamation which is actionable per se. Libel refers to the statement which intends
to lower the reputation of another person without any lawful excuse. The statement must be in printed form
capable of being reproduced like cartoons, drawings, recordings, etc.
SLANDER: Slander is an oral form of defamation where false and defamatory statements are made by words
spoken or gestures which intend to lower the reputation of a person.
Essential elements of defamation are as follows:

• Statement must be published


• It must be defamatory
• It must be false
• It must refer to the plaintiff
Defences against an action for defamation are as follows:
• Statements made about a public personality
• Statements which are true
• Fair comment
• Consent of the aggrieved

PRINCIPLE OF STRICT LIABILITY AND ABSOLUTE LIABILITY

STRICT LIABILITY:
At times a person may be held responsible for doing a wrong even though there had been no negligence on his
part or no intention to do such wrong or even if he had taken necessary steps to prevent such a wrong from
happening. This is known as the principle of strict liability and is based on a no fault theory.
“Anyone who in the course of “non-natural” use of his land “accumulates” thereon for his own purposes anything
likely to do mischief if it escapes is answerable for all direct damage thereby caused. It imposes strict liability on
certain areas of nuisance law.”
The essential elements of strict liability are as follows:
• There has to be some hazardous thing brought by the defendant on his land.
• Escape of the hazardous thing from the territory of the defendant.
• There must be a non-natural use of land.
Exceptions:
• Escape of the hazardous goods was because of plaintiffs’ own consent
• Act of god
• Act of a stranger
• Act done by any statutory authority
• Default of the plaintiff

ABSOLUTE LIABILITY
Absolute liability is a stricter form of strict liability. It refers to the no fault theory liability in which the wrongdoer
is held absolutely liable for the act of omission or commission without any defences which are available to the
rule of strict liability. It is applicable only to those people who are involved in hazardous or inherently dangerous
activity whereby they become absolutely liable to full compensation for the harm caused to anyone resulting
from the operation of such hazardous activity. The rule of absolute liability was first laid down in M.C Mehta v.
Union of India (Oleum gas case).

9. POSITION OF MINORS IN LAW OF TORT


In India, a minor is a person who is below the age of 18 years. They can sue just like adults but through their
parents and can also be sued like adults if they are old enough to form an intention to commit a tort.
CAPACITY TO SUE
A minor can sue for any wrong done to him through his ‘litigation friend’ who usually is his father. A minor
may even sue his parents for a negligent act. A child who sustained injury while in the mother’s womb can
also sue the guilty after coming to the world.
CAPACITY TO BE SUED
A minor is generally not capable of being sued if he commits a tort since he is incapable of reimbursing
damages, but in most of the cases he can be sued just like an adult. Also, a minor can be sued for
contributory negligence.
PARENTS LIABILITY FOR A MINORS TORT
Parents could be held liable for the tort committed by their children if they owed a direct duty of care
towards their child while he committed the tort. They are responsible for their children’s action the same way
as the employers are responsible for the harmful action of their employees.

TORT AFFECTING VALUATION


This topic will cover the legal liability of valuers and professional negligence, and areas where negligence cases
may arise and remedies for professional negligence. As this topic is such a broad area for research the project will
be limited as per the conditions and status of Tort studies in India.
In addition the legal principles of professional negligence will be investigated including ‘contract law’, ‘tort of
negligence’, ‘duty of care’ and ‘standard of care’. Past and present legal cases will be used to determine areas
where negligence cases may arise and how the property markets play a part in the alarming number of successful
claims in the law courts to award ever increasing damages to clients/plaintiffs against professionals. As a direct
result of recent legal cases and the subsequent decisions all ‘professional persons’ are increasingly aware of the
threat of ‘a claim for professional negligence’.

Claims made against valuers and other professional persons in the past ten years for professional negligence
have increased in both number and scale.
There are two main reasons for the dramatic increase in professional negligence claims. These reasons are: ·
"Consumerism": Clients are now more aware of their rights and increased media coverage has alerted the public
to making large claims for professional negligence. Although the media may record some of the larger and more
sensational settlements made in recent years, many claims are settled by negotiation with no publicity and are
not recorded.
The public who are potential clients are more educated today and increased media coverage allows details of the
more sensational damage claims to be published widely. In addition legal firms promote their practices in the
media and ‘success based” fee arrangements may be favourably negotiated. Payment of fees for legal cases may
be negotiated on the basis of “no win, no cost ” and these terms allow clients who would never have proceeded
to make a claim. In the valuation industry, ‘the overwhelming majority if these negligence claims have arisen from
valuations made for mortgage purposes’. As the law has evolved a client may choose the most advantageous
legal method to seek "redress" for the negligence suffered as the result of a valuer’s ‘lack of judgement’. It is also
apparent that when the property markets are unstable then a valuer's task is much more difficult.
It is very difficult to forecast the future because there are so many variables that a purchaser must take into
account when deciding to make an investment in property. It is easy to make valuations in a rising property
market and a valuation may appear to be rather conservative as time goes on, but the exact opposite is true in a
falling market. In a falling market the valuer's judgements may be more closely examined and hence more claims
of negligence may arise. It is therefore important to note that the number of negligence claims may rise when
there is a downturn in the market. Investors, mortgagees and other interested parties may not be able to realise
all of their funds on the disposal of their property and they will then cast around looking for some other party to
pay some or all of the equity lost.
Professional persons such as doctors, lawyers and valuers are legally liable for their carelessness that has caused
damage in both contract law and the law of tort. In the present millennium courts are getting tougher and
tougher on valuers and claims for negligence are steadily increasing.
The fact that a valuation figure has proven to be ‘incorrect’ by subsequent events such as a fall in the selling price
of the property may not be sufficient to make a successful claim against the valuer who valued the property.
Usually the valuation will be found to be deficient or ‘flawed’ in some identifiable respect, and that this deficiency
has lead to a foreseeable loss being sustained. When liability is established against a valuer, an assessment of
damages payable by the valuer to the Plaintiff is made. Over the years a body of principles and legal decisions
have developed in assessing the damages payable by a valuer
One of the main areas of law that allows a claim for negligence against a valuer falls within “the Law of
Contract”. The ‘contract for a valuation’ usually arises when a valuer is requested to undertake a valuation and
agrees to do so for a pre-arranged fee. ‘Letters of Instructions” are the primary source of the valuer’s duties,
obligations and liabilities. The terms and conditions of the contract are those that the parties have entered into
and these should be fully articulated in writing and clearly understood by all parties. Any breach of conditions
within that contract will be considered as a ‘breach of contract’ and the breaching party may terminate the
contract and seek damages. The person who has suffered the damage will be entitled to be placed in the same
position as if the contract had been performed properly and the court will assess damages for the loss suffered by
the valuer’s client. Generally, the principle of ‘privity of contract’ ensues that only the parties to the contract can
sue a valuer for ‘breach of contract’ for loss or injury suffered as a result of negligence. However there is a limited
scope for third parties to sue under the law of contract if they are beneficiaries.
Tort has been defined as ‘a Tort Law or the law of wrongs’. ‘Negligence is part of an area of law known as tort law
or the law of wrongs’. It is now an evolving principle that a valuer may be held liable to his client for ‘breach of
duty in negligence’ as well as ‘for breach of duty in contract’.
It is important to focus on the fact that valuers are usually sued for their breach of duty for negligent information
or advice that has caused economic loss. A mortgagee who has advanced funds relying upon the valuation report
often sustains this loss and when the property is sold there is a quantifiable loss for which damages can be
awarded.

LAWS OF ARBRITRATION & CONCILIATION

The Arbitration and Conciliation Act, 1996 improves upon the previous laws regarding arbitration in India namely
the Arbitration Act, 1940, the Arbitration (Protocol and Convention) Act, 1937 and the Foreign Awards
(Recognition and Enforcement) Act, 1961.Further, the new statute also covers conciliation which had not been
provided for earlier.The Act also derives authority from the UNCITRAL (The United Nations Commission on
International Trade Law)Model law on International Commercial Arbitration and the UNCITRAL rules on
conciliation. The Model law on International Commercial Arbitration was framed after taking into consideration
provisions regarding arbitration under various legal systems. Thus, it is possible to incorporate the model law into
the legal system of practically every nation. The Act of 1996 aims at consolidating the law relating to domestic
arbitration, international commercial arbitration, enforcement of foreign arbitral awards and rules regarding
conciliation.
The main objectives of the Act are as follows:
1. To ensure that rules are laid down for international as well as domestic arbitration and conciliation.
2. To ensure that arbitration proceedings are just, fair and effective.
3. To ensure that the arbitral tribunal gives reasons for its award given.
4. To ensure that the arbitral tribunal acts within its jurisdiction.
5. To permit the arbitral tribunal to use methods such as mediation and conciliation during the procedure of
arbitration.
6. To minimise the supervisory role of courts.
7. To ensure that an arbitral award is enforceable as a decree of the court.
8. To ensure that the result of conciliation proceedings may be treated as arbitral awards on agreed terms.
9. To treat awards given in a foreign country to which any one of the two international conventions apply as
followed by India as being a foreign arbitral award.
CONCEPT OF ARBITRATION
The term arbitration is not defined in The Arbitration and Conciliation Act, 1996. It only clarifies that it is not
necessary that to qualify as arbitration, the proceedings must be conducted by any permanent arbitral tribunal.
Hence legal authorities and decisions of courts are important in understanding the meaning and ambit of
arbitration. Arbitration is a private process of settling dispute between the disputing parties but without the
recourse to courts. A private arbitral tribunal is appointed to substitute the processes of a court. The Act defines
the arbitral tribunal as "sole arbitrator" or a "panel of arbitrators".
The decision is known as award and it binds those who were party to the arbitration proceedings. Decision in
Collins vs.Collins LJ Ch 186 and concludes the advantage of arbitration is a means to speedily settle the dispute
on the matters referred to the tribunal in a manner "so that they may not become the subject of future litigation
between the parties".

The 1996 Act adopts several of the sections and clauses suggested in the Model Law issued by the United
Nations Commission on International Trade Law (UNCITRAL). By virtue of sec. 36 of The Arbitration and
Conciliation Act, 1996, the award of an arbitral tribunal is enforceable like a decree of a Court. Unlike in the earlier
Act, there is no further need to present the arbitral award to the court and get it issued as its decree. Interestingly,
“a valuation award does not have the status of a court decree” and that “the court has no power to appoint an
umpire to resolve the difference between two valuers”and when valuation in the absence of umpire is not feasible
– the court may decide the valuation too.

STRUCTURE
The Arbitration and Conciliation Act of 1996, India is divided into four parts namely,
– Arbitration
– Enforcement of Foreign Awards
– Conciliation &
– Supplementary Provisions, which deal separately with the various aspects of both domestic and
international Commercial Arbitration.

PART – I
ARBITRATION
The Appraisal of law relating to Arbitration and Conciliation have been the topic of discussions since its inception.
It is not possible to deal with all the provisions in this Unit. The important sections needing attention are however
being discussed here so as to have an analytical appraisal of the development of law that has taken place in
respect of emergence of the method of alternative disputes resolution by way of Arbitration. This part applies in
all statutory (if not inconsistent with provisions of other enactment or statutory Rules) and/or non-statutory cases
and is the most part for the professional valuers.

Arbitration Agreement – (Section 7)


Arbitration agreement, under this provision, must be in writing and means an agreement by the parties to submit
to arbitration all or certain disputes which have arisen or may arise between them in respect of a defined legal
relationship. The said agreement, if not contained in the specific arbitration clause, may be inferred from
documents signed by the parties, by exchange of letters, telex, telegram or other means of communication and
also from admission of the parties not denying the existence of Arbitration. The arbitration clause has always to
be treated as an agreement distinct and independent of the main agreement.
Power to Refer Parties to Arbitration Where there is an Arbitration Agreement – (Section 8)
If a judicial authority, in a suit or action brought before it, comes to know about the existence of arbitration
agreement with respect to the dispute pending before it, it may refer the parties to arbitration provided the party
requiring arbitration so applies to such judicial authority before submitting his written statement on the
substance of a dispute. In Haryana Telcom Ltd. Vs. Sterlite Ltd. (1999 (5) SCC 688), the Supreme Court
placed an embargo and held that notwithstanding any agreement between the parties, the arbitrator would have
no jurisdiction to order windingup of a company which power vests under the Companies Act with the competent
court and thus a dispute relating to winding-up of a company cannot be referred to arbitration.

Interim Order – (Section/s 9 & 17)


The arbitral tribunal has the power to pass an interim order so as to protect the substance of the dispute during
the pendency of the arbitral proceedings under Section 17. Similarly the court has the power under Section 9 to
pass interim orders before, during and after the arbitral proceedings. In Sundaram Finance Vs. NEPC India Ltd.
(1999 (2) SCC 479), the Supreme Court affirmed this power and held that an interim order can be sought from
the court even before commencement of arbitration proceedings.

Composition of Arbitral Tribunal – (Section 10)


Under this section, the parties are free to determine the number of arbitrators provided that such number shall
not be even numbers. A sole arbitrator can be appointed. A dispute arose in MMTC Ltd. Vs. Sterlite Ltd. (1996
(6) SCC 716), where in the arbitration clause provided for nomination of the arbitrator by each of the parties and
the arbitrators so nominated were required to appoint an umpire. The MMTC Ltd. contended that since the
arbitration clause provided for appointment of even number of arbitrators, such clause was not valid in view of
provisions of Section 10(1). This controversy was resolved by the Supreme Court by stating that the validity of the
arbitration agreement does not depend on the number of arbitrators. The arbitration agreement though
specifying an even number of Arbitrator cannot be a ground to render the arbitration agreement invalid in as
much as the agreement satisfied the requirement of the section 7 of the Act and as such it is a valid agreement.
By nominating an umpire a valid arbitral tribunal of three persons (Odd Number) can be constituted satisfying the
requirement of the section 10 of the Act.

Appointment of Arbitrators – (S. 11)


Arbitrator can be appointed by agreement of parties and in case of failure of either of the parties, when request is
made for appointment, the aggrieved party may approach the Chief Justice of the concerned High Court in case
of Domestic Arbitration and the Chief Justice of India in case of International Commercial Arbitration for
appointment of arbitrator. All High Courts in India have framed a scheme for such purpose. While appointing the
arbitrator, the designated authority shall have due regard to any qualification fixed by the agreement of the
parties for the arbitrator and also to other considerations as are likely to secure the appointment of an
independent and impartial arbitrator as held by the Hon’ble Supreme Court in the case of ICICI Ltd. Vs. East
Coast Boat Building and Engineers (1998 (9) SCC 728). The decision with respect to appointment of arbitrator
by a designated authority is normally final and binding on the parties.

Sections 12, 13, 14 & 15.


The arbitrator has to disclose in writing any circumstances likely to cause justifiable doubts as to his
independence or impartiality and in case of any doubt about his independence or impartiality and of his
qualification, the authority of the arbitrator can be challenged by a party to the arbitration as per procedure laid
down in section 13 and, in case the challenge is unsuccessful, the arbitral tribunal shall continue the proceedings
and make an award. The mandate of an arbitrator shall terminate under Section 14 on his becoming de-jure or
de-facto unable to perform his function, or for his undue delay, or if he withdraws from his office, or the parties
agree to the termination of his mandate. In the event of termination of mandate, a substitute arbitrator may be
appointed according to the rules that were applicable to the appointment of the arbitrator being replaced. The
mandate of an arbitrator is however not terminated by death of any party by whom the arbitrator was appointed
[section 40(2)] and the legal heir/representative will substitute the demised party.
Jurisdiction of the Arbitral Tribunal (s.16)
The arbitral tribunal has power to decide on its own jurisdiction, including any objection with respect to the very
existence and validity of the arbitration agreement, and for that purpose, the arbitration clause which forms part
of the contract as an agreement independent from the other terms of contract. A decision by the tribunal that the
contract is invalid, shall not invalidate the arbitration clause, vide Olympus Superstructure Pvt. Ltd. Vs. Meena
Vijay Khetan (1999 (5) SCC 651).
Conduct of Arbitral Proceedings – (Section/s 18-27)
The parties to the arbitration have to be treated equally and have to be given full opportunity, by following the
principles of natural justice, to present their case (Section 18). The arbitral tribunal shall not be bound by the Code
of Civil Procedure or by Indian Evidence Act. Parties are free to agree on procedure and in case of non-
agreement, the tribunal shall conduct the proceedings in the manner it considers appropriate. Normally the
arbitration proceedings shall be deemed to have commenced on the date on which a request for reference of the
dispute to arbitration has been received by the respondent (Section 21). The arbitrator has power under Section
22 to appoint one or more expert/s to report on specific issue and also seek assistance of the court in taking
evidence.

Making of Arbitral Award and Termination of Proceedings – (S. 28-33)


No time limit is fixed for making the award, though it is always expected under the scheme of the Act that the
tribunal will resolve the dispute in accordance with the substantive law for the time being in force in India. In case
of international Commercial Arbitration, the tribunal shall decide the dispute, and failing any such agreement with
respect to the applicable law, the tribunal shall apply the rules of law which it considers to be appropriate in the
circumstances surrounding the dispute. In the case of tribunal consisting of more than one arbitrator, any
decision of the tribunal shall be made by the majority of the members of the tribunal. A crisis may arise when
there is no consensus between the members of the tribunal and when the members of the tribunal take divergent
views. In such circumstances, the Arbitration will fail. The parties may settle the matter during the arbitral
proceedings either by mediation, conciliation or by other proceedings and in such case the settlement will take
the form of award.
Form and Contents of Arbitral Award – (Section 31)
– The award should be in writing and signed by the members of the tribunal, and unless the parties have
agreed otherwise, it must state reason. This has further been affirmed by the Supreme Court in TNEB Vs.
Bridge Tunnel Construction (1997 (4) SCC 121), stating that it is mandatory for every award to state
reason in support of determination of liability/non-liability of the parties unless the parties have agreed
otherwise.
– The tribunal may at any time during the proceedings make an interim award in any matter with respect to
which it may make a final award.
– In case of an award for payment of money, the tribunal will award interest.
– The award shall state the date and place of arbitration, the cost of arbitration and the party entitled to
cost and the party who will pay the cost and the manner in which the cost shall be paid.

Termination of Proceedings – (Section 32)


The arbitral proceedings shall be terminated by the final award or by an order of the tribunal under Section (2),
i.e. in the circumstances when,
– The claimant withdraws
– The parties agree on the termination of the proceedings or
– When the tribunal finds that the continuation of the proceedings has, for some reasons, become
unnecessary and impossible.

Objection, Appeal, Finality and Enforcement of Award – (s. 34- 37)
Objection to Award (Section 34) The correctness of the arbitral award can be objected to under Section 34 in a
competent court of law within three months from the date of the award with another grace period of 30 days,
with reasonable explanation for not challenging the award within 3 months.
The award can be set aside only if –
1. the party complaining furnishes the proof of the following –
– He (i.e. the party) was under some incapacity (to present the case).
– The arbitration agreement was not valid under applicable law.
– He was not given proper notice of appointment of arbitrator or of arbitral proceedings or was otherwise
unable to present his case.
– The award is beyond scope of reference, vide R.K. Khanna vs. Union of India (1998 (7) SCC 129) and as
such the award deals with a dispute not contemplated by or not falling within the terms of the submission
to the arbitration.
– The composition of the tribunal is not in accordance with the agreement of theparties.
The Court finds that –
– The subject matter of the dispute was not capable of settlement by arbitration under the applicable law, or
– The arbitral award is in conflict with the public policy of India. Appeal (Section 37) An appeal under section
37 shall also lie from the following orders –
o Granting or refusing to grant any interim order under Sec. 9 &17;
o Setting aside or refusing to set aside an award under sec. 34;
o Accepting the plea of bar of jurisdiction under sec. 16; No appeal lies from an order passed under
section 37 except right to appeal to the Supreme Court by the special leave. Where the time for
making an application to set aside the award under sec. 34 has expired or it has been refused the
award shall be enforced in the same manner as if it were the decree of the court.
Limitation (Section 43)
Law of limitation squarely applies to arbitration, and for the purpose of limitation, arbitration shall be deemed to
have commenced on the date the request for reference is received by the other party. Besides, where an
arbitration agreement provides that any claim shall be barred unless some step to commence the arbitration
proceedings is taken within time fixed by the agreement of the parties, it must be commenced within that time,
and if it is not done, the leave of the court has to be obtained for extension of time to lodge the claim.

PART – II
ENFORCEMENT OF FOREIGN AWARDS
A “foreign Award’ means an arbitral award on a dispute of commercial nature and made on or after 28 th July 1924
if under Geneva Convention Award, or on or after October 1960, if made under New York Convention Award.
Enforcement of a foreign award can be refused by the court on somewhat similar grounds as found in Section 34
of the Act, besides that enforcement of an arbitral award may be refused if the court finds that:
– The subject matter of dispute is not capable of settlement by arbitration under Indian law, or
– Enforcement of award would be contrary to the public policy of India, i.e. if the making of award was
induced or affected by fraud or corruption, which may also mean to include an award contrary to the
fundamental principle of Indian law, justice and morality see. Refer: Renu Power Co. Vs. EC (AIR 1994 SC
860).
PART – III
CONCILIATION
The provision of conciliation is given under Sections 61-81 under Part III of the Arbitration and Conciliation Act.
The conciliation can be initiated by the consensus of the parties and once the machinery of conciliation is set into
motion, parties are precluded from initiating any arbitral or judicial proceedings in respect of a dispute which is
subject matter of conciliation proceedings except under Section 9 of the Act for interim protection of the
substance of the dispute. A Conciliator, who may be appointed by the parties, is required to assist the parties, in
an independent and impartial manner, in reaching an amicable settlement of dispute. A conciliator is required to
be guided by the principal of objectivity, fairness and justice, giving consideration to rights and obligations of the
parties, usages of the trade and circumstances surrounding the dispute, including any past business practice
between the parties. Any settlement arrived at pursuant to conciliation has the same status and effect as an
arbitral award.
Another important feature of the conciliation is that the conciliator and the parties are required to maintain
confidentiality in all matters related to conciliation proceedings and its fallout. If the conciliation fails then neither
of the parties may rely or take benefit of
1. Views expressed or suggestions made by the other party in respect of a possible settlement of the dispute,
2. Admission made by the other party in the course of conciliation proceedings,
3. Proposal made by the conciliator, and
4. The fact that the other party had indicated his willingness to accept a proposal for settlement made by the
conciliator.

PART – IV
SUPPLEMENTARY PROVISIONS
Power of High Court to make rules – (Section 82) The High Court may make rules consistent with this Act as to all
proceedings before the Court under this Act. Removal of Difficulties – (Section 83) The Central Government may,
by notification in the Official Gazette, make rules for carrying out the provision of this Act.

AUCTION
Auction: Authority of Auctioneer, Duties of Vendor, Purchaser and Public, Mis-description and Misrepresentation,
Advertisements, Particulars and Catalogues, Statements on the Rostrum, Conduct of Sale, Reservation of Price and
Right to Bid, Bidding Agreements, Memorandum of the Sale. The Deposit, Rights of Auctioneer against Vendor
and Purchaser.
FEW IMPORTANT DEFINITIONS
1. An auction is a process of buying and selling goods or services by offering them up for bid, taking bids, and
then selling the item to the highest bidder. The open ascending price auction is arguably the most common
form of auction in use today. Participants bid openly against one another, with each subsequent bid required
to be higher than the previous bid.
2. An auctioneer may announce prices, bidders may call out their bids themselves (or have a proxy call out a
bid on their behalf), or bids may be submitted electronically with the highest current bid publicly displayed.
3. Bid – An offer to buy at a certain price. A buyer’s premium is often added to a winning high bid to determine
the final contract price.
4. Absentee bid – A bid placed with the auctioneer or an auctioneer’s assistant in advance of the auction.
Usually offered at the auctioneer’s discretion to allow a bidder to participate without being present. An
alternative to online bidding.
5. Absolute auction – An auction where the property is sold to the highest qualified bidder without the seller’s
opportunity to reject the bid. An absolute auction is established by the auctioneer at the time the bidding is
opened. Sometimes called an auction without reserve.
6. Backup Bidder – The bidder with the second highest bid. In those rare instances where the high bidder fails
to complete the transaction (forfeiting payment), the backup bidder may have an opportunity to make a
purchase.
7. Bid Assistant (Ringman) – An auction staff member stationed in the audience to assist bidders and the
auctioneer as they communicate throughout the auction.
8. Bid calling – The verbal process used by the auctioneer to conduct the auction. Using a unique chant the
auctioneer acknowledges bids and asks for increments in the bidding.
9. Chant – The rapid talking of the auctioneer made up of words and sounds, used to acknowledge bids, ask
for increments, and communicate with Bid Assistants.
10. Closing – The time at which final funds are exchanged and the property changes hands. Usually handled by a
title company as prescribed in the Purchase Contract and occurring several weeks following the auction. The
closing process is essentially the same for an auction sale as for any other traditional real estate transaction.
11. Escrow – A legal arrangement in which the title company holds the deposit while preparing documents and
making arrangements for the closing.
12. Lot – One or more items offered for bidding. A lot could be a piece of real estate, and item in an estate, or a
group of items being sold together.
13. Minimum bid – Often used interchangeably with “reserve” and referring to the minimum bid which will be
acceptable to the seller. Not usually disclosed. Bidding may begin below the minimum bid and the seller may
accept a high bid that falls below his or her established minimum.
14. Onsite Auction – An auction in which bidders can participate live at a prescribed date, time, and location.
Often an onsite auction may also allow for simultaneous live online bidding.
15. Onsite bidder – During a live auction, the bidders who have chosen to come to the auction location to
participate in bidding.
16. Reserve, Seller’s Reserve, or Reserve Auction – The reserve is the minimum bid a seller has stated that he
or she will accept at the auction. Usually not disclosed. If the high bid falls above the reserve, the auctioneer
has the authority to declare the property sold. If the high bid falls below the reserve, the seller reserves the
right to accept or reject it.
17. Terms & Conditions – Rules of the auction that govern how the property is to be offered, how the bidding
will be handled, and how the ultimate purchase will take place. Bidders must read and agree to the terms &
conditions of the auction in order to register and receive a bidder’s number for participating in the auction.

AUTHORITY OF AUCTIONEER
The first and very obvious question comes in everyone's mind that What authority does the auctioneer have to
sign the sale contract on behalf of both the purchaser and the vendor and where does that authority come from?
Further from that: Who can sign a contract for the sale and purchase of land on behalf of the purchaser at an
auction?
The answer to the above said query is that the Auctioneer has complete authority to sign the Contract for the Sale
and Purchase of Land on behalf of both the vendor and the purchaser. This authority comes from common law
rather than through legislation and is sighted in the case Phillips v Butler, a 1945 English case, where a
successful bidder declined to sign a contract for the sale of land at the conclusion of an auction. The judge ruled
that there was an irrevocable authority to sign the contract, due to evidence proving that both parties had
finalised a contract and specific performance of the contract in this case was required.
An auctioneer serves as the agent of the seller who employs him or her, and the auctioneer must act in good
faith, advance the interest of the seller, and conduct the sale in accordance with the seller's instructions. Both the
buyer and the seller are bound by the announcement of the auctioneer concerning the identity of the property
and the terms and conditions of the sale.
In the absence of a statutory provision requiring authority to be in writing, an agent, pursuant to oral
authorization, can execute any contract required to be in writing.
Because of the trust and confidence the seller reposes in an auctioneer, the individual cannot delegate the power
to sell without special authority from the seller. The delegation of insignificant duties, such as the striking of the
hammer and the announcement of the sale, is allowable if conducted pursuant to the auctioneer's immediate
supervision and direction.
An auctioneer's authority normally terminates upon the completion of the sale and the collection of the purchase
price, but the seller can revoke the authority at any time prior to the sale. According to some authorities, the
buyer or seller can end the auctioneer's authority to sign a memorandum on his or her behalf between the time
of the fall of the hammer and the signing of the memorandum, but the prevailing view deems the auctioneer's
authority to be irrevocable. Private sales by an auctioneer are generally impermissible.
A public auction is an auction held on behalf of a government in which the property to be auctioned is either
property owned by the government, or property which is sold under the authority of a court of law or a
government agency with similar authority.
When the term "government auction" is used it generally means that specific auctioneers and agents are
contracted to deal with stock that needs to be liquidated by various government bodies.
Private property may be sold in a public auction for a number of reasons. It may be seized through a
governmental process to satisfy a judgment rendered by a court or agency, or to liquidate a mortgage
foreclosure, tax lien, or tax sale. Usually, prices obtained at a public auction to satisfy a judgment are distressed -
that is, they are much lower than the price which would be obtained for that property if the seller were free to
hold out for an optimal time to sell.

DUTIES OF SELLER, PURCHASER AND AUCTIONEER


SELLER
It is the duty of the seller to deliver the goods and the buyer to accept and pay for them, in accordance with
terms of contract of sale.
Unless otherwise agreed delivery of the goods and payment of price are concurrent conditions; that is to say that
seller shall be ready and willing to give possession of the goods to the buyer in exchange of the price.
The seller of goods has a duty of giving delivery according to the terms of the contract and according to rules as
specified.

PURCHASER
It is the duty of the purchaser to accept the goods and pay for them in accordance with the terms of the contract.
If the contract specifically provides for the delivery of the goods by the seller by instalments, the buyer shall
accept such a delivery.
If the buyer refuses to accept the goods, it is his duty to inform the seller about it.
If the seller delivers the goods as per the contract, it becomes the duty of the buyer to take delivery of the same
within a reasonable time. He remains liable to the seller for any loss arising on account of his refusal to take
delivery.
If the ownership rights have already been passed on to the buyer by the seller, the former has the duty to pay the
price as per the terms of the contract.
If the buyer wrongfully refuses to accept and pay for the goods, he will have to compensate the seller for
damages for non-acceptance.

Duty of Auctioneer
It is the duty of every auctioneer to state fully the terms and conditions upon which the sale will be made. Further,
an auctioneer shall announce the character, quality and description of the property offered for sale to the persons
present. The duty of an auctioneer generally includes, but is not limited to, the notification as to whether or not a
right to bid is reserved by or on behalf of the seller.
An auctioneer, by selling property for another at an auction, is the agent of the seller. In the absence of an
applicable statute to the contrary, the auctioneer’s rights and liabilities are governed by the general principles of
the law of agency.
As a general rule, an auctioneer owes a basic duty of competence and fairness to the seller. An auctioneer, as the
agent of the seller, is in a fiduciary position and has a duty to turn over proceeds of the auction sale in full, such
proceeds being tantamount to trust funds. An auctioneer must exercise ordinary care and skill in the performance
of the duties confided to him or her. If an auctioneer assumes a position that is entirely inconsistent with that of
his or her agency relationship, the auctioneer may lose his or her right to compensation for his/her services and
may be held accountable to the principal for any side profit received by him or her as a result of the sale which
the auctioneer did not disclose to the principal.

Misdescription And Misrepresentation


Misdescription refers to an inaccurate physical or legal description, often fraudulent of property, in a contract for
sale. Such a description usually misleads the other party. When a seller cannot transmit property which
corresponds to the description in the contract, a breach of contract occurs. Consequently, the buyer gets a right
to damages.
When there is a substantial misdescription, the seller will be unable to enforce the contract against the buyer.
However, when there is an innocent misdescription, which is not substantial, the contract may be enforced by the
seller. However, such contracts may be subject to a suitable reduction in the contractually agreed price.
Misrepresentation
An untrue statement of fact or law made by Party A (or its agent) to Party B, which induces Party B to enter a
contract with Party A thereby causing Party B loss. An action for misrepresentation can be brought in respect of a
misrepresentation of fact or law.
There are three types of misrepresentation:
Fraudulent misrepresentation: where a false representation has been made knowingly, or without belief in its
truth, or recklessly as to its truth.
Negligent misrepresentation: a representation made carelessly and in breach of duty owed by Party A to Party
B to take reasonable care that the representation is accurate. If no "special relationship" exists, there may be a
misrepresentation where a statement is made carelessly or without reasonable grounds for believing its truth.
Innocent misrepresentation: a representation that is neither fraudulent nor negligent.
The remedies for misrepresentation are rescission and/or damages. For fraudulent and negligent
misrepresentation, the claimant may claim rescission and damages. For innocent misrepresentation, the court has
a discretion to award damages in lieu of rescission; the court cannot award both.

CONDUCT AND VALIDITY OF SALE


The owner of the property has the right to control the sale until its conclusion. Unless conditions are imposed by
the seller, the auctioneer is free to conduct the sale in any manner chosen, in order to bar fraudulent bidders and
to earn the confidence of honest purchasers. The auctioneer cannot amend the printed terms and conditions of
the auction, but he or she is empowered to postpone the sale, if that is the desire. The auctioneer can modify the
sale terms of goods advertised in a catalog at any time during the sale, if announced publicly and all of the
bidders present are cognizant of it. The auctioneer may also retain the right to resell if there be an error or a
dispute concerning the sale property. The description of the property in the catalog must be unambiguous. A
significant error in a description might cause the cancellation of the sale, although trivial discrepancies between
the property and the description are not problematic. The seller can withdraw property until the acceptance of a
bid by an auctioneer.
A bid is an offer to purchase, and no obligations are imposed upon the seller until the bid is accepted. It can be
made in any manner that demonstrates the bidder's willingness to pay a particular price for the auctioned
property, whether orally, in writing, or through bodily movements, such as a wave of the hand. Secret signals
between the bidder and the auctioneer militate against equality in bidding and are thereby prohibited. The
auctioneer accepts a bid by the fall of the hammer or by any other perceptible method that advises the bidder
that the property is his or hers upon tendering the amount of the bid in accordance with the terms of the sale. An
auctioneer can reject a bid on various grounds, such as when it is combined with terms or conditions other than
those of the sale, or is below the minimum price acceptable to the owner.
As a general rule, any act of the auctioneer, seller, or buyer that prevents an impartial, free, and open sale or that
reduces competition in the bidding is contrary to public policy. An agreement among prospective buyers not to
bid has been held to void the sale to any buyer within this group. A purchase by a person who has not
participated in the illegal agreement remains in effect. A puffer or shill is a person who has no intention of buying
but is hired by the seller to place fictitious bids in order to raise the bidding of genuine purchasers. In general, if a
purchaser at an auction can prove that a puffer was employed, he or she can void the sale. Some jurisdictions
require the buyer to have been financially hurt by the puffer, but others permit an individual to void a sale even if
no harm occurred. Puffing and by-bidding are synonymous.
A deposit is not a pledge but a partial payment of the purchase price, usually made payable to the auctioneer
who retains it until the completion of the sale.
The property of one person should not be commingled and sold with the property of another by the auctioneer
unless notice is furnished to all interested parties, or it might constitute fraud.
An auctioneer who does not have the required license but who executes a sale can bePenalized, but the sale
remains valid; an auction is void, however, when it is conducted without the owner's consent.

STATEMENTS ON THE ROSTRUM


A raised platform on which the auctioneer may stand to conduct the auction.
An auction's appeal is frequently attributed to the thrilling experience and spectacle of the event. Auction houses
call on the services of a wide variety of support staff to make the event successful, from transportation experts
who make sure delicate products make it to the block unscathed to catalog production teams who photograph
and market the goods. Auction houses employ industry specialists to research and determine the authenticity of
goods, and keep these people on hand to counsel buyers and sellers on auction day.
The most important figure supplied by the auction house is the auctioneer. A skilled auctioneer can play a large
role in an auction's success through his or her ability to relate to the bidders. By perpetuating the myths of the
market, buyers are encouraged to follow their dreams, and the audience participates more readily in the bidding.
Much like the conductor of a symphony, an auctioneer must balance the excitement of performance with
technical control. Commanding the rostrum entails managing the emotions of bidders, setting the proper pace so
that prices increase quickly, yet not so fast that control is compromised. Some say that one either has the talents
of a good auctioneer or one doesn't, but this is not deterring would-be auctioneers from applying to auctioning
schools.
An auctioneer standing on the rostrum commands great duties and responsibilities towards the purchaser, seller
and also to public.

BIDDING AGREEMENT
Bidding Agreement is an agreement by which any two businesses who want to submit a tender for a contract can
do so. It is also called teaming agreement.This bidding agreement is designed for two businesses that want to co-
operate in tendering for a contract. If successful, one of them will be the lead contractor and the other will be a
subcontractor.
Main issues in these types of agreements
Tender submission. Both parties need to agree and sign off on the tender document. This means exchanging
pricing information.
Tender negotiations. The bidding agreement will specify which of them leads the negotiations and ensure that
the other is kept informed of negotiations and can participate when appropriate.
Tender costs. Usually each party is responsible for its own costs. But sometimes on a large tender, another
organisation will be introduced to provide support and, in that case, the costs might be shared.
Sub contract terms. Bidding agreement provides for one of the parties to be a subcontractor to the other if the
tender is successful. In consequence, the bidding agreement will set out the key terms of the sub contract so that
argument at the time the tender is awarded can be avoided. Alternatively, there may be a joint venture
agreement signed if the bid is successful.
Confidentiality is clearly important because sensitive commercial information is likely to be exchanged in the bid
process and if this is not successful, both parties want to protect that confidential information and data.
The bidding agreement generally includes the clauses on co-operation, submission of tender & pre-contract
negotiations, tender preparation costs, subcontract, further agreement, duration, confidentiality, obligations and
restrictions, assignment, governing law and disputes.

MEMORANDUM OF SALE
A memorandum of sale is a document recording the prospective buyer’s interest in purchasing the property and
the seller’s terms of the sale. It relates to the ‘sale agreed’ step when purchasing a property. While it’s not a
legally binding contract, it acts as written confirmation that a price has been agreed and declares the intention of
both parties to complete the sale. The document will include any special conditions or remarks that deviate from
the standard conditions of sale, including planning restrictions and specified rights concerning the land. The
memorandum of sale should contain:
Names and contact details of both the property seller and buyer
Details of the counsel/ mediator/ agent acting for each party
Property address, tenure, details of lease and special conditions
The agreed price for the property including proposed deposit amount/ token money
Any other details/ conditions.
Importance of memorandum of sale
Signing a memorandum of sale is generally seen as the first step of the buying/selling process. While it is not a
legally binding document, it acts as an informal agreement to the final contract and plays an essential part in
surveying and investigating the property. Most property sales are agreed over the phone and therefore a
memorandum of sale provides written evidence from the vendor that an offer has been accepted.

LAWS OF EVIDENCE: BURDEN OF PROOF,


PRESUMPTIONS, CONCLUSIVE PROOF
PRINCIPLES OF EVIDENCE
Facts: means anything or state of things or relations of thing which can be perceived by senses (see, touch, taste,
hear, and smell). Perception of another's 'state of mind' is also a fact – psychologicalfact.)
All facts other than electronic evidence can be proved by oral evidence (as per sec 59 of Evidence Act as amended
by the Information Technology Act 2000). Electronic records have to be proved by 65A and 65B.
In India evidence may be given to prove or disprove either
• Facts in issue and/ or
• Relevant facts
A fact in issue means something about the state or relation of thing(s) that can be sensed (perceived by any of
the five senses of a human being i.e. a Physical fact. As per Sec 3 of the Act it refers to those facts which can
establish right, duty, liabilities or obligations.
As per the Report of the Law Commission of India,in civil cases, facts in issue are decided in the manner provided
under order 14of the Code of Civil Procedure and in criminal cases, the charge constitutes and includes facts in
issue (chapter XVII of the Criminal Procedure Code, 1973)but for this course, understanding applicability of Civil
or Criminal Procedure Code is not required.
Relevant facts - A fact is said to be relevant to another when the one is connected with the other in any of the
ways referred to in the provisions of this Act. In other words, one fact is
relevant to another fact, if it is connected in any way as described in sections 6 to 55.
Section 5 states that 'evidence' may be given of 'facts in issue' and 'relevant facts' and the Act also defines 'facts
in issue'." All admissible evidence is relevant but all relevant evidence is not admissible on grounds of limitation
and several other considerations.
"facts of which evidence, being not admissible, cannot be given though they are 'relevant'" - such as the facts
referred to in sections 121 (magistrate/ judge is protected except to the extent of legally valid supervision of
superior), 122 (divulging communication during marriage without consent of the other), 123 (unpublished official
records related to the affairs of State), 126 (communication with advocate but does not protect discussion for
illegal object) etc. This is described as protection of privileged communication.
An important principle is Evidence is weighed for its merit and not counted (Sec.134). That is number of evidences
is not relevant. Hence section 134 does not specify any minimum number of witnesses to prove a fact.
As per the judgment of Ganesh K GulveVs. State of MaharastraCriminal Appeal No. 501 of 1999, the court held
that "In order to appreciate evidence, Court is required to bear in mind the set up and environment in which the
crime is committed, the level of understanding of witnesses, the over jealousness of some of near relations to
ensure that everyone even remotely connected with the crime be also convicted and that everyone has a different
way of narration of same facts. Then the evidence is required to be appreciated to find out what part out of the
evidence represents the true and correct state of affairs since it is for courts to separate grain from chaff."

Some more principles:


• Oral evidence that is not direct is challenged by the hearsay rule - unless the hearsay falls within the
exceptions allowed in the Act, it cannot be admitted.
• Oral evidence cannot generally prove the contents of documents (signature on document is often
proved by oral evidence and sec 6SB provides special provisions for electronic documents)
• A judgment/ order or the order sheet of Court is a case laws is accepted as public documents and do
not need to be proved - Benefit of doubt is available to accused in criminal cases. In his defense an
accused in a criminal case has only to create doubt in theminds of the court on the validity of the
evidence against him.
In criminal cases, accused is always presumed to be innocent till proven guilty beyond reasonable
doubt. But the prosecution has to provide evidence that is beyond the realms of doubt.

THE BURDEN OF PROOF AND ITS ADMISSIBILITY


Section 101 in The Indian Evidence Act, 1872 states "Whoever desires any Court to give judgment as to any legal
right or liability dependent on the existence of facts which he asserts, must prove that those facts exist. When a
person is bound to prove the existence of any fact, it is said that the burden of proof lies on that person."
Supreme Court held in Anil Rishi vs. Gurbaksh Singh Civil Appeal 2413 of 2006 "In view of Section 101 of the
Evidence Act, the initial burden of proof would be on the plaintiff.
The trial court and the High Court proceeded on the basis that the defendant was in a dominant position. The
fact that the defendant was in a dominant position must be proved by the plaintiff at the first instance.
In this case Anil Rishi case (supra) Court went on to hold "It should be borne in mind that a distinction exists
between burden of proof and onus of proof. Onus "assumes importance in the early stage of a case. The question
of onus of proof has greater force, where the question is which party is to begin.

Burden of proof is used in three ways:


1. to indicate the duty of bringing forward evidence in support of a proposition at the beginning or later,
2. To make that of establishing a proposition as against all counter evidence; and
3. An indiscriminate use in which it may mean either or both of the others. The elementary rule in Section 101 is
inflexible. In terms of Section 102 the initial onus is always on the plaintiff and if he discharges that onus and
makes out a case which entitled him to a relief the onus shifts to the defendant to prove those
circumstances, if any, which would disentitle the plaintiff to the same. The order reframing the issue is set
aside thus reviving the issue originally framed." The obligation to prove varies with the situation as described
below:
4. It is the duty of accused to prove his plea of alibi - that is he was in another place at the time of commission
of offence - The burden of proving mala fide is on the person who alleges it.
5. The burden of collusion lies on the person who alleges it.
6. When the facts prima facie proves negligence, the burden lies on the defendant to prove that in fact he was
not negligent
7. In suit for damages the burden lies on the plaintiff.
8. When the defendant pleads that suit of plaintiff is time-barred, the burden lies on the plaintiff to prove the
facts which would bring the suit within time.
9. The person who makes an allegation should prove it (Sec. 67)

Sometimes when an application is filed beyond the limitation period stipulated in any Act evidence must be led
that the filing is validly done and explain the delays. Then evidence on date of receipt of order is a valid
consideration. In Benarsi Krishna Committee &Ors. Vs. Karmyogi Shelters Pvt. Ltd arising from SLP.23860 of
2010 Supreme Court held that "since a signed copy of the Award had not been delivered to the party itself and
the party obtained the same on 15.12.2004, and the petition u/s 34 of the Act was filed on 3.2.2005, it has to be
held that the said petition was filed within the stipulated period of three months as contemplated u/s 34(3) of the
Act"
Sometimes liability to prove is statutorily reversed - death of a married woman within seven year of marriage
reverses the liability and requires accused to prove innocence.
Furnishing of evidence at the argument stage is automatically not bad. Supreme Court held in
DamaraVenkataMurali Krishna Rao vsGurujupalliSatvathamma (Civil Appeal arising out of SLP (C) No.18128 of
2006, Expert opinion was sought for comparison of signature of a witness at the stage of arguments in a suit for
recovery of money. This was rejected by trial court and confirmed by High Court on the ground that application
was filed to protract the litigation. Supreme Court held the fact of signature became relevant only when the said
witness was examined and evidence was sought to be presented on the very next date. Remanded the case to the
trial court to consider the matter on merit.
A Judge has to decide as to admissibility of evidence as per Sec. 136 - A judge may ask the party proposing to
give the evidence in what manner the alleged fact, if proved, would be relevant and accordingly decide on the
admissibility.

Some important rules on oral evidence :


1. No leading questions can be asked in Examination-in-Chief unless court has previously allowed it specifically.
It cannot be asked in re-examination under any situation.
2. A leading question is one which prompt respondent for the desired answer is built in the question itself. Re-
examination is the process of recalling you own witness for a further questioning.
3. As per section 149 questions are not to be asked in cross examination without reasonable grounds.
4. If Court opines that question is put by a counsel without reasonable ground, the court may report the matter
to High Court.
In civil cases, any person can be compelled to be a witness but in criminal cases, the accused cannot be
compelled to be a witness against himself. It is different matter that in India through custodial interrogation in
criminal cases self-implicatory (self-harming) admissions are extracted that is then substantiated by evidence the
accused himself may have helped to locate.
Self-harming statement in civil case can be called as "admission" and they are called as "confession" in criminal
case. But for confession in criminal case to be admissible it has to be validly made including the presence of
magistrate and warning that it will be used against him. In civil cases, admissions alone are relevant but not
confessions. But, in criminal cases, admissions as well as confessions are relevant.
Union of India vs. Moksh Builders and Financiers Ltd. and others 1977 (1) SCR 967and in Vathsala Manic
Kavasagam&Ors Vs. N Ganesan &Anr.1241 of 2005 "An admission, as defined in S.17 of the Evidence Act, 1872
constitutes a substantial piece of evidence and can be relied upon for proving the veracity of the facts
incorporated therein. Once the admission as noted in a statement, either oral or documentary, is found, then the
whole onus would shift to the party who made such an admission and it will become an imperative duty on such
party to explain it. In the absence of any satisfactory explanation, it will have to be presumed to be true."
The contents of a document may be proved either by primary or secondary evidence: Sec 64.
Document must be proved by primary evidence except in exceptional case provided for in that in Sec 65/ 65B:
1. But merelyfiling a document does not make it a part of the recordsof a case. The crucial test is whether a
request was made to court to take judicial note of them and taken on record.
2. The person who makes an allegation should prove it. Sec.67
3. Execution consists in signing the document written out, read over and understood the contents of
documents. Sec. 68.
4. Evidence cannot be given to remove the ambiguity of the language of the document or of the statute.
Sebastiao Luis Fernandes (Dead) through LRs &Ors. vs. K.v.P. Shastri (Dead) through LRs &Ors. 6183 of
2001 Sec.101 and 102 -It was upon plaintiff-appellants to furnish proof regarding ownership of one third share of
suit property and discharge their burden of proof as per ss. 101 and 102 - before they could invite the court to
address itself to the issue of their challenge to title of defendants- respondents to suit property - and claim
consequential relief of cancellation of registration in favour of defendants-respondents in respect of 1/3 share in
suit scheduled property.
... The plaintiff-appellants have not produced any document of title in relation to suit property, grant of decree in
their favour is erroneous in law. Therefore, High Court was justified in recording cogent and valid reasons to annul
the concurrent findings of courts below and in holding that non-appreciation of pleadings and evidence on
record by courts below rendered their finding on the contentious issues/points as perverse and arbitrary.
Presumption and conclusive proof:
Sec 4 of the evidence Act:when one fact is declared by the evidence act to be conclusive proof of another, the
court shall on proof of one presume the other proved and shall not allow evidence on it.
Presumption: -
Presumption may be defined "to be an inference, affirmative or dis-affirmative of the truth or falsehood of a
doubtful fact or proposition drawn by a process of probable reasoning from something proved or taken for
granted and this view has been taken in the judgement of Izhar Ahmad Khan v Union of India AIR 1962 SC
1060]
There are two kind of presumptions: one is rebuttable and another isirrebuttable presumption. Rebuttable
presumption can be overthrown by evidence to the contrary. But, irrebuttable presumption is conclusive or
irrebuttable.
1. Whenever it is provided by the Evidence Act that the court may presume a fact, it may either regard that fact
as proved, unless it is disproved, or may call for proof of it. The discretion is that of the Court.
2. But if the statute directs the court shall presume a fact, it shall regard that fact as proved, unless it is
disproved.
3. When one fact is declared by this Act to be conclusive proof of another, the court shall, on proof of the one
fact, regard the other as proved, and shall not allow evidence to be given for the purpose of disproving it.
• If a child of below seven year is directed by statute to be held as not responsible for his action. His age
proved, mental capacity cannot be proved.
• When law says a girl below 16 years consent to sexual intercourse is not consent that can be proved to
reduce liability, age of the girl being proved, the fact of absence of consent will be presumed to be
proved.
M.B. Ramesh (D) By LRS Versus K.M. Veeraje (D) By LRS. &Ors 1071 of 2006 and relying on JankiNarayanBhoir vs.
Narayan NamdeoKadam 2003 (2) SCC 91: 2002
1. "A Will, has to be executed in the manner required by s.63 of the Succession Act, 1925. Section 68 of the
Evidence Act requires the Will to be proved by examining at least one attesting witness." When as
contemplated in Section 71 of the Evidence Act the attesting witness denies or does not recollect the
execution of the document, its execution may be proved by other evidence then the Court held "Section 71 is
meant to lend assistance and come to the rescue of a party who had done his best, but would otherwise be
let down if other means of proving due execution by other evidence are not permitted. At the same time, the
section cannot be read to absolve a party of his obligation under Section 68 of the Evidence Act read with
Section 63 of the Succession Act to present in evidence a witness, though alive and available."
2. Non-possession of a document does not reduce duty to call opposite party to produce it when proving
forgery. Supreme Court held in Anil Rishi vs. Gurbaksh Singh Civil Appeal 2413 of 2006 decided on
02/05/2006) "With a view to prove forgery or fabrication in a document, possession of the original sale deed
by the defendant would not change the legal position. A party in possessionof a document can always be
directed to produce the same. The plaintiff could file an application calling for the said document from the
defendant and the defendant could have been directed by the Trial Judge to produce the same.
Conclusive proof: -
About Conclusive Proof (Aiyar, 2001) quotes Halsbury Laws of England 4th Ed "no contrary evidence shall be
effective to displace it unless the socalled conclusive evidence is inaccurate on its face, or fraud can be shown."
Some examples of Conclusive proof as per section of the Act are:
• Sec 41 states a final judgment, order or decree of a competent Court, in the exercise of its power for (i)
Probate of will, (ii) Matrimonial status declaration, (iii) Insolvency jurisdiction are conclusive proof
absolutely (i.e., in rem) against all.
• Evidence that a child is born during wedlock is sufficient to establish its legitimacy, and shifts the
burden of proof to the party seeking to establish the contrary. It is undesirable to enquire into the
paternity of a child whose parents have access to each other (as per sec. 112).
• If a judgment, decree or order declares that it had been or should be his property then this is when the
judgment/ decree when the decree is duly executed, this assertion becomes available.
SALIENT FEATURES OF THE INSOLVENCY
&BANKRUPTCY CODE, 2016
The Insolvency and Bankruptcy Code passed by the Parliament is a welcome overhaul of the existing framework
dealing with insolvency of corporates, individuals, partnerships and other entities. It paves the way for much
needed reforms while focusing on creditor driven insolvency resolution.
BACKGROUND
At present, there are multiple overlapping laws and adjudicating forums dealing with financial failure and
insolvency of companies and individuals in India. The current legal and institutional framework does not aid
lenders in effective and timely recovery or restructuring of defaulted assets and causes undue strain on the Indian
credit system. Recognizing that reforms in the bankruptcy and insolvency regime are critical for improving the
business environment and alleviating distressed credit markets, the Government introduced the Insolvency and
Bankruptcy Code Bill in November 2015, drafted by a specially constituted 'Bankruptcy Law Reforms Committee'
(BLRC) under the Ministry of Finance. Trilegal worked with the BLRC to assist with the drafting of the bill.
After a public consultation process and recommendations from a joint committee of Parliament, both houses of
Parliament have now passed the Insolvency and Bankruptcy Code, 2016 (Code). While the legislation of the Code
is a historical development for economic reforms in India, its effect will be seen in due course when the
institutional infrastructure and implementing rules as envisaged under the Code are formed.
THE CODE
The Code offers a uniform, comprehensive insolvency legislation encompassing all companies, partnerships and
individuals (other than financial firms). The Government is proposing a separate framework for bankruptcy
resolution in failing banks and financial sector entities.
One of the fundamental features of the Code is that it allows creditors to assess the viability of a debtor as a
business decision, and agree upon a plan for its revival or a speedy liquidation. The Code creates a new
institutional framework, consisting of a regulator, insolvency professionals, information utilities and adjudicatory
mechanisms, that will facilitate a formal and time bound insolvency resolution process and liquidation.

KEY HIGHLIGHTS
Corporate Debtors: Two-Stage Process
To initiate an insolvency process for corporate debtors, the default should be at least INR 100,000 (USD 1495)
(which limit may be increased up to INR 10,000,000 (USD 149,500) by the Government). The Code proposes two
independent stages:
Insolvency Resolution Process, during which financial creditors assess whether the debtor's business is viable to
continue and the options for its rescue and revival; and
Liquidation, if the insolvency resolution process fails or financial creditors decide to wind down and distribute
the assets of the debtor.
The Insolvency Resolution Process (IRP)
The IRP provides a collective mechanism to lenders to deal with the overall distressed position of a corporate
debtor. This is a significant departure from the existing legal framework under which the primary onus to initiate a
reorganization process lies with the debtor, and lenders may pursue distinct actions for recovery, security
enforcement and debt restructuring.
The Code envisages the following steps in the IRP:
1. Commencement of the IRP-A financial creditor (for a defaulted financial debt) or an operational creditor
(for an unpaid operational debt) can initiate an IRP against a corporate debtor at the National Company
Law Tribunal (NCLT).The defaulting corporate debtor, its shareholders or employees, may also initiate
voluntary insolvency proceedings.
2. Moratorium-The NCLT orders a moratorium on the debtor's operations for the period of the IRP. This
operates as a 'calm period' during which no judicial proceedings for recovery, enforcement of security
interest, sale or transfer of assets, or termination of essential contracts can take place against the debtor.
3. Appointment of Resolution Professional-The NCLT appoints an insolvency professional or 'Resolution
Professional' to administer the IRP. The Resolution Professional's primary function is to take over the
management of the corporate borrower and operate its business as a going concern under the broad
directions of a committee of creditors. This is similar to the approach under the UK insolvency laws, but
distinct from the "debtor in possession" approach under Chapter 11 of the US bankruptcy code. Under the
US bankruptcy code, the debtor's management retains control while the bankruptcy professional only
oversees the business in order to prevent asset stripping on the part of the promoters.
Therefore, the thrust of the Code is to allow a shift of control from the defaulting debtor's management to
its creditors, where the creditors drive the business of the debtor with the Resolution Professional acting
as their agent.
4. Creditors Committee and Revival Plan
The Resolution Professional identifies the financial creditors and constitutes a creditors committee.
Operational creditors above a certain threshold are allowed to attend meetings of the committee but do
not have voting power. Each decision of the creditors committee requires a 75% majority vote. Decisions
of the creditors committee are binding on the corporate debtor and all its creditors. The creditors
committee considers proposals for the revival of the debtor and must decide whether to proceed with a
revival plan or liquidation within a period of 180 days (subject to a one-time extension by 90 days).
Anyone can submit a revival proposal, but it must necessarily provide for payment of operational debts to
the extent of the liquidation waterfall.
5. The Code does not elaborate on the types of revival plans that may be adopted, which may include fresh
finance, sale of assets, haircuts, change of management etc.

Liquidation
Under the Code, a corporate debtor may be put into liquidation in the following scenarios:
• A 75% majority of the creditor's committee resolves to liquidate the corporate debtor at any time
during the insolvency resolution process;
• The creditor's committee does not approve a resolution plan within 180 days (or within the extended
90 days);
• The NCLT rejects the resolution plan submitted to it on technical grounds; or
• The debtor contravenes the agreed resolution plan and an affected person makes an application to
the NCLT to liquidate the corporate debtor.
• Once the NCLT passes an order of liquidation, a moratorium is imposed on the pending legal
proceedings against the corporate debtor, and the assets of the debtor (including the proceeds of
liquidation) vest in the liquidation estate.

Priority of Claims
The Code significantly changes the priority waterfall for distribution of liquidation proceeds.
After the costs of insolvency resolution (including any interim finance), secured debt together with workmen dues
for the preceding 24 months rank highest in priority. Central and state Government dues stand below the claims
of secured creditors, workmen dues, employee dues and other unsecured financial creditors. Under the earlier
regime, Government dues were immediately below the claims of secured creditors and workmen in order of
priority.
Upon liquidation, a secured creditor may choose to realize his security and receive proceeds from the sale of the
secured assets in first priority. If the secured creditor enforces his claims outside the liquidation, he must
contribute any excess proceeds to the liquidation trust. Further, in case of any shortfall in recovery, the secured
creditors will be junior to the unsecured creditors to the extent of the shortfall.
INSOLVENCY RESOLUTION PROCESS FOR NDIVIDUALS/UNLIMITED
PARTNERSHIPS
For individuals and unlimited partnerships, the Code applies in all cases where the minimum default amount is
INR 1000 (USD 15) and above (the Government may later revise the minimum amount of default to a higher
threshold). The Code envisages two distinct processes in case of insolvencies: automatic fresh start and insolvency
resolution.
Under the automatic fresh start process, eligible debtors (basis gross income) can apply to the Debt Recovery
Tribunal (DRT) for discharge from certain debts not exceeding a specified threshold, allowing them to start afresh.
The insolvency resolution process consists of preparation of a repayment plan by the debtor, for approval of
creditors. If approved, the DRT passes an order binding the debtor and creditors to the repayment plan. If the
plan is rejected or fails, the debtor or creditors may apply for a bankruptcy order.
INSTITUTIONAL INFRASTRUCTURE
The Insolvency Regulator
The Code provides for the constitution of a new insolvency regulator i.e., the Insolvency and Bankruptcy Board of
India (Board). Its role includes: (i) overseeing the functioning of insolvency intermediaries i.e., insolvency
professionals, insolvency professional agencies and information utilities; and (ii) regulating the insolvency process.

Insolvency Resolution Professionals


The Code provides for insolvency professionals as intermediaries who would play a key role in the efficient
working of the bankruptcy process. The Code contemplates insolvency professionals as a class of regulated but
private professionals having minimum standards of professional and ethical conduct.
In the resolution process, the insolvency professional verifies the claims of the creditors, constitutes a creditors
committee, runs the debtor's business during the moratorium period and helps the creditors in reaching a
consensus for a revival plan. In liquidation, the insolvency professional acts as a liquidator and bankruptcy trustee.

Information Utilities
A notable feature of the Code is the creation of information utilities to collect, collate, authenticate and
disseminate financial information of debtors in centralized electronic databases. The Code requires creditors to
provide financial information of debtors to multiple utilities on an ongoing basis. Such information would be
available to creditors, resolution professionals, liquidators and other stakeholders in insolvency and bankruptcy
proceedings. The purpose of this is to remove information asymmetry and dependency on the debtor's
management for critical information that is needed to swiftly resolve insolvency.
Adjudicatory authorities
The adjudicating authority for corporate insolvency and liquidation is the NCLT. Appeals from NCLT orders lie to
the National Company Law Appellate Tribunal and thereafter to the Supreme Court of India. For individuals and
other persons, the adjudicating authority is the DRT, appeals lie to the Debt Recovery Appellate Tribunal and
thereafter to the Supreme Court.
In keeping with the broad philosophy that insolvency resolution must be commercially and professionally driven
(rather than court driven), the role of adjudicating authorities is limited to ensuring due process rather than
adjudicating on the merits of the insolvency resolution.
SALIENT FEATURES OF THE COMPANIES
(REGISTERED VALUERS AND VALUATION) RULES,
2017
With above background, the Companies Act, 2013 (Act) had brought the concept of Registered Valuers to
regulate the practice of Valuation in India and to standardize the valuation in line with International standards.
However, the valuer’s qualification, experience, manner and process were left to be decided by the Rules.
After about 4 years, the Ministry of Corporate Affairs (MCA) has now issued the Companies (Registered Valuers
and Valuation) Rules, 2017 (Rules) on 18th October, 2017. Simultaneously, section 247 of the Act has now come
into force w.e.f. 18th October 2017.

These rules contain various aspects pertaining to Registered Valuers including:


1. Who can become Valuer (including Qualification, experience and clearance of Valuation examination) for
each Asset Class and the process involved;
2. Eligibility and Role of Registered Valuers Organization (RVO) for conducting educational courses, granting
membership, conducting training, laying code of conduct, monitoring the functioning of valuers and
addressing grievances including conducting disciplinary proceedings against valuers who are its members;
3. The Valuation Standards required to be adhered to while performing and reporting;
4. Contents of the Valuation Report including permissible caveats and limitations;
5. Professional competence and Due Care and Independence of Valuer;
6. Maintenance of record of each assignment for minimum 3 years and
7. Regulation of the profession including Model code of conduct for Registered Valuers.

The Registered Valuer under Companies Act, 2013 shall specifically do valuation of asset classes as required under
the Act. However, it is also clarified under the rules that the Registered Valuer provisions shall not automatically
become applicable to valuation required under any other law (like RBI, Income Tax, SEBI etc).
Accordingly, as of now, the Registered Valuer provisions shall cover the following Acts/Regulations - Companies
Act, 2013, Insolvency Code, 2016 and SEBI (REIT and InvIT) Regulations, 2016. However, in times to come it is
expected that other Regulators would also align their Act/Rules with these Registered Valuer provisions.
Though the Valuation Standards would also be framed shortly however it’s high time that Professionals interested
in practicing in this field to come forward and gain academic and practical knowledge of Business valuation
principles and concepts, valuation approaches and methodologies in general and also as prescribed under
various statutes like Income Tax, RBI and SEBI for different purposes, valuation case laws, ICAI business valuation
standard, ICAI Technical guide on Share Valuation, erstwhile CCI guidelines, guidance from Revenue Ruling 1959-
60 (USA) and international valuation standards, 2017 issued by International valuation standards council.
It is emphasize that though these rules have opened the doors of this profession to a wider set of professionals including
Graduate in Civil Engineering, Architecture or Town Planning for Land and Building, Mechanical/Electrical engineers for
Plant & Machinery (besides other organizations as may be prescribed) and a member of a Professional Institute including
CA, CS and CMA for Financial Assets, however it must be remembered that it is a very onerous task and has come with lot
of responsibilities as it now stands Regulated by the government.
However, our understanding of Valuation says that though we can make the valuation process more codified and scientific
however Valuation will always involve judgement based on experience of the valuers and would still be based upon the
“assumptions” and “limiting conditions” based on the “data” available on a “valuation date”. Accordingly, what the
Registered Valuers law is now telling us is essentially to learn, assess, analyze, document and disclose with transparency.
SALIENT FEATURES OF THE SARFAESI ACT, 2002
CONCERNING VALUATION
Valuation of and sale of immovable secured assets are referred under Rule 8 of Security Interest (Enforcement)
Rules, 2002 as under:
Before effecting sale of the immovable property referred to in sub-rule (1) of rule 9, the authorized officer shall
obtain valuation of the property from an approved valuerand in consultation with the secured creditor, fix the
reserve price of the property and may sell the whole or any part of such immovable secured asset by any of the
following methods:
1. By obtaining quotations from the persons dealing with similar secured assets or otherwise interested in
buying such assets; or
2. By inviting tenders from the public; or
3. By holding public auction; or
4. By private treaty.
Rule 2(d) of Security Interest (Enforcement) Rules, 2002 defines "Approved Valuer" which means a valuer
approved by the Board of Directors or Board of Trustees of the secured creditor, as the case may be.
In case of immovable property valuation by an approved valuer and fixing of reserve price is mandatory.

SECTION 5(N) OF THE BANKING REGULATION ACT,


1949
With this in view, in India the central government as early as 1949 made an attempt to define a "secured loan or
advance" under section 5(n) of the Banking Regulation Act, 1949. This is defined as-
"aloan or advance made on the security of the assets the market value of which is not at any time less than the
amount of such loan or advance".
The important term from a valuation point of view now is to find out what the term market value means. This is
defined as under as per International Valuation Standards (IVS).
"Market Valueis the estimated amount for which an asset should exchange on the date of valuation between a
willing buyer and a willing seller in an arm's length transaction after proper marketing wherein the parties had
acted knowledgeably, prudently and without compulsion"

THE COMPANIES ACT 2013: S. 192(2), 230 (1,2,3), 231,


232, 247(1), 281(1)
SECTION 192 RESTRICTION ON NON- CASH TRANSACTIONS INVOLVING
DIRECTORS
1. No company shall enter into an arrangement by which—
• a director of the company or its holding, subsidiary or associate company ora person connected with
him acquires or is to acquire assets for consideration other than cash, from the company; or
• The company acquires or is to acquire assets for consideration other than cash, from such director or
person so connected,unless prior approval for such arrangement is accorded by a resolution of the
company in general meeting and if the director or connected person is a director of its holding
company,approval under this sub-section shall also be required to be obtained by passing a
resolutionin general meeting of the holding company.
2. The notice for approval of the resolution by the company or holding company in general meeting under sub-
section (1) shall include the particulars of the arrangement along with the value of the assets involved in
such arrangement duly calculated by a registeredvaluer.
3. Any arrangement entered into by a company or its holding company in contraventionof the provisions of this
section shall be voidable at the instance of the company unless—
• The restitution of any money or other consideration which is the subjectmatterof the arrangement is
no longer possible and the company has been indemnified by any other person for any loss or
damage caused to it; or
• Any rights are acquired bona fide for value and without notice of thecontravention of the provisions of
this section by any other person.

SECTION 230(1,2,3) POWER TO COMPROMISE OR MAKE ARRANGEMENTS WITH


CREDITORSAND MEMBERS
1. Where a compromise or arrangement is proposed—
• Between a company and its creditors or any class of them; or
• Between a company and its members or any class of them, the Tribunal may, on the application of the
company or of any creditor or member of the company, or in the case of a company which is being
wound up, of the liquidator, order a meeting of the creditors or class of creditors, or of the members
or class of members, as the case may be, to be called, held and conducted in such manner as the
Tribunal directs.
Explanation.—For the purposes of this sub-section, arrangement includes areorganisation of the
company’s share capital by the consolidation of shares of differentclasses or by the division of shares into
shares of different classes, or by both of thosemethods.

2. The company or any other person, by whom an application is made under sub-section(1), shall disclose to the
Tribunal by affidavit—
• all material facts relating to the company, such as the latest financial positionof the company, the
latest auditor’s report on the accounts of the company and thependency of any investigation or
proceedings against the company;
• Reduction of share capital of the company, if any, included in the compromiseor arrangement;
• Any scheme of corporate debt restructuring consented to by not less thanseventy- five per cent of the
secured creditors in value, including—
o A creditor’s responsibility statement in the prescribed form;
o Safeguards for the protection of other secured and unsecured creditors;
o Report by the auditor that the fund requirements of the company afterthe corporate debt
restructuring as approved shall conform to the liquidity testbased upon the estimates provided to
them by the Board;
o Where the company proposes to adopt the corporate debt restructuringguidelines specified by the
Reserve Bank of India, a statement to that effect; and
o A valuation report in respect of the shares and the property and all assets,tangible and intangible,
movable and immovable, of the company by a registeredvaluer.
3. Where a meeting is proposed to be called in pursuance of an order of the Tribunalunder sub-section (1), a
notice of such meeting shall be sent to all the creditors or class ofcreditors and to all the members or class of
members and the debenture-holders of thecompany, individually at the address registered with the company
which shall beaccompanied by a statement disclosing the details of the compromise or arrangement, acopy
of the valuation report, if any, and explaining their effect on creditors, key managerialpersonnel, promoters
and non-promoter members, and the debenture-holders and theeffect of the compromise or arrangement
on any material interests of the directors of thecompany or the debenture trustees, and such other matters as
may be prescribed:
Provided that such notice and other documents shall also be placed on the website of thecompany, if any,
and in case of a listed company, these documents shall be sent to the Securitiesand Exchange Board and
stock exchange where the securities of the companies are listed, forplacing on their website and shall also be
published in newspapers in such manner as may beprescribed:
Provided further that where the notice for the meeting is also issued by way of anadvertisement, it shall
indicate the time within which copies of the compromise or arrangementshall be made available to the
concerned persons free of charge from the registered office of thecompany.

SECTION 231POWER OF TRIBUNAL TO ENFORCE COMPROMISE OR ARRANGEMENT


1. Where the Tribunal makes an order under section 230 sanctioning a compromise or an arrangement in
respect of a company, it—
– Shall have power to supervise the implementation of the compromise or arrangement; and
– May, at the time of making such order or at any time thereafter, give suchdirections in regard to any
matter or make such modifications in the compromise orarrangement as it may consider necessary for the
proper implementation of thecompromise or arrangement.
2. If the Tribunal is satisfied that the compromise or arrangement sanctioned undersection 230 cannot be
implemented satisfactorily with or without modifications, and thecompany is unable to pay its debts as per
the scheme, it may make an order for winding upthe company and such an order shall be deemed to be an
order made under section 273.
3. The provisions of this section shall, so far as may be, also apply to a company in respect of which an order
has been made before the commencement of this Act sanctioninga compromise or an arrangement.

SECTION 232 MERGER AND AMALGAMATION OF COMPANIES


1. Where an application is made to the Tribunal under section 230 for the sanctioning of a compromise or an
arrangement proposed between a company and any such persons as are mentioned in that section, and it is
shown to the Tribunal—
• that the compromise or arrangement has been proposed for the purposes of, or in connection with, a
scheme for the reconstruction of the company or companies involving merger or the amalgamation of
any two or more companies; and
• That under the scheme, the whole or any part of the undertaking, property or liabilities of any
company (hereinafter referred to as the transferor company) is required to be transferred to another
company (hereinafter referred to as the transferee company), or is proposed to be divided among and
transferred to two or more companies,the Tribunal may on such application, order a meeting of the
creditors or class of creditors or the members or class of members, as the case may be, to be called,
held and conducted in such manner as the Tribunal may direct and the provisions of sub-sections (3)
to (6) of section 230 shall apply mutatis mutandis.
2. Where an order has been made by the Tribunal under sub-section (1), merging companies or the companies
in respect of which a division is proposed, shall also be required to circulate the following for the meeting so
ordered by the Tribunal, namely:—
• The draft of the proposed terms of the scheme drawn up and adopted by the directors of the merging
company;
• Confirmation that a copy of the draft scheme has been filed with the Registrar;
• A report adopted by the directors of the merging companies explainingeffect of compromise on each
class of shareholders, key managerial personnel, promoters and non-promoter shareholders laying out
in particular the share exchange ratio, specifying any special valuation difficulties;
• The report of the expert with regard to valuation, if any;
• A supplementary accounting statement if the last annual accounts of any of the merging company
relate to a financial year ending more than six months before the first meeting of the company
summoned for the purposes of approving the scheme.
3. The Tribunal, after satisfying itself that the procedure specified in sub-sections(1) and (2) has been complied
with, may, by order, sanction the compromise or arrangement or by a subsequent order, make provision for
the following matters, namely:
• The transfer to the transferee company of the whole or any part of the undertaking, property or
liabilities of the transferor company from a date to be determined by the parties unless the Tribunal,
for reasons to be recorded by it in writing, decides otherwise;
• The allotment or appropriation by the transferee company of any shares, debentures, policies or other
like instruments in the company which, under the compromise or arrangement, are to be allotted or
appropriated by that company to or for any person:
• Provided that a transferee company shall not, as a result of the compromise orarrangement, hold any
shares in its own name or in the name of any trust whether on its behalf or on behalf of any of its
subsidiary or associate companies and any such shares shall be cancelled or extinguished;
• The continuation by or against the transferee company of any legal proceedings pending by or against
any transferor company on the date of transfer;
• Dissolution, without winding-up, of any transferor company;
• The provision to be made for any persons who, within such time and in such manner as the Tribunal
directs, dissent from the compromise or arrangement;
• Where share capital is held by any non-resident shareholder under the foreign direct investment
norms or guidelines specified by the Central Government or in accordance with any law for the time
being in force, the allotment of shares of the transferee company to such shareholder shall be in the
manner specified in the order;
• The transfer of the employees of the transferor company to the transferee company;
• Where the transferor company is a listed company and the transferee company is an unlisted
company: —
o The transferee company shall remain an unlisted company until it becomes a listed company;
o If shareholders of the transferor company decide to opt out of the transferee company, provision shall
be made for payment of the value of shares held by them and other benefits in accordance with a pre-
determined price formula or after a valuation is made, and the arrangements under this provision may
be made by the Tribunal:
Provided that the amount of payment or valuation under this clause for any share shall not be less than what
has been specified by the Securities and Exchange Board under any regulations framed by it;
• Where the transferor company is dissolved, the fee, if any, paid by the transferor company on its
authorized capital shall be set-off against any fees payable by the transferee company on its
authorized capital subsequent to the amalgamation; and
• Such incidental, consequential and supplemental matters as are deemednecessary to secure that the
merger or amalgamation is fully and effectively carriedout:Provided that no compromise or
arrangement shall be sanctioned by the Tribunal unless a certificate by the company’s auditor has
been filed with the Tribunal to the effect that the accounting treatment, if any, proposed in the scheme
of compromise or arrangement is in conformity with the accounting standards prescribed under
section 133.
4. Where an order under this section provides for the transfer of any property orliabilities, then, by virtue of the
order, that property shall be transferred to the transferee company and the liabilities shall be transferred to
and become the liabilities of the transferee company and any property may, if the order so directs, be freed
from any charge which shall by virtue of the compromise or arrangement, cease to have effect.
5. Every company in relation to which the order is made shall cause a certified copy of the order to be filed with
the Registrar for registration within thirty days of the receipt of certified copy of the order.
6. The scheme under this section shall clearly indicate an appointed date from which it shall be effective and the
scheme shall be deemed to be effective from such date and not at a date subsequent to the appointed date.
7. Every company in relation to which the order is made shall, until the completionof the scheme, file a
statement in such form and within such time as may be prescribed with the Registrar every year duly certified
by a chartered accountant or a cost accountant or acompany secretary in practice indicating whether the
scheme is being complied with in accordance with the orders of the Tribunal or not.
8. If a transferor company or a transferee company contravenes the provisions ofthis section, the transferor
company or the transferee company, as the case may be, shall be punishable with fine which shall not be less
than one lakh rupees but which may extend to twenty-five lakh rupees and every officer of such transferor or
transferee company whois in default, shall be punishable with imprisonment for a term which may extend to
one year or with fine which shall not be less than one lakh rupees but which may extend to three lakh rupees,
or with both.
Explanation.—For the purposes of this section,—
• In a scheme involving a merger, where under the scheme the undertaking,property and liabilities of
one or more companies, including the company in respect of which the compromise or arrangement is
proposed, are to be transferred to another existing company, it is a merger by absorption, or where
the undertaking, property and liabilities of two or more companies, including the company in respect
of which the compromise or arrangement is proposed, are to be transferred to a new company,
whether or not a public company, it is a merger by formation of a new company;
• References to merging companies are in relation to a merger by absorption, to the transferor and
transferee companies, and, in relation to a merger by formation of a new company, to the transferor
companies;
• A scheme involves a division, where under the scheme the undertaking, property and liabilities of the
company in respect of which the compromise orarrangement is proposed are to be divided among
and transferred to two or more companies each of which is either an existing company or a new
company; and
• Property includes assets, rights and interests of every description and liabilities include debts and
obligations of every description.

SECTION 247COMPANIES ACT, 2013,VALUATION BY REGISTERED VALUERS


1. Where a valuation is required to be made in respect of any property, stocks, shares, debentures, securities or
goodwill or any other assets (herein referred to as theassets) or net worth of a company or its liabilities under
the provision of this Act, it shall be valued by a person having such qualifications and experience and
registered as a valuer in such manner, on such terms and conditions as may be prescribed and appointed by
the audit committee or in its absence by the Board of Directors of that company.
2. The valuer appointed under sub-section (1) shall, —
• Make an impartial, true and fair valuation of any assets which may be required to be valued;
• Exercise due diligence while performing the functions as valuer;
• Make the valuation in accordance with such rules as may be prescribed; and
• Not undertake valuation of any assets in which he has a direct or indirect interest or becomes so
interested at any time during or after the valuation of assets.
3. If a valuer contravenes the provisions of this section or the rules made thereunder, the valuer shall be
punishable with fine which shall not be less than twenty-five thousand rupees but which may extend to one
lakh rupees:
Provided that if the valuer has contravened such provisions with the intention to defraud the company or its
members, he shall be punishable with imprisonment for a term which may extend to one year and with fine
which shall not be less than one lakh rupees but which may extend to five lakh rupees.
4. Where a valuer has been convicted under sub-section (3), he shall be liable to—
• Refund the remuneration received by him to the company; and
• Pay for damages to the company or to any other person for loss arising out of incorrect or misleading
statements of particulars made in his report.
SECTION 281(1)SUBMISSION OF REPORT BY COMPANY LIQUIDATOR
1. Where the Tribunal has made a winding up order or appointed a CompanyLiquidator, such liquidator shall,
within sixty days from the order, submit to the Tribunal, areport containing the following particulars, namely:

• The nature and details of the assets of the company including their location and value, stating
separately the cash balance in hand and in the bank, if any, and the negotiable securities, if any, held
by the company:
• Provided that the valuation of the assets shall be obtained from registered valuersfor this purpose;
• Amount of capital issued, subscribed and paid-up;
• The existing and contingent liabilities of the company including names,addresses and occupations of
its creditors, stating separately the amount of secured and unsecured debts, and in the case of secured
debts, particulars of the securities given, whether by the company or an officer thereof, their value and
the dates on which they were given;
• The debts due to the company and the names, addresses and occupations of the persons from whom
they are due and the amount likely to be realised on account thereof;
• Guarantees, if any, extended by the company;
• List of contributories and dues, if any, payable by them and details of any unpaid call;
• Details of trademarks and intellectual properties, if any, owned by the company;
• Details of subsisting contracts, joint ventures and collaborations, if any;
• Details of holding and subsidiary companies, if any;
• Details of legal cases filed by or against the company; and
• Any other information which the Tribunal may direct or the Company Liquidator may consider
necessary to include.
MULTIPLE CHOICE QUESTIONS
1) The Constitution of India came into force on:
a) 26 January 1950
b) 26 November 1949
c) 15 August 1947
d) 01 January 1950

2) Which constitution is the longest written constitution of any independent country in the world
a) Constitution of USA
b) Constitution of India
c) Constitution of Britain
d) Constitution of Canada

3) The original copy of the Constitution of India was handwritten and calligraphed in
a) Sanskrit
b) English
c) Hindi
d) Both English and Hindi

4) The original copies of the Constitution of India are kept in special helium-filled cases in
a) the personal custody of President of India
b) the Library of Supreme Court of India
c) the Library of the Parliament of India
d) None of the above

5) The beginning word ‘WE’ in the Preamble refers to the


a) Citizens of India
b) People of India
c) All adult people in India
d) All people above 25 years

6) The important test to identify the basic features of the Indian Constitution is
a) Directive Principles of State Policy
b) Preamble
c) Fundamental rights
d) Fundamental duties

7) India is a Sovereign Socialist Secular Democratic Republic. In the Indian Constitution, this expression occurs in
a) Preamble
b) Union & its Territory
c) Fundamental Rights
d) Directive Principles of State Policy

8) Two words added into Preamble by the 42nd Amendment are


a) Socialist, Democratic
b) Sovereign, Socialist
c) Sovereign, Secular
d) Socialist, Secular
9) The Preamble was amended by the
a) 43rd Amendment 1978
b) 44th Amendment 1978
c) 42nd Amendment 1976
d) 41st Amendment 1976

10) The Constitution provides


a) Powers
b) Responsibilities
c) Limitations
d) All the above

11) The Indian State is regarded as federal because the Indian Constitution provides for
a) Strong Central Government
b) Sharing of power between Centre and States
c) Residue Powers provide to Central Government
d) Head of the State is elected for a fixed period

12) The final interpreter to Indian Constitution is


a) President
b) Supreme Court
c) Parliament
d) Supreme Courts

13) Which one of the following States has a separate Constitution?


a) Sikkim
b) Arunachal Pradesh
c) Jammu & Kashmir
d) Maharashtra

14) Indian Constitution is called ‘Quasi-Federal’ because it has


a) Union of states
b) Single Citizenship
c) Single Judiciary
d) Residue Power to Central Government

15) The Constitution confers a special authority for the enforcement of Fundamental Rights on the
a) Parliament
b) Supreme Court
c) President
d) Election Commission

16) By which fundamental right other fundamental rights is protected?


a) Right to Constitutional Remedies
b) Right to live
c) Right to freedom
d) Right to equality

17) Which of the following has ceased to be a fundamental right in the Indian Constitution?
a) Right to live
b) Right to Constitutional remedies
c) Right to Property
d) Right to education for the children upto 14 years

18) Which is described as the ‘Heart and Soul of the Constitution’?


a) Preamble
b) Directive Principles of State Policy
c) Right to Constitutional Remedies
d) Right to freedom

19) The Right to Constitutional Remedies are


a) Fundamental Duties
b) Directive Principles of State Policy
c) Legal Rights
d) Fundamental Rights

20) Who is the Protector and Guarantor of the Fundamental Rights?


a) Supreme Court
b) President
c) Parliament
d) Prime Minister

21) In India sovereignty lies with


a) The Constitution
b) The Supreme Court
c) The Parliament
d) The People

22) Which one of the following subjects comes under the common jurisdiction of the Supreme court and the High
court?
a) Mutual disputes among states
b) Dispute between centre and state
c) Protection of the fundamental rights
d) Protection from the violation of the constitution

23) Who is head of Judiciary in the State?


a) Governor
b) Chief Minister
c) High Court
d) Supreme Court

24) The protection and improvement of environment including forests and wild life of the country is
a) Directive Principle of State Policy
b) Fundamental National Policy
c) Fundamental Duty of a Citizen
d) Both Directive Principles of State Policy and Fundamental Duty of a Citizen

25) The Indian Contract Act, applies to the:


a) Whole of India excluding Jammu & Kashmir
b) Whole of India including Jammu & Kashmir
c) States notified by the Central Government from time to time
d) None of the above

26) Every promise and every set of promise forming the consideration for each other is a/an
a) contract.
b) agreement.
c) offer.
d) Acceptance

27) Valid contracts


a) are made by free consent.
b) are made by competent parties.
c) have lawful consideration and lawful object.
d) all of the above.

28) Drawing cash from ATM, sale by fall of hammer at an auction sale, etc., are example of
a) express contract.
b) implied contract.
c) tacit contract.
d) unlawful contract

29) All Contract is a/an ……………


a) Offer
b) Agreement
c) Acceptance
d) Transaction

30) Every agreement and promise enforceable by law is …………….


a) Offer
b) Contract
c) Acceptance
d) Consideration

31) . ……………… is made by words written.


a) Express contract
b) Implied contract
c) Tacit contract
d) Unlawful contract

32) A contract which ceases to be enforceable by law becomes void when it ceases to be enforceable is called
a) Valid contract
b) A voidable contract
c) A void contract
d) None of above

33) An agreement which is enforceable by law at the option of one or more of the parties there to but not at the
option of other or others is
a) Valid contract
b) A voidable contract
c) A void contract
d) None of above
34) Undue influence makes a contract
a) Void
b) Voidable
c) Valid
d) None of above

35) The term "Fraud" means.


a) Active concealment of fact with knowledge and belief of the fact
b) Doing any act fitted to deceive
c) Both (a) and (b)
d) None of above

36) The suggestion as a fact of that which is not true by one who does not believe it to be true and the active
concealment of a fact by one having knowledge or believe of the fact is called
a) Cheating
b) Fraud
c) Misrepresentation
d) None of above

37) Mistake of fact


a) Makes a contract voidable
b) Does not make a contract voidable
c) Makes a contract void
d) None of above

38) The consideration or object of an agreement is lawful unless it is


a) Forbidden by law
b) Forbidden by contractors themselves
c) Forbidden by custom
d) None of above

39) Where both the parties are under mistake as to matter of fact the agreement is
a) Void
b) Voidable
c) Valid
d) None of above

40) Agreement without consideration is


a) Void
b) Voidable
c) Valid
d) None of above

41) A contract to do or not to do something if some event collateral to such contract does or does not happen is
called
a) Simple contract
b) Complex contract
c) Contingent contract
d) None of above
42) The term "Contingent" means.
a) Possible but not assured
b) Doubtful or uncertain
c) Both (a) and (b)
d) None of above
43) Indemnity" means
a) Security from damage or loss
b) Security for more profit
c) An act for protection
d) Both (a) and (b)

44) The term "Bailment" means


a) A delivery of a thing entrusted for some special purpose or object upon a contract
b) Delivery of goods free of cost
c) Delivery of goods without cost for welfare of public
d) None of above

45) An arbitrator:
a) is chosen and paid by the disputant
b) acts in accordance with privately chosen procedure so far as that is not repugnant to public policy
c) only (a) is correct
d) both (a) and (b) are correct.

46) An arbitral award:


a) must be connected with the subject-matter of the dispute arbitrated
b) must be founded on principle of trust
c) both (a) and (b) are correct
d) only (a) is correct

47) An arbitral award


a) has to be in writing but need not be signed
b) has to be in writing and signed by the members of the arbitral tribunal
c) may be oral
d) Either (a) or (b) or (c).
e)
48) An arbitral award
a) must state the reasons upon which it is based
b) must state the reasons upon which it is based only when the parties have agreed for the same
c) need not state the reason upon which it is based
d) may state or may not state the reasons upon which it is based as per discretion of the members of the
arbitral tribunal.

49) After the arbitral award is made, each party shall be delivered
a) the original award
b) a signed copy of the award
c) a photocopy of the award
d) an unsigned copy of the award.

50) An arbitral award shall be enforced in the same manner as if it were a decree of
a) local authority
b) the court
c) the tribunal
d) both (b) and (c).

51) Which of the following in the correct statement?


a) an arbitral award can be inferred with if it is contrary to the substantive provisions of the Act or against
the terms of the contract
b) an arbitral award can be set aside if the arbitral tribunal has not followed the mandatory procedure
prescribed under the Act
c) an arbitral award can be set aside if it is contrary to fundamental policy of Indian law, or the interest of
India, or justice or morality
d) all of the above.

52) Which one of the following is incorrect statement?


a) an arbitral award is a contract
b) an arbitral award must be in writing and signed.
c) an arbitral award includes an interim award.
d) none of the above.
INTRODUCTION TO STATISTICS
Statistics is mainly a branch of science or a branch of applied mathematics which uses certain techniques
and methods to make use of the best result from the data given.
Definition: Statistics is the science of collection, classification, analysis of data and get the result from the
analysis and interpreting, presenting the result.

DATA CLASSIFICATION AND PROCESSING


Classifications of data are mainly four types. They are
1. Quantitative classification
2. Qualitative classification
3. Chronological classification
4. Geographical classification

Quantitative classification: The classification based on quantitative characteristics is known as


quantitative classification. For example the data of students in a school is classified based on their height,
weight etc.

Qualitative classification: The classification based on qualitative characteristics in nature that is attributes
is known as qualitative classification. For example the data of students in a school is classified according to
their gender, religion, blood group etc.

Geographical classification: The classification of data based on geographical area or location is known as
geographical classification. For example the literacy rate is classified among different states, the
production of paddy or wheat among different nations etc

Chronological classification: The classification of data based on time period ( ex: year, month, days, etc)
is known as chronological classification. For example the data of population in different years, production
of paddy or wheat is classified according to different years etc.

DATA PROCESSING
This involve several steps
Validation – the process of checking the provided data is correct and relevant.
Sorting – it is the arrangement of data as different sequences or sets
Summarization – The process to reduce the complexity of the data
Aggregation – combing different data in a meaningful way
Classification – data are arranged into various categories according to different characteristics
Analysis – The process to get the useful information out of the collected data
Reporting – It is the step in which the results are reported in a standard manner.

PRESENTATION OF DATA
The classified data can be presented in two ways
1. Tabular presentation
2. Diagrammatical or Graphical presentation
Tabular presentation: The classified data or the statistical results can be presented in rows and columns
are called tabular presentation
Diagrammatical presentation: Diagrams are classified in different types. These are one dimensional
diagram, two dimensional diagram, three dimensional diagram, etc.

One dimensional diagram:


The diagram which is based on only one direction that is either height or width is called one dimensional
diagram
Examples are
1. Simple bar diagram
The simple bars with usually rectangular bars which are equidistant and the simple bar’s height
represent magnitudes or frequencies are known as simple bar diagram
Ex: Selling of ice cream scoops in different months in a shop

Selling of Ice creams (Scoops)


70
60
50
40
Selling of Ice
30 creams (Scoops)
20
10
0
May June July Aug Sept Nov

2. Multiple bar diagram


The interlinked data of various sets is expressed by multiple bar diagram. There will be adjacent bars
drawn and each with different shades or colors.
Ex: production of different agricultural products among different nations

Production of agricultural products


45
40
35
30
25 Rice
20 Wheat
15 Barley
10
5
0
India US China UK

3. Component bar diagram


A simple bar is drawn to represent the total magnitude and then the bar into components. The
different components are uses different shades or colors.
Ex: sales in different years as imports and exports

Sales in different years


140

120

100

80
Exports
60 Imports
40

20

0
2000 2005 2010 2015

4. Percentage bar diagram


Percentage bar diagram is another type of component bar diagram. Here instead of actual total value
we consider the total percentage as 100%. Then this 100% is divided as components.
Ex: sales in different years as imports and exports

Sales in different years


100%
90%
80%
70%
60%
50% Export
40% Import
30%
20%
10%
0%
2000 2005 2010 2015

TWO DIMENSIONAL DIAGRAMS:


The diagram which is based on two directions that is both the height and width are taken to express the
data.
Pie diagram:
A diagrammatic representation of circle is divided as sectors to represent the data is called pie diagram.
The data in the sectors are represents either the percentage or the angles. The total angle 360 degree is
divided into different sectors.
The angle for each sector is calculated based on the following equation
Individual Vlue
Sector angle = Total value
x 3600
Ex: state budget expenditure allocation among different fields

Budget allocations
7.3
8.7
Road
Health
43.7
Education
22.7
Water
17.5 Sanitation

GRAPHICAL REPRESENTATION
Graphs are used in statistics is mainly for the representation of the distribution of frequencies. The
important graphical representations are
1. Histogram: Continuous data in a class of intervals represented graphically. Where in the quantitative
data is divided into sub intervals are represented along x axis and their corresponding frequency
along y axis. Adjacent bars are drawn to represent the frequency.
2. Frequency Polygon: A histogram bars midpoints are joined with a straight line to get a frequency
polygon. Here we get a polygon shaped graphical representation which has many sides.
3. Frequency curve: It is obtained from a frequency polygon. A free hand smooth curve is drawn along
the lines joined by the bars midpoint in the histogram to obtain a frequency curve.
4. Ex: - The following shows distribution of number of students accordingly the scored marks in an
examination by them. It is shown by histogram, frequency polygon and frequency curve.

FREQUENCY DISTRIBUTION
The count or how many times a data point repeated is called frequency. The systematic representation of
frequencies for the values taken for a variable is termed as the frequency distribution.
The frequency distributions are two types.
– Discrete frequency distribution
– Continuous frequency distribution
The systematic representation of frequencies for the discrete values of the random variable termed as the
discrete frequency distribution. The systematic representation of frequencies taken for continuous random
variable class intervals then the distribution is called as a continuous frequency distribution

UNIVARIATE DATA ANALYSIS


CENTRAL TENDENCY
When we having a number of observations (data), this data show a natural tendency to concentrate or
tend towards a certain point called centre of the data (like centre of gravity). The central value is the
measure which represents the entire data. The central tendency values are,

Arithmetic mean:
It is a very common average and it is used most widely in many situations. It is calculated by taking the
entire sum of the data divided by total number of units in the set of data. When we are calculating
arithmetic mean we find the sum of all the values in the data. Hence arithmetic mean (AM) average
calculated based on giving equal priority or importance to every units of the data.
The main disadvantages of arithmetic mean is when there are extreme values the AM value is extremely
affected. The extreme value has high influence on the arithmetic mean.
The arithmetic mean is calculated based on the following equation
̅=sum of individual observations
X
total number of observations
∑X
̅
X=
n
Ex: - Calculate AM for the following marks of a student obtain in 6 subjects
75, 80, 62, 93, 88, 90
75+80+62+93+88+90
AM= 6
= 488/6
= 81.33

Weighted arithmetic mean:


The simple AM is calculated following the assumption that all the values have equal importance in the
data. But in some cases this is not applicable. There are cases in which some observations are having
relative importance compared to other observations. In those cases we are giving more weightage to the
relative important observation and calculating the average. This is called weighted arithmetic mean.

∑ W1 X1 + W2 X2 + ⋯ Wn Xn
̅
X=
W1 + W2 + ⋯ Wn
∑ WX
̅
X=
∑W
Median:
It is an important average which lies exactly the centre or middle most if the data observation is ordered or
arranged in descending or ascending order. The denotation of median is M. Thus the median observation
divides the data into two equal portions or parts. That is fifty percentages of the observation lies above
and below the value of median.
Unlike mean, the median is not computed based on values in the data. The extreme values are not
influencing the median value.
Median is calculated based on the following method
Step 1: Data is arranged in descending order or ascending order
𝑛+1 th
Step 2: Median, M is the ( 2
) observation, for odd number of observations
For even number of observations there will be two middle values, then those two middle values average is
taken as median.
Ex: - (1) Calculate median for the set of observations given below
21, 19, 15, 32, 45, 11, 8
Step 1: 8, 11, 15, 19, 21, 32, 45
Step 2= (7+1)/2 = 4th observation
Median = 32
(2) Calculate median for the set of observations given below
55, 58, 49, 66, 75, 80, 60, 78
Step 1: 49, 55, 58, 60, 66, 75, 78, 80
Step 2: (8+1)/2= 4.5th observation
Median = AM of (60, 66)
= (60+66)/2
Median = 63

Mode:
The frequent or repeated observation in the given data set is termed as mode. It is the easiest average that
can be calculated by merely inspecting the data. The notation of mode is usually, Z.
Some specialties of mode are,
1. The presence of extreme values does not influence the mode value
2. It is not based on every unit in the data.
3. It is the only average that can have more than one value for the same data or cannot be exist at all for
a data (if none of the observation is repeated or all the observation is repeated same number of
times)
Ex: - (1) Calculate mode for the following observations
25, 20, 20, 30, 32, 30, 20, 35
Mode = the most repeated observation
Mode = 20 (One mode)
(2) Calculate mode for the following observations
100, 152, 135, 120, 152, 100, 160, 196
Mode = 100, 152 (Two modes)
(3) Calculate mode for the following observations
20,22,22,18,20,18,16,15,1,5,16
Mode= No mode since each observation repeated is same.
Note: General relationship of mode, median and mean for non symmetrical distribution is,
Mode= 3 Median – 2 Mean

Geometric mean:
The main application of geometric mean is to measure the average change of growth. For example if we
are finding the average rate of increase of population of a place, geometric mean is the best average.
Geometric mean is the nth root of product of every the units of observations in a given set of data.
G = n√X1 X2 … X n
This average is also based on every the units of observations in a given set of data.
Ex: - Calculate GM for the set of observations given below
8, 2
GM = √2 ∗ 8
2

= √16
2

GM = 4
Note: The above equation is not applicable when the given data is very large.

Harmonic mean:
The HM is used to measure the average of different rates of observation producing the equal effect. That
is the average time taken to complete the same distance by different people. The harmonic mean is
defined as the reciprocal of the arithmetic mean of the reciprocal of the set of observation. It is denoted
by H.
That is
n
H= 1

x
Ex: - Calculate HM for the following observations
1/6, 1/8, 1/2, ¼
n
H= 1

x
4
= 1 1 1 1
∑ + + +
2 4 6 8

= 4/20
= 1/5
H = 0.5
Note: Relationship between the three means GM, AM and HM are
1. GM=√AM ∗ HM, for any two numbers
2. AM≥GM≥HM

MEASURES OF DISPERSION
Last part we have discussed the central value or the average value for the given data. But the central value
alone is not enough to describe the data entirely. Because there are many data with the same central value
but the data is entirely different in their nature. The variation or scatter of data from the central value or
the average is known as measures of dispersion.
Thus we can measure the variability within the data. Also we compare the variability of two or more data
sets.
Following are the different measures of dispersions
Range:
It is the simplest dispersion measure. The difference of the highest value and the lowest value has been
taken to obtain the value of range. That is
Range R= H-L
Where L = lowest value and H = highest value of a given observation set
The main demerit of range is that it does not consider the entire data. It is simply based on the lowest and
highest value of the data. Hence the extreme values are influence the range.
Mean deviation:
Mean deviation can be calculated from median, mean and mode. It is the average of the absolute
deviations of the given set of each observation from any measure of central value. Thus mean deviation
from mean, mean deviation from median and mean deviation from mode can be calculated from any set
of observations.
The main advantages of mean deviation are that MD is calculated on including all the data points.
Standard deviation:
The measure of dispersion which is used most common and the most important dispersion measure is
termed as standard deviation. The squared standard deviation is variance. It is computed by getting the
positive square root of the arithmetic mean of the squared deviations from each observation from their
mean. Hence the calculation of standard deviation is based on arithmetic mean of the observation.

MEASURE OF SKEWNESS
There are data sets which has the same measure of central value and the same measure of deviation, but
still the differ in their nature. Thus measures of central tendency and measure of dispersion together is not
enough to describe the characteristics of the distribution completely. Hence we study the measure of
skewness and measure of kurtosis.
In symmetrical distribution measures of central value like the mean, mode and median will coincide in a
point. And if we plot these in a graph we get a bell shaped curve which is symmetrical to the central
values.
The skewness is just opposite to the symmetrical distribution. That is it is the asymmetry or lack of
symmetry of the distribution. The skewness can be either negatively skewed or positively skewed. And if
the measure of skewness value is zero, then it is considered as a symmetrical distribution.
The skewness is explained by the following graph
Positively skewed distribution: the right tail of the frequency distribution is extends or the longer right
side of the frequency distribution is the asymmetry in the right side or positive skewness of the
distribution.
Here the relationship between the mean, median and mode are given by, mode<median<mean
Negatively skewed distribution: the asymmetry because of the longer left tail or the left side of the
frequency distribution is extended is called negatively skewed distribution.
Here the relationship between the mean, median and mode are given by, mean<median<mode

Following are the different measure of skewness


1. Karl Pearson’s method of skewness
2. Bowley’s method of skewness
3. Measure of skewness based on moments

MEASURE OF KURTOSIS
Even if the frequency distribution is symmetric about the mean, that is the frequency curve is bell shaped,
still the shape of the curve will be different based on the peakedness of the bell shaped curve.
Some curve shows normal peakedness. But there is symmetric curve which has more peakedness and has
flat curve also. The curve which has more peakedness is known as leptokurtic and the curve which has flat
peak is known as platykurtic. And the normal distribution is known as mesokurtic.

THEORY OF PROBABILITY AND PROBABILITY


DISTRIBUTIONS
The term probability is related to chances. In a daily life we often used the term chances of happening
something. For example there is a 90% chance that it may rain. There is a 50% chance that a person clears
the examination. So here we see that these chances are expressed in numerical number even though it is a
prediction.
Probability can defined as it is a statistical theory of finding the chance of occurring an event of
interest.

SOME BASIC TERMINOLOGIES ASSOCIATED WITH THEORY OF PROBABILITY


Random experiment: An experiment in which there will be more than one outcome and it can be
repeated under the similar conditions. A random experiment doesn’t have a unique outcome.
For example: Tossing of an unbiased coin, rolling an unbiased dice, measuring the body temperature of
persons at different time points. Etc.
Outcome: The results of the random experiment are termed as outcome of that experiment. For a random
experiment there will be two or more outcomes. A sample point is the single outcome of that random
experiment.
Sample space: It is the entire collection of the outcomes in a random experiment. It is the set of sample
points
Event: A whole or the portion of the sample space that is the subset of the entire outcome (sample space)
is termed as an event. Capital letters are usually used to denote an event. There are different types of
events. There are different types of events.
Simple event: An event which contains a single outcome (sample point) of interest is known as simple
event
Compound event: An event which contains more than one outcome is known as complex event
Null event: An event which has no outcome at all is called null event or impossible event. It is usually
denoted by the letter ϕ. There is a zero chance of occur that event.
Sure event: If all possible outcomes are included in an event it is termed as sure event. The sample space
since it contains all the outcomes is a sure event.
Equally likely events: When two or more events are said to be equally likely events, if the chances of
occurring of each and every event are the same. That is all the events have an equal probability of
occurring when conducting a random experiment.
Ex: -
1. While coin tossing experiment the two events occurring of head and occurring of tail have equal
chances of occurring
2. When a dice is rolled, the two events are A: An even number turned out and B : An odd number
turned out, then the events are
A= 2, 4, 6 B= 1, 3, 5

The chances or probability of occurring of these two events are the same
Union of events: union of two or more events is the occurrence of any one of the events. That is if A and
B are two events, then the union of events A and B is defined as the occurrence of either the event A or
the event B. The denotation of A union B is (A or B), (A U B) or (A+B).
Intersection of events: The intersection of two or more events is defined as the simultaneous occurrence
of all the events together. For the events A & B, their intersection is the occurrence of both the events A
and B together. The denotation of A intersection B is (A and B), (A∩B) or (AB)
Exhaustive events: The union or the totality of the events gives the sample space then those events are
termed as exhaustive events. That is the three events A, B and C are exhaustive events if their union is the
sample space, AUBUC=S
Mutually exclusive events: In a trial if the occurrence of any one of the events prevents the occurrence of
the rest of the events then those events are termed as mutually exclusive events. That is if the events are
mutually exclusive events only one event at a time can occur in a single trial. Thus their intersection will be
a null set.
Independent and dependent events: If the events are independent then the occurrence of an event does
not influence the occurrence of any other events.
In the opposite case if the occurrence of an event influence the occurrence of other event then these
events are dependent events.
Complement of an event: The complement of an event A is the non occurrence of that event. That is the
complement of an event A contains all the points which are not in the event A. The compliment of an
event A is denoted by A’, AC orA̅.

THE CLASSICAL DEFINITION OF PROBABILITY


When we perform a random experiment and let S is the sample space and A is the event of interest. That is
A is the subset of the sample space. Then the probability of the event A is obtained by
Number of favourable cases
P (A) = Total number of exhaustive cases
n(A)
P (A) =
n(S)

AXIOMS OF PROBABILITY
There are three important axioms of probability.
1. The probability of any event is a real number. That is P (A) ≥0.
2. The probability of occurrence of at least any one of the outcomes among all possible outcomes is
one. That is P(S) = 1
3. For any two mutually exclusive events, the occurrence of at least one of the events is the sum of
probability of occurrence of those two events. That is if A and B are two mutually exclusive events,
then the occurrence of either A or B is given by
4. P (AUB) =P (A) +P (B)
Note: The probability of any event lies between 0 and 1. That is if A is an event, then 0≤P (A) ≤1. That is
the probability of a null event is zero and probability of a sure event is one.

ADDITION THEOREM
The addition theorem states that if A, B are any two events and their corresponding probabilities are given
by P (A) and P (B). Then the occurrence of any one of these events is given by (at least one of the event)
P (AUB) = P (A) +P (B)-P (A∩B)
The addition theorem for two mutually exclusive events A and B is defined as the occurrence of any one of
the mutually exclusive events is given by
P (AUB) = P (A) +P (B)

CONDITIONAL PROBABILITY
The conditional probability is defined as the occurrence of one of the event under condition that the other
event has already been occurred.
That is if A, B are the two events of a random experiment, then the probability of occurrence of the event
A under the condition that the event B has already occurred is denoted by P(A/B).
P (A∩B)
P (A/B) = P(B)

MULTIPLICATION THEOREM
In a random experiment the two events A and B are dependent on each other, then the occurrence of both
these events is given by the multiplication theorem as
P (A∩B) = P (A) P (B/A).

PROBABILITY DISTRIBUTIONS
The probability distribution can be defined as a mathematical function which gives the probability or
chance of occurrence of events of a random experiment. The probability distribution is broadly divided
into two categories. They are
1. Discrete probability distribution
2. Continuous probability distribution
The probability distribution for a discrete random variable is known as discrete probability distribution and
when we consider the continuous random variable the distribution of such variable is known as continuous
probability distribution.
The following are the important discrete probability distributions
1. Bernoulli distribution
2. Binomial distribution
3. Poisson distribution
4. Hyper geometric distribution
Here we are explaining the first three distributions

Bernoulli distribution:
A single trial results the value either zero with a probability p or one with a probability 1-p is known as
Bernoulli trial. The distribution of the Bernoulli trial is known as Bernoulli distribution.
The distribution function is defined as p(x) = px (1-p) 1-x where x is zero or one.
Ex: - Number of heads obtained in tossing of a single coin

BINOMIAL DISTRIBUTION:
When the Bernoulli’s trial is repeated independently n number of times is called a binomial distribution.
Binomial distribution is a discrete distribution. The probability mass function is given by
P(x) = (𝑛𝐶𝑋 ) px (1-p) 1-x
Where, X is the binomial variate. X can take values 0, 1, 2….n.
The parameters of the binomial distributions are n and p. where n is the number of trials and p is the
probability of success.
The mean and variance of binomial distribution is np and np(1-p) respectively
Ex: - The number of defective items from a sample of 10 items manufactured by a company, number of
tails obtained when an unbiased coin is tossed 5 times etc

POISSON DISTRIBUTION:
The Poisson distribution is a distribution is a discrete probability distribution in which the probability of
success is very small. It is the distribution of number of events occurring in a given time period.
The probability mass function is given by
𝑒 −𝜆 𝜆𝑥
P(x) = 𝑥!
Where x is the binomial variate which can take values zero to infinity.
There is only one parameter fro Poisson distribution that is λ. The mean and variance of the Poisson
distribution is λ.
Ex: - number of road accidents occurred in a road on a particular day, demand of patients for service at a
health institution etc.
NORMAL DISTRIBUTION (CONTINUOUS DISTRIBUTION)
Normal distribution is a continuous probability distribution. It is the most useful distribution for the
continuous random variables. In real life situation most of the data have a tendency to follow a normal
distribution.
The normal distribution is given by
−1(𝑥−µ) 2
1
f (x) = 𝜎 𝑒 2 𝜎2
√2𝜋
Where, x is a normal variate which can take values from minus infinity to plus infinity.
The parameters of normal distributions are µ and σ2
Ex: - The weight of a group of individuals, the life length of a certain electronic equipment manufactured
by a company etc.
The curve of a normal distribution is known as normal curve. Which is a bell shaped curve.

The following are the properties of a normal curve,


1. It is a symmetrical curve
2. It is bell shaped curve
3. It is symmetric about the points mean, median and mode which coincide. That is
mean=median=mode
4. Normal distribution is a unimodal distribution. That is it has only one mode.
5. The normal distribution is mesokurtic
6. The total area under the normal curve is equal to one etc.

SAMPLING THEORY
The enumeration or study of complete unit of the population is called census survey. A representative
portion of the entire population is known as the sample. An enumeration conducted in the selected
representative sample of the population is called sample survey. The main thing which we have to take
care of is that the selected sample should represent the entire population. To ensure this we are using
some techniques and method of obtaining a representative sample from the population. This is called
sampling theory
The sampling can be broadly divided into two. They are
1. Probability sampling
2. Non probability sampling
The sampling units are taken from the population purely on random bases is known as probability
sampling. And the sampling is not done by random bases is known as non probability sampling.
The following are the important probability sampling techniques

Simple random sampling:


When the entire unit of the population is known in advance and the population is small we can adopt this
method of sampling. Here each and every unit of the population is assigning an equal and independent
chance of being selected. There are different methods to obtain simple random sampling. It can be done
either by random number table method or lottery method.
Systematic sampling:
The sample is taken from a systematically arranged data is called systematic sampling. From an arranged
data if want to select a sample of size “n” from a population size “N” units find a sample interval that is
K=N/n. first choose a number between 1 and K. that will be our first unit in the sample. Let it be a, then the
rest of the sampling units are a+k, a+2k……this is continued until we get the required number of sample of
n units.
Stratified sampling:
This sampling is opting when the population is heterogeneous in nature. First we are dividing this
heterogeneous data into homogenous groups based on the similar characteristics. Then we are taken
samples by simple random sampling from each homogeneous group. This makes sure that we have
representative samples from each homogenous group. The homogeneous groups are known as strata.
Cluster sampling:
In cluster sampling, initially the population is divided into different clusters based on particular
characteristics usually based on the geographical area. From these different clusters select few clusters
randomly. And from the selected clusters samples are taken. This helps in reducing the large area
population to a few selected clusters.

SAMPLING DISTRIBUTION
Any constant calculated based on the population is a parameter. Any constant or measure calculated
from the selected sample from the population is termed as a statistic. When we take a number of samples
from the population and from each sample we calculated the sample statistic, this sample statistic follows
a particular distribution is called as sampling distribution.
Different statistic calculated from samples follows different probability distribution. They are,
1. Sampling distribution of mean
2. Sampling distribution of proportion etc.

ESTIMATION
Estimation is a statistical theory used to obtain the unknown parameter of the population based on the
representative sample taken from the population. Here we estimate the most likely value of the unknown
parameter of the population based on the sample statistic.
The sample statistic or the mathematical function based on the sample values which is used to estimate
the unknown parameter of the population is known as estimator.
The value obtained from the estimator of the sample is known as an estimate of the unknown parameter
of the population.
There are two different types of estimation. They are
1. Point estimation
2. Interval estimation
Point estimation: When we are finding the value for the unknown parameter, we estimate a single unique
value for the unknown. And the single estimate of the population is called as point estimate
Interval estimation: When we are finding the unknown parameter value of the population instead of
estimating a single value we are estimating an interval in which our unknown parameter value expected to
lie. This estimation of proposing an interval of estimate is known as interval estimation.
TEST OF SIGNIFICANCE AND BI VARIATE ANALYSIS
SIMPLE TEST OF SIGNIFICANCE
The entire units or the totality of the units under consideration is called a population. A representative
part or a portion of the population is called a sample.
Any statistical constant of the population is called a parameter. Any statistical constant of the sample
taken from the population is called statistic.
Our main objective of sampling theory is to propose a value for the unknown parameter of the population
based on the sample. When we make a scientifically test to check any claim about the population
parameter is known as testof significance. And the claim about the population parameter is known as
hypothesis.

Statistical inference:
The statistical theory of making decision about the unknown population parameter based on the sampling
concept of probability is known as statistical inference.
Statistical hypothesis:
Any claim or a statement about the population parameter is known as statistical hypothesis. There are two
types of hypothesis. They are
1. Null hypothesis:
A null hypothesis is a hypothesis which is being tested for a possible rejection, under the assumption
that it is true. That is if we test a significance difference between the population parameter and a
sample statistic, then we define a null hypothesis of “no difference”. That is the population parameter
is equal to the sample statistic. The null hypothesis is denoted as H0.

2. Alternative hypothesis:
When we reject the null hypothesis, we accept a hypothesis against the null hypothesis is called an
alternative hypothesis. The alternative hypothesis is three types. They are not two tailed (population
parameter not equal to sample statistic), left tailed (population parameter is less than sample statistic),
right tailed (population parameter is greater than sample statistic).

Types of errors:
When we conduct a test of significance there are possibility to occur some errors. They are
Type I error: It is the error of rejecting the null hypothesis when it is actually true is called type I error.
Type II error: It is the error of accepting the null hypothesis when it is actually not true is called type II
error.
There are different test of significance and the test is choose based on the assumptions and the purposes.
For example t test is used to test the mean of the population; chi square test is used to test the
proportions etc.

CORRELATION
When we deal with the two or more continuous variables and the associated relative variation between
these variables we use the concept of correlation. For example a person tends to gain more weight when
his height increases. Here the height and weight are two variables (bivariate data) and there is a relative
variation in Weight as the height changes.
So in simple words the correlation is a linear relationship between two or more variables. We are studying
the interdependence of those variables.
Correlation can be
– Simple correlation
– Multivariate correlation and
– Partial correlation
When we are considering the relative variation between two variables it is called as simple correlation.
When we consider the relative variation among three or more variables it is called as multivariate
correlation or partial correlation. But in partial correlation we consider the relative variation of the two
variables at a time by making the effect of other variables constant.
Types of correlation:
1. Positive Correlation: If the relative variations are explained when variables change in direction is
same that is a change of increase of a variable may results the change of increase in the other
variable or the change of decrease of a variable may results the change of decrease in the other
variable is called as positive correlation or direct correlation.
Ex: - As the height increases then the weight increases
When the demand of a commodity increases the price of that commodity increases
2. Negative correlation: : If the relative variations are explained when the variables are moving
opposite in direction that is as one of the variables increases the other variable decreases or as one of
the variables decreases the other variable increases is called as negative correlation. It is also referred
as inverse correlation.
Ex: - When the supply decreases then the price increases, the demand being the same
When the atmospheric temperature increases the demand of woolen cloths decreases
3. No correlation: if no association or related variation among the two variables, then there is an
absence of correlation (no correlation) among those variables.

Perfect correlation:
Perfect correlation may be negative or positive correlation. That is if there is an exact variation (in the same
proportion) in one variable relative to the other variable then there exists a perfect correlation.
In real life situation there hardly exists such kind of perfect correlation.
Causation:
In the correlation concept we merely interested in finding the relative variation even though the change of
variation occurring in a variable may not be the reason for the variation of the other variable. If the
variation in one variable is a reason for the change occurring in the other variable then there is a cause
and effect relationship (Causation). But in some cases there may be a correlation among the variables
but there is an absence of cause and effect relationship. The variation in one variable may be because of
some other reasons. Such kind of correlation is called as non sense correlation or spurious correlation.

Method of measuring Correlation:


Broadly there are two methods of measuring correlation either by graphical approach or by
mathematical approach.
1. Scatter diagram
The graphical method of identifying the correlation of two variables is called scatter or dot diagram. If
there are two variables X and Y, take one variable say, X in the x-axis and the other variable Y in the y-axis.
For corresponding each pair of (X, Y) mark the point in the graph. Then we obtain a group of points which
is scattered in the diagram. By observing the direction of scattered points by an imaginary line we can say
about the correlation between those two variables. For example
The correlation between weight and height of certain group of persons is shown by the scatter diagram
below.
100

80

60

Weight 40

20

0
150 160 170 180 190
Height

The scatter diagram is the simplest method of identifying the correlation of two variables. But there are
some limitations also. Scatter diagram just give an idea about the correlation. The strength of relationship
cannot be measured using scatter diagram. And by scatter diagram only simple correlation (two variables
only) can be measured.
Different type of correlation shown by scatter diagram

2. Karl Pearson’s coefficient of correlation


This is a mathematical approach of finding the correlation. It identifies whether there exist a correlation
and the type of correlation. Also it the correlation strength can be measured if exists by a numerical value.
The equation for calculating the Pearson’s Correlation Coefficient is given by
Cov (x,y)
rxy= σx σy

Where “rxy”or simply “r” is the coefficient of correlation. The value of correlation coefficient lies between -1
and +1.
Since Karl Pearson coefficient of correlation is a mathematical approach it gives clear idea about the type
of correlation as well as the strength of correlation by interpreting the numerical value of correlation
coefficient, r.
Interpretation of the correlation coefficient value, r:
The correlation coefficient value lies between -1 and +1.
That is -1≤ r ≤ +1
A positive value of correlation coefficient indicates a positive correlation
A negative value of correlation coefficient indicates a negative correlation
A value zero indicates there is no correlation among the variables.
r=+1, perfect positive correlation
r=-1, perfect negative correlation
r=0, No correlation
r≥0.5, strong positive correlation
r≤0.5, strong negative correlation

3. Spearman’s coefficient of rank correlation:


The Karl Pearson a popular scientist introduced a coefficient of correlation that can be measured when
there are quantitative variable characteristics are under consideration. But in some cases we may need to
find the relative variations in the qualitative characteristics also. For example if we are interested in finding
the relative variation between the honesty and efficiency of people. When the distribution is not normal or
the shape of the distribution is not known instead of Karl Pearson’s method we are using Spearman’s Rank
Correlation.
This is a mathematical approach of finding the correlation like Karl Pearson’s method.

REGRESSION ANALYSIS:
In the previous section we discussed about correlation. Once we check the correlation and identified two
or more variables are closely related. If the correlation is strong enough and the correlation is cause-effect
relationship (causation), we may be interested in predicting one of the variables using the other variable/s.
The theory of estimation or prediction of one variable with respect to the change in the other
variable/s is called regression theory.
That is if there is a correlation between two variables, then we can predict the unknown value of the one
variable by using the value of the other variable which is known.
For example if we identify the amount of rain fall (X) and the yield of paddy (Y) is correlated, we may
interested in prediction of one of the variables. That is the amount of rain required to produce a certain
yield of paddy.
Dependent and Independent Variable:
In the regression analysis there are two types of variables, dependent and independent variable.
When there is a correlation between two or more variables we may be interested in predicting the value of
one variable, which is influenced by other variable/s, is called dependent variable. The dependent
variable is also called as regressed or explained variable.
In the regression analysis the variable/s which is/are influences the dependent variable or the variable/s is
used to predict the value of dependent variable is called as independent variable. Independent variable is
also called regressor.
For example, the cost of living and salary of persons are correlated. Here the cost of living is dependent on
the salary. Thus the cost of living is influenced by the salary of person. Then cost of living is the dependent
variable. And the salary is known as independent variable.

Line of regression:
When the two variables are correlated we can see that there will be a cluster of points and we can draw an
imaginary line along the pattern of the cluster of points. This straight line is known as line of regression.
This line of regression gives the best estimate for the dependent variable for a specific value of the
independent variable.
Regression equation:
The prediction or estimation of dependent variable by using the independent variable is done by certain
mathematical equation. The mathematical equation is called as regression equation.
There are two regression equations. They are
1. Regression equation of X on Y
In this case the X variable is the dependent variable and the Y variable is the independent variable. Thus
value of X is estimated for a corresponding value of Y.
The regression equation of X on Y is given by
(X-X
̅)=bxy(Y-Y̅)
2. Regression equation of Y on X
In this case the Y variable is the dependent variable and the X variable is the independent variable. Thus
value of Y is estimated for a corresponding value of X.
The regression equation of Y on X is given by
(Y-Y
̅)=byx(X-X̅)
Where X ̅ and Y̅ are the means of X and Y respectively. bxy and byx are the regression coefficients.

INTRODUCTION TO TIME SERIES


The data which are classified chronological bases is termed as a time series data. That is the data are
arranged according to different time periods. The main application of time series data analysis is for the
future prediction or forecasting the future events. Mainly the economists, business man, statisticians etc
are interested in this future prediction of even based on the past experiences.
TIME SERIES COMPONENTS
They are four in number
1. Seasonal variation
2. Secular trend
3. Cyclical variation
4. Irregular variation
5. Secular trend:
A time series data shows a general tendency of increasing or decreasing or it may remain constant over a
long period of time is a secular trend. And this secular trend may be because of population growth,
technological change, people taste changes etc.
The example for secular trend is the increase in the price of gold over many years, the usage of mobile
phones increases over many years etc

1. Seasonal variation:
The seasonal variation is the periodic variations and which is regular and a short term variation. Seasonal
variation usually occurs within a year. Since it is a regular variation it is predictable variation. The causes of
this regular and periodic variation are because of some customs or some traditions of the people. For
example the selling of sweets increases in the diwali season and selling of school stationary items will be
more in the school opening month.
Another cause of seasonal variation is climate and weather changes. That is in the winter season the selling
of sweaters and jackets will be more and in the summer season it will be less.

2. Cyclical variations:
The cyclical variation is also called business cycle. In most of the economic and business activities we can
observe cyclical variations. There are cyclical or oscillatory variations occur. There are four stages in the
cyclical variation. They are prosperity, depression, decline and recovery or improvement.
Since this kind of variation is occur mostly in the business field one can closely watch this variation in the
business and make the policies according to that.

3. Irregular variations:
The unexpected or sudden variation in the data is an irregular variation. This can neither be predicted nor
be controlled. This kind of variation is mainly caused by natural disasters.
For example the population in a place is suddenly decreased by the tsunami or earth quake.
The figure given below shows the pattern of different component of time series
INDEX NUMBERS
Index numbers is a very important concept in statistics. It uses almost every filed now a days. At some
time, everyone faces the question of how much something has changed over a different period of time.
The degree of change must be determined and defined. Typically, to measure such changes we use index
numbers.These are a specialized type of average which measures the relative change of quantity, price and
value of commodities.
Definition: It is a statistical device which measures the relative change in certain phenomenon like
quantity, price and value of commodities with respect to different time, different geographical area or
location etc.
Index numbers are even defined as economic barometers. Because the relative change occurred in the
economic activities can be measured using the index numbers. The economic activities such as imports,
exports and the quantity of goods produced over a different period of time can be measured using index
numbers.
It is a unique number expressed in percentage. This makes the comparison unit less irrespective of the unit
of different commodities and makes the comparison possible.

INDEX NUMBERS APPLICATION:


1. Index numbers such as the consumer or cost of living index numbers are often cited in news reports as
general indicators of the nation's economic condition and to determine the real buying power of
money is another example of how index numbers help increase knowledge of other factors.
2. Management uses index numbers as part of an intermediate computation to understand other
information better.
3. Seasonal indices to modify and improve estimates of the future.
4. The index numbers helps in studying the trend and tendencies.
5. Index number helps the government to formulate the policies.

Index numbers are not only making a price comparison of a commodity. It is used to compare the relative
change in the quantity or value of a single commodity or group of commodities. Based on this index
numbers are mainly classified into three types. They are
1. Price index number: Any commodity or a group of commodities relative change in the price
based on time, place etc are expressed by a unique number termed as price index number. Here
the price may be compared in the current year with the base year. P01 is the usual notation for price
index number.
2. Quantity index number: The relative change of the quantity of item or items, that is the volume of
goods produced, consumed, exported, imported etc are termed as quantity index numbers. Q01is
the usual notation for quantity index numbers.
3. Value index number: Relative change of the total money value (that is the product of price and
quantity) of the transaction takes place. V01 is the usual notation for value index numbers.
Some times index numbers are classified as accordingly number of commodities has taken into
consideration. They are
– Simple index number: if the relative change in the price, quantity or value of a single commodity
or item has taken at a time then the index number is called as simple index number.
– Composite index number: if the relative change in the price, quantity or value of a many
commodities is taken at a same time then the index number is called as a composite index number.

CONSUMER PRICE INDEX NUMBER


The wholesale (price) index number gives an idea about the common price level relative variations in the
economy. But when we wanted to find the relative changes in the cost for living in a particular class of
groups we calculate cost of living index number or consumer price index number. Here people of different
classes consume different commodities or the same commodities in different quantity. So the cost needed
for living for the people of different classes will be different.
This index number concept is important because if there is a relative change in the cost needed for living
of common people according to that their income allowances has to be structured.
The consumer price index is the index number of the cost met by to satisfy the basic needs that is to buy
the ‘basket of goods and services’ for a specified class of people or consumers.
The calculation of index number is broadly classified into two. They are
– Weighted index number
– Unweighted index number
When calculating unweighted index number it is giving equal importance to all the items. But in real
situation some items have higher priority or weightage compared to other items. Thus weighted index
number is the best and most commonly used index number.
WEIGHTED INDEX NUMBER
While computing weighted index number, the items have more importance assigned more weightage.
When we calculate price index number we use quantity as the weight. There are different equations for
calculating the price, quantity and value index number given by different scientists. Price index numbers
given by different scientists are,
1. Paasche’s Price index
2. Laspeyre’s Price index
3. Marshall-Edgeworth’s Price index
4. Dorbish-Bowley’s Price index
5. Fisher’s Price index
Fisher index number is considered as the most ideal index number.
ENVIRONMENTAL ISSUES IN VALUATION
ENVIRONMENT AND VALUATION
DIFFERENCES BETWEEN THE MARKET PRICE AND THE NEGATIVE VALUE
CONSEQUENT ON ENVIRONMENTAL IMPACT

Environmental Factors are increasing the importance of asset valuation, as the market has become aware
of the potential adverse effect of the contaminants in water, soil, land or other assets. Thus, it is important
for the valuer to become knowledgeable about contaminants and their effects on asset value.
THE VALUER’S DUTY
The environmental effect on property may be variable depending on the effects it has in that particular
case. Consequently, it is of fundamental importance that a valuer should be able to identify the basic
environmental risk involved in order to calculate the likely effect upon the property in question.
Valuers are not expected to be environmental experts. However, it is clear that they must have some basic
knowledge of the environmental issues. They must make initial enquires regarding factors which may
potentially affect the value of the property. One of the most important issue is that of contamination.

ENVIRONMENTAL ISSUES OF AIR POLLUTION,


WATER POLLUTION
Developmental activities such as construction, transportation and manufacturing not only deplete the
natural resources but also produce large amount of wastes that leads to pollution of air, water, soil, and
oceans; global warming and acid rains. Untreated or improperly treated waste is a major cause of pollution
of rivers and environmental degradation causing ill health and loss of crop productivity. In this lesson you
will study about the major causes of pollution, their effects on our environment and the various measures
that can be taken to control such pollutions.
Human activities directly or indirectly affect the environment adversely. A stone crusher adds a lot of
suspended particulate matter and noise into the atmosphere. Automobiles emit from their tail pipes
oxides of nitrogen, sulphur dioxide, carbon dioxide, carbon monoxide and a complex mixture of unburnt
hydrocarbons and black soot which pollute the atmosphere. Domestic sewage and run off from
agricultural fields, laden with pesticides and fertilizers, pollute water bodies. Effluents from tanneries
contain many harmful chemicals and emit foul smell. These are only a few examples which show how
human activities pollute the environment. Pollution may be defined as addition of undesirable material
into the environment as a result of human activities. The agents which cause environmental pollution are
called pollutants. Pollutants may be defined as a physical, chemical or biological substance unintentionally
released into the environment which is directly or indirectly harmful to humans and other living organisms.

AIR POLLUTION
Air pollution is a result of industrial and certain domestic activity. An ever increasing use of fossil fuels in
power plants, industries, transportation, mining, construction of buildings, stone quarries had led to air
pollution. Air pollution may be defined as the presence of any solid, liquid or gaseous substance including
noise and radioactive radiation in the atmosphere in such concentration that may be directly and indirectly
injurious to humans or other living organisms, plants, property or interferes with the normal environmental
processes. Air pollutants are of two types
1. Suspended particulate matter, and
2. Gaseous pollutants like carbon dioxide (CO2), NOx etc.

Suspended particulate matter


Some of the major air pollutants, their sources and effects are given in below table
Pollutant Sources Effects
Suspended Smoke from domestic, Depends on specific composition
particulate matter Industrial & vehicular soot. Reduces sunlight and visibility,
/dust increases corrosion, Pneumoconiosis,
asthma, cancer, and other lung
diseases
Fly Ash Part of smoke released from Settles down on vegetation, houses.
chimneys of factories and Adds to the suspended participate
power plants. matter (SPM) power plants in the air.
Leachates contain harmful material

Gaseous pollutants
Power plants, industries, different types of vehicles – both private and commercial use petrol, diesel as fuel
and release gaseous pollutants such as carbon dioxide, oxides of nitrogen and sulphur dioxide along with
particulate matter in the form of smoke. All of these have harmful effects on plants and humans. Below
Table lists some of these pollutants, their sources and harmful effects.

Pollutant Source Harmful effect


Carbon compound Automobile exhaust burning of Respiration problems
(CO and CO2) wood & coal Green house effects
Sulphur compounds Power plants and refineries Respiratory problems in human
(SO2, and H2S) volcanic eruptions Loss of chlorophyll in plants
(chlorosis)
Acid rain
Nitrogen Compound Motor vehicle exhaust Irritation in eyes and lungs
(NO and N2O) atmospheric reaction Low productivity in plants
Acid rain damages material
(metals and stones)
Hydrocarbons Automobiles and petroleum Respiratory problems
(Benzene, Ethylene) industries. Cancer causing Property
SPM (Suspended Thermal Power plants, Poor visibility, breathing
Particulate matter) Construction activities, problem
(Any solid & liquid Metallurgical processes and Lead interferes with the
particles suspended automobiles. development of red blood
in the air, flush, dust, diseases and cancer.
lead) Smog formation leads to poor
visibility and aggravates asthma
in patients.
Fibres, Cotton & Textile & Carpet weaving Lung Disorder
Wool industries
Prevention and control of industrial pollution Industrial pollution can be greatly reduced by:
1. Use of cleaner fuels such as liquefied natural gas (LNG) in power plants, fertilizer plants etc. which is
cheaper in addition to being environmentally friendly.
2. Employing environment friendly industrial processes so that emission of pollutants and hazardous
waste is minimized.
3. Installing devices which reduce release of pollutants. Devices like filters, electrostatic precipitators,
inertial collectors, scrubbers, gravel bed filters or dry scrubbers are described below:
– Filters – Filters remove particulate matter from the gas stream. The medium of a filter may be made
of fibrous materials like cloth, granular material like sand, a rigid material like screen, or any mat
like felt pad. Bag house filtration system is the most common one and is made of cotton or
synthetic fibres (for low temperatures) or glass cloth fabrics (for higher temperature up to 290oC).
– Electrostatic precipitators (ESP)- The emanating dust is charged with ions and the ionized
particulate matter is collected on an oppositely charged surface. The particles are removed from
the collection surface by occasional shaking or by rapping the surface. ESPs are used in boilers,
furnaces, and many other units of thermal power plants, cement factories, steel plants, etc.
– Inertial collectors – It works on the principle that inertia of SPM in a gas is higher than its solvent
and as inertia is a function of the mass of the particulate matter this device collects heavier
particles more efficiently. ‘Cyclone’ is a common inertial collector used in gas cleaning plants.
– Scrubbers – Scrubbers are wet collectors. They remove aerosols from a stream of gas either by
collecting wet particles on a surface followed by their removal, or else the particles are wetted by a
scrubbing liquid. The particles get trapped as they travel from supporting gaseous medium across
the interface to the liquid scrubbing medium. Gaseous pollutants can be removed by absorption in
a liquid using a wet scrubber and depends on the type of the gas to be removed e.g. for removal
of sulphur dioxide alkaline solution is needed as it dissolves sulphur dioxide. Gaseous pollutants
may be absorbed on an activated solid surface like silica gel, alumina, carbon, etc. Silica gel can
remove water vapour. Condensation allows the recovery of many by products in coal and
petroleum processing industries from their liquid effluents.

Apart from the use of above-mentioned devices, other control measures are:
1. Increasing the height of chimneys.
2. Closing industries which pollute the environment.
3. Shifting of polluting industries away from cities and heavily populated areas.
4. Development and maintenance of green belt of adequate width.

Control of vehicular pollution


1. The emission standards for automobiles have been set which if followed will reduce the pollution.
Standards have been set for the durability of catalytic converters which reduce vehicular emission.
2. In cities like Delhi, motor vehicles need to obtain Pollution Under Control (PUC) certificate at regular
intervals. This ensures that levels of pollutants emitted from vehicle exhaust are not beyond the
prescribed legal limits.
3. The price of diesel is much cheaper than petrol which promotes use of diesel. To reduce emission of
sulphur dioxide, sulphur content in diesel has been reduced to 0.05%.
4. Earlier lead in the form of tetraethyl lead was added in the petrol to raise octane level for smooth
running of engines. Addition of lead in petrol has been banned to prevent emission of lead particles
with the vehicular emission.
WATER POLLUTION:
Addition or presence of undesirable substances in water is called water pollution. Water pollution is one of
the most serious environmental problems. Water pollution is caused by a variety of human activities such
as industrial, agricultural and domestic. Agricultural runoff laden with excess fertilizers and pesticides,
industrial effluents with toxic substances and sewage water with human and animal wastes pollute our
water thoroughly. Natural sources of pollution of water are soil erosion, leaching of minerals from rocks
and decaying of organic matter. Rivers, lakes, seas, oceans, estuaries and ground water sources may be
polluted by point or non-point sources. When pollutants are discharged from a specific location such as a
drain pipe carrying industrial effluents discharged directly into water body it represents point source
pollution. In contrast non-point sources include discharge of pollutants from diffused sources or from a
larger area such as runoff from agricultural fields, grazing lands, construction sites, abandoned mines and
pits, roads and streets.
Sources of water pollution
Water pollution is the major source of water borne diseases and other health problems. Sediments
brought by runoff water from agricultural fields and discharge of untreated or partially treated sewage and
industrial effluents, disposal of fly ash or solid waste into or close to a water body cause severe problems
of water pollution. Increased turbidity of water because of sediments reduces penetration of light in water
that reduces photosynthesis by aquatic plants.
Pollution due to pesticides and inorganic chemicals
1. Pesticides like DDT and others used in agriculture may contaminate water bodies. Aquatic organisms
take up pesticides from water get into the food chain (aquatic in this case) and move up the food
chain. At higher trophic level they get concentrated and may reach the upper end of the food chain.
2. Metals like lead, zinc, arsenic, copper, mercury and cadmium in industrial waste waters adversely affect
humans and other animals. Arsenic pollution of ground water has been reported from West Bengal,
Orissa, Bihar, Western U.P. Consumption of such arsenic polluted water leads to accumulation of
arsenic in the body parts like blood, nails and hairs causing skin lesions, rough skin, dry and thickening
of skin and ultimately skin cancer.
3. Pollution of water bodies by mercury causes Mina Mata disease in humans and dropsy in fishes. Lead
causes dyslexia, cadmium poisoning causes Itai disease etc.
4. Oil pollution of sea occurs from leakage from ships, oil tankers, rigs and pipelines. Accidents of oil
tankers spill large quantity of oil in seas which kills marine birds and adversely affects other marine life
and beaches.
5. Thermal pollution: Power plants- thermal and nuclear, chemical and other industries use lot of water
(about 30 % of all abstracted water) for cooling purposes and the used hot water is discharged into
rivers, streams or oceans. The waste heat from the boilers and heating processes increases the
temperature of the cooling water. Discharge of hot water may increase the temperature of the
receiving water by 10 to 15 °C above the ambient water temperature. This is thermal pollution.
Increase in water temperature decreases dissolved oxygen in water which adversely affects aquatic life.
Unlike terrestrial ecosystems, the temperature of water bodies remains steady and does not change
very much. Accordingly, aquatic organisms are adopted to a uniform steady temperature of
environment and any fluctuation in water temperature severely affects aquatic plants and animals.
Hence discharge of hot water from power plants adversely affects aquatic organisms. Aquatic plants
and animals in the warm tropical water live dangerously close to their upper limit of temperature,
particularly during the warm summer months. It requires only a slight deviation from this limit to cause
a thermal stress to these organisms.
Discharge of hot water in water body affects feeding in fishes, increases their metabolism and affects
their growth. Their swimming efficiency declines. Running away from predators or chasing prey
becomes difficult. Their resistance to diseases and parasites decreases. Due to thermal pollution
biological diversity is reduced. One of the best methods of reducing thermal pollution is to store the
hot water in cooling ponds, allow the water to cool before releasing into any receiving water body.

Control of water pollution


Waste water from domestic or industrial sources or from garbage dumps is generally known as sewage. It
may also contain rain water and surface runoff. The sewage water can be treated to make it safe for
disposal into water bodies like rivers, lakes etc. The treatment involves three stages: primary, secondary
and tertiary. This includes: -
1. Sedimentation,
2. Coagulation/flocculation,
3. Filtration,
4. Disinfection,
5. Softening and
6. Aeration.
The first four steps are of primary treatment. The first three steps are involved in primary treatment
remove suspended particulate matter. Secondary treatment removes organic solids, left out after primary
treatment, through their microbial decomposition. Effluents after secondary treatment may be clean but
contain large amounts of nitrogen, in form of ammonia, nitrates and phosphorous which can cause
problem of eutrophication upon their discharge into a receiving water body such as river, lake or pond.
The tertiary treatment is meant to remove nutrients, disinfect for removing pathogenic bacteria, and
aeration removes hydrogen sulphide and reduce the amount of carbon dioxide and make water healthy
and fit for aquatic organisms. This treatment of waste water or sewage is carried out in effluent treatment
plants especially built for this purpose. The residue obtained from primary treatment one known as sludge.

The following measures can be adopted to control water pollution:


1. The water requirement should be minimized by altering the techniques involved.
2. Water should be reused with or without treatment.
3. Recycling of water after treatment should be practiced to the maximum extent possible.
4. The quantity of waste water discharge should be minimized.

Physical Site Inspection


In all cases of valuation, for whatever purposes and for whatever type of property, the valuer must carry
out a thorough site inspection. The valuer should invariably, during such an inspection, give due
importance to environmental or potential environmental matters.
Identification of potential environmental problems is a skill which has to be acquired by the valuer. Any
visible signs that may indicate an environmental problem should be studied further. For example, in
valuing an industrial site it is possible that the site may have been used as a storage for toxic chemicals in
the past or a commercial site has asbestos contamination. The valuer should, in particular, be cautious
about a cunning owner trying to avoid a particular plant area or storage area.
The valuer must pay special attention to storage areas, remote areas of the property or areas which exhibit
visible signs of possible contamination. Some of the signs may be soil discoloration, dead or stressed
vegetation, disturbed soil and unexplained or stained concrete. Other potential environmental hazards
may include Underground storage tanks, old dump yards and pollution by contaminated water streams
originating-from neighboring sites. The valuer should also be on the lookout for foul odors, intense noise,
etc.

Knowledge of Site History


Sometimes it may not be possible to get any indication of a property being environmentally hazardous.
Under such circumstances it would be advisable to find out or to look at the history of the land or the
history of the adjoining land as in the past such land could have been used for purposes that may have or
will generate environmental problems. One way of doing this is to examine the municipal records which
could provide a source of information as to how the site was used. In addition to making enquiries about
prior use of land, it is also prudent to enquire about present use and neighboring properties. Often one
polluting site contaminates the neighboring site and vice versa. An example of this may be where a valuer
is valuing an industrial property, located adjacent to a particular smelter operation which emits
contaminants.
If the site is located close to a cemetery, there will naturally be a stigma attached to the site particularly if
the site is to be developed as a residential area.
EFFECT OF ENVIRONMENTAL FACTORS ON PROPERTY VALUES
Measures to restore damage & cost to cure
Growth in technology and scientific research have multiplied the number of factories and led to the
elimination of agricultural land, forest land and green belts. Toxic waste and chemical dumping have
become a daily phenomenon in almost every part of India. Pollution and its contest have become an
urgent issue of every urban and rural department at the central and state levels.
In view of the above it is natural that environmental and locality affects land value. Plots near factories,
tanneries, chemical plants and abattoirs - where waste is emitted in horrendous proportions – are
understandably not the most sought-after plots - especially by people who can afford to choose.
Therefore, a valuer needs to consider the environmental hazards in such areas. For, in these areas, health
hazards will gradually increase.
Where a property is found to be affected because of adverse environmental factors, such property may be
referred to as contaminated. This is a general term where the land, building, structure or ground water, etc.
are adversely affected by hazardous matter. These may manifest themselves in any form, i.e., solid, liquid
or gaseous. Noise can also pose a hazard.
The effect of contamination on the property value can be classified under three separate heads:
1. Direct damage and the resultant contamination cost.
2. Indemnification.
3. Stigma

Contamination Cost
Contaminated land may place a direct financial burden upon the owners.

Productivity cost
The productivity of contaminated land is normally lower than uncontaminated land. This is particularly true
of agricultural land. The takers for such land might be few as the overall productivity would be low and/or
restricted only to certain type of crops.

Remediation cost
This refers to the cost associated with testing, cleaning, disposal and subsequent monitoring. The costs are
generally referred to as 'control' or 'cure' costs. Sometimes such remediation may be necessary where the
type or degree of contamination demands that it first be cured. Remediation costs are often substantial
and, therefore, affect property values. Sometimes the cost of remediation far exceeds the cost of the land
itself.

Operational cost
The cost of operating a business on contaminated land can be tremendous. A major cost in many cases
may be the insurance premium itself.

Indemnification
Many owners agree to retain responsibility for future and current contamination in the cost of property. In
case of a sale, the valuer should discount such properties at an appropriate rate.

Stigma
Stigma means an unfavorable perception that continued contamination remains after clean-up has been
affected. Such a perception may have the possibility of influencing value.
According to J.A. Chalmers', stigma is "impacts on value associated with the property and the effect of this
on marketability and financial ability." Loss in value due to stigma does not arise from any actual fiscal
effect but from the market perception of that contamination. It is that discount required to compensate
for risks associated with the property. This stigma may be associated even with the adjoining property
which may or may not be affected. For example, valuers may have to consider the impact on a property
which is located close to a tannery, an abattoir or an oil refinery from where unpleasant odors are released.

ASSESSMENT AND VALUATION TECHNIQUES


The presence of contaminants or pollutants within a property area may not necessarily reduce its value if
such contaminants form part of an industrial process. For example, a body of water within the protective
area of an industrial undertaking may contain toxic or otherwise harmful contaminants that form part of
an industrial process for which there is a market demand.
Generally, a special license is obtained from the appropriate competent authority for operation of such
industrial undertaking or maintaining these -harmful contaminants on site. These processes and the
property can continue to be used as it is.
Once a valuer is aware of the contamination or suspected contamination problem associated with the
subject property, one of the following techniques to the valuation problem may be adopted:
1. Unaffected valuation technique
2. Affected valuation technique
3. Environmental balance sheet technique
4. Comparative technique

Unaffected Valuation Technique


In this method, an assumption is made that the site is unaffected by contamination on a preliminary
analysis. However, in case of doubt, an indication to this effect should be stated including the basis for
such doubt. The report should recommend expert advice and qualify that valuation has been made on an
'unaffected' basis and should contamination be indicated at a later date by an expert, then the report
should not be relied upon and in that case fresh valuation report would be necessary.

Affected Valuation Technique


In this method, the nature and extent of contamination is assessed by the expert environmental consultant
including cost for remedial and cleaning up action with the client's prior approval. The discounted value of
the property is then calculated with the help of such information. This process should include estimates for
cleaning adjoining property if necessary, influence of 'stigma' after remedial measures on the
contaminated property, etc.
Sometimes the environmental balance sheet technique described on the next subsection may be helpful
but this may not always be possible. Nevertheless, it would be prudent on the part of the valuer to give as
detailed information as possible, including how and for what reasons the discount factor was selected. If
the discount selected is the opinion of an expert or someone else, this should be clearly mentioned. A
copy of the environmental expert's report should invariably be annexed to the valuation report.

Environmental Balance Sheet Technique


Like an accounts balance sheet, this reflects the positive and negative aspects affecting the market value of
a property. The 'asset entry' would indicate the value of the property considering no environmentally
hazardous effects whereas the 'liability entry' would include environmental factors tending to reduce the
property value. Such factors may consist of:
1. Cost of estimating the extent of contamination if any and/or if this problem is really present, i.e.,
preliminary investigation.
2. Cost of quantifying the extent of the problems from an environmental standpoint and offering
alternative action plans to tackle the problem. This would give freedom to the owner to choose from
the alternatives and identify remedial actions.
3. Cost of selecting an appropriate remediation scheme.
4. Actual estimated cost of locating pollutants.
5. Cost to the owner, as a result of prior actions.
6. Calculated present value of future remediation, operations, management expenses and related costs
affecting future cash flows.
7. Estimated stigma value or negative effects which are normally difficult to quantify.
Some of the above negative effects may become effective in the future and should, therefore, be properly
deferred in time. A discounted cash flow technique, where appropriate, may be resorted to.
It is suggested that valuers should not estimate environmental clean-up costs. These figures should be
obtained from the appropriate environmental expert and a copy of that expert's report should be annexed
to the valuation report.

Comparative Technique
The unaffected approach discussed above would, where sale instances are available, involve the
comparative method of valuation. Such an approach is also advisable in the valuation of environmentally
contaminated property. There will be only very few instances, if any, where such direct comparison of
contaminated property can be made for obvious reasons. However, efforts should be made to establish
such data if it is available. Valuers are advised to ensure that the properties being compared are truly
comparable in all aspects. Valuers should make their decisions and recommendations regarding
comparability of contaminated properties only after having previously studied an environment expert's
reports both for the subject property and for any properties used as comparable. This is an important
aspect of comparison as various environmental factors have to be taken into consideration.
It may sometimes happen that remediation cost may exceed the unaffected market value of the land. This
would imply a negative land value, at least theoretically. However, the normal practice is to adopt a figure
of no less than zero value. On that basis a valuer could advise a nominal or 'nil' value and discuss details
regarding additional liabilities or clean-up costs required for the land. It would be advisable for valuers to
consider valuation on the basis of the highest and the best use in these cases.
In normal valuation practice, the client's point of view is important. For example, mortgagees can be
worried even at the prospect of a very small contamination problem and should be informed by the valuer
even of the slightest suspicion of contamination of the property under valuation. Similarly, in valuing for a
purchaser, the valuer should inform the intending purchaser of any possible contamination problem.

ENVIRONMENTAL LEGISLATION
In the decade since the Bhopal disaster, there have been some new legislative enactments and some
changes in the existing acts. Briefly, these are as follows:

LEGISLATION
1. Important Legislation
– The Environment (Protection) Act, 1986
– The Environment (protection) Rules, 1986
– These are fresh enactments which seek to present and control various types of environmental
pollution.
2. Legislation on Pollution Control
– The Water (Prevention and Control of Pollution) Act, 1974
– The Water (Prevention and Control of Pollution) Rules, 1975
– The Air (Prevention and Control of Pollution) Act, 1981
– The Air (Prevention and Control of Pollution) Rules, 1982
– Hazardous Wastes (Management and Handling) Amendment Rules, 1999
– Prevention & Control of Pollution (Uniform Consent Procedure) Rules, 1999.
3. Other Regulatory Legislation
– The Factories Amendment Act, 1987, whereby the Factories Act, 1948 underwent
restructuring in its safety and health provisions.
– Manufacture, Storage and Import of Hazardous Chemicals (Amendment) Rules, 1999.
– Hazardous Micro Organisms & Genetically Modified Organisms (Manufacture, Use,
– Import, Export and Storage) Rules, 1999.
– Bio-Medical Wastes (Management and Handling) Rules, 1995
– Chemical Accidents (Emergency Planning, Preparedness and Response) Rules, 1996.
– The Central Motor Vehicle (Amendment) Rules, 1993.
4. Other Related Legislation
– The Water (Prevention and Control of Pollution) Cess Act, 1977
– The Water (Prevention and Control of Pollution) Cess Rules, 1978
– The Public Liability Insurance Act, 1991
– The Public Liability Insurance Rules, 1991
– National Environment Tribunal Act, 1995
– National Environment Appellate Authority (Appeal) Act, 1997
– National Environment Appellate Authority (Appeal) Rules, 1997
GUIDELINES FOR INDUSTRIES
1. The industrial site distance should be at least;
a) 25 kms. from ecologically sensitive area viz.
– Religious and Historic Places, Archeological Monuments (e.g. identified zone around Taj Mahal);
Scenic Area, Hill Resorts, Beach Resorts, Health Resorts.
– Coastal Area rich in Coral, Mangroves, Breeding Ground of Specific Species, Gulf Areas.
– Estuaries rich in Mangroves, Breeding Ground of Specific Species, Natural Lakes,Swamps.
– Biosphere Reserves, National Parks and Sanctuaries.
– Seismic Zone, Areas of Scientific and Geological Interest, Tribal Settlements.
– Defense areas of security importance and sensitive to pollution, Border Areas
(international) and Air Ports.
– Projected growth boundary of a major settlement (3,00,000 population).
b) 1/2 km. from-
– High Tide Line in coastal area.
– Flood plain or modified flood plain affected by dam in the upstream or by flood
control system.
– Highway and railway.
2. No forest land shall be converted into non-forest activity for the sustenance of the industry. No prime
agricultural land shall be converted into industrial site.
3. Land acquired shall be sufficiently large to provide space for appropriate treatment of waste water still
left for treatment after maximum possible reuse and recycle.
4. The green belt shall be 0.50 km wide around the battery limit of the Industry and for industry having
odor problem it shall be 1 km wide. The green belt between two adjoining large scale industries shall
be 1 km.
5. There must be physiographic barrier between the industry and the township.

ENVIRONMENT CLEARANCE NOTIFICATIONS


1. EIA Notification: [So O. 60(E)]
It imposes restrictions and prohibitions (for a list of 29 industries given in Schedule - I) on the
expansion and modernization of any activity or new projects being undertaken in any part of India;
unless Environmental Clearance has been accorded as per the procedure specified in the notification.
Subsequent amendments in the notification are:
– S. O. 230(E) dated 17.03.1994 & S. O. 356(E) dated 04.05.1994: Original Notification.
– S. O. 18(E) dated 10.04.1997: Introducing public hearing as a prerequisite. Procedure
for public hearing is also prescribed.
– S. O. 3l9(E) dated 10.04.1997: Delegating powers to States to grant environment
clearance for Thermal Power Plants.

2. Notified Areas:
Projects in notified/restricted areas need to obtain environment clearance from the Central
Government irrespective of list under Schedule - I of the EIA Notification.
– S. O. No. 136(E): Antop Hill in Mumbai
– S. O. No. 4l6(E): Dahanu Taluka, District Thane
– S. O. No. 20(E): MurudJanjira Area, Dist. Raigadh
– S. O. No. ILPIl087/2477/IND-2:Wardha, Bhatsa river basins
– S. O. No. 102(E): Doon Valley in Utter Pradesh
– S. O. No. 319(E): Specified area of Aravalli Range

3. Requirement of Site Clearance:


Following site specific projects will also have to obtain separate site clearance from the Central
Government.
– Mining Projects
– Pit Hear Thermal Power Stations
– River Valley Projects including hydel power, major irrigation and their combination
including flood control.
– Tourism Projects between 200-500 meters of High Tide Line and at location with an elevation of
more than 1000 meters.
– Ports and Harbours (excluding minor ports)
– Prospecting and exploring of major minerals (above 500 Ha.)
– Projects where support from the Central Government/International Agencies IS
envisaged.
– Project requiring industrial licenses.

INDIAN FOREST ACT 1927


The Indian Forest Act, 1927 was largely based on previous Indian Forest Acts implemented under
the British. The most famous one was the Indian Forest Act of 1878. Both the 1878 act and the 1927 one
sought to consolidate and reserve the areas having forest cover, or significant wildlife, to regulate
movement and transit of forest produce, and duty leviable on timber and other forest produce. It also
defines the procedure to be followed for declaring an area to be a Reserved Forest, a Protected Forest or a
Village Forest. It defines what is a forest offence, what are the acts prohibited inside a Reserved Forest, and
penalties leviable on violation of the provisions of the Act.
The Indian Forests Act of 1865 extended the British Colonial claims over forests in India. The 1865 act was
a precursor to the Forest Act of 1878, which truncated the centuries-old traditional use by communities of
their forests and secured the colonial governments control over the forestry. The act of 1865 empowered
the British government to declare any land covered with trees as a government forest and make rules to
manage it.

Reserve Forest
Reserved Forest is an area mass of land duly notified under the provisions of India Forest Act or the State
Forest Acts having full degree of protection. In Reserved Forests all activities are prohibited unless
permitted. Reserved Forest is notified under section 20 of the Indian Forest Act, 1927 [Act 16 of 1927] or
under the reservation provisions of the Forest acts of the [State Governments of the Indian Union]. The
manner in which a [Reserved Forest], shortly written as RF, has to be constituted is described in section 3
to 20 of the Act. It is within power of a State Government to issue a preliminary notification under section
4 of the Act declaring that it has been decided to constitute such land, as specified in a Schedule with
details of its location, area and boundary description, into a Reserved Forest. such a notification also
appoints an officer of the State Government, normally the Deputy Commissioner of the concerned district,
as Forest Settlement Officer. The Forest Settlement Officer fixes a period not less than three months, to
hear the claims and objections of every person having or claiming any rights over the land which is so
notified to be reserved. He conducts inquiries into the claims of rights, and may reject or accept the same.
He is empowered even to acquire land over which right is claimed. For rights other than that of right of
way, right of pasture, right to forest produce, or right to a water course, the Forest Settlement Officer may
exclude such land in whole or in part, or come to an agreement with the owner for surrender of his rights,
or proceed to acquire such land in the manner prescribed under the Land Acquisition Act, 1894 [Act 1 of
1894]. Once the Forest Settlement Officer settles all the rights either by admitting them or rejecting them,
as per the provisions of the Act, and has heard appeals, if any, and settled the same, all the rights with the
said piece land [boundaries of which might have been altered or modified during the settlement process]
vest with the State Government. Thereafter, the State Government issues notification under section 20 of
the Indian Forest Act, 1927 declaring that piece of land to be a Reserved Forest.

LAWS RELATED TO INDUSTRIAL HEALTH & SAFETY


National Policy on Safety, Health and Environment at Workplace (NPSHEW) On the basis of Directive
Principles as well as international instruments the Government of India, Ministry of Labor& Employment,
had declared the National Policy on Safety, Health and Environment at Workplace (NPSHEW) on 20th
February, 2009 and the policy document has been posted in the website of the Ministry of Labor and
Employment and DGFASLI at www.labour.nic.in and www.dgfasli.nic.in respectively. The purpose of this
National Policy is to establish a preventive safety and health culture in the country through elimination of
the incidents of work-related injuries, diseases, fatalities, disasters and to enhance the well-being of
employees in all the sectors of economic activity in the country.
The salient features of the Policy are as below: -
1. It recognizes safe and healthy working environment as a fundamental human right.
2. It aims at enhancing the well-being of the employees and the society at large by eliminating work
related injuries, diseases, etc.
3. It enumerates the goals to be achieved and brings into focus the objective of continuous reduction in
the incidence of work related injurious and diseases.

Major OSH Laws & Regulations


On the basis of these Directive Principles and international instruments, the Government of India declares
its policy, priorities, strategies and purposes through the exercise of its power. The Government of India
has enacted the statutes relating to Occupational Safety and Health (OSH) at workplaces. At present,
comprehensive safety and health statutes for regulating Occupational Safety and Health at workplaces
mainly exist in respect of the four sectors namely, manufacturing, mining, ports, and construction.
There are four main legislations that cover Occupational Safety and Health at workplace.
1. The Factories Act, 1948 , covering factories wherein the enforcement of safety at workplace is by the
Chief Inspector of Factories in the respective states,
2. The Mines Act, 1952 and Mines Rules, 1955 for mining industry where the enforcement is by the
Directorate General of Mines Safety (DGMS) under Ministry of Labour&Employment , Government of
India,
3. The Dock Workers (Safety, Health and Welfare) Act, 1986 followed by notification of the Dock
Workers (Safety, Health and Welfare) Regulations, 1990 dealing with the major ports of India and the
enforcement is by the Directorate General of Factory Advice Service &Labour Institutes (DGFASLI),
under Ministry of Labour&Employment, Government of India, and
4. The Building & Other Construction Workers (Regulations of Employment and Conditions of Service)
Act, 1996, covering construction workers at construction sites wherein the enforcement is by the
Directorate General Labour Welfare in the central sphere and by the Labour Commissioners/Factory
Inspectorates in the States/UTs.
REGULATORY AUTHORITIES
The key regulatory authorities are the:
– Ministry of Environment, Forest and Climate Change.
– Central Pollution Control Board.
– State Pollution Control Boards.

THE WATER (PREVENTIONS CONTROL OF POLLUTION) ACT 1974, (AMENDED


1988)
An Act to provide for the prevention and control of water pollution and the maintaining or restoring of
wholesomeness of water, for the establishment, with a view to carrying out the purposes aforesaid, of
Boards for the prevention and control of water pollution, for conferring on and assigning to such Boards
powers and functions relating thereto and for matters connected therewith. Plus The Water (Prevention &
Control of Pollution) Rules, 1975.

THE WATER (PREVENTION & CONTROL OF POLLUTION) CESS ACT, 1977


(AMENDED 2003)
An Act to provide for the levy and collection of a cess on water consumed by persons carrying on certain
industries and by local authorities, with a view to augment the resources of the Central Board and the
State Boards for the prevention and control of water pollution constituted under the Water (Prevention
and Control of Pollution) Act, 1974 and the Water (Prevention & Control of Pollution) Cess Rules, 1978.

THE AIR (PREVENTION & CONTROL OF POLLUTION) ACT, 1981 (AMENDED


1987)
An Act to provide for the prevention, control and abatement of air pollution, for the establishment , with a
view to carrying out the aforesaid purposes, of Boards, for conferring on and assigning to such Boards
powers and functions relating thereto and for matters connected therewith the Air (Preventions & Control
of Pollution) Rules, 1982.

THE ENVIRONMENT (PROTECTION) ACT, 1986 (AMENDED 1991)


An Act to provide for the protection and improvement of environment and matters connected therewith.
There are also various Rules which are listed below
1. The Environment (Protection) Rules, 1986 (amended 2010)
2. The Manufacture Storage and Import of Hazardous Chemicals Rules, 1989 (amended 2000)
3. The Rules for Manufacture, Use, Import, Export and Storage of Hazardous Micro Organisms,
Genetically Engineered Organisms or Cells, 1989 (amended 2010)
4. The Hazardous and Other Wastes (Management and Trans boundary Movement) Rules, 2016
[Suppressed the Hazardous Wastes (Management, Handling and Trans boundary Movement) Rules,
2008]
5. The Chemical Accidents (Emergency Planning, Preparedness and Response) Rules, 1996
6. The Bio-Medical Waste Management Rules, 2016 [Suppressed the Bio-Medical Waste (Management &
Handling) Rules, 1998]
7. The Recycled Plastics Manufacture and Usage Rules, 1999 (amended 2003)
8. The Noise Pollution (Regulation & Control) Rules, 2000 (amended 2010)
9. The Ozone Depleting Substances (Regulation & Control) Rules, 2000 (amended 2007)
10. The Solid Waste Management Rules, 2016 [Suppressed the Municipal Solid Wastes (Management &
Handling) Rules, 2000]
11. The Construction and Demolition Waste Management Rules, 2016 [Suppressed the Municipal Solid
Wastes (Management & Handling) Rules, 2000]
12. The Batteries (Management & Handling) Rules, 2001 (amended 2010)
13. The Environmental Impact Assessment Notification, 2006(amended 2013)
14. The Plastic Waste Management Rules, 2016 [Suppressed the Plastic waste Management and Handling
Rules, 2011]
15. The E-Waste (Management) Rules, 2016 w.e.f 1st October, 2016 [Suppressed the E-waste Management
and Handling Rules, 2011]
16. The Coastal Regulation Zone Notification, 2011.

MULTIPLE CHOICE QUESTIONS


1) Which of the following environmental law empowers the central and state pollution control boards
to meet with grave emergencies of air pollution?
a) Air (Prevention and Control of Pollution) Amendment Act
b) B. Environment (Protection) Act
c) C. Air (Prevention and Control of Pollution) Rules
d) D. Objective of Hazardous Waste (Management and Handling) Rules

2) Which of the following is the oldest law in India?


a) Wildlife Protection Act
b) River Board Act
c) Factories Act
d) Easement Act

3) Radioactive waste management in our country is governed under:


a) Hazardous Waste (Management, Handling and Trans-boundary Movement)
b) Atomic Energy Act, 1962
c) Environment (Protection) Act, 1986
d) Biomedical Waste (Management & Handling) Rules 1998

4) Which one of the following is a non-formal environment education and awareness programme?
a) Environmental appreciation courses.
b) National Environment Awareness Campaign.
c) Environmental Education in school system.
d) Environmental Management Business Studies.

5) According to Recycled Plastics (Manufacture and Usage) Rules 1999, the minimum thickness of
carry bags shall not be less than
a) 10 microns
b) 20 microns
c) 30 microns
d) 50 microns
PROFESSIONAL ETHICHS & STANDARDS
MODEL CODE OF CONDUCT AS NOTIFIED BY
MCA UNDER THE COMPANIES (REGISTERED
VALUERS AND VALUATION) RULES 2017

CODE OF CONDUCT FOR REGISTERED VALUERS


(See clause (g) of rule 7 and clause (d) of sub-rule (2) of rule 12)

1. PRIMARY DUTY AND OBLIGATION


– It is valuer's obligation to develop and describe the appropriate type of value or estimated
cost.
– It is valuer's obligation to develop appropriate and credible conclusion and opinions with a
high degree of accuracy depending on the purpose of the valuation assignment.
– A valuer shall abstain from deliberately giving false and misleading opinion and conclusion.
– A valuer shall attain high degree of competency in his work through education, training, study,
practice, and experience.

2. INTEGRITY AND FAIRNESS


– A valuer shall, in the conduct of his/its business, follow high standards of integrity and fairness
in all his/its dealings with his/its clients and other valuers.
– A valuer shall maintain integrity by being honest, straight forward, and forthright in all
professional relationships.
– A valuer shall endeavour to ensure that lie/it provides true and adequate information and
shall not misrepresent any facts or situations.
– A valuer shall refrain from being involved in any action that would bring disrepute to the
profession.
– A valuer shall keep public interest foremost while delivering his services.
3. PROFESSIONAL COMPETENCE AND DUE CARE
– A valuer shall render at all times high standards of service, exercise due diligence, ensure
proper care and exercise independent professional judgment.
– A valuer shall carry out professional services in accordance with the relevant technical and
professional standards that may be specified from time to time.
– A valuer shall continuously maintain professional knowledge and skill to provide competent
professional service based on up-to-date developments in practice, prevailing
regulations/guidelines and techniques.
– In the preparation of a valuation report, the valuer shall not disclaim liability for his/its
expertise or deny his/its duty of cart, except to the extent that the assumptions are based on
statements of fact provided by the company or its auditors or consultants or information
available in public domain and not generated by the valuer.
– A valuer shall not carry out any instruction of the client insofar as they are incompatible with
the requirements of integrity, objectivity and independence.
– A valuer shall clearly state to his client the services that he would be competent to provide
and the services for which he would be relying on other valuers or professionals or for which
the client can have a separate arrangement with other valuers.

4. INDEPENDENCE AND DISCLOSURE OF INTEREST


– A valuer shall act with objectivity in his/its professional dealings by ensuring that his/its
decisions are made without the presence of any bias, conflict of interest, coercion, or undue
influence of any party, whether directly connected to the valuation assignment or not.
– A valuer shall not take up an assignment if he/it or any of his/its relatives or associates is not
independent in terms of association to the company.
– A valuer shall maintain complete independence in his/its professional relationships and shall
conduct the valuation independent of external influences.
– A valuer shall wherever necessary disclose to the clients, possible sources of conflicts of duties
and interests, while providing unbiased services.
– A valuer shall not deal in securities of any subject company after any time when he/it first
becomes aware of the possibility of his/its association with the valuation, and in accordance
with the Securities and Exchange Board of India (Prohibition of Insider Trading) Regulations,
2015 or till the time the valuation report becomes public, whichever is earlier.
– A valuer shall not indulge in mandate snatching "or offering "convenience valuations" in order
to cater to a company or client's needs.
– As an independent valuer, the valuer shall not charge success fee.
– In any fairness opinion or independent expert opinion submitted by a valuer, if there has been
a prior engagement in an unconnected transaction, the valuer shall declare the association
with the company during the last five years.
5. CONFIDENTIALITY
– A valuer shall not use or divulge to other clients or any other party any confidential
information about the subject company, which has come to his/its knowledge without proper
and specific authority or unless there is a legal or professional right or duty to disclose.

6. VALUATION REPORTS
– A valuation report should convey every analysis, conclusion and opinion reached in a manner
which is clear and not misleading.
– The valuation investigations, methods, approaches, bases, premises and reporting should be
in compliance with the valuation standards in force.

OBJECTIVITY
1. The principle of objectivity imposes an obligation on the valuer not to compromise his
professional or business judgement because of bias, conflict of interest or undue influence of
others.
2. Some threats to objectivity arc incapable of avoidance or mitigation and where this is the case the
valuer should decline the assignment. However some potential threats to objectivity may be
either eliminated or effectively mitigated by safeguards. These safeguards can include appropriate
disclosure of the threat to the relevant parties and obtaining their consent to proceed with the
valuation assignment.

7. INFORMATION MANAGEMENT
– A valuer shall ensure that he/ it maintains written contemporaneous records for any decision
taken, the reasons for taking the decision, and the information and evidence in support of
such decision. This shall be maintained so as to sufficiently enable a reasonable person to take
a view on the appropriateness of his/its decisions and actions.
– A valuer shall appear, co-operate and be available for inspections and investigations carried
out by the authority, any person authorised by the authority, the registered valuers
organisation with which he/it is registered or any other statutory regulatory body.
– A valuer shall provide all information and records as may be required by the authority, the
Tribunal, Appellate Tribunal, the registered valuers organisation with which he/it is registered,
or any other statutory regulatory body.
– 32. A valuer while respecting the confidentiality of information acquired during the course of
performing professional services, shall maintain proper working papers for a period of three
years or such longer period as required in its contract for a specific valuation, for production
before a regulatory authority or for a peer review. In the event of apending case before the
Tribunal or Appellate 'tribunal, the record shall be maintained till the disposal of the ease.

8. GIFTS AND HOSPITALITY


– A valuer or his/its relative shall not accept gifts or hospitality which undermines or affects his
independence as a valuer.
– Explanation. - For the purposes of this code the term 'relative' shall have the same meaning as
defined in clause (77) of Section 2 of the Companies Act, 2013 (18 of 2013).
– A valuer shall not offer gifts or hospitality or a financial or any other advantage to a public
servant or any other person with a view to obtain or retain work for himself/itself, or to obtain
or retain an advantage in the conduct of profession for himself/ itself.

9. REMUNERATION AND COSTS


– A valuer shall provide services for remuneration which is charged in a transparent manner, is a
reasonable reflection of the work necessarily and properly undertaken, and is not inconsistent
with the applicable rules.
– A valuer shall not accept any fees or charges other than those which are disclosed in a written
contract with the person to whom he would be rendering service.

10. OCCUPATION, EMPLOYABILITY AND RESTRICTIONS


– A valuer shall refrain from accepting too many assignments, if he/it is unlikely to be able to
devote adequate time to each of his/ its assignments.
– A valuer shall not conduct business which in the opinion of the authority or the registered
valuer organisation discredits the profession.

11. PROFESSIONAL BEHAVIOUR


– A valuer shall accept a responsibility to act in the public interest. A professional valuer's duty is
not limited to meet the needs of a particular client or employer. There is also a need to
consider if professional decisions have a wider impact on unidentified third parties. While the
client's needs are normally paramount, a professional valuer shall avoid knowingly accepting
any instruction that appears to be prejudicial to the interests of the wider public and which
could discredit their own reputation and that of the profession generally.
– Valuer shall ensure that the service provided is in accordance with all legal, technical and
professional standards that are applicable to either the subject of the valuation, the purpose
or the valuation or both.
– A valuer should be honest and truthful and should not
• Make exaggerated claims for the services he is able to offer, the qualifications he
possesses, or experience he have gained; or
• Make disparaging references or unsubstantiated comparisons to the work of others.

ETHICAL CONSIDERATION UNDER TERMS OF


ENGAGEMENTS
1. Avoid conflicts of interest
2. Maintain your client's confidentiality
3. Avoid contributing to the perpetration of unlawful acts
4. Ensure your client is well informed; give comprehensive advice.
5. Ensure your client understands the advice, and has capacity to act
6. Be respectful. With older clients, beware of ageism and making an assumption that, because the
client is old and perhaps frail, they are not capable of making a valid decision.
7. Your client's best interests come first

Provides guidance on the appropriate response for professional valuers who come across suspected
unlawful activity on the part of their client or employer.

Where illegal activity is suspected, the valuer is advised to:


1. explain to the client or employer the potential illegality of the arrangement and get them to stop
that activity
2. suggest alternative, legal ways in which the client’s needs might be met
3. disclose the activity if there is a legal obligation to do so
4. if the client or employer does not change the suspect behaviour, then resign from the
engagement or employment
5. if disclosure is not mandatory, the valuer should consider whether, in their professional
assessment, disclosure is warranted in the public interest. If their professional assessment is that
disclosure is in the public interest, they are strongly advised to seek legal advice before
proceeding.

Cases of financial abuse of older people most usually involve "improper" conduct and not criminal
conduct. During the course of the interview with the older client, however, the practitioner may
suspect that unlawful acts have been committed, such as theft, or physical abuse. Whether an act is
unlawful is often difficult to determine and is not the role of the valuer to so determine. It is
important however that the suspicions be followed up.
1. The practitioner should explain to the client that the suspected unlawful act is no less unlawful
because it has been committed by a family member or friend, and recommend to the client that
the matter be reported to police, and could offer to call the local statewide elder abuse service
with the older person, to get their input.
2. The valuer may themselves refer to external agencies for general advice on how to advise a client
in this situation, while respecting confidentiality.
3. If the valuer suspects something illegal or dangerous, he or she should express that to the client,
minute it in the record of meeting and confirm it to the client in writing.
4. If the client is the perpetrator, the valuer should explain why they cannot assist the client any
further unless the potentially unlawful acts are rectified and discontinued (see below for
suggested phrases).
5. All practitioners will have a Letter of Engagement and it is recommended that it limits
confidentiality in matters where there is a clear breach of the law. However before relying on any
such clause, the valuer should first seek legal advice.
6. If the valuer considers that an unlawful act has been committed and is considering reporting the
matter to an authority or body, thereby potentially overriding confidentiality, it is strongly
recommended that he or she first seeks legal advice on the matter.

Suggested statements when bringing a suspected unlawful act to the attention of the client who may
be the perpetrator
1. "It appears that this act may be unlawful."
2. "The consequence of this act, if unlawful, are …"
3. "If I am involved, the consequences for me may be highly significant."
4. "It may be highly detrimental for me if I were found to be involved in any way."
5. "Here are some options for going about this which are not unlawful ..."

The conversation with the client, whether abused or perpetrator, should be confirmed in writing.
If the practitioner is in doubt as to how to proceed, it is recommended that the statewide elder abuse
services be contacted for general advice, or alternatively, contact Ethical.
Possible criminal offences
1. Duress
2. Fraud
3. Obtaining financial advantage by deception
4. Forgery
5. Assault or the threat of assault

PROFESSIONAL CONSIDERATIONS
Research has shown that many professionals, when coming across a situation where financial abuse is
suspected, take no action. The reasons for this sort of response appear to be:
1. perceived confidentiality issues
2. belief that their actions would not improve the situation.

Additional reasons for lack of action may be:


1. uncertainty that what is being observed is abuse
2. uncertainty as to what to do about it
3. concern about potential conflict of interests
4. lack of willingness to take action when it is the client who may be the perpetrator
5. a desire not to get involved.
Professionalism and the Code of Ethics
Valuer are required to be objective and not allow bias, conflicts of interest, or undue influence of
others to override professional or business judgements. They must also exercise professional
competence and due care.
VALUER requires that its members, as trusted professionals and as members of a highly respected
professional association, respond professionally and with integrity. Where financial abuse is
suspected, the VALUER can use this toolkit to assist them to respond in a way that meets their
professional and ethical obligations, that is, to advise the older person that they may be experiencing
abuse, provide advice on how to avoid or remediate abuse, and to be cautious in ensuring that they
do nothing to contribute to the abuse.
Confidentiality
Much information is provided to valuers which is personal and confidential to the client.
Confidentiality must be respected. It is recommended, however, that your terms and conditions of
engagement specifically exclude from promises to maintain confidentiality, any events where
contravention of the law is evident.
Where information has been provided which indicates potential financial abuse, but does not involve
clear breaches of the law, the valuer must bring the potential abuse to the attention of the client and
seek their instructions. If the client is the older person, and lack of capacity is suspected, advice
should be sought from one of the statewide elder abuse services.
Client confidentiality must be respected at all times, except where clear breaches of the law are
involved. However, advice concerning potential abuse should be provided to the client in writing, and
the valuer should not do anything to facilitate the arrangements, unless requested to do so in writing
by the older person concerned, and only then when that older person demonstrates capacity and has
been fully advised of the potentially negative outcomes.
To maintain confidentiality, and to ensure that you get the whole picture, interviews with the older
person should be conducted without the presence of other interested parties. Where the practitioner
considers that an unlawful act may have been committed, and is considering reporting the matter to
the police, it is recommended that they first seek legal advice before doing so.
Conflicts of interest
The VALUER must not act for both clients where there is an actual or potential conflict of interest, for
example, between an older person and their adult child.
Where the client is the adult child (or the potential abuser):
– the VALUER should recommend that the older person get advice from another qualified party
– that other party may be a different partner within the same firm, or from a different firm
altogether
– the VALUER may assist the older person by recommending alternative qualified professionals.

Where the client is the older person:

– the VALUER should refer the adult child (or the potential abuser) to another qualified
professional
– you need to be absolutely independent of the party with the influence or control over the
older person
– your advice and the subsequent instructions given to the VALUER by the client should be
confirmed in writing, and sent to the older person, not to their children or other family
members, not even if those family members have an enduring power of attorney.

Loss of business
Your client may not welcome you bringing the potential for financial abuse to their attention. In many
cases, it is the client’s children who are the potential abusers and the client, if the older person, may
not accept that there is cause for concern. After all, you are advising them to be wary of the potential
(or suspected) actions of their own children.
If it is the potential abuser who is your client, providing advice brings with it additional challenges.
The practitioner may feel that their advice will be unwelcome, will probably be ignored and may result
in the client going elsewhere for more palatable advice. The temptation to ignore the situation, to
turn a blind eye can be substantial. The practitioner has a clear duty of care to their client, and has an
ethical and professional duty to address concerns of potential abuse.
This toolkit provides some suggestions as to how to go about providing advice in difficult situations
involving potential financial abuse which, when used with skill, may strengthen the relationship with
the client. At a minimum however, potential loss of business is not a valid reason for turning a blind
eye.
Other considerations
Financial abuse of an older person can be a confronting and difficult challenge for the professional.
– Give comprehensive and independent advice so that the client can make their own informed
decision
– Ensure your client has the capacity to make the decision and is not acting under undue
influence
– Ensure your client receives and understands your advice
– Be aware of the impact of cultural diversity and any language barrier
– Intervene, but be careful that your intervention is respectful and does not make matters worse
– Be true to your professional integrity and your moral responsibility
– Seek assistance and advice from your colleagues and from statewide elder abuse services
whenever you feel ill equipped to proceed
When recommending a course of action, clarity of engagement and of the costs associated with
proposed action are important in reducing future dispute or loss of the client. This is best achieved
through an engagement letter.
Will my actions really improve outcomes?
Adults of any age have the right and, unless proven to the contrary, are presumed to have the
capacity to make their own decisions, even bad ones that ignore, or open up potential for, financial
abuse. It is not the adviser's role to prevent the abuse. Rather it is the adviser's role to ensure that the
client is fully aware of the issues and of their rights and remedies, and that they knowingly and
willingly accept the risks of the abuse, or with the adviser's help, take steps to mitigate them.
A VALUER who, faced with a situation of actual or potential financial abuse, fulfils that role, will be
making a significant contribution to the improvement of outcomes for older people.
LAW RELATED TO REAL ESTATE

LAND ACQUISITION

The Right to Fair Compensation and Transparency in the Land Acquisition, Rehabilitation and
Resettlement Act, 2013. The Right to Fair Compensation and Transparency in Land Acquisition,
Rehabilitation and Resettlement Act, 2013:

1) Application of Act(S.2);
2) Definition (S.3);

3) Notification and Acquisition (ChapterIV);


4) Rehabilitation and Resettlement Award (ChapterV);

5) Acquisition of part of house or building(S.94);

6) Exemption from Income Tax, stamp duty and fees(S.96).

Eminent Domain is the right asserted by the Sovereign State to do anything in public interest
and it is extended to the Sovereign State's right to acquire private property for public purpose.
The Constitutional Amendment in 1978 took away the fundamental right to property when it
repealed Art 19(1)(f) (i.e. the right to acquire, hold and dispose property) and Art 31 (which
earlier regulated compulsory acquisition of property and restricted acquisition only to public
purpose). An Article300A was inserted in 1978 about acquisition of property only under the
authority of law - thus right to property became a constitutional right.

Art 31A deals with exempting laws that provide for acquiring estates from being deemed void
on grounds of violating Art 14 or Art 19. Art 31A proviso dealt with land which is part of an
estate and is within personal cultivation of and within ceiling limit applicable to the person
along with building / structures on it cannot be acquired except on payment of compensation
not be less than the market value.
Sri RadhayShyam (Dead) Through L.Rs. vs. State of U.P. & others CA 3261 of 2011 (Arising
out of Special Leave Petition (C) No.601 of 2009) In this a large chunk of land measuring
4.3840 hectares was not acquired apparently because the same belong to an ex-member of
the legislative assembly while land of poorer farmers were acquired and justified as land of
neighboring village was also acquired under provisions of Land Acquisition Act, 1894: "while
examining the land owner's challenge to the acquisition of land in a petition filed under Article
226 of the Constitution, the High Court should not adopt a pedantic approach, and decide the
matter keeping in view the constitutional goals of social and economic justice and the fact
that even though the right to property is no longer a fundamental right, the same continues
to be an important constitutional right and in terms of Article 300-A, no person can be
deprived of his property except by authority of law ...the High Court should insist upon filing
of reply affidavit by the respondents and production of the relevant records and carefully
scrutinize the same before pronouncing upon legality of the impugned notification/action
because a negative result without examining the relevant records to find out whether the
competent authority had formed a bona fide opinion on the issue of invoking the urgency
provision and excluding the application of Section 5-A is likely to make the land owner a
landless poor and force him to migrate to the nearby city only to live in a slum.

If the acquisition is intended to benefit private person(s) and the provisions contained in
Section 17(1) and/or 17(4) are invoked, then scrutiny of the justification put forward by the
State should be more rigorous in cases involving the challenge to the acquisition of land, the
pleadings should be liberally construed and relief should not be denied to the petitioner by
applying the technical rules of procedure embodied in the Code of Civil Procedure and other
procedural laws. The Court ordered payment of 5 lakh compensation towards litigation cost
and required reconsideration of the case.

The earlier Land Acquisition Act 1894 did not have any concept of consultation, all acquisition
was forced. It had provisions of an urgency clause but it was not defined. The concept of
public purpose was also not defined. There was little scope for appeal or a hearing. Resorting
to the writ jurisdiction of the High Court was the only option. There was no designated
process for computing compensation. There was nothing included in rehabilitation in that Act.

The Right to Fair Compensation and Transparency in Land Acquisition, Rehabilitation and
Resettlement Act (RFCTLARR), 2013 was notified in the gazette on 27th Sept. 2013. Prior to
this date the earlier Land Acquisition Act of 1894 passed by British Parliament applied on the
matter of land acquisition. However, on the Rehabilitation & Resettlement of families affected
and displaced as a result of land acquisition this is the first pan-India.
This Act came into force on 01st January, 2014 by repealing the Land Acquisition Act, 1894.
This Act allows States to enact laws to increase or add to the entitlements enumerated under
this Act. The Department of Land Resources (DoLR), Ministry of Rural Development,
Government of India is administering the Act. MORD notified the Right to Fair Compensation
and Transparency in Land Acquisition, Rehabilitation and Resettlement (Compensation,
Rehabilitation and Resettlement and Development Plan) Rules, 2015 on 18th December,2015.

The Amendment Bill was introduced in the Lok Sabha by the Minister for Rural Development
on February 24, 2015. The Bill amends the Right to Fair Compensation and Transparency in
Land Acquisition, Rehabilitation and Resettlement Act, 2013 (LARR Act, 2013). The Bill replaces
the Right to Fair Compensation and Transparency in Land Acquisition, Rehabilitation, and
Resettlement (Amendment) Ordinance, 2014.

This bill not only provides for land acquisition but also for rehabilitation and resettlement
(R&R). The provisions of this bill shall be applicable where the government acquires land
either for its own use or for the use of any private company for public purposes. The earlier
Act only allowed land acquisition by the government for government-owned companies as
well as schemes run by societies/authorities/co-operative societies while the new Bill allows
acquisition for a public purpose by privet companies and Public Private Partnership (PPPs)
apart from government-controlled organizations.

According to this bill, the private companies are entitled to provide rehabilitation and
resettlement if they acquire land through private negotiations. R&R shall be equal to or more
than 100 acres in rural areas and 50 acres in urban areas. As per this bill, the term
―public purpose‖ includes:
1) National security and strategic defense purposes
2) Roads, railways, and ports built by government and PSEs
3) Project affected people
4) Planned development or improvement of villages
5) Residential purposes for the poor
6) Government projects benefiting public.

This Bill creates five special categories of land use which are exempted from certain provisions.
This includes:
1) Defense
2) Rural infrastructure
3) Affordable housing
4) Industrial corridors
5) Infrastructure projects including PPPprojects.
However, under the LARR Act, 2013 land could be acquired for all these purposes only when
80% of the project affected people give their consent. But this bill exempts the above-
mentioned land uses from this consent clause. This Bill stipulates the appointment of an R&R
committee to review R&R progress in the case where land acquired is 100 acres or more for
the public purpose.

APPLICATION OF ACT

As per Sec 2, this act will apply when:

a) Government acquires land for its own use, or to hold and control.

b) Where Government acquires land for private companies or for specified public
purpose - require prior consent of 80 per cent of the affected families.

c) Government acquires land for Public Private Partnership Projects - consent of 70


percent of the affected families is obtained, MORD Notification dated 9thFebruary
2016, states "the limit of extent of land referred to in sub-section (1) of section 46 of
the said Act shall be 20 (twenty) hectares in urban areas and 40 (forty) hectares in rural
areas"

The rules state that rehabilitation and resettlement under the Act would apply where a private
company purchases land equal to or more than 20 hectares (50 acres) in urban areas and forty
hectares (100 acres) in rural areas.

As per the 2013 Act, in the Scheduled Areas (tribal areas to which the fifth schedule of the
constitution applies, no land acquisition can happen in contravention of any law relating to
land transfer, prevailing in such Scheduled Areas.

In fact any order or judgment of a High Court which has become final would also need to be
adhered to. As per this provision in scheduled areas no land acquisition under even Land
Acquisition Act 2013 can happen in contravention of Panchayat (Extension to the Scheduled
Areas) Act, 1996(PESA).

PESA was enacted to extend the Panchayat raj provisions contained in the Part IX of the
Constitution to the Scheduled Areas with exceptions and modification. PESA states in, "Sec 4(i)
"the Gram Sabha or the Panchayat at the appropriate level shall be consulted before making
the acquisition of land in the Scheduled Area as for development projects and before re-
settling or rehabilitating persons affected by such projects in the Scheduled Areas; the actual
planning and implementation of the projects in the Scheduled Areas shall be coordinated at
the State level.

Sec 4 (k) the recommendations of the Gram Sabha or the Panchayat at the appropriate level
shall be made mandatory prior to grant of prospecting license or mining lease for
minor minerals in the Scheduled Areas; Sec 4 (I) the prior recommendation of the Gram Sabha
or the Panchayat at the appropriate level shall be made mandatory for grant of concession for
the exploitation of minor minerals by auction"

Sec 4 (m) (iii) the power to prevent alienation of land in the Scheduled Areas and to take
appropriate action to restore any unlawfully alienated land of a Scheduled Tribe;The Scheduled
Areas have been designated by the President of India and are located in Andhra Pradesh,
Chhattisgarh, Gujarat, Himachal Pradesh, Jharkhand, Madhya Pradesh, Maharashtra, Odisha,
Rajasthan and Telangana.

State Areas

Andhra Pradesh Visakhapatnam, East and West Godavari, Adilabad, Srikakulam,


Vizianagaram, Mahboobnagar, Prakasam (only some mandals)Dumka,
(including Telangana)
Godda, Deogarh, Sahebgunj, Pakur, Ranchi,Singhbhum (East &West),
Jharkhand Gumla, Simdega, Lohardaga,parts of Palamu, Garwa)

Chhattisgarh Sarguja, Bastar, Raigad, Raipur, Rajnandgaon, Durg, Bilaspur,Shahdol,


Chindwada, Kanker

Himachal Pradesh Lahaul and Spitidistricts, Chamba district (Kinnaur,


PangitehsilandBharmoursub-tehsil)

Madhya Pradesh Jhabua, Mandla, Dhar, Khargone, Betul, Seoni, Balaghat,Morena, East
Nimar (khandwa), Sailana tehsil in Ratlam district,

Gujarat Parts of the districts of Surat, Bharauch, Dangs, Valsad,

Panchmahl, Sadodara, Sabarkanta

Maharashtra Parts of the districts of Thane, Nasik, Dhule, Ahmednagar,Pune, Nanded,


Amravati, Yavatmal, Gadchiroli, Chandrapur

Odisha Mayurbhanj, Sundargarh, Koraput (fully scheduled area in these

threedistricts), Raigada, Keonjhar, Sambalpur,

Boudhkondmals, Ganjam, Kalahandi, Bolangir, Balasor (parts ofthese


districts only)

Rajasthan Banswara, Dungarpur (which are fully tribal districts),


Udaipur,Chittaurgarh, Siroi (partly tribalareas)
DEFINITIONS

Some definitions from section 3 of the Act are enumerated below:

• "Displaced family" means any family, who on account of acquisition of land has to be
relocated and resettled from the affected area to the resettlementarea;

• "Family" includes a person, his or her spouse, minor children, minor brothers and
minor sisters dependent on him. An adult of either gender with or without spouse or
children or dependents shall be considered as a separate family. Widows, divorcees
and women deserted by families shall be considered separatefamilies;

• "Market value" means the value of land determined in accordance with section 26;

• "Agricultural land" means land used for raising nursery, cultivation of crops, trees,
horticulture, grass, garden produce, dairying, poultry farming, pisciculture, sericulture,
seed farming, breeding of livestock, medicinal herbs and also open land used for the
grazing ofcattle
• "Cost of acquisition" includes - solatium (payment to compensate injured feelings or
emotional pain), any enhanced compensation ordered by the Land Acquisition
Authority or the Court and interest, demurrage payable for damages caused in the
process of acquisition to the land and standing crops. It also includes the cost of
acquisition of such land and building for the settlement' of displaced or adversely
affected families and the cost of development of infrastructure and amenities at such
resettlement areas;
There are 13 other Act which deal with acquisition and are listed in the fourth schedule and
vide a departmental notification the provision the RFCTLARR Act, 2013 will become applicable
to acquisition under those Acts also. These are:
1) The Ancient Monuments and Archaeological Sites and Remains Act, 1958 (24 of 1958).

2) The Atomic Energy Act, 1962 (33 of1962).


3) The Damodar Valley Corporation Act, 1948 (14 of1948).

4) The Indian Tramways Act, 1886 (11 of1886).


5) The Land Acquisition (Mines) Act, 1885 (18 of1885).

6) The Metro Railways (Construction of Works) Act, 1978 (33 of1978).


7) The National Highways Act, 1956 (48 of1956).

8) The Petroleum and Minerals Pipelines (Acquisition of Right of User in Land) Act,1962 (50
of1962).

9) The Requisitioning and Acquisition of Immovable Property Act, 1952 (30 of1952).

10) The Resettlement of Displaced Persons (Land Acquisition) Act, 1948 (60 of1948).
11) The Coal Bearing Areas Acquisition and Development Act, 1957 (20 of1957).

12) The Electricity Act, 2003 (36 of2003).

13) The Railways Act, 1989 (24 of1989).

• "Affected family"includes

1) A family whose land or other immovable property has been acquired;


2) A family which does not own any land but whose primary source of livelihood stand
affected by the acquisition of land:

a. Are agricultural laborers,b.any kind of tenants


c. holding of usufruct right d.share-croppers
e.traditional artisans

3) People who may be working in the affected area for three years prior to the acquisition of
the land,

4) Family of gatherers of forest produce, hunters, fisher folk and boatmen - whose primary
source of livelihood for three years prior to the acquisition of the land is dependent on
forests or water bodies and is affected due to acquisition of land

5) Holders of Community forest rights recognized under the Scheduled Tribes and Other
Traditional Forest Dwellers (Recognition of Forest Rights) Act, 2006 due to acquisition of
land
6) A member of the family who has been assigned land by the State or the Central
Government under any scheme and such land is under acquisition;

7) A family residing on any land in the urban areas for preceding three years or more prior to
the acquisition of the land or whose primary source of livelihood for three years prior to
the acquisition of the land is affected by the acquisition of such land;

NOTIFICATION AND ACQUISITIONINCLUDING COMPUTATION OF COMPENSATION


AWARD (CH. IV)

Chapter IV (sections 11 to 31) of the Act deals with notification for acquisition, declaration and
also computation of Compensation Award. A preliminary notification of the proposed
acquisition under section 11(1) of the 2013 Act must be published in the following manner,
namely,

1) In the Official Gazette;

2) In two daily newspapers circulating in the locality of such area of which one shall be in the
regional language;
3) In the local language in the Panchayat, Municipality or Municipal Corporation, as the case
may be and in the offices of the District Collector, the Sub-divisional Magistrate and the
Tehsil;

4) Uploaded on the website of the appropriate Government;


5) In the affected areas, in such manner as may be prescribed (rules provide for
announcement by beating of drums)

The date of this notification shall be the date for determination of market value. The
notification must be issued in the form II annexed to the rules. It includes the following points:

• "Social Impact Assessment Study was carried out by Social Impact Assessment (SIA)
Unit and a report submitted / preliminary investigation was conducted by a team
constituted by Collector as laid down under rule 4. The summary of the Social Impact
Assessment Report/ preliminary investigation is as follows...". A copy of the report must
also be attached to this preliminary notification.

• The notification puts a bar on subsequent transfer or create encumbrance through


pledge and mortgage: "Under section 11(4) of the Act, no person shall make any
transaction or cause any transaction of land i.e. sale/purchase, etc., or create any
encumbrances on such land from the date of publication of such notification without
prior approval of the Collector."
• It must call for objections thus "Objections to the acquisition, if any, may be filed by the
person interested within 60 (sixty days) from the date of publication of this notification
as provided under section 15 of the Act before Collector."

• The concerned gram Sabha(s) at the village level, municipalities and the Autonomous
Councils in Sixth Schedule areas shall be informed of the contents of the notification in
a special meeting.
• No person shall make or cause any transaction or create any encumbrances on such
land from the date of publication of such notification till the completion of the process.
• Collector shall ensure updating of land records as prescribed within a period of two
months

• As per sec 12 of the Act, on publication of notice, in the presence of the property
owner it shall be lawful for designated officials to enter upon and survey and take
levels of any land in that area; to dig or bore into the sub-soil, set out boundaries of
the land proposed to be taken; placing marks and cutting trenches and where
otherwise the survey cannot be completed and the levels taken and the boundaries
and line marked, to cut down and clear away any part of any standing crop, fence or
jungle. Entry in absence of owner can be done only after noticing him 60 days prior to
entry. A seven days’ notice is to be given prior to entering dwellings or enclosed
garden under occupation. As per sec 13 any damage caused during such assessment
shall be compensated to the owner.
Social Impact Assessment is mandatory but its report has a shelf life of 12 months and must
be redone if the preliminary acquisition notification is not issued within such period. The
appropriate government after recording in writing the reason may extend the period of
12months.

A rehabilitation and resettlement scheme have to be drawn up and duly publicized for calling
for objections and for information. This scheme must take account census details and
following particulars of:

• Lands and immovable properties being acquired of each affected family;

• livelihoods lost in respect of land losers and landless whose livelihoods are primarily
dependent on the lands being acquired;

• A list of public utilities and Government buildings which are affected or likely to be
affected, where resettlement of affected families is involved;

• The amenities and infrastructural facilities which are affected or likely to be affected,
where resettlement of affected families is involved; and
• any common property resources being acquired

The plan must give particulars of the rehabilitation and resettlement entitlements of each land
owner and landless whose livelihoods are primarily dependent on the lands being acquired
along with a timeline of execution of the Scheme. Where resettlement of affected families is
involved - then a list of Government buildings to be provided in the Resettlement area and
the details public amenities and infrastructural facilities to be provided must be collected.

Within a period of twelve months from the date of the publication of preliminary notification,
a declaration stating the district or other territorial division in which the land is situated, the
purpose for which it is needed; its approximate area shall be made along with a declaration of
an area identified as the "resettlement area". Along with it the declaration the Collector shall
publish a summary of the Rehabilitation and Resettlement Scheme. Any period or periods
during which the proceedings for the acquisition of the land were held up on account of any
stay or injunction by the order of any Court shall be excluded from the period of 12 months.
The appropriate government can extend the period of 12 months.

The declaration and summary of resettlement etc. shall be publicized in the same manner as
listed under publication of preliminary notification detailed above. Once published this
declaration shall be conclusive evidence that the land is required for a public purpose. Prior to
the declaration, the land requisitioning agency must deposit an amount, in full or part, as may
be prescribed by the appropriate Government toward the cost of acquisition of the land. If the
application for acquisition itself specified stages for the rehabilitation and resettlement, then
all declarations shall be made according to the stages so specified.
Public Hearing must be conducted in every affected Gram Sabha and Municipality where
more than twenty-five per cent of the land belonging to that Gram Sabha or Municipality is
being acquired

Within 12 months of publication of declaration, the Collector must make the award unless the
period is extended by the State government.

As per Sec 25 any land acquisition process initiated under the earlier 1894 act but not
completed can be lapsed in some conditions. If award was made under earlier Act then the
earlier Act shall apply even in the courts. If award was made but most did not take it then the
new Act will apply. Where an award has been made five years or more prior to the
commencement of this Act but the physical possession of the land has not been taken or the
compensation has not been paid the said proceedings shall be deemed to have lapsed.

Rule 20 of Right to Fair Compensation and Transparency in Land Acquisition, Rehabilitation


and Resettlement (Compensation, Rehabilitation and Resettlement and Development Plan)
Rules, 2015 describes the manner of return of land which remains unutilized for a period of
five years. It states the same shall be returned to the original owner or owners or their legal
heirs, as the case may be, or to the Land Bank by issuing a notice to the Requiring Body for
whom the land was acquired and by giving an opportunity of being heard and by passing
necessary order in writing by the Collector in this behalf for this purpose. The land is taken
over by the Collector first prior to transferring.

CASE LAWS IN LAND ACQUISITION

The Divisional Bench of Supreme Court in the Working Friends Co-operative House
Building Society Ltd. Vs the State of Punjab and Ors., (CA 8468/ 2015 decided on Oct. 12,
2015) considered the question "whether the compulsory acquisition of the ... land under the
Land Acquisition Act, 1894 lapses in view of the provisions of Section 24(2)" of the new Act of,
2013". It was held that the acquisition proceedings initiated by the land acquisition
notifications dated 12th November, 1992 and 21" July, 1993 followed by the award dated
22nd February, 1995 have lapsed only in so far as the appellant is concerned. It is based on a
conclusion of fact "The admitted position is that the compensation of Rs. 35,52,528/- was
neither paid to the appellant i.e. the Working Friends Cooperative House Building Society Ltd.,
nor was it deposited in the Reference Court. It was admittedly deposited in the Government
Treasury of the State". The deposit was made only after the Act came into force and was done
perhaps with a view to get over the provisions of Section 24(2) of the Act. It observes that
even if it were deposited in the reference Court on 26th June, 2014 it would not have aided as
under any circumstances and cannot be taken as "deemed payment".
A three-judge bench of the Apex Court in Union of India and Ors vs. Shiv Raj &OrsCA NOS.
5478-5483 of 2014 (Arising out of S.L.P.(C) Nos. 24297-24302 of 2007decided on 7 May, 2014)
- "The facts and circumstances which have arisen in this appeal are that the land, the subject
matter of the appeal, stood notified under Section 4 of the Act 1894 on 25.11.1980. The other
persons whose land had also been acquired by the same notification had challenged the
validity of the notification under Section 4 of Act 1894 by filing the writ petitions and its
validity was upheld by the judgment and order dated 15.11.1983. It was during the pendency
of the acquisition proceedings that the present appellant had purchased the land vide
registered sale deeds dated 6.5.1985 and 24.5.1985". ..."... various orders in various litigations
pending before the High Court had been passed. The writ petition filed by the present
appellant was dismissed vide impugned judgment and order dated 17.12.2004.‖... "In view of
the fact that the other land covered by the same notification ...Review Petition etc. had been
filed, which was dismissed on 27.7.2007". A point rose before it was that many stays had been
issued by High Court etc. To which it opines "... It is evident from the orders passed by the
High Court that it had granted stay of dispossession during the pendency of the writ petition
as well as the review petition, though no interim order has been passed by this court.The
respondent did not take possession of the land in dispute though award had been made in
the year 1987-1988, and the High Court had decided against the appellant in the year 2007.
Thus, a period of 7 years has lapsed without any stay of proceedings and yet no action has
been taken by the respondents in pursuance to the award". The court concluded the
acquisition was hit by sec 24(2) of the new Act.

The period during which the proceedings for acquisition of land was held up on account of
any stay or injunction issued by any court is excluded for the purpose of calculation of five
years period.

DETERMINATION OF THE MARKET VALUE OF LAND

Under the earlier 1894 Act, the Collector decided the quantum of compensation payable to
those displaced. The present Act provides a clear process.

The date for determination of market value shall be the date on which the notification has
been issued under section 11. The Collector shall adopt the following criteria in assessing and
determining the market value of the land including all assets attached to the land, namely:

1) The market value, if any, specified in the Indian Stamp Act, 1899 for the registration of
sale deeds or agreements to sell, as the case may be, in the area, where the land is
situated; or
2) The average sale price for similar type of land situated in the nearest village/ vicinity area.
This will be based on sale price in the sale deeds or the agreements to sell registered for
similar type of area in the near village or near vicinity area during immediately preceding
three years of the year in which such acquisition of land is proposed to be made. One-half
of the total number of sale deeds or the agreements to sell in which the highest sale price
has been mentioned shall be taken into account. However, any price paid as
compensation for land acquired under the provisions of this Act on an earlier occasion in
the district shall not be taken into consideration. Any price paid, which in the opinion of
the Collector is not indicative of actual prevailing market value may be discounted for the
purposes of calculating market value. The market value so calculated shall be multiplied
by a factor one for urban areas and two for rural areas.

3) Consented amount of compensation as agreed upon under sub-section (2) of section 2 in


case of acquisition of lands for private companies or for public private partnership
projects, whichever is higher.

Where the market value cannot be determined for the reason that:

o The land is situated in such area where the transactions in land are restricted
by or under any other law for the time being in force in that area; or

o The registered sale deeds or agreements to sell as mentioned in clause


(a) of sub-section (1) for similar land are not available for the immediately
preceding three years; or
o The market value has not been specified under the Indian Stamp Act, 1899 by
the appropriate authority,

4) The State Government concerned shall specify the floor price or minimum price per unit
area of the said land based on the price calculated in the manner specified in sub-section
(1) in respect of similar types of land situated in the immediate adjoining areas

5) If the body that requires the land offers its shares to the owners of the lands (whose lands
have been acquired) as a part compensation, for acquisition of land, such shares in no
case shall exceed twenty-five per cent of the value nor can it compel land owners to
accept share.

6) The land or property of an educational institution established and administered by a


religious or linguistic minority shall be such as would not restrict or abrogate the right to
establish and administer educational institutions. This requirement was inserted vide Art
30(1A) in the Constitution of India in1978.

7) For determination of value of things attached to land or building and other immovable
property or assets attached to the land or building which are to be acquired, use the
services of competent professionals:
o For buildings - engineer or any other specialist in the relevant field, as may be
considered necessary by him.
o Trees can be assessed by use the services of experienced persons in the field of
agriculture, forestry, horticulture, sericulture, or any other field, as may be
considered necessary by him.
o Value of the standing crops damaged during the process of land acquisition,
may use the services of experienced persons in the field of agriculture as may
be considered necessary by him.

AS PER THE FIRST SCHEDULE ONTHE MINIMUM COMPENSATION FOR LAND OWNERS
AND TENANTS

Compensation package Manner of determination of value


component
Multiplier Factor of the In the case of urban areas - one and In the case of Rural areas - One
market value to two based on the distance of project from urban area, as may be
notified by the appropriate Government [Vide MORD Notification
dated 9th February, 2016 S.O. 425(E) - .
[F No.13011/04/2015-LRD]

Solatium Equivalent to one hundred percent of the market value of land


multiplied by the appropriate factor mentioned above plus value of
assets attached to land or building
Final award in rural/ urban Market value of land multiplied by the appropriate factor for rural/
areas urban areas plus value of assets attached to land or building
mentioned plus solatium as above

ACQUISITION OF PART OF HOUSE OR BUILDING(S.94)

The provisions of this Act cannot be invoked for severing land and acquiring a part only of any
house, factory or other building when the owner desires that the whole of such house, factory
or building shall be acquired. Whether it is or is not part of a house, manufactory or building
within the meaning of this section will be decided by the Authority to whom the Collector
refers the matter which will decide whether the land is critical to "full and unimpaired use of
the house, manufactory or building"

If due to severing of a part of the land being acquired from remainder of the land of a owner
if the appropriate Government is of opinion that the claim is unreasonable or excessive, it
may, at any time before the Collector has made his award, order the acquisition of the whole
of the land of which the land first sought to be acquired forms a part. No fresh declaration is
needed even if this additional area is included in the award.
AWARD FOR LAND ACQUISITION

An award under this Act is computed for each eligible family and consists of the following
elements:

1) Market value as computed above for land, building and things attached to land and
building. This is the compensation amount- Solatium which is an amount equivalent to
one hundred per cent of the market value and given in addition

2) twelve per cent per annum on such market value computed for the period commencing
on the date of publication of the notification of the Social Impact Assessment study and
till the date of the award of the Collector or the date of taking possession of the land,
whichever is earlier

3) 25 percent of the total compensation can be through shares in the Requiring Body if the
projected affected family consent

4) If land is acquired for urbanization then 20 percent of the acquired land must be reserved
for landowning project affected people and offered to them at a price equal to cost of
acquisition
5) If a person previously displaced is being displaced again in another project from the
rehabilitation site - they will be entitled to 75 percent additional compensation.

EXEMPTION FROM INCOME TAX, STAMP DUTY AND FEES

As per sec 97 no income tax or stamp duty can be levied on any award or agreement made
under this Act, except under section 47 and no person claiming under any such award or
agreement shall be liable to pay any fee for a copy of the same on or after the 5th September
2011 as per Sec 47 amongst others provides when any person other than a specified person is
purchasing land through private negotiations for an area equal to or more than such limits
specified for applicability of the Act, he must apply to the Collector. On application Collector
shall pass individual orders.

If the same land is acquired within three years from the date of commencement of this Act,
then, forty per cent of the compensation paid for such land acquired shall be shared with the
person(s) who was the owner as on 5th, 2011.
Figure 1: The format of Land Acquisition Award specified in the Rules

FORM VI
See rule III
Land Acquisition case No:

1 Name of the Project -


2 Number and date of declaration under which the land is to be acquired

Situation and extent of the land in hectares, the number of field plots on the survey map,
3
the village in which situated with the number of mile plan if any.

Description of the land, i.e., whether fallow, cultivated, homestead, etc. If cultivated, how
4
cultivated? Source of irrigation

5 Names of persons interested in the land and the nature of their respective interests.
6 Aadhaar No. of such persons
7 Amount allowed for the land itself, without trees, buildings etc., if any
8 Amount allowed out of such sum as compensation for the tenants interested in the land.
9 Basis of calculation:
10 Amount allowed for trees, houses or any other immovable property
11 Amount allowed for crops.
12 Additional compensation on the market value under section 30(3)
13 Damages under section 28 of Act 30 of 2013
14 Solatium u/s 30(1)
15 Total of amounts
Particulars of abatement of Government Revenue, or of the capitalised value paid, the date
16
from which the abatement takes effect.
Apportionment of
the amount of Amount
Serial Name of Aadhar Bank A/c
17 compensation. payable *Remarks
No. claimants No. No.
to each
Area (in hectares)

Date on which possession was taken u/s. 38(1) and 40(I) of Act 30 of 2013.

If under section 40(1) the number and date of the order of Government giving authority to do
so.

Date: Signature

Bank account details to be collected in all cases where Aadhaar number is not available or
Aadhaar is not seeded in the bank account of the claimant.
BUILDING RULES & REGULATIONS

Building Rules and Regulations of Local Bodies as well as Development Control Rules &
Regulations of different urban development authorities for feasibility of
Development/Redevelopment on the Land – Rules for Open Space, FSI and Plinth Area
Restrictions.

SOURCE OF BUILDING LAW ANDRULES

Building laws and rules are derived from different sources:

1) National Building Code,2016

2) Model Building ByeLaws

3) Master Plan &Zoning Rules

1) NATIONAL BUILDING CODE,2016

The National Building Code of India (NBC), a comprehensive building Code, is a national
instrument providing guidelines for regulating the building construction activities across the
country. It serves as a Model Code for adoption by all agencies involved in building
construction works be they Public Works Departments, other government construction
departments, local bodies or private construction agencies. The Code mainly contains
administrative regulations, development control rules and general building requirements; fire
safety requirements; stipulations regarding materials, structural design and construction
(including safety); building and plumbing services; approach to sustainability; and asset and
facility management.

The Code was first published in 1970 at the instance of Planning Commission and then first
revised in 1983. Thereafter three major amendments were issued to the 1983 version, two in
1987 and the third in 1997. The second revision of the Code was in 2005, to which two
amendments were issued in 2015.Due to large scale changes in the building construction
activities, such as change in nature of occupancies with prevalence of high rises and mixed
occupancies, greater dependence and complicated nature of building services, development
of new/innovative construction materials and technologies, greater need for preservation of
environment and recognition of need for planned management of existing buildings and built
environment, there has been a paradigm shift in building construction scenario. Considering
these, a Project for comprehensive revision of the Code was taken up under the aegis of the
National Building Code Sectional Committee, CED 46 of BIS and its 22 expert Panels; involving
around 1 000 experts. As a culmination of the Project, the revised Code has been brought out
in 2016 as NationalBuildingCodeofIndia2016reflectingthestate-of-the-art and contemporary
applicable international practices. The comprehensive NBC 2016 contains 12 Parts some of
which are further divided into Sections totaling 33 chapters (see Annex 1).The salient features
of the revised NBC (see Annex 2) include, apart from other changes made, the changes
specially in regard to further enhancing our response to meet the challenges posed by natural
calamities. The major changes incorporated in this third revision of the Code are as follows:
• Provisions for association of need based professionals and agencies have been updated
to ensure proper discharge of responsibilities for accomplishment of building project.
• With a view to ensuring ease of doing business in built environment sector, a detailed
provision for streamlining the approval process in respect of different agencies has been
incorporated in the form of an integrated approval process through single window
approach for enabling expeditious approval process, avoiding separate clearances from
various authorities.
• Further, with a view to meeting the above objective, the provision on computerization of
approval process has been detailed, enabling online submission of plans, drawings and
other details, and sanction thereof, aiding in speedier approval process.
• The mechanism of ensuring certification of structural safety of buildings by the competent
professional and peer review of design of buildings, have been further strengthened.
• Requirements for accessibility in buildings and built environment for persons with
disabilities and the elderly have been thoroughly revised and updated.
• Provisions on fire and life safety have been thoroughly revised to meet the challenges of
modern complex building types including the high-rises.
• Latest structural loading and design and construction codes including those relating to
wind load, earthquake resistant design of buildings, steel design and foundations have
been incorporated with a view to ensuring structural safety of buildings including against
disaster.
• Provisions relating to all building and plumbing services have been updated keeping also in
view the latest international practices as related to the country.
• Provisions have been updated to ensure utilization of number of new/alternative
building materials and technologies to provide for innovation in the field of building
construction.
• Construction management guidelines have been incorporated to aid in timely completion
of building projects with desired quality in a safe manner within the budgeted cost.
• Guidance has been provided for making buildings and built environment energy efficient
and environmentally compatible, through the newly introduced and updated chapter on
sustainability, namely Part 11 ‗Approach to Sustainability‘
• New chapters have been added on structural use of glass; escalators and moving walks;
information and communication enabled installations; solid waste management; and asset
and facility management.

The Code has been published in two volumes containing all the Parts and Sections.

ANNEX 1

CONTENTS OF NBC 2016 VOLUME 1


PART 0 INTEGRATED APPROACH – A PRE-REQUISITE FOR APPLYING THE PROVISIONS OF THE
CODE
PART 1 DEFINITIONS
PART 2 ADMINISTRATION
PART 3 DEVELOPMENT CONTROL RULES AND GENERAL BUILDINGREQUIREMENTS PART 4
FIRE AND LIFE SAFETY
PART 5 BUILDING MATERIALS PART 6 STRUCTURALDESIGN
Section 1 Loads, Forces and Effects Section 2 Soils and Foundations Section 3 Timber and
Bamboo
3A Timber 3B Bamboo
Section 4Masonry
Section 5Concrete
5A Plain and Reinforced Concrete 5B Prestressed Concrete
Section 6 Steel
Section 7 Prefabrication and Systems Building and Mixed/Composite Construction
7A Prefabricated Concrete
7B Systems Building and Mixed/Composite Construction Section 8 Glass and
Glazing
VOLUME 2
PART 7 CONSTRUCTION MANAGEMENT, PRACTICES AND SAFETY PART 8 BUILDING
SERVICES
Section 1 Lighting and Natural Ventilation Section 2 Electrical and Allied Installations
Section 3 Air Conditioning, Heating and Mechanical Ventilation Section 4 Acoustics,
Sound Insulation and Noise Control Section 5 Installation of Lifts and Escalators and
Moving Walks
5A Lifts
5B Escalators and Moving Walks
Section 6 Information and Communication Enabled Installations PART 9 PLUMBING
SERVICES (INCLUDING SOLID WASTEMANAGEMENT)
Section 1 Water Supply
Section 2 Drainage and Sanitation Section 3 Solid Waste Management Section 4 Gas
Supply
PART 10 LANDSCAPE DEVELOPMENT, SIGNS AND OUTDOOR DISPLAY STRUCTURES
Section 1 Landscape Planning, Design and Development Section 2 Signs and Outdoor
Display Structures
PART 11 APPROACH TO SUSTAINABILITY PART 12 ASSET AND FACILITY MANAGEMENT

ANNEX 2

Salient Features of National Building Code 2016 (NBC 2016)

1) Detailed provision for streamlining the approval process in respect of different agencies
in the form of an integrated approval process through single window approach thereby
avoiding separate clearances from various authorities, with a view to ensuring ease of
doing business in built environment sector.
2) Progressive computerization of approval process, for enabling online submission of
plans, drawings and other details, and sanction thereof.
3) Updated mechanism of ensuring certification of structural safety of buildings by the
competent professional and peer review of design of buildings.
4) Defining the roles and responsibilities of all professionals and contractors involved in a
building construction project.
5) Comprehensive planning norms for minimum amenities to be provided in a city/town.
6) Detailed provisions relating to requirements for accessibility in buildings and built
environment for persons with disabilities and the elderly.
7) Planning and development norms, such as, Transferable Development Rights (TDR) and
Accommodation Reservation(AR).
8) Provisions for underground or multi-storeyed parking as also mechanized
parking of vehicles.
9) Norms for solar energy utilization.
10) Requirements for buildings on podium for ensuring fire and life safety in such buildings.
11) Fire and life safety in modern complex buildings including the high rises, glazed
buildings, atria, commercial kitchen and car parking facilities.
12) Updated structural design provisions for wind and seismic loads, imposed load due to
helipad, and blast loads, for safe design and construction of buildings with due focus on
ductile detailing.
13) Latest research and development inputs and provisions on concrete, steel and masonry
buildings with a view to ensuring disaster resilient buildings.
14) Assessment of liquefaction potential of a site and ground improvement techniques for
maximum utilization of land resources including at seismically vulnerable sites.
15) Updated provisions on engineered use of bamboo in housing and other building
construction.
16) Promotion of use of agricultural and industrial wastes including construction and
demolition wastes in building construction without compromising the quality and safety.
17) Inclusion of provisions on self compacting concrete, high performance concrete and steel
fibrereinforced concrete.
18) Updated provisions on prefabricated construction technique for speedier construction.
19) New chapter on structural use of glass in buildings.
20) New and alternative building materials, and technologies for building construction
such as, reinforced masonry, confined masonry building construction and masonry wall
construction using rat-trap bond.
21) Construction project management guidelines for timely completion of building projects
within the budgeted cost with desired quality.
22) Habitat and other welfare requirements for workers at construction site.
23) Inclusion of modern lighting techniques such as LED and induction light and their
energy consumption.
24) New provisions on compact substations and updated provisions on installation of energy
meters.
25) Comprehensive provisions relating to lightning protection of buildings.
26) Provisions on aviation obstacle lights; electric vehicle charging and car park
management.
27) Protection of human beings from electrical hazards and against fire in the building due to
leakage current.
28) Use of refrigerants for air conditioning addressing zero ozone depletion potential
(ODP) and ultra-low global warming potential(GWP).
29) Inclusion of new and energy efficient options of air conditioning, heating and
mechanical ventilation, such as variable refrigerant flow system, inverter technology,
district cooling system, hybrid central plant using chilled beams, radiant floor
components, and geo-thermal cooling and heating.
30) Thrustonenvelopeoptimizationusingenergymodelling,daylightingsimulation,
solar shade analysis and wind modelling software to optimize the air conditioning load.
31) Air conditioning, heating, and ventilation (HVAC) provisions considering adaptive
thermal comfort conditions for energy efficiency.
32) Provisions pertaining to metro train ways and metro stations with respect to fire and
life safety; and air conditioning, heating and ventilation for metro stations.
33) HVAC requirements for data centres and healthcare facilities; refrigeration for cold
stores; efficient strategies for winter heating using reverse cycle operation, solar heating
systems, ground source heat pump and electric heat pump; and modern system of
mechanical ventilation for industries, commercial kitchen and underground car parking.
34) Updated provisions on building automation system to include the latest practices for
web-based monitoring and control of performance parameters.
35) High speed lifts for tall buildings.
36) New chapter on escalators and moving walks for comfortable and safe movement of
people.
37) New chapter on information and communication enabled installations in
buildings.
38) Updated provisions on water supply, drainage and sanitation for modern high rise
buildings and complexes.
39) Provisions relating to swimming pools covering hygiene and safety.
40) Updated provisions on rainwater harvesting.
41) New chapter on solid waste management covering various solid waste management
systems within the building and building complexes.
42) Updated provisions on piped gas supply in houses, and in hospitals for medical
purposes.
43) Promoting quality of outdoor built environment through updated provisions on
landscape planning, design and development.
44) Promoting sustainability in buildings and built environment in tandem with relevant
sustainable development goals.
45) New chapter on asset and facility management to cover provisions relating to
management of building assets and associated services, also covering responsibilities of
occupants for maintenance of facilities, such as structures, equipment and exterior
property.
2) MODEL BUILDING BYE LAWS 2016

Building Bye-Laws are legal tools used to regulate coverage, height, building bulk, and
architectural design and construction aspects of buildings so as to achieve orderly
development of an area. They are mandatory in nature and serve to protect buildings against
fire, earthquake, noise, structural failures and other hazards. In India, there are still many small
and medium sized towns which do not have building bye-laws and in the absence of any
regulatory mechanism, such towns are confronted with excessive coverage, encroachment and
haphazard development resulting in chaotic conditions, inconvenience for the users, and
disregard for building aesthetics, etc. It is in this context,
TCPOhasmadeanefforttoprepare―ModelBuildingBye-Laws-2016‖fortheguidance of the State
Governments, Urban Local Bodies, Urban Development Authorities, etc. which is an
improvement over the previous Model Building Bye Laws brought out in 2004.
In 2003, the Ministry of Urban Development desired that Model Building ByeLaws be
prepared, in view of Bhuj Earthquake that occurred in 2001, to lay focus on structural safety of
buildings and for the guidance of the State Governments. Accordingly, the MBBL 2004
incorporated the provisions of structural safety and other provisions like rainwater harvesting
and waste water recycling, solar assisted heating, barrier free public buildings and fire safety.
The Bye-Laws were circulated to all the State Governments and Union territories and out of 36
States and UTs, wherein 22 States and UTs have undertaken comprehensive revision of their
respective Building Bye-Laws since2004.

The Reason deter for revising the Bye-Laws are as under:


• Growing Environmental concerns

• Increased Safety and Security measures

• Technological Developments

• Swachh Bharat Mission

• Focus on Ease of Doing Business In2015,

It was further desired by the Ministry of Urban Development that the Model Building Bye-
Laws, 2004 needs to be revised and updated keeping in view the emerging issues like Norms
for Rooftop Solar PV Installation, Segregated sanitation facilities for visitors in public buildings,
Additional provisions in Building regulations for natural hazard prone areas, Conservation of
heritage sites including heritage buildings, heritage precincts and natural feature areas, Bye-
laws for safe use of glass, barrier free environment for disabled, children and old persons and
Mitigation of the effects of electromagnetic radiation on built spaces.
The Draft Model Building Bye-laws, 2016 were circulated to Central Governments agencies/
institutes like National Disaster Management Authority, Bureau of Indian Standards, National
Building Construction Corporation, Delhi Development Authority, National Capital Region
Planning Board, Indian Institute of Public Administration, Municipal Corporation of Delhi
(South), Housing and Urban Development Corporation, Schools of Planning and Architecture,
State Town and Country Planning Departments, selected Urban Development Authorities,
selected Urban Local bodies and associations like CREDAI and NAREDCO for obtaining their
comments and suggestions.

A National Workshop on Draft Model Building Bye Laws, 2016 was held on 18.2.2015 mainly
to obtain comments / suggestions from the State Town and Country Planning
Departments/Urban Development Authorities/ Municipal Corporations and Schools of
Planning and other related stakeholders like Association of Real Estate Developers and private
consultants.

The workshop was attended by 110 participants and comments/suggestions on following


issues emerged out in the day long deliberations-
• Norms for High Rise Buildings (Chapter 5)

• Norms for differently abled, senior citizens and children (Chapter8)


• Sustainability and Green buildings (Chapter10)
• Streamlining building pla approvals, Ease of Doing Business (Chapter13)
• Norms for Low income housing
• Parking Norms (public parking space for vehicles inCBD)
• Flexible FAR prescriptions
• Incorporation of new terms in definitions
• Updations of all IS Codes ofBIS
• Updating fire provisions with respect to staircase and exits.

A review meeting was held under the chairmanship of Secretary (UD) on 06.07.2015 wherein
Joint Secretary (UD) made a presentation on the draft MBBL, 2016. In the meeting, it was
decided that the draft may also be circulated to related Ministries involved in granting NOCs
for construction projects. Comments were received from the Ministries of Culture, Consumer
Affairs, Food and Public Distribution, Housing and Urban Poverty Alleviation, Power, Water
Resources and River development and Ganga Rejuvenation and Civil Aviation. The relevant
comments received from all the concerned have been duly incorporated in the MBBL, 2016.
On18th March, 2016, a revised set of Model Building Bye Laws, 2016 (‗2016 Bye-Laws‘) was
released.
THE SALIENT FEATURES OF MBBL- 2016 ARE GIVEN AS UNDER –

Safety and security (Chapter-6): includes Structural Safety, Disaster management as per
Prof. Arya Committee Report and BIS Codes including Structural Design Basis
Report(SDBR)forvariousbuildingtypes.Preventionmeasuresagainst―SoftStoreys‖ in multi-
storeyed buildings and Proof Checking of Structural Design for buildings.
• Barrier Free Environment: Provisions for Differently abled, Elderly and Children
including Site development, Access Path/ Walk Way, Parking, Building requirements,
Stair, Lifts, Toilets, Drinking Water, Refuge and signage.

• Environmental Concerns: Additional Chapter-10 incorporated for Green Buildings and


Sustainability provisions, Rainwater Harvesting, Wastewater Reuse and Recycle and
installation of Solar Roof Top PVnorms.

• Adoption for Modern Construction Technology: Additional Chapter-5 incorporates


Definitions, Structural Safety and other provisions for Highrise Building regulations with
Parking, Peripheral Open Spaces including set-backs. Disaster Management &Fire
Safety.

• Swachh Bharat Mission: Revised Norms for adequate toilet facilities for women and
Public Conveniences in Public Buildings and Mandatory Provisions for Segregated toilet
facilities for visitors in Public Buildings.
• Ease of Doing Business: Additional Chapter-14 includes Provision for Online Building
plan approval process adopting automated systems of plan scrutiny, generation of
reports/approvals and integrated systems of intimation of approval, Compliance report
from drawings for automatic generation of Completion certificate and integration of
various clearances at Master Plan levels and introduction of Integrating ―Single
Window‖ process, Empowering Architects, Outsourcing non discretionary verification
jobs and formulation of Citizen‘s Charter.

• Rain Water Harvesting: Additional Chapter-9 Indicative Provisions for harvesting in


various types of buildings along with responsibility of ULBs for RWH in public spaces,
provisions for Enforcement and Monitoring.

• Effects of Communication Technology: Additional Annexure –I, Draft Guidelines for


mitigation of Electro Magnetic Radiations in built spaces by identifying emission
appliances/ sources. Guidelines for safe spacing of indoor and outdoor appliances and
equipment.

3) MASTER PLAN AND ZONING RULES

Where the Town and Country planning Acts are operational usually the Directorate of
Town and Country Planning oversees planning and development in urban and rural areas by
way of issuance of Master Plans prepared for the urban centers and notified rural areas by
indicative Land Use Plans. Such Directorates exist in almost all states viz Andhra Pradesh,
Odisha, Assam, Tamil Nadu, Chhattisgarh, Haryana,and
Himachal Pradesh. Jharkhand has a Town and Country Planning Organization (TCPO) under
the Urban Development Department and the Master Plans for Jharkhand was prepared for the
Urban Development Dept. by a private agency.

The Town and Country Planning Organization (TCPO), technical arm of the Ministry of Urban
Development, Government of India, is an apex technical advisory and consultant organization
on urban and regional planning strategies and monitoring of central government schemes
and development policies.

Master Plan derives authority from the parent Act. In Jharkhand Master Plan is a document
prepared under the Jharkhand Regional Development Authority Act. 2001. The Delhi
Development Act 1957 gives sanctity to the master plan and zonal development plans. As per
sec 14 of Delhi Development Act use of land or building other than in conformity with the
Master Plan is prohibited. The current version as on the date of registration of Sale Deed or
date of application for sanction of plan applies. There is no vested right to purchaser for
claiming NOC for sanction of building under earlier Master Plan on the basis of date of Sale
Agreement or otherwise. Further no sanction
hadbeencommunicatedtoapplicantunderearlierplan(DewanandSonsInvestments
(P) Ltd. vs. DDA, AIR 1997 Del 388). In case of Mumbai earlier the Development Plans were
issued under Sec 26 (1) Maharashtra Regional and Town Planning Act, 1966. Later these were
issued under the Metropolitan Development Authority constituted under the Mumbai
Metropolitan Region Development Authority Act1974.

Master Plans do not exist in every state/ city. But especially after Jawaharlal Nehru National
Urban Renewal Mission (JNNURM) the larger cities included under JNNURM has Master Plans
which amongst others divide the city into zones to regulate the type of usage the land in that
zone can be put to. There is usually no penalty in every state for such violations. The Zoning
and Development Promotion Regulations issued by Hyderabad Metropolitan Development
Authority has following zones:

• Residential Use Zone

• Commercial Use Zone, including commercial along notified commercial roads; strip
commercial along roads as earmarked in the master plan and areas earmarked as
Commercial use in the MasterPlan

• Multiple Use Zone, including areas covered in Transit-Oriented Development


(TOD)zone

• Public and Semi - Public Use Zone

• Work Centre Use Zone

• Open Space Use Zone - Parks, Playgrounds, Exhibition grounds, Green bufferzone
• Water bodies Use Zone - River, stream, Nalah, Storm Water Drains,Lakes

• Transportation Use zone (Road, rail, Airport, Bus depots, Terminals, Workshops, Truck
terminals, Warehouses, Parking areas/Parking lots/Parking complex

• Special Reservation Use zone

• Sites specifically earmarked as heritage conservation - buildings and precincts/areas,


rocks & hillocks/Natural heritage

• Defence/Military lands

• Burial grounds, Cremation groundsheet

• Special Area Development Plan (SADP)areas

Hyderabad's zoning provision overtly clarifies the proposed activities and uses in the
proposed buildings/premises in addition to the conformity with these regulations shall be in
conformity with the Hyderabad revised building rules, 2006 provisions. The notification
provides where there is an inconsistency between the building rules and the zoning
regulations, the provisions of the zoning regulations shall prevail

In Bangalore the BDA layouts are completely planned layouts, with access roads, planned
transportation hubs, major and minor roads of necessary width, sites set apart for schools and
hospitals, etc... There are 62 layouts. ... Sites allotted under the BDA scheme may only be used
for the specific purpose that they are planned for – residential, commercial, or for providing
social infrastructure.

Draft Development Control Regulations for Development Plan applicable in municipal


areas of Pune, every application for a building permission/ commencement certificate shall be
accompanied by the following "for verifying the proof of ownership.

• Latest 7/12 extract (or the property register card) indicating date of issuance within six
months, showing the area in words and figures

• Measurement Plan issued by authority, having date of demarcation within 4 years, the
only exception being recently sanctioned plotted layout.

• Title Search report by advocate who is enrolled with the Bar council.

• Other documents including no objection certificate as requisitioned by the Municipal


Commissioner"

These municipal provisions are mandatory since these requirements set out in executive
orders are issued under powers allowed by the parent act. OswalAgro Mills Ltd. Vs.
Hindustan Petroleum Corporation Ltd. etc. Civil Appeal No. 10933 of 2013 the suit land
had been indicated as being earmarked for housing since the 1992 plan. SC through its
majority judgment directed the Municipal Commissioner to reconsider the application
made by OswalAgro Mills Ltd. keeping in view the objections raised by the Police Department,
Ministry of Petroleum, Ministry of Environment and Intelligence Bureau and the Security
Control Regulations issued by the State of Maharashtra.

But master plans may not integrate well with plans of other authorities like the Regional
Development Authority, any Mineral Area Development Authority (MADA) or the Mines Board.

EXISTING BUILDINGS USE

As per the bye-laws of the Ranchi Municipal Corporation, an Existing Buildings is a building,
structure or its use as sanctioned/approved/ regularized by the competent authorities. The
existing buildings are classified into different types as follows.

Residential For normal residential purposes with or without cooking or dining or


Buildings both facilities. It includes one or two or multi-family dwellings,
apartment houses and flats.
But the definition under the relevant Act must be referred to avoid
confusion. For instance, as per Delhi Rent Control Act for the limited
purpose of computing Standard Rent public hospitals, educational
institutions, public libraries, reading rooms and orphanages are
considered as residential premises.
Institutional Used for medical or other treatment or care of persons suffering from
Buildings physical or mental illness/disease or infirmity, care of infants,
convalescents or aged person and for penal or correctional detention
in which the liberty of the inmates is restricted. Institutional buildings
ordinarily provide sleeping accommodation for the occupants and
include hospitals, sanatoria, jails, prisons, mental hospitals,
reformatories.
Educational These shall include any building used for school, college or day care
Buildings purpose for more than an hour per week and involves assembly for
instruction, education or recreation.

Assembly where people congregate or gather for amusement, recreation,


Buildings religious, patriotic, civil, travel and similar purposes. E.g. theatres,
motion picture houses (Cinema), assembly halls, auditoria, exhibition,
halls, museums, skating rinks, gymnasium, restaurants, places of
worship, dance halls, club rooms, passenger stations, and terminals of
air, surface and other public transportation services, recreation places
and stadium.
Business These shall include any building or part of a building which is used for
Buildings transaction of business, for keeping accounts and records for similar
purposes, doctor‘s service facilities, City halls, Town halls are classified
in this group in so far as principal function of these is transaction of
public business and the keeping of books and records.

Mercantile/ Office, shops, stores, market, for display and sale of merchandise
Commercial either wholesale or retail (includes stores and service facilities
Buildings incidental to the sale of merchandise and located in the same
building); hotels, dormitories and lodging houses shall also come
under this category.
Industrial products or materials of all kinds and properties are produced/
Buildings manufactured E.g. assembly plants, Laboratories, power plants, smoke
houses, refractory, gas plants, mills, dairies, factories etc.

Storage whole or part of building used for storage and sheltering of goods
Buildings: e.g. warehouse, cold storages, freight depots, transit sheds, store
houses, garages, aero plane hangars, truck terminals, grain elevators,
barns and stables.

Hazardous highly combustible or explosive materials are stored handled


Buildings:

The category of existing building use is a very material consideration when allowing
conversion to other building use.

In, ‗Nihal Chand Lallu Chand Pvt. Ltd. vs. Pancholi Cooperative Housing (AIR 2010 SC 3607)‘
Supreme Court held that any requirement for giving an undertaking contrary to Development
Controlled Regulations for Greater Bombay 1991(DCR) will not be binding either on the flat
purchasers or the Society.

Generally, the additional area in the precincts of monument under Archaeological Survey of
India is demarcated and fenced/ bounded. But future development needs, around the site
needs to be kept in mind and development rights restricted of the properties nearby.

Existing properties may be notified for being vacated since lying within certain distance of
shoreline/ river line etc.

Some specific provisions are:

1) PARKINGSPACE

Providing parking for visitor's vehicle is increasingly a requirement. Development Control


Rules of Greater Mumbai - in Table 15 Regulation 36 - requires that 25 percent (earlier
10 percent) of the parking space has to be kept for visitor's cars.

According to Regulation 36 of the Development Control Regulations (1991) for Greater


Bombay, whenever a property is developed or redeveloped, parking spaces have to be
provided in accordance with the guidelines in the DCR. Before a builder can develop or
redevelop a project, he needs to obtain the development permission and a commencement
certificate. "To get these approvals, he needs to submit a building plan, which includes the
details of the parking spaces

OS Bajpai's case supra clarifies that parking space in stilt area of building cannot be sold by
builders.

2) OFFSET

Supreme Court decided in Anil Sen Gupta and Dhirendra N Basak 1998 SCC 547 the offset to
be kept from the side wall privately committed to the selling Cooperative Society is a private
covenant, even if the quantum provided in it is higher than that specified in the applicable
building rules. Court clarified that on a plain reading the building rule R 40 of Bengal
Municipal Act, 1932 only specifies the minimum norm of offset and anything higher in a
private covenant must be honoured.

3) NO OBJECTION CERTIFICATE

Approval or permissions are needed under various Act Works of Defence Act, 1903, The
Explosives Act (for e.g. petrol pumps, stock depots of firecrackers, explosion), State Pollution
Control Board, National Green Tribunal etc. Permission may also be required from Station
Commander of the local defence establishment of Armed Forces, Border Security Forces or
Central Reserve Police Force if construction is within a specified distance of those
establishments. Airport Authority of India may need to approve if location is within a certain
distance or within the takeoff or landing pathway.

The Pune's Development Control Regulation, 2013 in point 6.2.13 requires application for
development permission to be accompanies by No objection Certificate obtained from the
authorities like "Civil Aviation Authorities, Railway, Directorate of Industries, Maharashtra
Pollution Control Board, Central Pollution Control Board, District Magistrate, Inspectorate of
Boilers and Smoke Nuisance etc."
RENT CONTROL LAWS

Sections pertaining to Occupancy Rights of Tenants, Freezing of Rent and Protection against
Eviction of Tenant and its effect on value of property.

Rent and tenancy is regulated by state acts for e.g. some of the state level acts are
enumerated below:

1) West Bengal Premises Tenancy Act, 1997 as amended in 2002 initially notified on 28th
December, 1998 and applicable to Kolkata, Howrah and some other areas (which repealed
West Bengal Premises Tenancy Act,1956)

2) Tamil Nadu Buildings (Lease and Rent Control) Act,1960

3) Jharkhand Building (Lease, Rent & Eviction) Control Act, 2011

4) Bihar Building (Lease, Rent and Eviction) Control Act, 1982 (which replaced The Bihar
Building (Lease, Rent and Eviction) Control Act,1977)

5) Maharashtra Rent Control Act, 1999 (operational since March 2000. Earlier three Acts
were operational in the state of Maharashtra: Bombay Rent, Hotel and Lodging House
Rent Control Act for Bombay region; the Central Provinces and Berar Act were prevailing
in the area of Vidarbha and Hyderabad House (Rent, Eviction and Lease) Control Act 1954
for Marathwada Region.

6) Haryana Urban (Control of Rent and Eviction) Act,1973

7) The Himachal Pradesh Urban Rent Control Act, 1971 (Act No. 23 of1971)

8) The Jammu and Kashmir Houses and Shops Rent Control,1966

9) Uttar Pradesh Urban Buildings (Regulation of Letting, Rent and Eviction) Act,1972

10) Delhi Rent Control Act, 1958 is still applicable in Delhi - it applies to tenancies with Rs.
3500 or less paid as monthly rent. Another Act Delhi Rent Control Act 1995 was enacted
by Parliament and received presidential assent but was not implemented (as it had
triggered a major agitation by tenants in Delhi) and steps initiated to repeal it in 2013
through introduction of a bill in the Rajya Sabha has not progressed. The 1958 Act is the
subject of a petition by an association of landlords with Advocate Shoba Agarwal as a
member and being heard by Delhi High Court on enacting 1995 Act. But central
government in 2015 has said it wishes to repeal the 1995 Act and enact another.

These legislations are enacted to regulate the rent payable be tenant. Many acts seek to
prevent it from exceeding standard rent, regulate repair and maintenance of the property and
also regulate eviction of tenants. Rent Control Acts of different states favor
thetenants.TheRentControlActgenerallyonlyappliesonleaseagreementsofatleast
12 months, and hence an eleven-month agreement helps landlords to take a pre-
emptive measure of avoiding filing eviction suit against tenant to claim back possession of
premise. Sec 106 of Transfer of Property Act has limited application where ever a State level
legislation regulating rent and lease prevails.

LANDLORD/ TENANT DEFINED

Landlord - As per the West Bengal Premises Tenancy Act, 1997 the definition includes any
person who, for the time being, is receiving, or is entitled to receive, the rent for any premises,
whether on his own account or for another as a trustee/ guardian or court receiver or on
behalf of another. The term 'Landlord' also includes one who would have received the rent as
described here if the premises were let to a tenant.

The term "premises" means whole of part of any building or any hut or part of a hut that is let
out separately, and includes the gardens, grounds and out-houses appurtenant to it. It also
includes any furniture/ fixtures/ fittings supplied by the landlord for the use of the tenant (but
do not include a room in a hotel or a lodging house).

A lease for running a factory comprising costly machineries for the purpose of manufacturing
is not a premise - Alliance Jute Mills v Alliance & Co., 82 CWN 1055; Venkaya v Subba Rao, AIR
1957 AP 619

Any space on roof parapet on the outer wall let out for hoarding or advertisement is not a
premise for which a tenancy can be granted- Durjendra v K. Shaw, AIR 1953 Cal 147: ILR
(1954)1 Cal201.

Tenant refers to a person who has to pay rent or on whose behalf rent is payable for any
premise or would have had to pay rent but for a special contract. It includes

1) Any person continuing in possession after termination of his tenancy

2) If tenant dies, for residential properties, the definition also includes, for up to five years
from the date of death members of his family, who were his dependents ordinarily staying
with him and owning no other residential premise (namely his son, daughter, parent and
the widow of his predeceased son). The definition includes spouse and no bar of five years
applies. They shall also have a right of preference for tenancy in a fresh agreement on
payment of fair rent.

3) In respect of premises let out for non-residential purpose his spouse, son, daughter and
parent who were ordinarily living with the tenant up to the date of his death as members
of his family, and were dependent on him or a person authorized by the tenant who is in
possession of such premises

4) Does not include any person against whom any decree or order for eviction has been
made by a Court of competent jurisdiction
PREMISES EXEMPTED FROM PROVISIONS OF TENANCY ACT

Some premises are exempted from the provisions of rent control/ tenancy regulation as per
the West Bengal Premises Tenancy Act, 1997 as

1) As per Sec 3, the Act does not apply to any premises owned by the Central or State
Government, or a Government undertaking or enterprise, or a Cantonment Board or a
local authority or other statutory body which is not a local authority

2) Any tenancy created by the Central Government, or any State Government in respect of
the premises taken on lease or requisitioned by that Government

3) Any tenancy where the lease with due consent of the tenant has been registered under
the Registration Act, 1908 (16 of 1908), after the commencement of this Act, and the fact
of such consent has been recorded in the instrument so registered
4) If premise is rented to a foreign mission or international agency; or a tenancy created by a
foreign mission or an international agency, either by way of lease or otherwise
5) A residential premise located within limits of Kolkata or Howrah Municipal corporation for
which Rs. 6500/- monthly rent is payable or monthly rent of Rs.3500/- to other areas
where the Act extends. While for a non-residential premise located within limits of Kolkata
or Howrah Municipal Corporation for which Rs 10000/- as monthly rent is payable or Rs.
5000 as monthly rent in other areas to which this Act extends. If the tenancy is for dual
purpose then respective rate shall apply in proportion to respective areas

Central and State government properties are exempted in most Act. But state acts may have
specific provisions too. For example, as per the Jharkhand Building (Lease, Rent & Eviction)
Control Act, 2011 the provisions of tenancy contained therein shall not apply to any building
belonging to Jharkhand State Shwetamber Jain Trust Board or Jharkhand State Digamber Jain
Trust Board or Wakf Board or Hindu DharmikNyas Board or any other Religious and Charitable
Trusts or institution run by the Central or State Government.

OBLIGATIONS OF LANDLORD AND TENANT

1) OBLIGATION OFLANDLORD

• Issuance of a written receipt in the prescribed form signed by landlord/ authorised


agent on receiving the rent or any charge for the maintenance of premises
• Keep the premises in good and tenantable condition and ensure due maintenance
of essential supply/service
• No further charge can be claimed if tenant authorised (within tenancy provision)
way sublets the premise held by him
2) OBLIGATIONS OFTENANT

• Pay rent to the landlord or his authorised agent within the prescribed period.

• Use the premise for what it was let out to the tenant

• The landlord or his authorised agent will be allowed to enter upon the premises
and inspect the condition after due service of a notice on tenant of the intention to
doso

• Not add, alter the premises without the written consent of the landlord

• Without written consent of the landlord not to sublet, or transfer or assign his right
in the tenancy wholly or partly

• Pay charges on amenities of the premises at the rate of ten per cent of the fair rent
or the agreed rent and pay his share of municipal tax as an occupier of the
premises as required in the appropriate Municipal Act (in Bengal it is Kolkata
Municipal Corporation Act, 1980 or the West Bengal Municipal Act, 1993.

"Occupier" includes any person at that time liable for paying or actually paying to the owner
the rent or fee or any part thereof or paying damages on account of the occupation of any
land or building, and also includes a rent-free tenant: However an owner living in or otherwise
using his own land or building shall be deemed to be the occupier thereof.

PROTECTION OF UNLAWFUL EVICTION OF TENANTS

"One half of the lis between landlord and tenant would not reach courts, if tenant agrees to
pay the present prevalent market rate of rent of the tenanted premises to the landlord. In that
case landlord would also be satisfied that he is getting adequate, just and proper return on
the property. But the trend in the litigation between landlord and tenant shows otherwise.
Tenant is happy in paying the meagre amount of rent fixed years ago and landlord continues
to find out various grounds under the Rent Acts, to evict him somehow or the other."

(Obiter dicta in Mohammad Ahmad &Anr vs. Atma Ram Chauhan & Ors.CA No.4422 of 2011
[Arising out of SLP.(C) No.6319 of 2007 decided by Supreme Court Justices Dalveer Bhandari
and Deepak Verma]

The West Bengal Premises Tenancy Act says its provisions will prevail over other acts in
matters of suits for eviction of tenants filed by the Owner as the Courts have been barred from
passing any order/ decree in favour of the landlord against the tenant normally. So if rent
control act was not applicable on a premise, an owner can proceed against a
tenant on expiry of the notice of eviction served under Section 106. But once rent control
applies, the owner must prove grounds permitted under rent control act entitling him to evict
the tenant. The grounds on which landlord can file a suit for eviction of tenant usually after
serving a month's notice on the tenant as per the West Bengal Act are:

1) Without consent of owner in writing if the tenant has sublet/ assigned/ parted with the
whole or part of the property or used it for purpose other than purpose it was rented.

2) Not paid rent for three months in a period of 12 months or thrice in a three year where
rent is not payable monthly.

3) Where the premises are required by the landlord for building or rebuilding or adding or
altering substantial portion and which cannot be carried out without the premises being
vacated. Or if the landlord or any beneficiary of the premise needs it for self- occupation
and the landlord does not have any suitable accommodation within the same Municipal
Corporation/ Municipality or in any other area within ten kilometers from such premises
where this Act extends. However, if the competent court deems that ordering vacating
only a part of the premise for building/ altering etc. or for self- occupation will serve the
purpose of the landlord it may accordingly order if the tenant/ subtenant agrees. If
subtenant agrees but tenant does not then too subtenant can continue in part of the
portion.

4) Where the tenant has given notice to quit but not given vacant possession of the
premises to the landlord

5) Tenant or person residing therein have violated provisions of sec 108 of the TPA (like
provisions of clause (m) to restore property in the condition received and repair within
three months defects caused by lessee according to directions of lessor; clause (o) i.e. use
property as permitted in the lease or clause (p) i.e. not erect any permanent structure
without lessor's permission of section)
6) Allowing use of premise for immoral or illegal purpose:

• Tenant is in default and has failed to prevent deterioration of the premises


• Tenant is causing nuisance or annoyance to the neighbor including the landlord;
• For tenant who have rented for residential purpose and if the tenant has acquired or
constructed, or allotted, a house or flat (one year is allowed for vacating the premises)
• If the landlord is a member of the Armed Forces and requires it for occupation of his
family and produces a certificate of the prescribed authority certifying he is serving at a
non-family post or in special conditions
• If the rented property is not used and kept locked by the tenant or persons claiming
through him for 10 out of 12months.
• Where landlord got the property via transfer, he may file a suit within a year of such
transfer for the recovery of possession of the premises on the ground of needing it for
building or rebuilding or addition or alteration or for own occupation
• A two-year bar on litigation on property to which an earlier Act applied
• When a tenanted premise is transferred then the transferor's right to evict a tenant on
grounds of subletting as per Sec 13(1)(a) not supported by tenancy deed is also
transferred.

In Mohammad Ahmad &Anr vs. Atma Ram Chauhan & Ors.CA No.4422 of 2011 [Arising out of
SLP.(C) No.6319 of 2007] Supreme Court Justices Dalveer Bhandari and Deepak Verma
formulated "guidelines and norms …so as to minimize landlord-tenant litigation at all levels".
Its ratio was when rent is paid on time despite any agreement between the parties to the
contrary; the landlord cannot evict the tenant for five years. It recognizes need of premise for
personal use of landowner as an exception to the five-year rule. Further rent must be
increased by ten percent every three years or as agreed by the parties. Being a Supreme Court
judgement, it will apply across the country and to the courts hearing related matters Hence it
is important to review the guidelines as formulated in the case which are as follows:

• The tenant must enhance the rent according to the terms of the agreement or at least by ten
percent, after every three years and enhanced rent should then be made payable to the
landlord. If the rent is too low (in comparison to market rent), having been fixed almost 20 to
25 years back then the present market rate should be worked out either on the basis of
valuation report or reliable estimates of building rentals in the surrounding areas, let out on
rent recently.
• Apart from the rental, property tax, water tax, maintenance charges, electricity charges for the
actual consumption of the tenanted premises and for common area shall be payable by the
tenant only so that the landlord gets the actual rent out of which nothing would be
deductible. In case there is enhancement in property tax, water tax or maintenance charges,
electricity charges then the same shall also be borne by the tenant only.
• The usual maintenance of the premises, except major repairs would be carried out by the
tenant only and the same would not be reimbursable by the landlord.
• If any major repairs are required to be carried out then in that case only after obtaining
permission from the landlord in writing, the same shall be carried out and modalities with
regard to adjustment of the amount spent thereon would have to be worked out between the
parties.
• If present and prevalent market rent assessed and fixed between the parties is paid by the
tenant then landlord shall not be entitled to bring any action for his eviction against such a
tenant at least for a period of 5 years. Thus, for a period of 5 years the tenant shall enjoy
immunity from being evicted from the premises.
• The parties shall be at liberty to get the rental fixed by the official valuer or by any other
agency, having expertise in the matter.
• The rent so fixed should be just, proper and adequate, keeping in mind, location, type of
construction, accessibility with the main road, parking space facilities available therein etc.
• Care ought to be taken that it does not end up being a bonanza for the landlord.
These are some of the illustrative guidelines and norms but not exhaustive, which can be
worked out between landlord and tenant so as to avoid unnecessary litigation in Court.

Supreme Court of India in Ranjit Kumar Bose &Anr vs Anannya Chowdhury &AnrCA No.
3334/2014 held that Section 6 of the Tenancy Act overrides a contract between the landlord
and the tenant … that Sec 6 of the Tenancy Act is one such law which clearly bars arbitration in
a dispute relating to recovery of possession of premises by the landlord from the tenant. Since
the suit filed by the appellants was for eviction, it was a suit for recovery of possession and
could not be referred to arbitration because of a statutory provision in Section 6 of the
Tenancy Act. This upholds decision in Natraj Studios (P) Ltd. v. Navrang Studios & Anr,1981 1
SCC 523 which concluded that arbitration agreements between parties whose rights are
regulated by the Bombay Rent Act cannot be recognized by a court of law since the exclusive
jurisdiction is vested in Bombay Small Causes Court.

In Rahman JeoWangnoo vs. Ram Chand (AIR 1978 SC 413) Supreme Court held that the
provision contained in the West Bengal Premises Tenancy Act, 1956 mandates the court to
consider whether partial eviction as contemplated therein should be ordered or the entire
building should be directed to be vacated. Supreme Court also held in Anamika Roy v
JatindraChowrasiya&ors CA No. 4539 of 2013 that in a three-bed room flat with one kitchen, it
is too harsh to consider partial eviction and order to divide a flat between parties. Besides
partial eviction meeting the needs of the building owner had also not been argued by the
defendant at any stage.
Dr Suhas H Pophale v Oriental Insurance Company Limited and its Estate Officer Deals
with the applicability of the Public Premises (Eviction of Unauthorized Occupants) Act, 1971
and the State's rent control act. DrPophale was the tenant of Indian Mercantile Insurance Co.
(MIC) and was a lawful tenant under the Bombay Rent Act 1947 (now Maharashtra Rent
Control Act, 1999). MIC was merged with the Oriental Insurance Company a government
company, with effect from 1 January 1974 and the management of MIC was taken over by the
Central Government in 1971. Eviction notice was served on DrPophale. For any premises to
become public premises the relevant date will be 16 September 1958 or whichever is the later
date on which the concerned premises become the public premises (as belonging to or taken
on lease by Public Authorities like OIC). All those persons failing within the definition of a
tenant occupying the premises prior thereto will not come under the ambit of the Act and
cannot therefore be said to be persons in "unauthorized occupation". If possession of their
premises is required, that will have to be resorted to by taking steps under the respective
State Acts.

PROVISION REGARDING RENT

1) Despite any contract between parties, rent in excess of fair rent cannot be charged

2) Fair rent is to be paid within the time fixed by the contract or, in the absence of any such
contract, by the fifteenth day of the next month following the month for which it is
payable, provided the tenant may pay the rent payable for any month at any time during
such month before it falls due.
3) Other than one month's rent, no other premium in addition to rent can be charged by the
landlord at the time of grant, renewal or continuance of a tenancy of any premises,
whether in cash or in kind in addition to rent without the previous permission of the
Controller appointed under the Act. Any advertisement setting a requirement for any
payment in excess of a month‘s rent as advance is banned

4) The tenant or any other person acting on his behalf cannot claim or receive any payment
for relinquishment, transfer or assignment of the tenancy

5) Sale of furniture on premise to the tenant cannot be made a condition for the grant,
renewal or continuance of the tenancy of such premises. Any advertisement advertising
such a requirement is also banned
6) Where any sum in excess of provisions of the Act is paid, on application to the Controller
within a period of six months from the date of such payment, the Controller can direct the
landlord to refund such sum or adjust against future dues.

FIXATION OF FAIR RENT (SEC 17)

1) The landlord or the tenant can apply to the Controller for fixation of the fair rent of any
premise.

2) For a property constructed and let out after the year 1984 - fair rent will be six and three-
fourth per cent per annum of the amount of the actual cost of construction and the
market price of the land on the date of commencement of construction

3) The cost of construction of premises includes the cost of water supply, sanitary and
electric installation and as per the Public Works Department rates of the State
Government as applicable for the area. A ten percent variation (+/-) is allowed by the
Controller.
4) For computing the market value of land, the controller is to consider only that portion of
the site on which the premises were constructed and as an amenity, add a further sixty per
cent of the appurtenant vacant land.

5) As per sec. 17(3) of the West Bengal Premises Tenancy Act, for premises constructed in or
before the year 1984,

a. If a tenancy subsists for twenty years or more, amount considered for fair rent, is at
most three times (in case of residential property) and five times (in case of
commercial purposes) the rent as on 1.7.1976, and then in case of residential purpose
only deducting the increase, if any, in the manner provided in Schedule II

b. tenancy subsists for ten years or more but less than twenty years, at most two times
(in case of residential property) and three times (in case of commercial purposes) the
rent as on 1.7.1986 and then in case of residential purpose only deducting the
increase, if any, in the manner provided in Schedule

c. Or in both cases by accepting the existing rent if such rent is more than the
increased rent determined according to that Schedule. If above processes do not
apply then the fair rent shall be such as would be reasonable, having regard to the
situation, locality and condition of the premises and the amenities and considering
rent payable in respect of other similar premises.

6) In case the landlord contravenes provision, tenant may file a complaint under Sec. 28 of
the Act.

7) As per the statute the fair rent initially fixed shall be automatically increased by five per
cent every three years unless the state government notifies another rate of increase. Fair
rent as fixed by the Controller takes effect from the month of tenancy next after the date
of application.

8) Any increase of rent by the owner has to be preceded by a notice as provided in the Act
to the tenant. Usually the Rent Controls Acts have provisions like the landlord or the
tenant can approach Rent Control Court for revision of rent at an interval of years (three
or five years).

9) Where the landlord does not accept any rent, the tenant shall remit the rent to the
landlord by postal money order within fifteen days of such refusal. If the postal money
order is returned to the postal authority as undelivered, either on account of the landlord
having refused to accept the payment thereof or for any other reason, the tenant may
deposit such rent with the Controller within fifteen days from the date on which it is so
returned to the tenant. If there is doubt as to the person or persons to whom rent is
payable, the tenant may deposit such rent with the Controller. Deposit to Controller has to
be supported by an affidavit of the tenant identifying the property, address of landlord
and the reason for depositing the same with the Controller. The Controller then informs
the tenant the fact of payment.

10) According to Sec 4(2) and (3) of the West Bengal's Act, a landlord is bound to keep the
premises in good condition. The low rent cannot be an excuse to not repair the premise.
He is bound to take measures to ensure essential supply or service comprised in the
tenancy. For effecting essential repairs, tenant may make an application to the rent
controller under Sections 35 and/or 36 of the Act.
11) Supreme Court has held in Ranjit Kumar Bose &Anr vs Anannya Chowdhury &AnrCA No.
3334/2014, in matters of tenant eviction the state act shall prevail. Cases under Rent
Control Act are heard by the Rent Control Act where constituted and generally Munsif's
Court executes the order passed by the Rent Control Court.

CONSEQUENCE OF RENT CONTROL IN INDIA


Following adapted from an interesting analysis in a working paper by (Dev & Dey, 2006) are
some of the consequences of Rent Control and thus point to the need for change.

1) Rent control places an arbitrary cap on income operates as disincentive for people with
resources and otherwise capable of to investing in creating rental housing

2) Rentcontrolintroducedunderstresssituationlikewar,partitionofcountryleading
to heavy influx of people tends to take on permanent character.

3) Leads to rapid deterioration of houses as it has eroded all interest in investing in the
upkeep of the existing apartment

4) Fear of losing control on property brought under rent control forces owners to withdraw
their vacant premises from tenancy market

5) Difficult to evict tenant in rent control premise and selling house with tenant reduces the
marketability of the title

6) Property tax is reduced as it is computed as a percentage of Annual Ratable value or other


rent computation as the income that the property fetches gets capped under rent control
act irrespective of the locality or demand for housing there

7) State must invest much to administer rent control act and enforce its provision, itself
earning little revenue from the taxes

8) Operation of tenancy persisting across generations in India leads to locking out of the
house property from the market

9) Selective applicability of rent control reduces its ambit - rent control may not apply to
new properties for a period of say ten years

10) Landlords refuse to issue rent receipt in a bid to prevent creation of proof of tenancy and
ends up fueling black money economy

Where the Act permits, the landlord can improve the property without the consent of the
tenant, in turn increasing the standard rent and thus the burden on the vulnerable

MESNE PROFITS OF PROPERTY

Is collective term for profits which a person in wrongful possession of a property actually
receives or with ordinary diligence could have received together with interest thereon. But if
any improvements to property were made by the person in wrongful possession and profits
accrued due to such improvement then it is not included. Prayer of mesne profits is prayed for
in conjunction with a decree for possession and mesne profits under Order 20 Rule 12 of the
Civil Procedure Code, 1908 on following grounds:

Where a suit is for the recovery of possession of immovable property and for rent or
mesne profits, the court may pass a decree—
1. for the possession of the property;
2. for the rents which have accrued on the property during the period prior to the
institution of the suit or direction an inquiry as to such rent;
3. for the mesne profits or directing an inquiry as to such mesne profits;
4. directing an inquiry as to rent or mesne profits from the institution of the suit until—
i. the delivery of possession to the decree holder,
ii. the relinquishment of possession by the judgment debtor with notice to the
decree holder through the court, or
iii. the expiration of three years from the date of the decree, whichever event first
occurs.

Where an inquiry is directed under clause (b) or clause (c), a final decree in respect of
the rent or mesne profits shall (b)

RIGHT OF WAY, SECTION 52 -LICENSE UNDER THE INDIAN EASEMENT


ACT,1882

A license is a personal right granted to a person to do something upon immovable property


of the guarantor and does not amount to the creation of an interest in the property itself. It is
purely a permissive right and is personal to the guarantee. It creates no duties and obligations
upon the person making the grant and is therefore, revocable except in certain circumstances
expressly provided for in the Act itself. The license has no other effect than to confer a liberty
upon the licensee to go upon the land which would otherwise be unlawful.

Section52oftheIndianEasementActdefineslicenseasfollows:―Whereoneperson grant to another,


or to a definite number of other persons, a right to do, or continue to do, in or upon the
immovable property of the grantor, something which would, in the absence of such right, be
unlawful, and such right does not amount to an easement or an interest in the property, the
right is called license.‖

CHARACTERISTICS OF LICENSE

1) No transfer of interest:- A license is a permission to do some act which, without such


permission, would be unlawful.

2) No interest in accretions: - A licensee has no interest in the property and therefore, he


acquires no right by accretion

3) Neither transferable nor heritable:- A license is neither transferable; nor heritable.

4) A license is a matter purely personal between grantor and grantee.


5) Section 52 of Easement Act does not require any consideration, material or
non-material, to be an element of the definition of license, nor does it require that the
right under the license must arise by way of contract or as a result of mutual promises.
6) The person who grants the license must be the owner of the property. The other person
who gets the permission must be a stranger or have no right in the property.

7) License creates no duties and obligations upon the person making the grant and is
therefore revocable except in certain circumstances expressly provided in the Act itself.
8) A license is usually revocable by grantor, except in the two cases mentioned in the section
60 of Easement Act.

9) A subsequent transfer of the property terminates license.

10) A licensee cannot sue trespassers and strangers in his own name.

11) A license is terminated by death of either party.

SALIENT FEATURES OF RERA,2016


Salient Features of Real Estate (Regulation & Development) Act, 2016 and Real Estate
Regulating Authorities Established under the Act.

The Real Estate (Regulation and D1evelopment) Act, 2016 aims to regulate and promote the
real estate sector by regulating the transactions between buyers and promoters of residential
as well as commercial projects. It also has provisions for establishing a regulatory authority at
state level called "Real Estate Regulatory Authority" (RERA) for monitoring the real estate
sector and adjudicating disputes relating to Real Estate Projects. The main aim of the Act is to
protect buyers and help investment in Real Estate Sector.

MAIN OBJECTIVES

1) Enhance transparency and accountability in real estate and housing transactions;

2) Providing uniform regulatory environment to ensure speedy adjudication of disputes and


orderly growth of the real estate sector;

3) Boosting domestic and foreign investment in the Real Estate sector;


4) Promote orderly growth through efficient project execution and standardization;

5) Offer single window system of clearance for real estate projects.

SALIENT FEATURES OF THE ACT

• Establishment and incorporation of Real Estate Regulatory Authority (RERA) at every State
in India for monitoring and adjudicating disputes relating to real estate projects (Section
20).

• Establishment of fast track dispute resolution mechanism for settlement of real


estate disputes through dedicated adjudicating officers and Appellate Tribunal (Section 43
& 44).

• Registration of all real estate projects is made mandatory with RERA having territorial
jurisdiction over such projects. No sale in a real estate project can be made without
registration of the project with RERA (Section 3). RERA can also refuse to register a
project, if the same is not compliant with provisions of the Act. Registration of a project
can even be cancelled, in case, RERA receives any complaint and the same is found to be
correct after inquiry.

• It is mandatory for a promoter to upload details of proposed project on the website of


RERA, including details of registration, types of apartments or plots booked, list of
approvals taken and the approvals which are pending subsequent to commencement
certificate, status of the project, sanction plan, layout plan etc. (Section 11).

• RERA shall approve or reject the application for registration within 30 days, failing which it
shall be deemed to have accepted the application for registration (Section 5).

• Any promoter shall not accept a sum more than ten per cent of the cost of the apartment,
plot, or building as the case may be, as an advance payment or an application fee, from a
buyer without first entering into a written agreement for sale with such person and
register the same. (Section13)
• It has been made obligatory for the promoters to deposit 70% of the money collected
from buyers for a particular project in a separate account that will cover the cost of land
and construction and the same can be withdrawn only after certification from an engineer,
an architect and a chartered accountant. [Section 4(2)(l)(D)].
• It is now obligatory for all the promoters to obtain insurance in respect of title of the land
and buildings and construction of every project. (Section16).

• The promoter shall not transfer or assign his majority rights and liabilities in respect of a
real estate project to a third party without obtaining prior written consent from at least
2/3rd no. of Allottees, except the promoter, and without the prior written approval of
RERA [Section15(1)].

• Both promoter and buyer are liable to pay equal rate of interest in case of any default
from either side (Section 2(za)(i)).

• The promoter shall compensate the buyer in case any loss caused to him due to defective
title of the land, on which the project is being developed or has been developed, in the
manner as provided under this Act, and the claim for compensation under this subsection
shall not be barred by limitation provided under any law for the time being in force.
(Section18)

• An aggrieved person may file a complaint with RERA, as the case may be, for any
violationorcontraventionofprovisionsofthisActorrulesandregulationsmade
there under against any promoter, buyer or real estate agent. (Section 31)

• During the pendency of enquiry, RERA can restrain any promoter, buyer or agent from
continuing with the act complained of. (Section36)

• A person aggrieved by any direction or decision or order made by RERA or by an


adjudicating officer under this Act may prefer an appeal before the Appellate Tribunal
having jurisdiction over the matter. (Section43)

• If a promoter continues to violate the provisions of Section 3, he shall be punished with


imprisonment for a term which may extend to three years or fine which may extend to ten
percent of the estimated cost of the project or both [Section59(2)].

• If a promoter fails to comply with orders or directions of RERA, he shall be liable to a


penalty, which may extend up to five percent, of the estimated cost of the project as
determined by the Authority (Section 63).
• If a promoter fails to comply with the orders or directions of the Appellate Tribunal, he
shall be punished with imprisonment for a term which may extend to three years or fine,
which may extend up to ten percent of the estimated cost of the project, or with both
(Section64).

• Where an Offence under this Act has been committed by a company, every person who,
at the time, the offence was committed was in charge of, or was responsible for the
conduct of business of the company, as well as the company, shall be deemed to be guilty
of the offence and shall be liable to be proceeded against and punished. (Section 69)

• No civil court shall have jurisdiction to entertain any suit or proceeding in respect of any
matter which RERA or the adjudicating officer or the Appellate Tribunal is empowered by
or under this Act to determine and no injunction shall be granted by any court or other
authority in respect of any action taken or to be taken in pursuance of any power
conferred by or under this Act. (Section79)

REGULATORY FRAMEWORK–REAL ESTATE (REGULATION AND DEVELOPMENT) ACT,2016

• Establish the Real Estate Regulatory Authority for regulation and promotion of the real
estate sector.
• Ensure sale of plot, apartment of building, as the case may be, or sale of real estate
project, in an efficient and transparent manner.
• Ensure protect the interest of consumers in the real estate sector.
• Establish an adjudicating mechanism for speedy dispute redressal and also to establish
the Appellate Tribunal to hear appeals from the decisions, directions or orders of the Real
Estate Regulatory Authority(RERA)
• Regulates transactions between buyers and promoters of residential real estate projects
• Establishes state level regulatory authorities called Real Estate Regulatory Authorities
(RERAs)
• Residential real estate projects, with some exceptions, need to be registered with RERAs
• Promoters cannot book or offer these projects for sale without registering them. Real
estate agents dealing in these projects also need to register with RERAs
• Registration, the promoter must upload details of the project on the website of the RERA.
These include the site and layout plan, and schedule for completion of the real estate
project
• Amount collected from buyers for a project must be maintained in a separate bank
account and must only be used for construction of that project. The state government can
alter this amount
• Right to Legal Representation on behalf of Client by Company Secretaries or chartered
accountants or cost accountants or legal practitioners
• Imposes stringent penalty on promoter, real estate agent and also prescribes
imprisonment.

TRANSFER OF PROPERTY ACT (TPA), 1882

The Transfer of Property Act came into existence in 1882. Before that, the transfer of
immovable property was governed by principles of English law and equity. The preamble of
Act sets out the objectives of the legislation. Scope of this Act is limited. It applies only to
transfer by the act of parties and not by operation of law. Also, this Act deals with a transfer of
property inter vivos, i.e., a transfer between living persons. It contains transfer of both
movable and immovable property, but a major portion of the enactment is applicable to the
transfers of immovable properties only. The Act is not exhaustive.

STATUTORY DEFINITIONS (SECTION 3)

• IMMOVABLEPROPERTY

Definition in Section 3 is not exhaustive. It says only that ‗immovable property‘ does not
include standing timber growing crops or grass. Definition of immovable property in Section
3(26) of General Clauses Act, 1897, is also not exhaustive. It defines immovable property as it
shall include land, benefits to arises out of land, and things attached to earth. Thus, we find
that while Transfer of property excludes certain things. General
Clauses Act includes certain things under the head ‗immovable property‘. By combing both
definitions, we may say that, the term includes land, benefits to arises out of lands, and things
attached to the earth, except standing timber, growing crops and grass.

1. Land: It means a determinate portion of the earth‘s surface, which may be covered by
water, the column of surface above the surface, the ground beneath the surface. All the
objects which are on or under the surface in its natural State are included in the term land.
Also, all objects placed by human agency on or under the surface with the intention of
permanent annexation are immovable property, e.g., Building, wall, fences.

2. Benefits to arise out of land: Apart from physical point of view, every benefits arise out
of land is also regarded as immovable property. Registration Act also includes as
immovable property benefits to arise out of land, hereditary allowances, right of way,
lights, ferries and fisheries. In Anand Behera v. State of Orissa, AIR 1956 SC 17, the right to
catch away fish from chilka lake, over a number of years, was held to be an equivalent of
profits a pendrein England and a benefits to arise out of land in India. Similarly, a right to
collect a rent and profits of immovable property, right to collect dues from a fair or heat
or market on a land are immovable property.

3. Thingsattachedtoearth:Section3oftransferofpropertydefinestheexpression
‗attachedtoearth‘asincluding(1)thingsrootedintheearth,(2)thingsembeddedin the earth, (3)
things attached to what is so embedded, and (4) chattel attached to earth or building.

• Things rooted in earth include trees and shrubs, except standing timber, growing crops
and grasses (Section 3, TPA). Whether tress regarded as movable or immovable
depends upon the circumstances of the case. If the intention is that trees should
continue to have the benefit of further sustenance or nutriment by the soil (land), e.g.,
enjoining their fruits, then such tree is immovable property. But if the intention is to
out them down sooner or later for the purpose utilising the wood for building or other
industrial purpose, they would be timber and of accordingly be regarded as movable
property (Shanta Bai v. State of Bombay, AIR 1958 SC 532) determining whether the
tree is movable or immovable, the intention if party is important if the parties intend
that the tree should continue to have the benefit of further nutriment to be afforded by
soil, the tree is immovable property. But if intention is to withdraw the tree from land,
and the land is providing it only as a warehouse, it is to be treated as movable
property.

• Things embedded in earth: It includes such things as house, buildings, etc., however
certain things like an anchor imbedded in the land to hold a ship is not a immovable
property‘ to determine whether such things are movable or
immovable property, depends upon circumstances of each case and there are two
main conditions to indicate intention:
1. the degree or mode of annexation, e.g. tie-up seats fastened to the floor of
cinema halls are immovable property on brick-work and timber and tapestries;
2. the object of annexation, for, e.g., Blocks of stone placed one on the top of
other without any mater or cement for the purpose of forming a dry wall, will
become part of land, so immovable property, but not the stones deposited in
the builder‘s yard.

• Things attached to what is so embedded must be for the permanent beneficial


enjoyment of the to which it is attached, as section says for, e.g., door and windows of
a house are immovable property to be permanent, like electric fans or window blinds,
they are movable property.

• Chattel attached to earth or building if a chattel, i.e., movable property is attached to


earth or building, if is immovable property. The degree, manner, extent and strength of
attachment are the main features to be regarded in determining the question. Standing
timber, growing crops and grasses are regarded as severable from land and they are
regarded as movable property. However if they and the land on which they stand is
sold, such standing timber, growing crops or grasses will pass to purchases.

4. Standing timber: The word standing timber includes Babool Tree, Shisham, Nimb, Papal
Banyan, Teak, Bamboo, etc. The fruit berating tree like Mango, Mahua, Jackfruit, Jamun,
etc., are not standing timber, and they are immovable properties ( Fatimabibi v. Arrfana
Begum, AIR 1980 All 394). But if intention is to cut them down sooner or later for the
purpose utilising them as timber, and not to use them for the purpose of enjoying their
fruits, they are regarded as movable property. (T.A. Sankunni v. B.J. Philips, AIR 1972
Mad272).

5. Growing crops: Growing crops includes creepers like pan, angoor, etc., millets (Wheat,
Sugarcane, etc.), Veg like Lauki, Kaddo, etc. These crops don‘t have any own independent
existence beyond their final produce.

6. Grasses: It can only be used as fodder, and no other use is possible. Therefore it is
movable. But a contract to cut grass will be an interest in chattel, so is immovable
property. The following has been judicially recognised as immovable property:

• Right to collect rent of immovable property.

• Right to dues from a fair on a piece of land.

• A right offisheries.
• A right ofterry.

• A right ofway.

• Hereditary offices.

• The interest of a mortgagee in immovable property.

7. Minerals: Upon transfer of immovable property, things not only rooted to it, but also
anything found deep down below the property goes along with the transfer. All minerals
below the land sole are immovable property.

• MOVABLEPROPERTY

Transfer of property does not define movable property. In General Clauses Act, it is defined as
―Property of every description except immovable property‖. Some examples are right of
worship, royalty, machinery not attached to earth which can be shifted, a decree for arrear of
rent, etc.

1. Attestation

Attestation has been defined in Section 3 of Transfer of Property Act asunder:—


‖attested‖, in relation to an instrument, means and shall be deemed always to have meant
attested by two or more witnesses each of whom has seen the executant sign or affix his mark
to the instrument, or has seen some other person‘s sign the instrument in the presence and by
the direction of the executant a personal acknowledgment of his signature or mark, or of the
signature of such other person, and each of whom has signed the instrument in the presence
of the executant, but it shall not be necessary that more that one of such witnesses shall have
been present at the same time, and no particular form of attestation shall be necessary.

To ‗attest‘ means to sign and witness any fact of execution by the executant. It means that a
person has signed the document by way of testimony of the fact that he saw it executed.

Following are the essential requisites of a valid attestation-

1. There must be two or more attesting witness.


2. Each witness must see—(a) the executant sign or affix his mark (thumb impression) to
the instrument; (b) or, see some other person sign the instrument in the presence, and
by the direction, of the executant; (c) or, receive from the executant a personal
acknowledgment of his signature or mark or of the signature of such other person.

3. Each witness must sign the instrument in the presence of executant.


4. Each witness must sign only after the executant iscomplete.

5. It is not necessary that more than one of such witnesses should be present at the
sametime.

6. No particular form of attestation isnecessary.

7. Attester Should be sui generis, i.e., capable of entering intocontract.

8. The witness should have put his signature animoattestandi(intention toattest).

9. A. person who is a party to transfer cannot attest it as a witness. The object of


attestation is to protect the executant form being required to execute a document by
other party thereto by force, fraud, or undue influence, a party to the transaction
cannot laid down in Kumar Harish Chandra v BanshidharMahanty, AIR 1965 SC1738

Effect of invalid attestation it makes the deed of transfer of property involved, and therefore
no property invalid and therefore no property passes under it. The document cannot be
enforced in the court of law.

2. Notice

A person is said to have a notice of a fact when he actually knows that fact, or when, but for
wilful abstention from an inquiry or search which he ought to have made, or gross negligence,
he would have known it (Section 3).

Section 3 of Transfer of Property Act enumerates three kinds of notices—

• Actual or express notice

• Constructive or implies notice


• Imputed notice.

Actual notice: A person is said to have actual notice/express notice of a fact if he actually
knows it. It must be definite information given in the course of negotiations by person
interested in the property. A person is not bound to attend vaguerumours.

Constructive notice: It is a notice which treats a person who ought to have known a fact, as if
he actually knows it. A person has constructive notice of all the f acts of which he would have
acquired actual notice had he made those inquiries which he ought reasonably to have made.
Constructive notice has roughly been defined as knowledge which the court imputes to a
person upon a presumption so strong that it cannot be rebutted that the knowledge must be
obtained.
Legal presumption of knowledge arise when—(1) There is wilful abstention from an enquiry or
search. It means wilful or deliberate abstention to take notice of a fact which a reasonable
man would have taken in the normal cause of life. It is such abstention from
enquiry or search as would show want of bona fides in respect of a particular transaction.

Illustration

a) A contract to sell his house to B. The house is on rent and B knows that the tenants
have been paying the rents to C. B has constructive notice of the right of C to take
rents from the tenants.

b) A propose to sell his property to B, who at the same time knows that rents due in
respect of the property are paid by the tenants to a third person X. B will be fixed with
notice of the right of X. [Hunt v. Lack, (1902) 1 Ch429.]

c) A refuse a registered letter, which contains information relating to property which A


propose to purchase. A will be deemed to have notice of the contents of the latter.
[Ismail Khan‘ v. Kali Krishna, (1901) 6 Cal WN134]

Gross negligence: Negligence means carelessness or omission to do such act which a man of
ordinary prudence would do. Doctrine of constructive notice applies when a person, but his
gross negligence would have known the fact. Mere negligence is not penalised. It should be
high degree of neglect. In Hudston v. Vincy, (1921) 1 Ch 98, Eve J. said, ―Gross negligence
does not mean mere carelessness, but means carelessness of so aggravated a nature as to
indicate a attitude of mental indifference to obvious risk.‖ It can be described as ‗a degree of
negligence so gross that a court of justice may treat it as evidence of fraud, impute a
fraudulent motive to it and visit it with the consequences of fraud‘.

In Ltoyds Bank Ltd. v. P.E. Guzders and Co. Ltd., (1929) 56 Cal 868, a person A deposited title
deeds of his house in Calcutta with Bank. N to secure the loan he had taken from the bank.
Subsequently, A represented the Bank that intending purchases of the house wanted to see
the title deeds. The bank returned the deeds to A who deposited the deeds with the plaintiff
bank in order to secure a loan. It was held that the Bank N, on account of gross negligence in
parting with the deed has lost its prior rights with respect of the house.

In Imperial Bank of India v. U. Raj Gyaw, (1923) 50 IA 283, a purchases was informed that the
title deeds were in possession of a bank for safe custody and omitted to make any inquiry
from bank It was held that he was guilty of gross negligence and was deemed to have notice
of the rights of the bank which had the custody of the deeds.

Registration as notice: Explanation I to Section 3 provides that ‗where any transaction


relating to immovable property is required by law to be and has been effected by a registered
instrument, any person acquiring such property or any part, or share or interest in such
property shall be deemed to have notice of such instrument as from the
dateofregistration,‗Thusanypersoninterestedinthetransactionwhichisregistered
under the provisions of the Indian Registration Act, 1908 cannot plead that he has no notice
of the transfer made under the deed.

In order that, registration may be treated as constructive notice of its content, following
conditions must be satisfied:

1. The instrument must be compulsorilyregistrable.


2. All the formalities prescribed under the Registration Act are duly completed in the
mannerprescribed.

The instrument and particulars must be correctly entered in the registers.

After registration, document becomes a public document and the title can be confirmed in the
Registrar‘s office.

Actual possession as notice of Title: Explanation II of Section 3 provides that, ―any person
acquiring any immovable property or any share or interest in such property shall be deemed to
have notice of the title, if any, of any person who is for the time being in actual possession thereof.
―Thus in order to operate as constructive notice, possession must be actual, i.e., de facto
possession. It amounts to notice of title in another, e.g., A leased a house and garden to B who
takes possession of the properties. A then sells the said properties to C. C is deemed to have
constructive notice of B‘s rights over these properties, i.e., C cannot plead that he had no
knowledge (notice) of the fact of B‘s possessionontheproperties[Denielsv.Davison,(1809)16Ves240].

Imputed notice

Explanation III to Section 3 provides that, ―A person shall be deemed to have had notice of
any fact if his agent acquires notice thereof whilst acting on his behalf in the course of
business to Which that fact is material:

Provided that, if the agent fraudulently conceals the fact, the principal shall not be charged
with notice thereof as against any person who was a party to or otherwise cognizant of the
fraud‖

This is based on the maxim Qui facit per aliumfacit per se, i.e., he who does by another, does
by himself. In MohoriBibee v. D. Gliosh, (1903) 30 Cal 539, held that although the principle was
absent from Calcutta and did not take part in the transaction personally, his agent in Calcutta
stood in his place for the purposes of the transaction and the acts and knowledge of the latter
were the acts and knowledge of the principal.

3. ActionableClaim

Section 3 of the Transfer of Property Act, 1882 defines actionable claim as, ―it means a
claim Actionable Claim to any debt, another then a debt secured by mortgage of immovable
property or by hypothecation or pledge of movable property or to any beneficial interest in
movable property , not in the possession, either actual or constructive, of the claimant, which
the civil courts recognise as affording grounds for relief, whether such debt or beneficial
interest be existent, accruing, conditional or contingent.‖
Thus, according to Section 3, actionable claim means—

1. a claim to unsecured debt,or


2. a claim to beneficial interest in movable property not in possession of the claimant.

Debt: A debt is an obligation to pay a liquidated sum of money. The amount of money must
be certain, otherwise it is not debt. For an actionable claim, a debt must be unsecured debt,
for secured debts are excluded from the definition of actionable claim, e.g., A owes Rs. 10.00
to B. B‘s claim is an actionable claim.

Debits secured by a mortgage of immovable property or by a pledge of movable property are


excluded from the definition of actionable claim, because they are secured debt.

Claim to beneficial interest not in possession of the claimant: Actionable claim includes a
claim to any beneficial interest in movable property, not in the possession of the claimant,
interest or the right of possession of claimant is recognised by the court. (However a claim for
demands, i.e., for an unascertained. sum of money or a claim for mesne profits does not come
within the definition if actionable claim), e.g., A agrees to sell to B bales of cotton deliverable
on a future day B has a beneficial interest in the goods and it is an actionable claim.

Some instance of actionable claim:


• Claim for arrears ofrent.
• Claim for the money due under insurance policy (Shamdas v. Savitri, AIR 1937Sind 24)
• Claim for the return of earnestmoney.
• A share inpartnership.
• maintenance allowance payable infuture.
• fixed deposit in abank.
• right to the proceeds ofbusiness.
• Hire-Purchaseagreement.
• Instances of claims which are notactionable:
• Acopyright,
• Claim for mesneprofits,
• Secureddebt,
• Right to get damages,
• A debt which has passed into adecree.

Transfer of Actionable Claims

Actionable claims are transferable properties; thus it can be transferred by way of sale,
mortgage, gift or exchange. Section 130 of Transfer of Property Act provides the mode of
effecting the transfer of actionable claims, and its effect. Itprovides—

The transfer of an actionable claim whether with or without consideration shall be


effected only by the execution of an instrument m writing signed by the transferor or his duty
unauthorized agent, shall be complete and effectual upon the execution of such instruments,
and thereupon all the rights and remedies of the transferor, whether by way of damages or
otherwise, shall rest in the transferee, whether such notice of the transfer as is hereinafter
provided be given ornot:

Provided that every dealing with debt or other actionable claim by the debtor or other person
from or against whom the transferors would, but for such instrument of transfer as aforesaid,
have been entitled to recover or enforce such debt or other actionable claim, shall (save where
the debtor or other person is a party to the transfer or has received express notice thereof as
hereinafter provided) be valid as against such transfer.The transferee of an actionable claim
may, upon the execution of such instrument of transfer as aforesaid, sue or institute
proceedings for the same in his own name without obtaining the transferor‘s consent to such
suit or proceeding and without making him a party thereto.‖ Section 131 of the Act provides
that the notice should be in writing signed. Itprovides:

―Every notice of transfer of an actionable claim shall be in writing signed by the transferor or
his agent duly authorised in this behalf, or in case the transferor refuses to sign, by the
transferee or his agent, and shall state the name and address of the transferee.‖

Section 132 of the Act provides the liability of transferee of actionable claim. It reads,
―Thetransfereeofanactionableclaimshalltakeitsubjecttoalltheliabilitiesandequities to which the
transferor was subject in respect thereof at the date of the transfer‖.,

E.g., A transfers to C a debt due to him by B, A being then indebted to B. C sues B for the debt
due by B to A. In such suit it is entitled to set off the debt due by A to him; although C was
unaware of it at the date of such transfer.
• TRANSFER OF IMMOVABLE PROPERTY: SALE, MORTGAGE, GIFT, EXCHANGE,
ASSIGNMENT, CHARGE, LIEN, TENANCIES/ SUB-TENANCIES.
Concept of Moveable and Immoveable Property

As per the Securitisation and Reconstruction of Financial Assets and Enforcement of Security
Interest Act, 2002 (SARAFESI) the term "property" means -

1. immovableproperty;

2. movableproperty;
3. any debt or any right to receive payment of money, whether secured or unsecured;

4. receivables, whether existing orfuture;


5. intangible assets (know-how, patent, copyright, trade mark, License, franchise or any
other business or commercial right of similar nature likegoodwill)

TheSARAFESIActalthoughlittleoutofplaceherebuttheambitoftheterm―property" in it is relevant
for our understanding. However, valuation for secured lending centres more on immoveable
property and certain moveable assets like plant and machinery and vehicles.

The term immoveable property is defined in several other statutes: Sale of Goods Act, 1930,
General clauses Act 1897 (Sec 3(26)), Registration Act, 1908 (sec 2(6), Income tax Act, 1961
(sec 269UA (d)), Transfer of Property Act, 1882 (sec 3). The definitions varyfrom one statute to
another in terms of the detailcovered.

The General Clauses Act states Immoveable property includes land, benefits to arise out of
land and things attached to the earth or permanently fastened to anything attached to it.
Hence, land per se and mines, agriculture and forests being benefits from land are covered.
Any machinery fastened securely to land would also be covered but not wooden poles etc.
that is similarlyfixed.

The sec 3 of Act TPA defines immoveable property as something that does not include
"standing timber, growing crops or grass". The definition is criticised for focusing on what it is
not, rather than focusing on defining it but it stresses on severability from the earth as a
paramount consideration in classifying moveable or immoveable property. This Act deals with
transfer of property as a whole and covers moveable property too. In some cases, like the
transfer of moveable property, transaction is complete with delivery of property to the buyer/
recipient but not so in case of immoveable property. When the Registration Act 1908, and
Stamp Duty Act and Transfer of Property Act areread
together in case of immoveable property it is not enough to give possession, registration of
the transfer document and the payment of due stamp duty is also necessary. But the party in
possession and supported by unregistered sale deed, does have some rights stemming from
the doctrine of part performance (Sec 53A of the Transfer of Property Act) read with Sec 49 of
the Registration Act (i.e. an unregistered document can at best be admitted as secondary
evidence under the Indian Evidence Act to prove title) and cannot be dispossessed without
due process of law.

Transfer of Property Act, 1882 requires registration of all such deeds which transfer real estate
of value of over Rs. 100/-. The Promoter's Act requires registration of all the Deed related to
transfer of flat from Promoter to purchaser and some other agreements too.

• TRANSFER
Transfer of Property Act, 1882 is crucial to understanding the laws of transfer of Real Estate
and provides for transfer through sale, exchange, mortgage, lease, lien and gift.

Some crucial points of pertaining to definition of transfer under section 5 of Transfer of


Property Act are:

1. Governs transfer of property between one or more living persons on either side (inter
vivo). The section itself clarifies that the concept of living persons includes company or
other registered or unregistered association of persons or body of individuals.
However, it further clarifies that TPA provisions are not in derogation or contradiction
of provisions in other statute also affecting transfer ofproperty.

2. Transferor of property can also be the transferee (recipient) of the property either
alone or along with other living persons. For e.g. if the property is transferred to a
Trust created to manage the property for the benefit of the transferor and who is also
aTrustee
3. The line "conveys Property in present or future" in the definition leads to some
confusion. One court decision held that "present or future" qualifies the term
"conveys" - as determined by SC in 1955 in Jugal Kishore Saraff v Raw Cotton Co. Ltd
AIR 1955 SC 376. But in 1969 SC conceded there can be contract to transfer future
property under sec 40 & 43 of TPA in Bharat Nidhi Ltd. vs. Takhatmal AIR 1969 SC 313.
Thus, property in question may be in existence or of a future property (from will).

4. It is deliberate and is the result of the intention of the parties involved (i.e. the Act
does not cover situations like transfer of property due to forfeiture of property given
as security in mortgage, when property is impounded on any order of court, intestacy
(succession following of holder dying without making a will or a trust for
management). However, a point worth noting here is - TPA also does not cover "family
settlement"/"family arrangement" or a "family partition"
(whichalsooccursattheintentionofthepartieswhohavepre-existingtitleor
claim on property). A family settlement between all eligible adults only tries to pre-
empt and prevent future disputes around a property by determining the rights of
successors or those having other bases for claim in the property and hence is usually
not transfer under TPA. In a partition, one person who is a joint owner gets some
specific property in lieu of his rights on common property and thus no new property is
really being transferred and only pre-existing joint title is being severed and
reallocated, that too is not a transfer under TPA]

5. Can cover transfers of entire bundle of rights (as in sale) or only carved out part of
total rights (e.g. leasehold,mortgage).

The transfer of property is dealt in other Acts as well. The Allahabad High Court in Misc No.
5937 of 2015 Sahara Grace Apartments Owners Assoc vs. State of U.P. has stated the
"provisions of the Transfer of Property Act, 1882, in view of Section 29 of the U.P. Apartment
Ownership Act, 2010, shall in so far as they are not inconsistent with the provisions of the Act
apply to the transfer of any apartment together with its undivided interest in the common
areas and facilities appurtenant thereto made by the owner of such apartment, where such
transfer is made by sale, lease, mortgage, exchange, gift or otherwise as they are applied to
the transfer of any immovable property." That is TPA shall apply only if not inconsistent with
the Apartment Act. Some Acts overtly assert the applicability of TPA.

• WHAT CAN (NOT) BETRANSFERRED?

This Sec 6 of TPA does not really details what can be transferred under it. This section says
everything can be transferred but then goes on to list what cannot be transferred. One's most
relevant to real estate valuation only are described below:

1. Spec successionisi.e. the mere expectation/ possibility of succeeding as heir to a


property of a living person at a future date. This future right is transferable in UK but
TPA bars it in India. In the same class is the chance of being left a legacy by some
living person which is similarly barred. Even if the living person whose property is the
subject matters gives consent, even then the right to the property is nottransferable.

2. Lessor and lessee are governed by the lease deed which may provide for conditions
that the lessee has to honour for continued enjoyment of the property. It will also
provide that any of these conditions are subsequently breached then the lease stands
cancelled (i.e. lessor gets right of re-entry into the leased property). The mere
possibility of a condition being breached by lessee and property reverting to lessor is
a mere chance and therefore cannot be transferred by lessor. However, if the whole
interest in the property including the interest that will revert to lessor on the end of
the lease period is being transferred then such right of re-entry will also stand
automaticallytransferred.
3. As we shall see in the chapter on easement - there are two properties involved in an
easement - the dominant heritage and servient heritage. The property, owner of which
is enjoying the easement right, is the dominant heritage and the property on which
that right is asserted is the servient heritage. Easement right is defined in Easement
Act "a right which owner or occupier of certain land possesses as such for the
beneficial enjoyment of that land, to do and continue to do something or to prevent
and continue to prevent something being done in or upon or in respect of certain
other land, not his own". An easement right by itself cannot be transferred unless the
dominant heritage is also transferred.
4. Any interest in property that is restricted to personal enjoyment cannot be transferred.
E.g. If B has a life interest in i.e. right of maintenance from a property but no right to
transfer the property, then the right of maintenance cannot be transferred by B to
anyone else.

5. A right to receive maintenance cannot be transferred, but any arrears not paid can be
transferred. However, an annuity i.e. when a fixed amount is payable, is transferable.
Maintenance amount can vary depending on the needs of that time period and hence
is not quantifiable. Since such a charge can be placed on income accruing from a
property hence in that sense it is relevant to understanding real estate.
6. Mere right to sue is not transferable. Sue means to pursue a matter legally in
appropriate court/ forum. So merely transferring right to recover mesne profits is not
valid. But if the property is sold together with right to recover mesne profits then it is
valid.

In Amirtham vs. Sarman AIR 1991 SC 1256 - Supreme Court has clarified if a property
belonging to a minor is transferred by his guardian without legal necessity or for meeting
minor's wants/ benefit and also without court's permission then within three years of the
minor attaining majority he can file a suit to set aside the transfer. If instead, on attaining
majority the said minor transfers the right to sue for his alienated property to a third person -
then that transfer is not hit by sec 6 for a period of time till three years after minor attains
majority. The transferee from minor X is competent to file a suit within three years of the
minor's attaining majority to set aside the sale.

• PERSONS COMPETENT TOTRANSFER

Sec 7 gives essential competency to execute a transfer of property deed. As per definition the

persons competent to transfer are the following

1. Persons competent to contract: to understand which we have to refer to sec 11 of the


Indian Contract Act which in turn enumerates a person may contract if he has

a) attained the age of majority (i.e. a minor cannot contract and thus cannot transfer his
property by any method)
b) is mentally sound and
c) not disqualified under law from contracting

2. Entitled to transfer: person having title to the property including for example
coparceners in a joint property if a partition between the holders is first affected; a Hindu
father/ grandfather or great-grandfather of the joint property held respectively with sons,
grandsons, great-grandsons to pay off a personal debt of the senior that was contracted
earlier and is not immoral.

3. Includes any person not holding title to the property but having valid authority to
transfer - e.g. (1) vide duly executed power of attorney a person is permitted to alienate
property of another then it is valid (2) a member of a housing cooperative registered
under WB Cooperative Societies Act, 1973 does not own the flat he purchased but can
nominate a person to take his flat and his share in the cooperative in return for money (3)
Managing Partner does not own the property of the Partnership firm but if the deed
permits it he can alienate the property by way of mortgage etc.

Since we are on the transferor's competency it may be good to see limits, if any, on who may
be a transferee under the Transfer of Property Act

• Sec 136 - prohibits judges, legal practitioners, mukhtar and other officers connected
with the courts of justice from purchasing any actionable claim (and thus cannot be
transferee).
• Order 21, rule 73 Civil Procedure Code prohibits decree holders from bidding in
auction sale without prior permission of the executing court.

Therefore, a minor can be a transferee of a property so long the minor is not required to
covenant in return to comply with some terms/ conditions. Hence a lease deed in favour of
minor is not valid if the child has to agree in return to pay rent at required interval etc.

Courts have held a de facto guardian i.e. one who is not a natural guardian, nor appointed by
court cannot transfer property of a minor unless it is for necessity or benefit of minor or minor
consents to it on attaining majority (Hari Satya Banerjee vs Mahadev Banerjee AIR 1983 Cal 76
(81/80)DB)

• OPERATION OFTRANSFER
Sec 8 deals with "Operation of Transfer". As per the Act in normal course when a transfer is
done the following will be presumed to be included, in case of:

1. Land - the easement annexed thereto, the rents and profits thereof accruing after the
transfer, and all things attached to the earth;'

2. House - the easement annexed thereto, the rents and profits thereof accruing after the
transfer, lock, key bars, doors, windows and all things provided for permanent use.
3. Machinery fixed to earth - include the moveable parts also

This summarization in the Transfer of Property Act helps to reduce the drafting load. Only if
express intentions to mean something else is manifest from the deed justifying limiting the
meaning of house/ land or such an implication is apparent on plain reading will the alternate
meaning will apply. For e.g. Sarkar, 2000 asserts citing case decisions in A Jogendra vs Miasa
51 IC 360 held that when grantee of a land for a certain term in years transfers his interest, the
right to renewal of the grant also goes to the transferee along with the land unless the express
or implied intention is different.

Subsequent court decisions have expanded the term land to include "trees, shrubs and
saplings" as on date of agreement. Other Acts like Easement Act also clarifies the conception
of house/ land that with transfer of dominant heritage the easement benefit will also change
hands

This is best understood by the statement "A vendor cannot convey a title better than what he
has" - meaning what rights the vendor (whether as the seller or the lessor or the mortgagor
etc.) possesses only that he can transfer, he cannot transfer a title better than what he has. So,
vendor can transfer all the interest he possesses and any legal ability to enforce his interests
along with the property.
Vested and Contingent interest
Vested Interest Contingent Interest
Governing sections Sec - 19 of TPA and Sec 119 of Indian Sec 21 of TPA
Succession Act, 1925.

When transfer This is a certainty. In fact, has already Transfer is possible but not
happens happened and often even certain to occur.
communicated to transferee.
Note: despite being a certainty some
additional event in future may cause
the dissipation of the vested right.

Transfer of interest It covers both situation - present right The right to enjoyment takes
when takes effect to currently enjoy or is present right to effect on the happening or not
future enjoyment happening of some future
uncertain event or the doing or
not doing of some activity
described in the transfer
instrument of the property

Example 'A' transfers a life interest in A transfers life interest in


property to 'B' via will/ family property to B via will/ family
settlement and after B's death the settlement and after B's death the
property will go to C and D. But if B's property will go to B's children. In
has a child the property will go to her her lifetime B enjoys the
child and not to C & D. In this case C property. But her children only
& D have vested interest till no child have interest contingent on their
is born to B. The time a child is born surviving their mother B. So,
to B their vested interest is while the death of B is certain, the
dissipated/ eroded into nothingness. order of death of B and her
children is not known.

Vested interest is defined in Sec - 19 of TPA and Sec 119 of Indian Succession Act, 1925. In
case of vested interest while the property is transferred but the transferee's interest does not
take immediate effect nor is it clear from the transfer deed when it will take effect. The transfer
is certain. Even if the transferee expires before getting the property, his claim survives till the
time the necessary event occurs and his heirs and assignees can step into his rights.

It does not mean that existence of vested interest is inferable automatically where the
enjoyment of the property rights is postponed to sometime in future. Nor does it mean
that a prior interest is given to another. If income from property is not given to the beneficiary
but accumulated till the happening of a future event that too does not mean that existence of
vested interest is confirmed.

Contingent interest - the property is transferred but it will take effect on fate of an uncertain
event or action. Interest is vested once the desired event/ action take place. Sec 21 itself states
if a property is to be transferred to B when B attains a certain age and if till then her needs are
to be met from the income of the property then it's not a contingent transfer.

• SALE
As per sec 54 of TPA Act: Sale is a transfer of property in exchange for a money value - the
price may have been paid, promised to be paid or partly paid and partly promised. The
completion of payment is not a condition precedent to transfer of any property.

Sale Transfer under TPA is not always absolute.

1. Sec 39 of the TPA deals with third person's right of maintenance from the property or
to get a provision for marriage, these rights would be enforceable against the
transferee who had due notice ofit.
2. Sec 7 defines who may contract to transfer a property and one condition by reference
to the Contract Act, 1972 is that the person must be a major. Hence a minor cannot
transfer. His guardian can transfer to meet his needs/ in furtherance of his benefit or it
can be alienated on permission of the court. Within three years of attaining majority
the minor has a right to apply to court to repudiate the sale of his property restoration
of his property tohim.

The date of sale is crucial in determining liability for charges and taxes. It is payable by seller
till that date, unless otherwise agreed. The seller is also entitled to rents and profits till
ownership transfer to buyer during the process of transfer.

Since the liability of stamp duty led to people undercutting the price recorded in sale deeds,
government per force introduced - Circle Rates/ Guideline rates. The following is not from TPA
and relevant to it but is none the less important in understanding concept of Sale at market
price. Other methods are also used to dodge the requirements of paying appropriate stamp
duty.

In Suraj Lamp & Industries (P) Ltd. vs. State of Haryana the Apex court called for an end to
the 'pernicious practice' of sales via Sale Agreement/General Power of Attorney/Will which it
said are resorted to 'avoid payment of stamp duty and registration charges on deeds of
conveyance and/or to avoid payment of capital gains on transfers, and thus to invest 'black
money' while avoiding payment of `unearned increases' to
Development Authorities. Submissions by the states of Delhi, Haryana, Punjab and Uttar
Pradesh were that such sales led to loss of revenue and increase in litigation due to defective
title. This decision will not affect transfers before the date of the instant judgment. It will also
not affect genuine transactions involving Power of Attorney to immediate/ close relatives
(spouse, son, daughter, brother, sister or a relative) to manage his affairs or to execute a deed
of conveyance. It also continues to permit land owner to execute development agreement
with a land developer or builder giving Power of Attorney empowering the developer to
execute agreements of sale or conveyances in regard to individual plots of land or undivided
shares in the land relating to apartments in favour of prospective purchasers.

• RIGHTS AND LIABILITIES OF BUYERS ANDSELLERS

Unless the buyer and seller decide on a modified or different set of rules in part or in whole the
provisions of sec 55 TPA which defines Rights and Liabilities of Buyers and Sellers will apply.

DUTIES OF SELLER

1. Update the buyer on the defect of the seller's title to the property which the seller
knows and the buyer cannot discover by ordinary diligence.
2. If requested by buyer, produce all title related and allied documents in his possession
or which is within his power to procure

3. Respond to the questions on the property/ title put by the buyer based on best
information known to him

4. On receipt of price vide payment or by other tender, to execute proper conveyance


deed provided by the buyer

5. Between the date of the deed and the date the property is delivered to the buyer, the
seller undertakes to take proper care of the property and related documents as any
other careful man would have done
6. Give possession to the property when required to the buyer or the representative
indicated by the buyer

7. Pay all charges or dues till the date of sale

By executing a sale deed the buyer is holding out that the interest is subsisting and he has
legal capacity to transfer it vide the deed (this is referred to in judicial decisions as the
"Statutory Warranty of Title" or also as "Covenant of Title" and this cannot be waived by
agreement of parties. Another attendant covenant read into this is the "Covenant of quiet
possession" which goes with the "Warranty of Title" - that is right to quiet enjoyment by
transferee on transfer of title).
When a Trustee or Receiver or some person holding fiduciary capacity is transferring then he
is also thereby holding out that the seller has not done anything that puts a bar/limit on his
capacity to transfer. He is also holding out that the property is not encumbered in any manner
other than what was declared.

The section clarifies that both the interest in the property and the transfer of the right to its
benefit is presumed on execution of the deed enforceable by anyone having the interest.

Seller’s Rights

1. Seller retains rights to the profit/ rent of the property till the date of the transfer unless
prior rents are also specifically transferred.

2. If the property is transferred before whole of the agreed amount is received by seller
then the seller has a charge on the property so transferred. The buyer, every
subsequent transferee who got without payment and such other transferee who has
notice of this non-payment will be subject to the charge on the property.

BUYER'S DUTIES

1. Disclose such aspects of the property that enhances its potential price/ value or
disclose the full extent of seller's interest in the property which buyer is aware of but
are unknown to seller.

2. To pay/ tender to the buyer or his representative the price of the property at time/
place where sale was completed. If there is any encumbrance on the property which
was not disclosed (or sale was made on unencumbered basis) then when paying he
can retain the amount payable to the encumbrance holder.

3. After title has passed to the buyer he is liable to suffer for any subsequent loss or
destruction to the property which was not caused by the seller.

4. He is liable for all charges payable on the property. He is also liable to pay off any
encumbrance (both principal and interest) of which he had notice of when purchasing.

Buyer's Rights

1. Right to all rents and profit and also benefit stemming from any improvement in the
property or increase in its value.

2. Buyer has a right to a charge being levied on the property against the seller and
others claiming through the seller in these following situations
i. If buyer has paid any part of the purchase money in anticipation of delivery
ii. Any earnest money paid
iii. Any cost awarded to buyer by the court
iv. However, this right to levy a charge will not hold if the purchaser had declined to
take possession either because he failed to pay full price or other conditions set in
contract would constitute "improperly declined to take possession".

• MORTGAGE
Mortgage (sec - 58) is transfer of interest in a particular, identifiable immoveable property for
securing money already advanced or to be advanced as loan or other type of debt or any
engagement that can result in a financial liability. The transferee is the mortgagee while the
person transferring is the mortgagor. It is not necessary that mortgagor is always the owner of
the property. He can be for example leaseholder with right to mortgage for general or
specified purpose. Sec 59A further clarifies that the term mortgagor or mortgagee includes all
people claiming through him unless it is specifically barred.

Sec 58 of the Transfer of Property gives details of different types of mortgage. But Banks
resort to mostly following types of mortgage:

Simple Mortgage (sec 58(b)) property continues in possession of the borrower (mortgagee),
only a personal undertaking acknowledging the loan and agreeing to repay on a specified
date is with the mortgagor (bank).

Mortgage by deposit of title deed (sec 58(f)) where original title documents are deposited
with the mortgagee/ bank or an agent specified them to create a security. This TPA Act
provides for this option of mortgage in Kolkata, Chennai and Delhi. It leaves discretion to the
State government to extend provision to other towns as well.

English Mortgage (Sec 58(e)) - Mortgagor absolutely transfers the mortgaged property with
absolute ownership rights with intention to create a mortgage and also commits to repay the
amount on a certain day. But the absolute transfer is done subject to a condition that - the
transferee will return the property on return of the money by the mortgagor.
Other mortgages detailed in this section of TPA are: Mortgage by Conditional Sale (58(c))
- the property is conditionally so - if the amount obtained against this mortgaged is not
returned by the specified date, the conditional sale shall become final. Instead if the amount is
repaid then the sale shall be cancelled, or it may provide that the property will be
retransferred back to the seller. Explicitly stating when the amount advanced is returned the
property will be returned to seller - only then the deed can operate as a one of mortgage by
conditional sale.
Mortgaging has led to the development of a special class of Property Buyers
known as “Vulture Buyers” in the United States who as per (Mitchell et al.,
2010) target owners of distressed properties worried about potential forced
sales. Such property owners who have fallen behind on their loan payments
know a foreclosure action (where the lender moves in to assert claim on the
property given as security) may destroy or severely damage their credit rating
and make future loans impossible. The vulture buyers are able to reap
handsome profits by purchasing the property directly from the distressed
owner either prior to or after a foreclosure action has been initiated merely
to help them retain their creditscores.

Bankers in India are referring credit scores of potential buyers generated by


Credit Information Bureau Limited (CIBIL) - the credit rating scores more to
know existing level of indebtedness. Do you think when CIBIL reports are used
more to assess credit worthiness of return capability from past repayment
behavior will such a class of buyers also be seen in India.

Usufructuary Mortgage - in this possession of the mortgaged property is transferred to the


person advancing the money along with the right to collects rent, other income accruing from
the property till the repayment fully or partly of the money advanced and interest on it.
Usufruct mortgage is different from "profit a pendere" which is not a mortgage or a lease but
essentially a License to enter and only collect something from within it e.g. to harvest crop/
pluck fruits.

Anomalous Mortgage - A mortgage that does not fall in any of the foregoing categories is
an AnomalousMortgage.

Why the other types of mortgage are not resorted to by banks will be best known to the
financial institution. Banking Regulation Act (Sec 9 on Non-banking asset) states 'no banking
company shall hold any immovable property howsoever acquired, except such as is required
for its own use, for any period exceeding seven years from the acquisition thereof". Perhaps
some forms of mortgage like mortgage by conditional sale or Usufructuary mortgage may not
be as common with banks because of this provision and the requirement.

Mortgages involving principal amount of Rs. 100 or more has to be executed by a registered
document which is signed by two witnesses (except a mortgage by deposit of title deed which
does not have to be registered). It is important to remember that by its very definition,
mortgage refers to an immoveable property. A pledge refers to a "moveable property"
which is delivered to the pagdee as a security. In case of default in due repayment, in pledge,
the lender i.e. the holder of the immoveable asset can sell
it.Incaseofhypothecationtoomoveableassetisinvolvedbutunlikeinpledgeitisnot
delivered to the person taking the loan (i.e. lender), it remains with thehypothecator.

ICICI Bank Ltd. Vs. Sidco Leathers Ltd. DOJ 28/04/2006, SC held that the dues of the workmen
and the debt due to the secured creditors are treated pari passu with each other, but that
does not dilute the concept of inter se priorities amongst the secured creditors. Such a
valuable right, therefore, must be held to have been keptpreserved.

The State's Debts ranks above bank dues: This is derived from the English rule of Crown debt
preceding others. In Central Bank Of India vs. State of Kerala DOJ 27/02/2009 the Supreme
Court held the Recovery of Debts Due to Banks and Financial Institutions Act, 1993 and the
Securitisation and Reconstruction of Financial Assets and Enforcement of Security Interest Act,
2002 (SARAFESI or Securitisation Act) do not create first charge in favour of banks, financial
institutions and other secured creditors.

As per Sec 48 of the Transfer of Property Act, 1882 Act, the claim of the first charge holder
shall prevail over the claim of the second charge holder. Hence property valuation reports
need to reflect the nature of the security and the value of the earlier charge on it.

Mortgage default insurance is meant to cover normal risks as also catastrophic risks - latter
being foreclosure occurring due to economic depression. The risk horizon is long - 15 years or
more. Inflation here reduces the risk of mortgage insurer's loss (Blood, n.d.).

Credit Rating Information Services of India Ltd. CRISL as per (Blood, n.d.) has staff and
competency in rating new general lines insurers and carriers that choose to engage in writing
mortgage default insurance. CRISIL has rated HFCs, banks and builders.

Entities like Housing Development Finance Corporation (HDFC) amongst the largest Housing
Finance Company in India - are reported to have studied mortgage insurance business
anticipating deregulation of insurance sector and allowing of other entities to engage in it, yet
such mortgage default insurance is not common. As of now borrowers are insured for default
due to death through the financial institution mandating an insurance policy.

• RIGHTS OFMORTGAGOR

Sec 60 TPA deals with the right to redeem the mortgaged property and associated
documents and a court case filed to enforce this right is called a suit for redemption. At any
time after the principal money has become due, the mortgagor has a right, on payment or
tender, at a proper time and place, of the mortgage-money, to require the mortgagee (a) to
deliver to the mortgagor the mortgage-deed and all associated documents of the mortgaged
property till then with the mortgagee, (b) to deliver possession of the property where
applicable to the mortgagor and at the cost of the
mortgagor either to re-transfer the title of the mortgaged property to him or to such third
person as he may direct, or to execute and if mortgage was vide a registered deed, then also
register a mortgage release deed.

For this act to apply the parties should not have acted to extinguish mutual rights stemming
from lease deed. If the time of payment of principal is passed or not fixed, the mortgagee in
equity is entitled to a reasonable notice before payment or tender of such money
necessitating release of the property/ documents.

When there is more than one mortgagor of the property, part release by one mortgagor or
any person interested through him by paying his proportionate share of the mortgage
amount is not allowed.

Sec 60A states that by virtue of his redemption rights a mortgagor may assign the mortgage
debt and transfer the mortgaged property to a third person; and the mortgagee shall be
bound to accordingly transfer to the third person as directed by mortgagor. The section also
details the right can be enforced by or the mortgagor or by any encumbrance holder. But
encumbrance holder's claim is prior to that of mortgagor and between encumbrance holders,
the requisition of a prior encumbrance shall prevail over that of a subsequent encumbrance.
Section does not apply to a mortgagee who was or has been inpossession.

As long as a mortgagor's right of redemption subsists, mortgagor shall be entitled at all


reasonable times after requesting can make copies at this cost or inspect the documents
relating to the mortgaged property which are in the custody or power of the mortgagee.

Sec 61 clarifies where two or more property are mortgaged to same mortgagee at the option
of the mortgagor, he can seek to redeem one first or can redeem both together.

Sec. 62 Right of Usufructuary mortgagor to recover possession on clearing dues.

In a Usufructuary mortgage, the mortgagor has a right to recover possession of the property
together with the mortgage-deed/ documents with the mortgagee on payment of the
mortgaged money from rents/ profits or if he is authorised to pay himself from such rents and
profits or if the period has expired but the money not recovered then when mortgagor pays
or tenders to the mortgagee the mortgage-money balance thereof or deposits it in court.

As per sec 63where there is any in the mortgaged property in possession of the mortgagee
redemption shall, in the absence of a contract to the contrary, be entitled as against the
mortgagee to such accession.

Sec 63A where mortgaged property in the possession of the mortgagee has, during the
continuance of the mortgage, been improved, the mortgagor, upon redemption, shall, in the
absence of a contract to the contrary, be entitled to the improvement. If such improvement
was paid for by the mortgagee and was necessary to preserve the property from destruction
or deterioration or was necessary to prevent the security from becoming insufficient, or was
ordered by a government or a public authority, the mortgagor shall, in the absence of a
contract to the contrary, be liable to pay the proper cost thereof as an addition to the
principal money with interest at the same rate as is payable on the principal, or, where no such
rate is fixed, at the rate of nine per cent per annum, and the profits, if any, accruing by reason
of the improvement shall be credited to the mortgagor.
Where mortgaged property is a lease, and the mortgagee obtains a renewal of the lease, the
mortgagor, upon redemption, shall, in the absence of a contract by him to the contrary, have
the benefit of the new lease.

Liabilities of Mortgagor

Sec 65 sets out the implied contracts by mortgagor. It states in the absence of a contract to
the contrary, the mortgagor shall be deemed to have contracted the following with the
mortgagee-

1. The mortgagor has the power to transfer the said interest in the property and the
interest continues to subsist

2. Should it be needed the mortgagor will defend his interest during the mortgage and
when the mortgagee is in possession of the property, then mortgagor will help the
latter to defend title
3. Till the time mortgagee is in possession of the mortgaged property, the mortgagor
must pay all public charges

4. If a leased property is mortgaged, the mortgagor is deemed to be holding out that the
dues payable and condition set under the lease deed have been duly complied,
including the condition about renewal of deed. Till the mortgagee is in possession, the
mortgagor shall clear all dues. For all past dues not paid, the mortgagor will indemnify
the mortgagee against all the claims that may ensue

5. Sometimes a mortgaged property is re-mortgaged again: in such case the mortgagor


undertakes to clear interest dues accruing under the prior encumbrance on the
property and also discharge the principal when due.
6. Rights of mortgagee may be enforced by every person in whom that interest is for the
whole or any part thereof from time to time vested for whatever reason.

Sec 65A deals with powers of a mortgagor who is in possession to further lease the property
and it shall be binding on mortgagee. The lease will take effect within six
months from date of leasing. The lease shall comply with local custom and rules. No premium
shall be paid or promised and no rent shall be payable in advance nor make any commitment
about process of renewals

In the case of a lease of buildings, whether leased with or without the land on which they
stand, the duration of the lease shall in no case exceed three years and including a condition
of re-entry if rent is not paid on time.

As per TPA a property offered as security is deemed insufficient unless the value of the
mortgaged property exceeds by one-third, or, if consisting of buildings, exceeds by one- half,
the amount for the time being due on the mortgage.

• RIGHTS OFMORTGAGEE

Sec 67 TPA read with the explanation deals with the right of mortgagee by conditional sale or
a mortgagee under an anomalous mortgage having right to foreclose, to foreclose or sell the
mortgaged property as per process set in TPA. To foreclose means to take away the right of
mortgagors to redeem their mortgaged property. Unless the mortgage deed states otherwise,
and subject to the conditions set in TPA, at any time after the mortgage money has become
due and before the money is actually paid or a court passes a decree for the redemption of
the mortgaged property, the mortgagee has a right to obtain from the court a decree that the
mortgagor shall be absolutely debarred of his right to redeem the property, or a decree that
the property should be sold. Such a suit is called a suit for foreclosure.

Liabilities of Mortgagee

1. The mortgagee by conditional sale cannot institute a suit for foreclosure as such
before the happening of thecondition.

2. A mortgagor who holds the mortgagee's rights as trustee or legal representative of


mortgagee, cannot institute a suit for foreclosure even if he had the right to sue for a
sale of the property as per trust deed/ mortgagedeed

3. Mortgagee in case of a railway, canal, or other work in the maintenance of which the
public are interested, to institute a suit for foreclosure orsale

4. When there is more than one mortgagee in a deed, a suit for foreclosure cannot be
filed for one mortgagee's interest in the mortgaged property till the time they have
first separated severed their interests under themortgage.

TPA also casts a duty on the person acquiring immovable property or any share/interest by
mortgage or otherwise as being presumed to have notice of the title of any other person who
was in actual possession of such property.
All sales, mortgages (other than by way of deposit of title deeds) and exchanges of immovable
property are required to be registered by virtue of the Transfer of Property Act.
• CHARGE
Where immovable property of one person is by act of parties or operation of law made
security for the payment of money to another and when it does not amount to a mortgage,
the latter person is said to have a charge on the property. The provisions of Simple Mortgage
apply. Sec 100 deals with charge and it states this section does not apply to the charge of a
trustee on the trust-property for reimbursing expenses properly incurred in the execution of
his trust. No charge shall be enforced against any property once it is transferred for
consideration without prior notice of the existence of charge (except as provided in law).

• LEASE

In Sec 105 TPA the term "lease" is defined. A lease of immoveable property is a transfer of a
defined right to enjoy such property, made for a certain time, or in perpetuity. It could be
clearly stated or implied and in consideration of a price (paid or promised), or of money, a
share of crops, service or any other thing of value, to be rendered periodically or on specified
occasions to the transferor by the transferee, who accepts the transfer on suchterms.

The transferor of the right to enjoy such property, is called the lessor, the transferee is called
the lessee. The price paid or promised is called the premium. The 'money, share, service or
other thing to be so rendered usually at predetermined periodicity in return is called the 'rent'.

Lease License

Governingstatut Sec 105 of Transfer of Property Sec 52 of Easement Act, 1882


ory provision Act and also relevant sections of
the corresponding Rent
Control/ Premises tenancy Act
of the State governing lease of
premises
Granted to To an identified individual/ To an identified individual/ body of
body of individuals individuals

What is the Delivery of exclusive possession A person, permits another a right to door
lesser/ grantor of the property continue to do upon a property whichin
Allowing along with a defined interest in absence of such right would beunlawful
the property. Transfer of Only a permission to use the immoveable
interest is the defining property of the grantor. No interest in the
distinction said property is created.
Normally without right to exclusive
possession. However even if defined
exclusive possession is granted other
elements of License like mere permission to
use/ no interest created is point of
determination
Nature of the The rights are assignable/ License rights are not attached to the
rights inheritable within terms of the property and do not travel along with
lease deed change of ownership of the underlying
property. It is granted to a person. Hence
License is not inheritable nor can it be
assigned. The licensee's right dies with the
license holder.
Unless specifically permitted or an integral
element of it, License rights cannot be
transferred nor exercised by assignees,
servants or employees of License holder.
Exception for example include a movie ticket
which gives License to enter theatre to watch
the chosen movie in a manner acceptable to
the theatre manager. Its nature is such that it
can be transferred. But other Licenses on
immoveable property are
not transferable.
Can be created not necessarily only by the
owner, but even by tenant/ leaseholder but
not exceeding their own tenure (period).
Life interest holder too can create but not for
a period beyond his life time
Fate on Continuation governed by If granted for specific period, on expiry of
termination provision of the lease deed and lease period licensee loses right to continue
otherwise the TPA. to occupy. It expires with death of the
holder.
Lease deed may get
automatically continued for the On termination of license by passage of time,
same period again on the exiting licensee can seek time to leave
termination of the leaseperiod if the property and also to remove building
monthly payment continues to materials etc.
bereceived.
In some lease it may convert to If licensed property is transferred by the
year-on-year lease when licensee to some other person against the
payment continues to be terms of the License, the transferee is a
accepted. trespasser against the transferor/ property
It is possible to transfer the owner/ original grantor.
right of re-entry on expiry of the
lease or on breach/ violation
subsequently of a condition set
during grant of lease

Example • State continues to levy a • At times a license can be converted to


leasehold rent on flats lease by way of one deed - adeveloper is
developed in landowned by granted license to build and then after
Delhi Development completion will convert as leaseholder
Authority for part of the building to run office or
otherpurpose
• A person cultivates the land ofanother,
shares the produce of agriculture with
the owner but has no right on theland
• Right to cut grass, collect and takeaway
leaves, bamboo are examples oflicense
• Allowing person to continue in
possession till the time the land isused
for the exclusive purpose of runninga
school

In distinguishing Agreement to Lease vs. Agreement for Lease Supreme Court has held a
confirming third party (in this case a company) in agreement to lease a property did not
create a lease subject to Bombay Stamp Duty Act 1958 as relationship of lessor and lessee was
not established between the company and the State. It was only an agreement to lease and
thus does not fall under sec 2(n) of Bombay Stamp DutyAct
1958 [State of Maharashtra vs Atur India (P) Ltd. 1995(2) Bom CR 31 (SC))

The disrepair for which a leaseholder or tenant is usually liable for mainly commercial
properties when he has agreed to return premises in good repair is called Dilapidations. No
property can usually be returned in the same condition as received. But when the building
continues in possession of leaseholder/ tenant, deterioration may continue. It is important to
track if the building is notified for being pulled down. For example, section
354 of the Brihan Mumbai Municipal Corporation (BMC) Act deals with removal of dangerous
structures authorises demarcating even private buildings and for evacuation to prevent
human lives from being compromised due to shaky structures.

A tenant or lessee has liability to maintain hedges, boundaries, doors, premises etc. It is thus
breach of lease covenants relating to the condition of a property, and the process of
remedying those breaches. Landlord can demand being made good for the loss or demand
actual repairs being done. This is of special significance in UK jurisprudence where in the event
of disputes appraisers review case laws to argue their preferred position.

Lease deeds were traditionally executed by two ways: written document where the lessor
executes a lease patta with recipient signing or the recipient i.e. lessee executing a
kabuliyatadmitting to the receipt or orally along with delivery of possession.

Section 106 of TPA gives the period of lease in the absence of contract or local law or usage
stating otherwise

A lease of immovable property for agricultural or manufacturing purposes is deemed to be


year to year that can be terminated by either side on six month notice

A lease of immovable property for any other purpose is a lease from month-to-month that
can be terminated by either side on fifteen days' notice.

It also states the start date of notice of six month/ 15 days will be counted from date of
receipt of notice by the other side. If the period of notice falls short of the required period
then it is not a failing if the suit seeking recovery has been filed with delay. The notice has to
be served in writing, duly signed by issuer and sent by post or delivered in person to the other
side or handed to one of his family members or servants at his residence or affixed to a
conspicuous part of the property.

• RIGHTS AND LIABILITIES OF LESSOR ANDLESSEE

Rights and liabilities of lessor and lessee are given in Sec 108 TPA the rights and
liabilitiesoflessorandlessee.Therightsaretemperedbytheliabilitiesapplicableon
lessor/ lessee as are applicable to the property leased:

Liabilities of the Lessor

1. Must disclose to the lessee any material defect in the property considering the usage
to which it will be put and known to lessor and which the lessee could not with
ordinary care discover

2. When lessee asks to put lessee in possession of the property he mustcomply

3. On due payment of rent as agreed in the lease contract and complies with all
conditions set by lessor he may hold the property without interruption for the time till
the lease determines during the time limited by the lease. Anyone who claims through
the lessee as the lease interest is vested in him will be entitled to the benefit of the
leasedeed.

Rights of the Lessee

1. During continuation of the lease period any improvement/ accession in lease property
shall be deemed to be included in hislease;

2. Due to climate elements like 'fire, tempest or flood' or due to attack by any army or
mob, if the lease property is wholly or partly destroyed or becomes unfit for the
purposes it was taken on lease, at the option of the lessee the lease can cease/become
void. This protection will not be available if the injury is result of the wrongful act or
default of the lessee, he shall not be entitled to avail himself of the benefit of
thisprovision;

3. If the lessor does not carry out necessary repairs, the lessee may get the repair done
and deduct the expense of such repairs with interest from the lease rent payable to
lessor, or otherwise recover it from thelessor;
4. The lessee can pay off compulsory liabilities of the lessor which if left unpaid will be
recovered from lessee being the occupier or from the property, and deduct principle
and interest paid from the lease rent payable to or otherwise recover from thelessor;

5. Once the lease determines (i.e. ends) but lessee continues in possession, the lessee
may remove, all things which he has attached to the earth but he has to restore the
property in the state in which he receivedit;

6. If a lease whose period is not definite, terminates for no fault of the lessee, then even
after its end lessee or his representative shall have free access to the land to harvest
and carry them

7. The lessee may transfer absolutely or mortgage or sub-lease the whole or any part of
his interest in the property and his transferee can further transfer it likewise. But sub-
lease and sub-transfer will not abate the liability of the lessee to comply with the
terms of the lease. A tenant with an un-transferable right of occupancy cannot
transfer/ lease. A farmer of an estate which has defaulted in paying revenue or estate
of one whose assets are being managed by the Courtof
Wards cannot also assign his interest as such tenant, farmer or lessee.

Liability of the Lessee

1. The lessee must disclose to the lessor any fact known to lessee as to the nature or
extent of the interest which the lessee is about to take, of which the lessor is not,
aware, and which materially increases the value of suchinterest;
2. The lessee is bound to pay or tender the premium or rent to the lessor or his agent in
thisbehalf

3. The lessee is bound to keep property in good condition and on the termination of the
lease to restore, the property in the same condition as it was in at the time when he
was put in possession, except dilapidation cause by normal wear and tear or
irresistibleforce,

4. Has to allow the lessor and his agents, at all reasonable times during the term, to enter
and inspect the condition of the property and if any information is given of any defect
in the property due to action or default of lessee or his agents he will make good the
loss within three months of suchnotice
5. On learning of any legal action against the property or encroachment or others which
amounts to interference with the lessor‘s rights on the lease property, lessee must
notify the lessor within reasonable is bound to give, with reasonable diligence, notice
thereof to thelessor
6. Like an ordinarily careful person would use his own property, a lessee may use the
lease property and its products. Further he must not use or permit another to use, the
property for a purpose other than that for which it was leased - mining, quarrying
cannot be started if land was not leased for that purpose. Nor must he cut or sell
sanding timber or pull down or damage any buildings belonging to the lessor. Broadly
any act that is destructive or permanently injurious to the property is not allowed.
7. Without the lessor‘s consent, permanent structure, except for agricultural purposes
cannot beerected

8. On the determination of the lease, the lessee is duty bound to put the lessor into
possession of theproperty.

• EXCHANGE
Sec 118 refers to a specific type of mutual two-way transfers. In exchange two persons are
involved where each transfers their ownership of one thing for the ownership of another's
non-money item. This distinction is important - in sale a thing is exchange for money or a
promise of money. But in exchange it's more a barter of kind. Even if the exchange deed
mentions the value of the items being exchanged from either side, that will not change the
character of exchange. A transfer of property in completion of an exchange can be made only
in manner provided for the transfer of such property by sale.
Family partition, family settlement, handing over of property in any compromise do not
constitute an exchange since condition of mutual transfer of property by the two parties is not
fulfilled.

• GIFT AND HOW TRANSFER VIDE GIFT ISAFFECTED


As per sec. 122 TPA, a "Gift‖ is the transfer of certain existing moveable or immoveable
property made voluntarily and without consideration, by one person, called the donor, to
another, called the donee, and accepted by or on behalf of the donee. The gift has to be to a
defined number and definite number of people, to be a valid gift.

Acceptance must be made during the lifetime of the donor and while he is still capable of
giving to be a valid deed. Conversely, if the donee (recipient) dies before acceptance, the gift
is void. It is possible that gift was accepted and its acceptance can be proved but the
transferor soon after died and hence the transfer in interest was not registered via mutation
during lifetime of the transferor. Then the transfer vide gift is held asvalid.

A transfer of property is critical to constitute a "gift". Thus creation of a trust and transferring
property to it for the benefit of someone is not gift to the recipient if the trust generally gives
beneficial interest to the beneficiary while the property remains with the trust. But a gift to an
idol or for religious institution is valid (Santi Swaroop vs. Radhaswami Satsang Sabha AIR 1969
All 248). Under Mitakshara property system, the co-parceners hold the property jointly. One or
some of the co-parceners cannot transfer their share without consent of the other co-
parceners.

Sec. 123 clarifies transfer via gift deed of immoveable property is to be made only through a
registered instrument signed by the donor or his representative, and attested by at least two
witnesses.

In case of a gift of movable property, registration of the deed is optional. It can also be done
by delivery of the property in a manner goods in sales are delivered.

Since the days of Transfer of Property Act, the concept of "property" has evolved. Transferable
Development Rights (TDR) is only a tradable privilege that originates in one property which
can be transferred to another area and property:

TDR is a certificate issued against land acquired for public purposes either by the Central or
State Government in consideration of surrender of land by the owner without monetary
compensation and free from all encumbrances or by way of declared incentives by State
Government, which are transferable in part or whole. It is issued by the ULB's Urban Local
Body (Municipal body, Urban Improvement Trust, Urban DevelopmentAuthority).
Care is taken to back TDR rights legally. Government of Karnataka amended the Karnataka
Town and Country Planning Act, 1961 in order to empower the local bodies (Corporations /
Planning Authorities) to permit additional FAR for the land. Likewise, a new sec 14B was
inserted in the A.P Town and Country Planning Act 1920. The land prohibited by order of any
court or lands under acquisition are not eligible. TDR certificate can have name of a nominee
and is usable by nominee on the death of the holder and is tradable.

Maharashtra regulations required part of the FSI increase is be used in remainder portion of
the same plot whose portion of which was acquired for road widening etc. If needed a further
FSI can be purchased from other holders and used

As per a note on TDR prepared by Ministry of Urban Development, GOI in Aug 2015 for
Telengana Govt., TDR is resorted to for the following reasons:

• To facilitate development of affordable houses under State Affordable Housing Policy In


lieu of Floor Area Ratio (FAR) granted as per the Policy - TDR shall be calculated on the
total plot area being reserved for affordable housing project (including Economic Weaker
Section EWS / Low Income Group LIG) subject to the norms as prescribed in Affordable
Housing Policy. The maximum TDR shall not be more than 1.5 times of the total land area
but in case of Affordable Housing projects the developer shall be provided double of the
permissible FAR (e.g. at the time of launch of Affordable Housing Policy permissible FAR
was 1.67/1.80 hence the double FAR for both the cases will be 3.50) in case of slum
rehabilitation scheme under Slum Development Policy, 2012 maximum permissible FAR is
4.0 but TDR shall not be more than 1.5 and developer shall consume the maximum FAR on
the proposed project site provided all planning parameters and provisions of Building
Regulations are fulfilled. The unutilized FAR subject to maximum 1.5 can be taken asTDR
• Development of Green spaces - Parks/ Open Spaces/Playgrounds /Water Bodies etc. as per
the provision of Master Plan/ SectorPlan.
• Development of Master Plan/ Sector Plan roads including roadwidening
• Development of Public Parkinglots.
• Development of City level Facilities/other public purposes as per Master Plan proposals.
• Slum rehabilitation scheme under SlumDevelopment
• In lieu of land surrendered for other purposes as specified by StateGovernment

TDR policy can help to reduce time and money needed to acquire land - notification, hearing,
payment of compensation etc. By one estimate for the land acquisition for the Peripheral ring
road to be financed by a Japan agency, Bangalore Development Authority needs Rs. 8100
crores a kind of resource it does not have. Often TDR is preceded by
reduction of the FAR applicable to the area. The idea is to introduce TDR: "For instance, the
floor area that includes balconies and staircases built on a 30ftX40ft plot should not exceed
1200 sqft. To make any addition to this floor area, one will have to buy TDR at twice the cost
of guidance value as proposed in the new amendments." "Most parts of Mumbai, where space
is in high premium and has been traditionally a vertical city, the FAR is 1. Bengaluru is growing
horizontally. A FAR of 1 creates a huge market for TDR making it a viable option for land
losers‖

Telangana government is considering this route for acquiring half an acre land for a 40 feet
road along Banjara Hills of the Annapurna Studios in Hyderabad, owned by the late Tollywood
actor Akkineni Nagarjuna, which is part of 22 acres allotted at a nominal price ranging from Rs
7,500 to Rs 8,000 per acre. The land is now worth up to Rs 30 crore per acre.

TRANSFER OF IMMOVABLE PROPERTY, LEASE GRANTED BY


PRIVATE AND STATUTORYBODIES- IMPACT OF EACH ON
VALUATION.
1) LEASE

A lease of an immovable property is a transfer of a right to enjoy such property for a certain
time (express or implied), or in perpetuity, in consideration of a price paid or promised, or of
money, a share of crops, service, or any other thing of value, to be rendered periodically, or on
specified occasions, to the transferor by the transferee, who accepts the transfer on such
terms.

The price is called the premium. The money, share, service or other thing to be rendered
periodically is called the rent. The transferor is the lessor, and the transferee is the lessee. The
lease of immovable property is governed by Sections 105 to 117 of the Transfer of Property
Act.

2) LEASE GRANTED BY PRIVATE & STATUTORY BODIES – IMPACT OF EACH ON


VALUATION

The Transfer of Property Act does not define property but gives only a negative definition of
immovable property (Sec. 3). It states that "immovable property does not include standing
timber growing crops and grass". To determine precisely the meaning of movability or
immovability we have to resort to judicial decisions. The following definition given by Justice
Holloway in the 1972 Madras case of SukryKurdepa vs.
Condikulisacceptedasastandarddefinition.Movabilityisthecapacityinathingto
suffer alteration with reference to its location without injury or damaging its surroundings;
while immovability is the characteristic of a thing to suffer alteration with reference to its
location by injuring or destroying its surroundings.

SECTIONS: 3, 5, 6, 7, 25, 53 AND 53A OF TRANSFER OF PROPERTY


ACT 1882.

1) SECTION 3

In this Act, unless there is something repugnant in the subject or context: -

1. ―immoveableproperty‖doesnotincludestandingtimber,growingcropsorgrass:

―instrument‖ means a non-testamentary instrument:


―attested‖, in relation to an instrument, means and shall be deemed always to have
meant attested by two or more witnesses each of whom has seen the executant sign or
affix his mark to the instrument, or has seen some other person sign the instrument in the
presence and by the direction of the executant, or has received from the executant a
personal acknowledgement of his signature or mark, or of the signature of such other
person, and each of whom has signed the instrument in the presence of the executant;
but it shall not be necessary that more than one of such witnesses shall have been present
at the same time, and no particular form of attestation shall be necessary:
―registered‖ means registered in 6[ Bangladesh] under the law for the time being in
force regulating the registration of documents:

―attached to the earth‖ means:

• Rooted in the earth, as in the case of trees andshrubs;


• Imbedded in the earth, as in the case of walls or buildings;or
• Attached to what is so imbedded for the permanent beneficial enjoyment of that to which
it isattached:
• ―Actionable claim‖means a claim to any debt, other than a debt secured by mortgage of
immoveable property or by hypothecation or pledge of moveable property, or to any
beneficial interest in moveable property not in the possession, either actual or
constructive, of the claimant, which the Civil Courts recognize as affording grounds for
relief, whether such debt or beneficial interest be existent, accruing, conditional
orcontingent:
• ―apersonissaidtohavenotice‖ofafactwhenheactuallyknowsthatfact,orwhen, but for willful
abstention from an inquiry or search which he ought to have made, or gross negligence,
he would have knownit.
Explanation I.-Where any transaction relating to immoveable property is required by law to be
and has been effected by a registered instrument, any person acquiring such property or any
part of, or share or interest in, such property shall be deemed to have notice of such
instrument as from the date of registration or, where the property is not all situated in one
sub-district, or where the registered instrument has been registered under sub-section (2) of
section 30 of the Registration Act, 1908, from the earliest date on which any memorandum of
such registered instrument has been filed by any Sub- Registrar within whose sub-district any
part of the property which is being acquired, or of the property wherein a share or interest is
being acquired, issituated;

Provided that-

1. The instrument has been registered and its registration completed in the manner
prescribed by the Registration Act, 1908, and the rules madethereunder,

2. The instrument or memorandum has been duly entered or filed, as the case may be, in
books kept under section 51 of that Act,and
3. The particulars regarding the transaction to which instrument relates have been
correctly entered in the indexes kept under section 55 of thatAct.

Explanation II.-Any person acquiring any immoveable property or any share or interest in any
such property shall be deemed to have notice of the title, if any, of any person who is for the
time being in actual possessionthereof.

Explanation III.-A person shall be deemed to have had notice of any fact if his agent acquires
notice thereof whilst acting on his behalf in the course of business to which that fact is
material:

Provided that, if the agent fraudulently conceals the fact, the principal shall not be charged
with notice thereof as against any person who was a party to or otherwise cognizant of the
fraud.

Enactments relating to contracts to be taken as part of contract Act

2) SECTION 5

Defines transfer of property as a transaction whereby the possession or ownership of a


property undergoes a change by the acts of the parties to the said transaction. Examples of
such a change of ownership or possession may be in the shape of a sale (where the ownership
is totally effected), .a mortgage (where possession of the property is used) as a security for
payment of a loan or a lease (where possession of a property is enjoyed on payment of a rent)
amongothers.

Ownership, as seen earlier, consists of a bundle of rights. The rights which so make up
absolute ownership (e.g., right of possession or right of engagement) are called interests in
property and a transfer of property is either a transfer of absolute ownership or a transfer of
one or none of those interests.

Transferability is the general rule in the law of property and the right to property includes the
right to transfer the property to another person. Subject to this general principle, the Transfer
of Property Act lists certain properties which cannot be transferred.

3) SECTION 7

Deals with competency to be a transferor. It states that a transferor must (a) be competent to
contract, and (b) have title to the property, or authority to transfer the property if the property
is not his own. In this connection, reference is made to Section 11 of the Indian Contract Act,
1872 which specifies the competency to contract and states that a person is competent to
contract if he is (a) of the age of majority according to the law to which he is subject, (b) of
sound mind, and (c) not otherwise disqualified from contracting. It is thus clear that the power
to transfer depends on the power to contract, as without a contract to give and take, there can
be no transfer at all. Further, it is also a well-established principle of law that a minor lacks the
competence to contract and consequently the competence to transfer, and what is more, a
minor does not even have the competency to authorize another to enter into a contract on
hisbehalf.

Although a minor cannot be a transferor, a transfer made to a minor is a valid one. In other
words, a minor lacks the competence to be the transferor but he possesses the competence to
be a transferee.

4) SECTION 8

of the Act confers stability on title and removes speculations in a transfer. It elaborates the
operation of a transfer and states that unless a different intention is expressed or implied, a
transfer of a property passes immediately to the transferee. Section 8 also elaborates the legal
incidents involved in the transfer. Some of these are:

1. Where the property is a land - the easements annexed thereto, the rents and profits
accruing after the transfer and all things attached to theearth.

2. Where the property is machinery attached to the earth - all movable parts thereof.

3. Where the property is a house - the easements annexed to it, the rent accruing after
the transfer as well as locks, keys, doors, windows and all other things provided for
permanentuse.
5) SECTION 25

Conditional transfer. —An interest created on a transfer of property and dependent upon a
condition fails if the fulfilment of the condition is impossible, or is forbidden by law, or is of
such a nature that, if permitted, it would defeat the provisions of any law, or is fraudulent, or
involves or implies injury to the person or property of another, or the Court regards it as
immoral or opposed to public policy.Illustration

1. A lets a farm to B on condition that he shall walk a hundred miles in an hour. The lease
is void.

2. A gives Rs. 500 to B on condition that he shall marry A‘s daughter C. At the date of the
transfer C was dead. The transfer isvoid.

3. A transfers Rs. 500 to B on condition that she shall murder C. The transfer isvoid.

4. A transfers Rs. 500 to his niece C, if she will desert her husband. The transfer is void.

5. Other sections under this head which deal in detail regarding the legal aspects of the
Transfer of Property Act are beyond the scope of thistext.

FRAUDULENT TRANSFER

6) SECTION 53

1. Every transfer of immoveable property made with intent to defeat or delay the
creditors of the transferor shall be voidable at the option of any creditor so defeated
or delayed. Nothing in this sub-section shall impair the rights of a transferee in good
faith and for consideration. Nothing in this sub-section shall affect any law for the time
being in force relating to insolvency. A suit instituted by a creditor (which term
includes a decree-holder whether he has or has not applied for execution of his
decree) to avoid a transfer on the ground that it has been made with intent to defeat
or delay the creditors of the transferor shall be instituted on behalf of, or for the
benefit of, all thecreditors.

Every transfer of immoveable property made without consideration with intent to


defraud a subsequent transferee shall be voidable at the option of such transferee.
For the purposes of this sub-section, no transfer made without consideration shall be
deemed to have been made with intent to defraud by reason only that a subsequent
transfer for consideration wasmade.]
SECTION 53A
Part performance.—Where any person contracts to transfer for consideration any immoveable
property by writing signed by him or on his behalf from which the terms necessary to
constitute the transfer can be ascertained with reasonable certainty, and the transferee has, in
part performance of the contract, taken possession of the property or any part thereof, or the
transferee, being already in possession, continues in possession in part performance of the
contract and has done some act in furtherance of the contract, and the transferee has
performed or is willing to perform his part of the contract, then, notwithstanding that where
there is an instrument of transfer, that the transfer has not been completed in the manner
prescribed therefore by the law for the time being in force, the transferor or any person
claiming under him shall be debarred from enforcing against the transferee and persons
claiming under him any right in respect of the property of which the transferee has taken or
continued in possession, other than a right expressly provided by the terms of the contract:
Provided that nothing in this section shall affect the rights of a transferee for consideration
who has no notice of the contract or of the part performance thereof.

LAW RELATING TO INHERITANCE/ SUCCESSION

• MOHAMMEDAN: MUSLIM PERSONALLAW

Muslims in India are governed by The Muslim Personal Law (Shariat) Application Act, 1937.
This law deals with marriage, succession, inheritance and charities among Muslims. The
Dissolution of Muslim Marriages Act, 1939 deals with the circumstances in which Muslim
women can obtain divorce from their husbands and to provide for matters connected
therewith. These laws are not applicable in Goa state, and are also not applicable to Muslims
who have married under the Special Marriage Act, 1954

RULES OF INHERITANCE

1. A son gets double the share of the daughter wherever they inherittogether.

2. The wife gets one-eighth of the share if there are children and one-fourth of the share if
there are no children. In case the husband has more than one wife, the one- eighth share
will be divided equally among all wives. The husband gets one fourth of the share of his
dead wife's property, If there are children and one-half if there are no children.
3. If the parent has more than one girl children, only two-third of the property shall be
divided equally among girl children. If the parent has only one daughter, half of the
parent's property is inherited by her.

4. The mother gets one-sixth of her dead child's property if there are grandchildren, and
one-third of the property if there are nograndchildren.

5. Parents, children, husband and wife must, in all cases, get shares, whatever may be the
number or degree of the otherheirs.

6. Slavery, homicide, difference of religion and difference of allegiance, exclude from


inheritance.

MAHR

Mahr can be defined as a mandatory payment, in the form of money (it can also be anything
agreed upon by the bride such as jewelry, home goods, furniture, a dwelling or some land) or
possessions paid by the groom, to the bride at the time of marriage, that legally becomes her
property. "Dower" is equivalent terminology to that of Mahr, and can be termed as deferred
Mahr, as "dower" refers to the payment from the husband or his family to the wife, especially
to support her in the event of hisdeath.

Mahr is distinct from Dower in two ways:

1. Mahr is legally required for all Islamic marriages while dower wasoptional,
2. Mahr is required to be specified at the time of marriage (when a certain amount is
promised, if not paid immediately), while dower is not paid until the death of the
husband.
3. Mahr also can be classified as a form of "bridewealth", and is paid directly to the bride
and not to herparents.

Will

The law on Muslim wills is different from that of other Indians and this is because, under
Muslim Law, testamentary disposition of property is considered to be divine in nature and
draws on the Quran. The Muslim will is not governed by the Indian Succession Act, 1925. The
Muslim personal laws in India, or the Shariat law, decrees certain rules and regulations and
ways in which an individual can dispose off his/herproperty.

CERTAIN IMPORTANT POINTS

1. Any person, who is 18 years of age or above, and is of sound mind, is eligible to make
their will, according to Shariatlaw.
2. TheMuslimwillcanbemadeeitherorallyorwiththeuseofpenandpaper.
There is no well defined format to draft a will. However, one needs to remember that
the instructions left in the document should be clear and concise, and legible, to
ensure it is followed after theirdeath.

3. According to the Shariat law, a person can only leave one-third of their property to
anyone they wish. The remaining two-thirds will, by law, go to their heir or heirs,
equally shared between them.

For example, if A has total asset of Rs. 7 lakhs and he owes Rs. 50k of someone and let his
funeral expenses to be added of Rs. 50k than A can only left an amount of Rs. 6 lakhs from
which he can give an amount of Rs.2 lakh through WILL (one-third of the asset left), and
remaining Rs. 4 lakhs will go to his legal heirs.

1. Muslim law does not allow a property to be bequeathed to an unborn child. However,
in case the mother is pregnant with the child, and is born within six months of the
death of the person making the will, the child has all rights to inheritit.

2. Muslim law decrees that a person can cancel the will at his convenience without giving
any reasons, anytime before hisdeath.

3. At the time of making the will, an individual needs to pick the persons who might
execute his will. The request is taken into account at the time of disposal of assets. The
person picked as an executor of the will has the right to dispose off the assets as
specified in thewill.

GIFT

A gift is a transfer of property in which the interest in the property will be transfer from one
living person to another, without any consideration and is purely gratuitous in nature. This is
the general definition that is accepted by all the religions, including Muslim law. As per the
Muslim Law, a gift is called asHiba.

Under the Muslim Law, a gift is a transfer of property or right by one person to another in
accordance with the provisions provided under Muslim law. Hiba is an immediate and
unconditional transfer of the ownership of some property or of some right, without any
consideration or with some return; and the term ‗hiba‘ and ‗gift‘ are often indiscriminately
used but the term hiba is only one of the kinds of transactions which are covered by the
general term ‗gift‘. The other types of gifts include Ariya, where only the right to enjoy the
property is transferred and Sadqah where the gift is made by the Muslim with the object of
acquiring religiousmerit.
• THE HINDU SUCCESSION ACT 1956, THE HINDU SUCESSION (AMENDMENT)
ACT, 205 (39 OF2005)

The Act lays down a uniform system of inheritance equally applicable to persons governed by
the Mitakshara and Dayabhaga schools as also those in the Southern India who are governed
by the Marumakkattayam, Aliyasanthana and Nambudri systems of Hindu law. The Act applies
to all Hindus and the term Hindu includes Buddhists, Jains and Sikhs.

It has further been extended even to those persons whose parents are Hindu, Buddhist, Jain and
Sikh and who are brought up as Hindus (Section 2). The Act does not apply to the property of a
person to whom the provisions of the Special Marriage Act, 1954 apply (Section 5).

1. Section 4 of the Act gives overriding effect to the provision of the Act. It abrogates all
the rules of the law of succession hitherto applicable to Hindus, whether by way of any
text, customs or usage, having force of law. Any other law contained in the Central or
State legislation shall cease to have effect in so far as it is inconsistent with any of the
provisions contained in theAct.
2. The Act has abolished impartible estate and the special mode of itssuccession.

3. The Act has extensively affected the entire concept of Mitakshara coparcenary which
was governed by the rule of survivorship. In this scheme female heirs did not have any
place and the property devolved only on the male heirs of the coparcenary on the
death of a male member under the Act. The rule of survivorship has a limited
application. It would apply only in those cases where a male member on his death left
coparcenersonly.

4. In case such male member of a Mitakshara coparcenary dies intestate leaving behind a
female heir mentioned in Class I of the Schedule, the property of the deceased would
devolve not according to the rule of survivorship but according to the provisions of
this Act, which provides for a specific share to such female heir.

ORDER OF SUCCESSION
The order of succession provided by the Act is broadly based on the doctrine of propinquity
or nearness of blood and accordingly tine heirs are divided into four categories instead of
three, which are as follows:
1. Heirs in class I of the Schedule

i. Agnates
ii. Agnate means a person related to wholly through males either by blood or by adoption.
The agnatic relation may be a male or a female. One‘s father‘s brother, or father‘s brother‘s
son or father‘s son‘s son or father‘s son‘s daughter are agnates. So the final relationship
may be male or female but it must be through males. The relationship can also be in
ascending or descendingline.
iii. Cognates
iv. Cognates means a person related not wholly through males. Where a person is related to
the deceased through one or more females, he or she is called a cognate. Thus son‘s
daughter‘s son or daughter, sister‘s son or daughter, mother‘s brother‘s son, etc. are
cognates. Here also the final relationship may be male or female but there is at least one
female in that line ofrelationship.
v. Note that if there are more than one Widow‘s , then they get one share only and then
divide it between themselves and a person immediate family will also be considered as one
unitonly.
The property devolves firstly upon the sixteen preferential heirs mentioned in class I of the
Schedule to the Act and failing such heirs upon the second will prevail. The outstanding
feature of the above division is that the heirs in class I of the Schedule inherit the property
simultaneously and according to the doctrine of representation in case of predeceased sons
ordaughter.
Another important point to be noted is that class I of the Schedule contained that a list of
sixteen heirs out of which eleven are females and five are males of which one male can claim
through a female. All of them inherit in equal shares. All the heirs in class II do not succeed
simultaneously but the heirs placed in one Entry would be entitled to inherit simultaneously.
In the case of third and fourth categories of heirs, i.e., agnates and cognates, the rules of
preference have been adopted in order to determine thepriority.
1. The Act has abolished Hindu women‘s limited estate and made her absolute owner of
the property irrespective of its source of acquisition. Any property acquired by a Hindu
female in any lawful manner whatsoever and possessed by her became her absolute
property and she enjoys absolute power to dispose ofit
in a way she desires. (Section 14 of Hindu Succession Act, 1956).

2. The Act has also provided uniform order of succession with respect to property of
female Hindu. On her dying intestate her property shall devolve on her children and
husband and thereafter upon her parents and the heirs of parents. In absence of any
issues to her, the property inherited from her parents would revert back to the heirs of
parent instead of devolving upon the husband or heirs ofhusband.
3. The right of child in womb at the intestate‘s death and subsequently born alive shall
relate back to the date of intestate‘s death. (Section20)

4. The Act lays down some general rules of succession inter alia to the effect that heirs
related to a male or female intestate by full blood are to be preferred to those related
by half blood if the nature of relationship is the same in every other respect (Section
18). Another rule is that if two or more heirs succeed to the property of an intestate,
they shall take their share per capita and not per stripes. Such heirs take the property
as tenants-in-common and not as joint tenants. (Section 19)

5. The Act has thoroughly revised the law relating to exclusion from inheritance. Section
28 discards all the grounds of exclusion based on physical defects, deformity or
disease. The disqualifications are confined to the case of remarriage of a widow of a
predeceased son, widow of a predeceased son of predeceased son and widow of
thebrother.
6. Another disqualification stated in the Act relates to a murderer who is excluded on the
principles of justice and equity. Conversion is no longer a ground to exclude a person
from inheriting the property but a converts descendants have been disqualified from
inheriting the property of their Hindurelatives.

7. The right of illegitimate children to inherit the property of their mother has been
preserved but such children are disqualified to succeed to their father‘sproperty.

The Hindu Succession (Amended) Act, 2005 (39 of 2005)

This amendment came into force on September 9, 2005 as the Government of India issued a
notification to this effect. The Act removed gender discriminatory provisions in the previous
Hindu Succession Act of 1956 and gave the following rights to daughters:

The daughter of a coparcener becomes a coparcener in her own right in the same manner as
the son. (Coparcener is a person who has equal right in the inheritance of an undivided
property.)

The daughter has the same rights in the coparcenary property as she would have had if she
had been a son;

The daughter shall be subject to the same liability in the said coparcenary property as that of a
son;

The daughter is allotted the same share as is allotted to a son;


According to Supreme Court of India, Hindu female inheritors not only has the succession
rights but also the same liabilities fastened on the property along with the male members. A
new Section (6) provides for parity of rights in the coparcenary property among male and
female members of a joint Hindu family on and from September 9, 2005. This is a crucial date
for the followingreason:

This Act applies to a daughter of the coparcener, who is born before September 9, 2005 (and
alive on 9 September 2005) on which date the amendment came into force. It does not matter
whether the daughter concerned was born before 1956 or after 1956 (when the actual Act
came into force) since the date of birth was not a criterion for application of the Principal Act.
And there is also no dispute about the entitlement of daughters born on or after September
9,2005.

• THE INDIAN SUCCESSION ACT,1925


The Indian Succession Act, 1925can be broadly be divided into two categories, testamentary
succession and intestate succession. Testamentary succession is applicable when a written
Willhas been created. Intestate succession is applicable when a person dies without a Will and
his properties are to be distributed as per the applicable laws based on his religion.
For Christians, the laws relating to both testamentary and intestate succession is as per the
Indian Succession Act, 1925.

For Jains, Buddhists and Sikhs, they have been given a wider meaning under the Indian
Succession Act. Hence, laws relating to testamentary succession as per the Indian Succession
Act, 1925 is applicable for them also. Intestate succession is applicable to them as per the
Hindu Succession Act, 1956.

• WILL &TESTAMENT

Making a Will helps ensure one's property devolves as wished and the right heirs receive their
fair shares. Under the Indian Succession Act 1925, a Will is a legal declaration of the intention
of the testator, with respect to his property which he desires to be carried into effect after his
death.
After the death of a person, his property devolves in two ways - according to his Will i.e.
testamentary, or according to the respective laws of succession, when no Will is made. In case
an individual dies intestate (no Will is made), the laws of succession come into play. A Will is a
legal declaration. Certain formalities must be complied with in order to make a valid Will. It
must be signed and attested, as required by law. A Will is intended to dispose off property.
There must be some property which is being given to others after the death of thetestator.
A Will becomes enforceable only after the death of the testator. It gives absolutely no rights to
the legatee (the person who inherits) until the death of the testator. It has no effect during the
lifetime of the testator.

A testator can change his Will, at any time, in any manner he deems fit. Every person of
sound mind, and not a minor, can make a Will. If a person is of unsound mind at the time of
making a Will, the Will is not enforceable.

A Will, obtained by force, coercion or undue influence, is a void Will as it takes away the free
agency of the person. A Will, made under influence of intoxication or in such a state of body
or mind, sufficient to take away free agency of the testator, is void. A Will can be
made at any time in the life of a person. There is no restriction on how many times a Will can
be made by a testator. However, only the last Will made before his death is enforceable. A Will
has to be executed by the testator, by signing or affixing his thumb impression on it. It should
be attested by two or more witnesses, each of whom should have seen the testator signing
theWill.

Though the registration of a Will is not compulsory, it can be registered with the sub-
registrar. If, at any time, the testator wishes to withdraw the Will, he can do so. A Willalso can
be sealed and kept in safe custody.

On the death of the testator, an executor of the Will or a heir of the deceased testator can
apply for probate. The court will ask the other heirs of the deceased if they have any
objections to theWill.

If there are no objections, the court will grant probate. A probate is a copy of a Will, certified
by the court, and is conclusive evidence that the Will is genuine.

If a testator intends to make a few changes to the Will, without changing the entire Will, he
can do so by making a codicil to the Will. The codicil can be executed in a similar way as the
Will.
One must note that a Will or codicil is not unalterable or irrevocable. They can be altered or
revoked at any time. In case any objections are raised by any of the heirs, a citation has to be
served, calling upon them to consent. This has to be displayed prominently in thecourt.

If no objection is received, the probate will be granted. It is only after this that the Will comes
into effect.

One can make some provision for a faithful servant, a nurse, a friend in need of money and so
on.
Further one can fulfill his spiritual desires like creating a trust, donating to good causes like
orphanages, temples, old age homes, hospitals, educational institutions, social service
organisationsetc.
Any one of sound mind and not being a minor may dispose of his/her property by Will.
Registration of a Will is purely optional and it is not compulsory. It
cannot be ordinarily be tampered with, destroyed, mutilated, lost or stolen. It is kept
in the safe custody of the office of the Registry. If an unregistered Will is lost, the testator's
wish cannot be given effect as it will be difficult to trace the Will. Probate is a document issued
under the seal and signature of a Court officer, certifying that a particular Will was
proved, with a copy of the will annexed. The Supreme Court has recently
held that petition for probate or lettersof
administration of the Will of a testator must be filed within three years from the date of death
of the testator. No probate is necessary for Christian and Muslim Wills.Under Muslim
law, male and female can make Will. Will by Pardanasin woman is also valid but stronger
evidence is needed to prove the genuineness of the same. The executor is the most
important person in the Will. An executor has a duty to collect and realise the estate of the
deceased, pay his debts and distribute the legacies as mentioned in the Will by the testator.
The duty of the executor is to probate the Will in a manner known to law. The court shall grant
probate only to an executor who has been named in theWill.

All Wills can be revoked, either impliedly or expressly, either by conduct or by a specific
document. By conduct, the Will can be presented to be revoked by the testator. For
example, a testator may make bequest of property in his Will to a person, but he may dispose
of the said property even during his lifetime.
This is called implied revocation. Suppose, the testator makes a bequest of a vacant land in his
Will, but subsequently the testator himself constructs a dwelling house therein, in such
circumstance, the Will can be deemed to have been expressly revoked by the testator.
It has been experienced that when there is a Will, painful litigation in the family of the testator
is prevented. Only in a very few cases, litigation crops up questioning the genuineness of the
Will.
SUCCESSION CERTIFICATE

Definition

A succession certificate is given to the successor of a deceased person who dies without
making a will in order to establish the authenticity of the successor and also to give an
authority over the deceased person‘s debts and securities.

Who issues a succession certificate?

A succession certificate is issued by the district judge of the relevant jurisdiction. The relevant
jurisdiction would be where the deceased person ordinarily resided at the time of his death or
if no such place is available, the jurisdiction within which any property belonging to the
deceased may be found.

What are the particulars to be included in the petition for succession certificate?

The petition for the succession certificate made to the district judge must be signed and
verified by the applicant/successor and include the following details:

1. The date, time and place of the death of the deceased and death certificate will be
attached, ifavailable.

2. The details of ordinary residence of the deceased at the time of his death; and if such
details are not available, then the details of the property that is within the jurisdiction
of the district judge to whom such an application ismade.
3. The family or other near relatives of the person deceased andtheir respective
residences.

4. The rights of thepetitioner.

5. The absence of any reason to invalidate the grant of thecertificate

6. The debts and securities in respect of which the application for such a certificate
ismade.

7. Prayer Clause.

Procedure to obtain a succession certificate

The legal procedure to obtain the succession certificate is mentioned as below:

1. Step 1: First draft the petition, verify and sign the same and submit it to the district
judge in the appropriate jurisdiction after paying the appropriate court fees.

2. Step 2: The district judge will read and hear upon the application and if the same is
admitted, he shall fix a day for the hearing in respect of the same and also send notice
of the hearing to the defendants/otherparties.

3. Step 3: After hearing all the parties concerned, the judge will decide if the applicant is
within his right to apply for the application and shall grant the certificate to him
ifsatisfied.
4. Step 4: The district judge may also require the applicant to provide a bond with one
or more sureties or any other security so as to make good any possible loss arising out
of the use or misuse of suchcertificate.
Validity of the Certificate

The succession certificate stands valid anywhere within India. However, where a certificate has
been granted to a person who is a resident of a foreign country, by an Indian representative
(as appointed by the government), of such foreign country, the certificate will stand valid only
if properlystamped.

What is the effect of the Certificate?

The main purpose of this certificate is to provide protection to all parties paying debts where
such payments are made in good faith. The certificate holder is also empowered to receive
any interest/dividend on the securities and negotiate or transfer such securities as mentioned
in the certificate. Thus all payments made to and by the certificate holder on behalf of the
deceased person will be legally valid. However, this does not necessarily mean that the
certificate holder is the owner of the securities or the legal heir. The legal heir/heirs are
determined by a separate procedure oflaw.
What are the differences between Succession Certificate and the Legal Heir Certificate?

The following are the main differences between Succession Certificate and Legal
HeirCertificate:
Succession Certificate Legal Heir Certificate
Applicability To gain authority to obtain the debts To stake a claim as a rightful heir
and securities of the deceased where a to the estate of the deceased
will has not been drawn up
Contents Relationship of the applicant to the Lists all the legal heirs of the
deceased and the list of debts and deceased
securities sought.
Function Establishes authority of certificate holder Identifies and establishes living
to inherit debts and securities of the heirs of a deceased person
deceased and provides protection
to parties paying debts
Effect The holder may not be the ultimate The holder is entitled to inherit
beneficiary of the estate of the the estate
deceased
VALUATION OF REAL ESTATE
1. COST, PRICE AND VALUE
Cost, Price and Value are three basic terms which are frequently used in valuation. These three
terms are distinctly different from each other and never substitute one another. We must
thoroughly understand the meaning of these words and should not use these words in wrong
context.

COST: Cost is the amount required to acquire or create the asset. When that asset has been
acquired or created, its cost is a fact. Price is related to cost because the price paid for an asset
becomes its cost to the buyer.

PRICE: Price is the amount asked, offered or paid for an asset. Because of the financial
capabilities, motivations or special interests of a given buyer or seller, the price paid may be
different from the value which might be ascribed to the asset by others.

VALUE: The word ―value‖ refers to the judgment of the valuer of the estimated amount
consistent with one of the bases of value set out in IVS 104 Bases of Value.

2. TYPES OFVALUE
MARKET VALUE
Market Value is the estimated amount for which an asset or liability should exchange on the
valuation date between a willing buyer and a willing seller in an arm‘s length transaction, after
proper marketing and where the parties had each acted knowledgeably, prudently and without
compulsion. The definition of Market Value must be applied in accordance with the following
conceptual frame work:
1. ―The estimated amount refers to a price expressed in terms of money payable for the
asset in an arm‘s length market transaction. Market Value is the most probable price
reasonably obtainable in the market on the valuation date in keeping with the market
value definition. It is the best price reasonably obtainable by the seller and the most
advantageous price reasonably obtainable by the buyer. This estimate specifically
excludes an estimated price inflated or deflated by special terms or circumstances such
as typical financing, sale and leaseback arrangements, special considerations or
concessions granted by anyone associated with the sale, or any element of value
available only to a specific owner or purchaser.

2. ―An asset or liability should exchange‖ refers to the fact that the value of an asset or
liability is an estimated amount rather than a predetermined amount or actual sale
price. It is the price in a transaction that meets all the elements of the Market Value
definition at the valuation date.
3. ―On the valuation date‖ requires that the value is time-specific as of a given date.
Because markets and market conditions may change, the estimated value may be
incorrect or inappropriate at another time. The valuation amount will reflect the market
state and circumstances as at the valuation date, not those at any other date.
4. ―Between a willing buyer‖ refers to one who is motivated, but not compelled to buy.
This buyer is neither over eager nor determined to buy at any price. This buyer is also
one who purchases in accordance with the realities of the current market and with
current market expectations, rather than in relation to an imaginary or hypothetical
market that cannot be demonstrated or anticipated to exist. The assumed buyer would
not pay a higher price than the market requires. The present owner is included among
those who constitute ―the market

5. ―And a willing seller is neither an over eager nor a forced seller prepared to sell at any
price, nor one prepared to hold out for a price not considered reasonable in the current
market. The willing seller is motivated to sell the asset at market terms for the best
price attainable in the open market after proper marketing, whatever that price may be.
The factual circumstances of the actual owner are not a part of this consideration
because the willing seller is a hypothetical owner.

6. ―In an arm‘s length transaction‖ is one between parties who do not have a particular or
special relationship, e.g., parent and subsidiary companies or landlord and tenant, that
may make the price level uncharacteristic of the market or inflated. The Market Value
transaction is presumed to be between unrelated parties, each acting independently.
7. ―After proper marketing‖ means that the asset has been exposed to the market in the
most appropriate manner to effect its disposal at the best price reasonably obtainable
in accordance with the Market Value definition. The method of sale is deemed to be
that most appropriate to obtain the best price in the market to which the seller has
access. The length of exposure time is not a fixed period but will vary according to the
type of asset and market conditions. The only criterion is that there must have been
sufficient time to allow the asset to be brought to the attention of an adequate number
of market participants. The exposure period occurs prior to the valuation date.

8. ―Where the parties had each acted knowledgeably, prudently presumes that both the
willing buyer and the willing seller are reasonably informed about the nature and
characteristics of the asset, its actual and potential uses, and the state of the market as
of the valuation date. Each is further presumed to use that knowledge prudently to
seek the price that is most favorable for their respective positions in the transaction.
9. Prudence is assessed by referring to the state of the market at the valuation date, not
with the benefit of hindsight at some later date. For example, it is not necessarily
imprudent for a seller to sell assets in a market with falling prices at a price that is lower
than previous market levels. In such cases, as is true for other exchanges in markets
with changing prices, the prudent buyer or seller will act in accordance with the best
market information available at the time.

10. ―And without compulsion‖ establishes that each party is motivated to undertake the
transaction, but neither is forced or unduly coerced to complete it.

INVESTMENT VALUE
Investment Value is the value of an asset to a particular owner or prospective owner for
individual investment or operational objectives. Investment Value is an entity-specific basis
of value. Although the value of an asset to the owner may be the same as the amount that
could be realised from its sale to another party, this basis of value reflects the benefits
received by an entity from holding the asset and, therefore, does not involve a presumed
exchange. Investment Value reflects the circumstances and financial objectives of the entity
for which the valuation is being produced. It is often used for measuring investment
performance.

FAIR VALUE
Fair value is defined as the price that would be received to sell an asset or paid to transfer a
liability in an orderly transaction between market participants at the measurement date.

A fair value measurement is for a particular asset or liability. Therefore, when


measuring fair value an entity shall take into account the characteristics of the asset or
liability if market participants would take those characteristics into account when
pricing the asset or liability at the measurement date. Such characteristics include, for
example, the following:
i. the condition and location of the asset; and

ii. restrictions, if any, on the sale or use of the asset.

A fair value measurement assumes that the asset or liability is exchanged in an orderly
transaction between market participants to sell the asset or transfer the liability at the
measurement date under current market conditions.
A fair value measurement assumes that the transaction to sell the asset or transfer the
liability takes place either:
i. in the principal market for the asset or liability; or

ii. in the absence of a principal market, in the most advantageous market for the
asset or liability.
An entity shall measure the fair value of an asset or a liability using theassumptions
that market participants would use when pricing the asset or liability, assuming that
market participants act in their economic bestinterest.
Fair value is the price that would be received to sell an asset or paid to transfer a
liability in an orderly transaction in the principal (or most advantageous) market at the
measurement date under current market conditions (ie an exit price) regardless of
whether that price is directly observable or estimated using another valuation
technique.
A fair value measurement of a non-financial asset takes into account a market
participant's ability to generate economic benefits by using the asset in its highest and
best use or by selling it to another market participant that would use the asset in its
highest and best use.
The highest and best use of a non-financial asset establishes the valuation premise
used to measure the fair value of the asset.

SPECIAL VALUE
An amount above the Market Value that reflects particular attributes of an asset that are
only of value to a special purchaser is called Special Value. A special purchaser is a
particular buyer, or a restricted class of buyers, for whom a particular asset has special value
because of advantages arising from its ownership that would not be available to general
purchasers in the market. Special value can arise where an asset has attributes that make it
more attractive to a particular buyer than to the general body of buyers in a market. These
attributes can include the physical, geographic, economic or legal characteristics of an
asset. Market value requires the disregard of any element of special value because at any
given date it is only assumed that there is a willing buyer, not a particular willing buyer.
When special value is reported, it should be clearly distinguished from marketvalue.

The attributes of an asset that could be of value to a special purchaser include any element
of synergistic value that would be generated by its acquisition.

SYNERGISTIC VALUE
It is an amount that reflects particular attributes of an asset that are only of value to a
special purchaser. It is the result of a combination of two or more assets or interests where
the combined value is more than the sum of the separate values. If the synergies are only
available to one specific buyer then Synergistic Value will differ from Market Value. The
added value above the aggregate of the respective interests is often
referredtoas―marriagevalue.

LIQUIDATION VALUE
Liquidation Value is the amount that would be realised when an asset or group of assets
are sold on a piecemeal basis. Liquidation Value should take into account the costs of
getting the assets into saleable condition as well as those of the disposal activity.
Liquidation Value can be determined under two different premises of value:
• an orderly transaction with a typical marketing
• period,or
• a forced transaction with a shortened marketingperiod A Valuer must disclose which

premise of value isassumed.

RELAISABLE VALUE
Market value minus all costs related to (1) holding costs during the expected marketing period,
(2) all selling costs related to disposition of the property and (3) the cost of funds or rent loss
during the anticipated marketing period. Holding costs include, but are not limited to, real
estate taxes, property insurance, liability insurance, utilities, and normal repairs and
maintenance. Selling costs include, but are not limited to, brokerage commissions, closing
costs, title work, and surveys. This is also called Net Realisable Value (NRV).

FORCED SALE VALUE


The term―forced sale is often used in circumstances where a seller is under compulsion to sell
and that, as a consequence, a proper marketing period is not possible, and buyers may not be
able to undertake adequate due diligence. The price that could be obtained in these
circumstances will depend upon the nature of the pressure on the seller and the reasons why
proper marketing cannot be undertaken. It may also reflect the consequences for the seller of
failing to sell within the period available. Unless the nature of, and the reason for, the
constraints on the seller are known, the price obtainable in a forced sale cannot be realistically
estimated. The price that a seller will accept in a forced sale will reflect its particular
circumstances, rather than those of the hypothetical willing seller in the Market Value definition.
A ―forced sale‖ is a description of the situation under which the exchange takes place, not a
distinct basis of value.

If an indication of the price obtainable under forced sale circumstances is required, it will be
necessary to clearly identify the reasons for the constraint on the seller, including the
consequences of failing to sell in the specified period by setting out appropriate assumptions. If
these circumstances do not exist at the valuation date, these must be clearly identified as
special assumptions.

A forced sale typically reflects the most probable price that a specified property is likely to bring
under all of the following conditions:

1. consummation of a sale within a short time period,

2. the asset is subjected to market conditions prevailing as of the date of valuation or


assumed timescale within which the transaction is to be completed,

3. both the buyer and the seller are acting prudently and knowledgeably,

4. the seller is under compulsion to sell,

5. the buyer is typically motivated,


6. both parties are acting in what they consider their best interests,

7. a normal marketing effort is not possible due to the brief exposure time, and

8. payment will be made in cash.

9. The value estimated by fulfilling all the above conditions is termed as Forced Sale Value.

SALVAGE VALUE
The value of an asset that has reached the end of its economic life for the purpose it was made
is known as salvage value. The asset may still have value for an alternative use or for recycling. It
applies to a dismantled building but is not applicable to land.

SCRAP VALUE
This is the amount that is likely to be obtained for a portion or a component of a material item
when sold at the end period of its useful life. It is a kind of salvage value. For example, if a
building is demolished then certain old material that may be sold will bring in money in the
form of scrap value.

RESIDUALVALUE
The anticipated value of an asset at the expiration of its useful life.

REINSTATEMENT VALUE
Reinstatement Value means the cost of replacing or reinstating on the same site, property of
the same kind or type but not superior to or more extensive than the insured property when
new.

USE VALUE

In stark contrast to market value and fair value, use value is the value a specific property has for
a specific use. In estimating use value, a valuer focuses on the value the real estate contributes
to the enterprise of which it is a part or the use to which it is devoted, without regard to the
highest and best use of the property or the monetary amount that might be realized from its
sale.

Real property has both a use value and a market value, which may be the same or different
depending on the property and the market. For example, an older manufacturing plant that is
still used by the original owner may have considerable use value to that owner but only a
nominal market value for another use. Use value may vary depending on the management
of the property and external conditions such as changes in business operations. For example, a
factory designed around a particular assembly process may have one use value before a major
change in assembly technology and another use value afterward.

Use value valuation assignments may be performed to value assets (including real property) for
mergers, acquisitions, corporate financial reporting, or securities issues. These types of
assignments are sometimes encountered in valuing industrial real estate when the existing
business includes real property.

PUBLIC INTEREST VALUE

Historically, public interest value has been used as a general term covering a family of value
concepts that relate the highest and best use of property to noneconomic uses. (Other terms
for similar concepts include natural value, intrinsic value, aesthetic value, scenic value, and
preservation value.) The analysis of public interest value tends to be driven by social, political,
and public policy goals rather than economic principles.

3. BASIC ELEMENTS OF VALUE - MARKETABILITY, UTILITY, SCARCITY, AND


TRANSFERABILITY
MARKETABILITY AND VALUE
The marketability of a property is determined by how ready it is to sell. Property‘s capacity to
deliver services to meet human needs, house economic activities, and supply satisfaction and
amenities determine eagerness of the buyers to buy it. Only value of marketable real estate
depends upon marketability. Non-marketable real estate value does not depend upon
marketability. In case of marketable real estates, the more and more it becomes marketable, the
number of buyers will increase and the value will increase. Therefore, the value of a property is
directly proportional to its marketability.

The following defects make a property less marketable and thus reduce the value considerably:

1. Outstanding mortgages/liens
2. Restrictive covenants

3. Outstanding future interests of others in the property, i.e. a "reverter".


4. Encumbrances

5. Easements on the property


6. Variations in the names of grantors and grantees

7. Variations in the chain of title

8. Adverse possession claims

9. Structural encroachments

10. Existing violations of an equitable servitude or covenant


11. Zoning restriction violations

UTILITY AND VALUE UTILITY means the state of being useful, profitable, or
beneficial. Therefore, with the increase in utility the value of real properties increases.

SCARCITY AND VALUE If demand of a commodity is high and price of the commodity is
high being short in supply then that particular commodity flows from other parts of the market
to that part where it is scarce and thus brings down the price in a traditional economic market.
Such a movement of price is possible because goods and commodities can physically flow from
one part of the market to other. But things are quite different with the landed property. You
cannot produce land and so the question of increasing the supply of land through industrial
production does not arise. Total area of land is limited by nature and you have to get this thing
accepted in any study of the real estate market. So is the supply of land limited in a particular
locality. When the demand for land rises, the supply being fixed, the value of land also rises
sharply. Therefore, with increase in demand and fixed supply of land as well as other real
properties (because of stringent development control rules) a scarcity of land as well as built-up
spaces is created in the market. This results in increase in value of the properties in that specific
area. Therefore, value of properties is directly proportional to the scarcity of that property in the
real estate market.

TRANSFERABILITY AND VALUE

In an economic market goods and commodities are traded freely and flown from one part of
the market to another in response to a demand and supply condition. But things are quite
different with the landed property. Landed property cannot be transferred physically from one
part of the market to another like other goods and commodities. As a result, even when overall
supply of developed land or real estate is increased through investment, supply may remain low
at specific pockets due to total non availability of developable land in such areas. This
characteristic of non-transferability of real estate causes a rise in value in some locations while
value may go down in other locations due to over-supply.

If we discuss ‗transferability‘ of real estate as an inherent legal character of it, we may


declare that value of a real estate is directly proportional to its ease of transferability. The more
easily it could be transferred, the better value the owner of the estate could expect.

4. FACTORS AFFECTING VALUE - PHYSICAL, ECONOMIC, LEGAL AND


SOCIAL
PHYSICAL FACTORS AFFECTING VALUE OF REAL ESTATE:

Location: It attributes most in determining the value of a real estate. The more and more the
real estate is located nearer to centres from where the residents can fulfill their every day needs,
the more and more would be the value.

Infrastructure: The ease of receiving water, electricity, roads, sewerage system, drainage
system, solid waste management etc. to the residents of a real estate project increases its value.

Quality of Construction: The better the quality of construction the higher shall be the value of
that real estate.

Planning: The better the planning the better shall be the value of that real estate. For example,
if a real estate housing project provides parking facilities for the visitors shall fetch more value
than one not having this facility.

Amenities and facilities: The more the amenities and facilities (e.g. swimming pool, jogging
track, club, gymnasium, indoor sports centre etc.) the better shall be the value of that real
estate.

ECONOMIC FACTORS AFFECTING REAL ESTATE


Land is a physical entity with inherent ownership rights that can be legally limited for the good
of society. Land is also a major source of wealth, which, in economic terms, can be measured in
money or exchange value. Land and its products have economic value only when they are
converted into goods or services that are useful, desirable, paid for by consumers, and limited
in supply. (A product with unlimited supply will have a low value.) The economic concept of
land as a source of wealth and an object of value is central to value theory.
The economic concept of land reflects a long history of thought on the sources and bases of
value, which is referred to as value theory. Value theory contributes to the value definitions
used in valuations and valuation literature, and it is an important part of the philosophy on
which professional valuation practice is founded.
Inflation: Real estate is said to be a hedge against inflation. The characteristic of real estate
prices to overcome inflation makes real estate a suitable investment option in any economic
environment. During inflationary times investment in long-term government securities or other
fixed interest securities is not secured in real terms. The real value of money associated with
such investments falls with time causing a loss of capital. But value of real estate continues to
grow with time at a faster rate than the rate of inflation. As a result, the real value of money in
real estate investment is maintained in real terms. Therefore, during inflationary times there is
an increase in demand for investment in real property from a section of investors to maintain
the real value of money.

Population changes: As more and more people migrate in an area, there is an increase in
demand for land. But since total area of land is fixed by nature, the per capita allocation of
space for any use reduces. Supply of land being low and demand remaining high the value of
land sharply rises

Economic Growth: The economic growth is indicated by the growth rate of GDP, industrial
production, employment data etc. Studies indicate that when these indicators are vigilant the
economy of the country is of very sound health and real estate demand is high. Conversely,
when these indicators give low values the economy slows down in growth and demand for
landed property shrinks. Economic growth leads to the increase in real value of the income of
people. Owing to the rising income people are able to spend a larger percentage of their
earnings on home loans. Thus there is a rise in demand for residential real estate. The converse
is true of a time of slowing down of economic growth. Thus when economy slows down and
there is rise of unemployment rate or fear of unemployment, lesser number of people bids for
houses and demand shrinks.

LEGAL FACTORS AFFECTING REAL ESTATE

Land use reflects the needs and values of organized society. In countries where the ownership
and marketability of land are not free, government often dictates the use of land. In free market
economies, land use is regulated within a framework of laws. To understand how the various
forces affecting land operate, the basic role of law must be recognized.
The cultural, political, governmental, and economic attitudes of a society are reflected in its
laws. The law does not focus on the physical characteristics of land but on the rights and
obligations associated with various interests in land.

Clear Title: Marketable title (real estate) is a title that a court of equity considers to be so free
from defect that it will legally force its acceptance by a buyer. If one cannot produce a clear title
of deed to the property, then the prospective buyer should expect to lose in a specific
performance action. Absolute clear title of a real property fetches maximum value for it.
Rights and interests: Freehold right fetches the maximum value of a property followed by
leasehold right, assignor‘s right, right of a life interest. A tenant or a licensee doesn't have any
valuable right of a real property. Freehold rights are also sometimes curbed by many legal
factors.

Covenants: Covenants agreed between two parties in a contract has a substantial effect on
valuation. Negative covenant viz. ‗not to sublease‘ substantially reduces the value of leasehold
right of a property.

Statutes: Existing statues e.g. building bye laws, acquisition act, rent control laws, taxation acts,
transfer of property act etc. have a great effect on valuation of real estates. For a small change
in Floor Area Ratio (FAR) / Floor Space Index (FSI) makes a considerable change in the value of
real estate in urban areas.

SOCIAL FACTORS AFFECTING REAL ESTATE


Modern society has become increasingly concerned with how land is used and how rights are
distributed. The supply of land is fixed, so increased demand for land exerts pressure for land to
be used more intensively. Conflicts often arise between groups that hold different views on
proper land use. Some believe that land is a resource to be shared by all. Some want to
preserve the land‘s scenic beauty and important ecological functions. Others view land primarily
as a marketable commodity; they believe society is best served by private, unrestricted
ownership. For example, the developer of a proposed shopping center or a business park may
view a particular parcel of land as developable in a desirable and affordable location serving a
definable market area. On the other hand, local residents may argue that, as the site of a
significant freedom fighter‘s residence, the parcel deserves government protection (if taxpayers
could be persuaded to support such a public investment in historical preservation). These
conflicting views do not alter the constitutional rights of ownership or market concepts of land.
Rather, they reflect controversies that arise between the property rights of the individual and
those of society. As a resource, land may be protected for the good of society. As a marketable
commodity, the ownership, use, and disposal of land are regulated so that individual rights are
not violated.

Customs: Social customs sometimes override the Acts which also has substantial effect on
valuation.

Rearrangement of family structure: Joint family structure in India is now shifting towards
nuclear family leading to increase in demand for individual flats.

Urge to reside close proximity to celebrities: Urge to reside close to celebrity‘s house
increases demand vis-à-vis value in local basis.
5. HIGHEST AND BEST USE, VALUE IN USE, VALUE INEXCHANGE
HIGHEST AND BEST USE
Highest and best use is the use, from a participant perspective, that would produce the highest
value for an asset. Although the concept is most frequently applied to non-financial assets as
many financial assets do not have alternative uses, there may be circumstances where the
highest and best use of financial assets needs to be considered. The highest and best use must
be physically possible (where applicable), financially feasible, legally allowed and result in the
highest value. If different from the current use, the costs to convert an asset to its highest and
best use would impact the value. The highest and best use for an asset may be its current or
existing use when it is being used optimally. However, highest and best use may differ from
current use or even be an orderly liquidation. The highest and best use of an asset valued on a
stand-alone basis may be different from its highest and best use as part of a group of assets,
when its contribution to the overall value of the group must be considered. The determination
of the highest and best use involves consideration of the following:

1. To establish whether a use is physically possible, regard will be had to what would be
considered reasonable by participants.

2. To reflect the requirement to be legally permissible, any legal restrictions on theuse of


the asset, e.g. town planning/zoning designations, need to be taken into account as well
as the likelihood that these restrictions will change.
3. The requirement that the use be financially feasible takes into account whether an
alternative use that is physically possible and legally permissible will generate sufficient
return to a typical participant, after taking into account the costs of conversion to that
use, over and above the return on the existing use.

4. Market value should be the result of the highest and best use of the property that
maximizes its potential. Such highest and best use should have regard to what the
market participants have considered best before making a bid and what is physically
possible, legally permissible and financially feasible.
5. As a matter of fact, on many occasions the market value shall be the existing or current
use value of the asset. But if an alternative use is possible, and that is legally permissible
then cost of putting the property in that use by conversion or redevelopment should
have an impact on the value of the asset and if the most benefits generated as a result
of conversion or redevelopment exceeds the previous benefits arising before conversion
or redevelopment and is considered sufficient then that alternative use might lead to
the highest and best use.

6. So, in order to estimate the market value of the property, in its highest and best use, the
Valuer has to consider all the steps described in the preceding two
paragraphs.Heistobeconsciousofthefactthatanalternativeuseofaproperty
may lead to the highest and best use provided they meet the criteria set down in the
preceding paragraphs.

VALUE IN USE
International Financial Reporting Standards define value in use as ―the discounted present
value of estimated future cash flows expected to arise from the continuing use of an asset and
from its disposal at the end of its useful life.‖

VALUE IN EXCHANGE
It is the amount of goods and services which we may obtain in the market in exchange of a
particular thing. In other words, it is the price of a particular good which can be sold and
bought in the market. For instance, if one kg of rice can be obtained in exchange of one dozen
of banana, then we may say that value of one kg of rice is equal to one dozen of banana. Thus,
value-in-exchange depends on two things:

1. Time: Value-in-exchange depends on time element. That is, with the change in time
value- in-exchange for a commodity in respect to other commodity willvary.

2. Place: Value-in-exchange also depends from place to place. Value-in-exchange for a


particular commodity varies from one market to other markets. Hence, it varies
according to time and place. Again, one commodity may have immense use value but
no exchange value or vice versa. For example, water has immense use value but not
exchange value. On the contrary, diamond has huge exchange value but no use value.

6. REAL PROPERTY: RIGHTS AND INTERESTS IN REAL ESTATE, TYPES OF


OWNERSHIPS AND TYPES OF OCCUPANCY IN REAL ESTATE
REAL ESTATE AND REAL PROPERTY

Real estate and real property are much discussed in all knowledgeable circles in modern times.
The International Valuation Standards Council (IVSC) has defined real estate as a physical asset
which includes land and all things that are a natural part of the land e.g. trees and minerals as
well as things that are attached to the land by people, e.g. buildings and site improvements.
Whereas, real property has been defined by the IVSC as all the rights, interests and benefits
related to the ownership of real estate. Real property is a legal concept distinct from real estate
which is a physical asset. This distinction between real estate and real property has not been
rigidly followed in the use of the terms in the
property and valuation circles in this country. However, real estate is an economic commodity
and is produced and supplied for purchase and sale in an economic market but unlike other
economic goods and commodities it has some peculiar characteristics that distinguish it from
other goods and commodities. These peculiar characteristics often dominate the price and
hence value of real estate. We will take up their short introduction in the following paragraphs.

The peculiar characteristics of real estates are:—

1. Fixity in supply: Total quantity of land is fixed in supply by nature. Land is not
produced in factory by human effort. So, if the population in an area increases it has the
effect of putting an upward pressure on price because only those that can pay more by
competition will be able to procure it.

2. Non-transferability: Land/real-estate cannot be transferred physically from one place


to another unlike other goods and commodities. Real estate is a commodity that is
produced and consumed in situ. So, if demand for real estate in an area is increasing
(may be due to a population growth or increased income of people) land/real-estate
being non-transferable supply cannot be augmented to meet the demand and so the
price rises. Of course, in the short run supply can be somewhat increased by more
intensive development of existing properties. E.g. old dilapidated properties are
demolished, and site re-developed with greater quantity of floor area. But ultimately a
point is reached when demand overtakes the supply. This again has the effect of
pushing up the price of real estate.

3. Durability: Real estate is a highly durable commodity and land is virtually


indestructible. So, life of real estate tends to be perpetual. Again, real estate is produced
and consumed in situ. Once therefore, real estate has been produced it cannot be
withdrawn from the market except by demolition. So, there is a risk of capital loss if
prediction of the market trends could not be properly made and market downturns
after the constructional works are complete.

4. Heterogeneity: Goods and commodities as produced in factories are often


homogeneous in all aspects. But, real estate is developed in situ, each site as well as the
buildings having their special features. Even when in the same building block some
difference must exist between two apparently identical apartments. As a matter of fact
no two properties are identical in all respects in the whole world. This characteristic of
real estate is called heterogeneity. Heterogeneity makes comparison of value between
two properties difficult and requires experience and expertise of the Valuer.
BUNDLE OF RIGHTS

Realestateownershipisassociatedwithanumberofrightsoftenreferredtoasthe―bundle of rights or
a ―bundle of sticks, each stick representing a property right. The real estate market is also
referred to as the land market or the property market. The paradox of the land market is that it
does not deal in land. Because, what a transferred in real estate transactions are nothing but
property rights. Real estate is physically non-transferable unlike other commodities. Only the
rights associated with real estate are transferred. When real estate (that is, the rights associated
with real estate) is transferred, a conveyance deed on appropriate stamp paper is drawn. A deed
of conveyance describes fully the rights, privileges and encumbrances of the real estate unit
transferred. It is the transferred rights, privileges and encumbrances that dictate the market
value of the real estate unit as much as the physical aspects of the land and buildings do. A.M.
Honor identified 11 sticks in the bundle of property rights. These are: the right to possess, the
right to use, the right to manage, the right to the income of the thing, the right to the capital,
the right to security, the rights of incidence of transmissibility, right to absence of term, the
prohibition of harmful use, liability to execution and the incident of residuality.

The right to possess This right empowers the owner to exclusive control of the property
physically or excluding others from the benefit or use of the property. This right has been
described by Honor as the ―foundation on which the whole superstructure of ownership
rests‖. Indeed, from a very early stage of evolution of ownership of property rights, possession
has been regarded as the nine points of law.

The right to use the right to use acknowledges the liberty of the owner to use and enjoyment of
the property personally.

The right to manage This right acknowledges power of the owner to decide how and by whom
the property should be used.

The right to the income This right empowers the owner to receive the benefits derived from the
property by allowing it to be used by others on parting with possession and personal use.

The right to capital This right entitles the owner to alienate the property, that is, he can sell and
give away otherwise to another person. This right also entitles him to appropriate, waste or to
modification or destruction of the property.

The right to security This right empowers the owner to retain his rights against expropriation,
that is, acquiring by state or other authorities.

The power of transmissibility This power entitles the owner to bequeath the property that
is, allow to pass on the property to somebody upon death.

The absence of term This right recognizes the perpetual continuity of the ownership right
making it of indeterminable length.

The prohibition of harmful use This property stick imposes a duty on any person using the
property not to use it in a manner so that it causes harm to others.

Liability to execution This stick inflicts a liability on the owner for allowing the property to be
taken away from him for the repayment of a debt.

The incident of residuality. However, the owner may grant some rights to other persons and yet
may remain an owner of the property because he still owns many other rights left. For example,
if the freehold owner of a property grants an easement right to pass through his land to the
owner of the neighboring property, the owner of the easement right is his neighbor having an
enforceable right on the servient property. Yet the freehold owner remains an owner of the
property by virtue of owning the remaining rights. Similarly, when a freehold owner mortgages
his property to a bank and then allows his near relative to occupy the property he still remains a
owner as to the property subject to the terms of the mortgage.

THE PRINCIPAL TYPES OF INTEREST IN REAL ESTATE


The principal types of interest in real estate / property are as follows:

Freehold interest
A freehold interest is of perpetual duration and is the highest form of ownership prevailing in
this country and many other countries like UK and the USA, etc. The bundle of property rights
comprising of all the 11 property incidents/sticks are available in the highest form of freehold
ownership. However, the freehold owner may grant rights over his property to any other
person, such as granting an easement right to walk through his land to access the property by a
neighbour. Or, he may grant a leasehold right to a lessee forgoing his right to occupy the
property and granting many other advantageous rights according to the lease agreement and
reserving some others for his own use as negative covenant. In each case the freeholder will still
remain the owner because many other rights will still be left for his exercise. But, the value of
the interest in the property will reduce depending upon the nature of the encumbrance.

Leasehold interest
A leasehold interest usually lasts for a term of years which may be a short term, a long term, or
a very long term. The rights enjoyed by a lessee depend upon the lease agreement between the
lessor and the lessee. The grant or of the lease is called the lessor who may also be the
freeholder or may be called the landlord. Whereas, the lessee may also be called the
leaseholder or the tenant. The freeholder may curtail any of his property interests described
above as bundle of rights. The agreement between the lessor and the lessee determines
whether the lessee has been granted a marketable title. The rights and liabilities of the lessor
and lessee are determined by a written agreement. In the absence of a written agreement, local
usages determine the rights and liabilities. When both a written agreement and local usages are
absent The Transfer of Property Act (1882), section 108 determines the rights and liabilities of
the lessor and lessee. These rights and liabilities under a lease are binding on both lessor and
lessee. Some of the standard forms of rights that can be of importance to the lessee are as
follows. The right to sub-let; the right to assign or transfer of the lease; the right to mortgage
the leasehold interest; the right to develop the property and construct buildings, etc. The
freeholder‘s or the lessor‘s right to receive back the property from the lessee after the lease is
over is called reversion.

The life interest


The freeholder of a property may grant a life interest in the property to any person which last
so long as the person lives. Like the leasehold interest therefore the life interest is also another
terminable interest in a property. However, whereas the leasehold interest is for a definite term
to be mentioned in the lease agreement, a life interest is for an uncertain duration and hence of
unsound value. Obviously, there is no power of the recipient of a life interest to bequeath any
property on his death. Whereas, the owner of a freehold interest may grant an interest in
property even on his death, because a freehold interest is of perpetual duration. Even, a lessee
may bequeath an interest to a person on his death where the leasehold interest is of very long
term duration and extends far beyond the life of the lessee when the lease agreement does not
provide anything to the contrary. The legal successors of the lessee can step in his shoes and
become the owner of the remaining part of the leasehold interest on the death of the lessee.
But this opportunity is not available to the holder of life interest as he does not hold any right
in the property on his death and all his rights ceases there. If the freeholder, at the time of
granting the life interest grants a right to the life interest holder to transfer his property to
another person then the life interest holder can transfer his right in the life interest to an
outsider for the term of his life only. His interest ceases on his death and then the transferee
loses his title in the property. Life interests are valued according to the tables based on
mortality of people at various ages.

Co-ownership right
The following are the rights and liabilities of the co-owners as decided by Punjab and Haryana
High Court in Sant Ram Nagina Ram case:
1. A co-Owner has an interest in the whole property and also in every parcel of it.
2. Possession of the joint property by one co-owner is in the eye of law, possession of all
even if all but one are actually out of possession.

3. A mere occupation of a larger portion or even of an entire joint property does not
necessarily amount to ouster as the possession of one is deemed to be on behalf of all.

4. The above rule admits of an exception when there is ouster of a co-owner by another.
But in order to negative the presumption of joint possession on behalf of all, on the
ground of ouster, the possession, of a co-owner must not only be exclusive but also
hostile to the knowledge of the other, as, when a co-owner openly asserts his own title
and denies that of the other.

5. Passage of time does not extinguish the right of the co-owner who has been out of
possession of the joint property except in the event of ouster or abandonment.

6. Every co-owner has a right to use the joint property in a husband like manner not
inconsistent with similar rights of other co-owners.

7. Where a co-owner is in possession of separate parcels under an arrangement consented


to by the other co-owners, it is not open to anyone to disturb the arrangement without
the consent of others except by filing a suit for partition.

8. The remedy of a co-owner not in possession, or not in possession of a share of the joint
property, is by way of a suit for partition Or for actual joint possession, but not for
ejectment. Same is the case where a co-owner sets up an exclusive title in himself.

Where a portion of the joint property is, by common consent of the co-owners, reserved for a
particular common purpose, it cannot be diverted to an inconsistent user by a co- owner; if he
does so, he is liable to be ejected and the particular parcel will be liable to be restored to its
original condition. It is not necessary in such a case to show that special damage has been
suffered.

Easement right
An easement is a right which the owner or occupier of certain land possesses, as such, for the
beneficial enjoyment of that land, to do and continue to do something, or to prevent and
continue to prevent something being done, in or upon, or in respect of, certain other land not
his own. By default a property owners attains the right of way, the right to air or right to light,
the right to build, the right to the uninterrupted flow of water. All these are known as the
examples of a property owner's easement rights. The Indian Easement Act says that if a person
has enjoyed these over a period of time, they have a valid right without any restriction, almost
as though it were a privilege.

Transferable Development Rights (TDR)


Transferable development right means transfer the right to develop a land to government, local
authority or corporation. So, when an owner of land transfers his rights of developing
a land to a government, local authority, corporation or government use the same land for
infrastructure projects such as road widening, metro rail projects, park, garden, schools or may
be for making new roads or for any other projects of public utility. DRC (Development rights
certificate) will then be issued to owner of the land, the main purpose of whole process is to
acquire the required amount of land in hassle free manner. Now this DRC will allow the land
owner an additional built up area in return of the area for which he has relinquish his rights and
enables him to develop the given area by himself of transfer his rights for consideration. DRC
issued to land owner if transferable is known as transferable development rights (TDR), which
can be transferred to another entity.

Rights under adverse possession


Adverse possession is a legal principle under which a person who does not have legal title to a
piece of property—usually land (real property)—acquires legal ownership based on continuous
possession or occupation of the land without the permission of its legal owner. The Limitation
Act, 1963, is a key piece of legislation, elaborating on adverse possession. The Act prescribes a
period – 12 years for private properties and 30 years for government- owned ones – within
which you have to stake claim on your property. Any delay may lead to disputes in the future.
The principle on which the Limitation Act is based is that 'limitation extinguishes the remedy,
but not the right'. This means that in case of an adverse possession, the original owner may
have the title over the property but he loses the right to claim such right through a court oflaw.

Some essential requirements to be proved for claiming under adverse possession are:

1. Hostile possession: The intention of the possessor of the property must be to acquire
rights through means of adverse possession. These rights are acquired at the expense
of the rights of the original owner. There must be an express or implied denial of the
owner's title by the possessor. Constructing a boundary wall around the property can be
means of asserting this possession.

2. Public knowledge: The public at large must be aware about the possession of the
claimant. This condition is put in place so that the actual owner has adequate means to
know that someone is in possession of his property and gets reasonable time to act.
However, one is not bound to inform the original owner about it.

3. Actual possession: There must be actual possession throughout the period of


limitation. Physical acts like harvesting crops, repairing the building, planting trees,
erection of shed, etc, could be means through which actual possession can be
determined. The possessor could not claim possession over the property without being
physically possessing it.
4. Continuity: The possessor must be in peaceful, unbroken, uninterrupted and
continuous possession of the property. Any break in the possession will extinguish his
rights.
5. Exclusivity: The possessor must be in sole possession of the property. The possession
cannot be shared by different entities or persons for the claimed time duration.

TYPES OF OCCUPANCY IN REAL ESTATE


Tenancy
Tenancy does not create any valuable right over the property but provided only occupancy
right. Tenancy in India is governed by different rent control legislations in different states. The
rent control legislation is not a central act but is a state act. Therefore, its structure and
purviews are different for different states. Moreover, a number of states have abolished this
legislation. Therefore, to estimate value of the tenant‘s occupancy right, if there be any, Valuer
shall have to be thorough enough about that specific act prevailing in that state. The detail of
tenancy is elaborately discussed in the section Rent Control Laws: Sections pertaining to
Occupancy Rights of Tenants, Freezing of Rent and Protection against Eviction of Tenant and
its effect on value of property of the subject Law–Real Estate‘.

License
Where one person grants to another, or to a definite number of other persons, a right to do, or
continue to do, in or upon the immovable property of the grantor, something which would, in
the absence of such right, be unlawful, and such right does not amount to an easement or an
interest in the property, the right is called a license. A license may be granted by anyone in the
circumstances and to the extent in and to which he may transfer his interests in the property
affected by the license. The grantor of license is called Licensor‘ and the acceptor is called
Licensee‘. Unless a different intention is expressed or necessarily implied, license to attend a
place of public entertainment may be transferred by the licensee; but, save as aforesaid, a
license cannot be transferred by the licensee or exercised by his servant or agents. When the
grantor of the license transfers the property affected thereby, the transferee is not as such
bound by the license Licensee‘s rights on revocation: Where a license is revoked, the licensee is
entitled to a reasonable time to leave the property affected thereby and to remove any goods
which he has been allowed to place on such property. Licensee‘s rights on eviction: Where a
license has been granted for a consideration, and the licensee, without any fault of his own, is
evicted by the grantor before he has fully enjoyed, under the license, the right for which he
contracted, he is entitled to recover compensation from the grantors. In short, license does not
create any type of right over the property. Even it does not even create any occupancy right.
Therefore, valuation of licensee‘s right is not possible.
7. ANNUITIES, CAPITALISATION, RATE OF CAPITALISATION, YEARS’
PURCHASE, SINKING FUND, REDEMPTION OF CAPITAL,
REVERSIONARY VALUE
ANNUITY
It is defined as the Net Annual payment (Return on investment) for the capital invested in an
immovable property or any other form of investment. Rent from land or house, interest on bank
fixed deposit or yield on government security are the examples ofAnnuity.

CAPITALISATION
The conversion of a periodic income to an equivalent capital value is known as capitalisation.

RATE OF CAPITALISATION
The return represented by the income produced by an investment, expressed as a percentage is
known as rate of capitalisation. To illustrate, if a person deposits Rs. l,00,000 in Bank fixed
deposit and Bank offers interest of 8% on FD., rate of 8% is called rate of capitalisation. If an
investor yields 8 % return on his investment in a rented house property, by way of net rent
income, 8 % is called rate of capitalisation.

YEAR’S PURCHASE
Years purchase is defined as capitalised value required to be paid once and for all, in order
toreceiveannualincomeofRe.1forspecifiedperiodoftimeatspecifiedrateofreturn.If
‗C‘ is capital value and ‗N‘ is the net income per annum of a property, Years‘ Purchase(Y.P)
= C / N.

ANNUAL SINKING FUND


It can be defined as an annual recurring fund (amount) required to be set aside every year, for a
given period of time, at the given rate of interest, to recoup capital invested in a property,
interest or return from which would cease after a given period of time.

RATE OF REDEMPTOPN OF CAPITAL


This rate of return is normally adopted when income is a terminable income. To recoup (get
back) the capital invested in such type of property (like leasehold property where income
would cease after some years), this rate of interest is adopted. This rate is also called as Rate of
Recoupment of Capital or Accumulative rate of interest.

REVERSIONARY VALUE
This is generally defined as the present amount of value which is obtained after a specified
period. For calculating the reversionary value of land, the present rate of land is usually adopted
at a certain rate of interest for a deferred period of time. For example, suppose a building stood
on land and it was proposed to demolish the building after four years. It was estimated that
cost of demolition would offset the value of scrap material to be sold. Land value at present is
`10,00,000. Then considering 8% compound interest, reversionary value of land would be 0.735
x 10,00,000 i.e. `7,35,000.

8. CONSTRUCTION AND USE OF VALUATION TABLES


INTRODUCTION
The valuation tables are used as tools of Valuers in drawing their conclusion on carefully
considered estimate of worth of landed property. Without their help the Valuer in each case
would have to be involved in long laborious calculations of mathematical processes. However,
the invention of computers has made the task of entering into laborious calculations rather
simple. But that does not dispense with the use of valuation tables altogether. Indeed, for
practical purposes the tables are still useful tools to Valuers. The valuation tables are based on
compound interest principles as the derivation of their formulae will indicate in the calculations
that follow under this chapter. The use of the tables cuts short the risk of mathematical errors
and that is why they are extensively used by Valuers and their use is likely to be continued
infuture.

SIMPLE INTEREST AMOUNT WORKING


Interest is the fee paid on an amount of money, whether it's loaned, borrowed, or invested.
Simple interest is determined by multiplying the daily interest rate by the principal by the
number of days that elapse between payments.
Simple Interest = P x I x N
Where, P = Principle; I = Interest and N = Number of days between the payments.

Problem
Mr. Dave purchased a car from an automobile dealer in 2018 on easy simple interest installment
basis. As on May 01, 2020 that has a `1,50,000 principal balance. The next monthly installment
payment due date is on June 01, 2020. But Mr. Dave wants to close the
loan account on May 20, 2020. How much amount Mr. Dave has to deposit to close this loan
account, if the simple interest rate is 8% per annum.

Solution
Interest tobepaid = `1,50,000 x (0.08/365) x20
= `658
Total amount tobedeposited = `(1,50,000 +658)
= `1,50,658

COMPOUND INTEREST AMOUNT WORKING


(Amount of `1 table)
The purpose of this table is to indicate the amount to which an investment of `1 shall
accumulate at a given rate of compound interest after a given number of years. Starting simply,
let us assume that interest rate on an investment of `1 is i at the end of 1 year (where i is a
fraction of `1).

Then at the end of 1 year, the amount to which ` 1 is accumulated is `(1+i) At the end of
two years, the amount accumulated is `(1+i)(1+i) or `(1+i)2
At the end of three years, the amount accumulated is `(1+i)2 (1+i) or `(1+i)3
Similarly, at the end of n years the amount accumulated is `(1+i)n
Therefore, the amount of `1 at the end of n years at i rate of interest is (1+i)nProblem
Mr. Ramaswami, an investor, has invested an amount of ` 1,000,000 in purchasing
an urban site on taking a loan at 9% compound rate of interest from a bank. What amount will
he have to repay to the bank after three years when he finds the purchaser for his land?

Solution
The amount to which ` 1 is accumulated at i rate of compound interest after n years is `
(1+i)n
Given that, i=0⋅9 and n=3
So, the amount to which his loan is accumulated
= 1,000,000×(1+i)n
= 1,000,000 (1+0⋅09)3
= 1,000,000×1⋅295
= ` 1,295,000

Therefore, the investor has to refund an amount of ` 1,295,000 to the bank out of the proceeds
of the land.
PRESENT VALUE OF RUPEE WORKING
This table is intended to indicate the present value of ` 1 to be received after a given number of
years at a given rate of compound interest.
As before, assume that interest rate of an investment of ` 1 is i at the end of 1 year. (i is a
fraction of `1).
It is known from Amount of ` 1 Table that,
` 1 invested today accumulates to (1+i) at the end of 1 year and,
` 1 invested today accumulates to (1+i) nat the end of n years.
So, if (1+i)nis the amount after n years, its present value today is ` 1. Therefore, if `1 is the
amount after n years, its present value today is `1/(1+i)n Therefore, present value of `1 to
be received after n years is 1/(1+i)n

Problem
Mr. Datta, a developer plans to develop a large area of land and estimates the residual value of
the site to be ` 10,000,000. But the site will be handed over to him only after 3 years. What is the
present value of the site considering interest rate at 12% perannum?

Solution
Present value of ` 1 = 1/(1+i)n
Where, i is the interest rate on ` 1 and n= number of years. Here, n=3 years" and i=0⋅12
Therefore, present value
= 1/〖(1+0⋅12)3 × `10,000,000
= 0.712× `10,000,000
= `7,120,000
Therefore, present value of the site = `7,120,000

AMOUNT OF RS. 1/YEARWORKING


This table is purported to indicate the accumulated amounts of a series of installments of ` 1
each invested at the end of each year for a given number of years at a compound rate of
interest.
Let the rate of compound interest on ` 1 be I per annum. Let n be given number of years
for the series.
Then the first dose of ` 1 invested at the end of year 1 shall accumulate to (1+i)(n-1)
after n years.
Similarly, the second dose of ` 1 invested at the end of year 2 shall accumulate to (1+i) (n-2)
after n years, and so on …
So, if S denotes the summation of all the accumulated amounts of installments /doses of `
1 for n years deposited as above the amount to which this sum Sis accumulated is given by the
expression,
S= (1+i)(n-1)+(1+i)(n-2)+(1+i)(n-3)… …+ (1+i)(n-r) + (1+i)2+(1+i)+1
or, S= 1+(1+i)+(1+i)2+...(1+i)(r-1)+⋯+(1+i)(n-3)+ (1+i)(n-2)+(1+i)(n-1)
This is a G.P. series the summation of which is given by the formula, S= a(R n-1)/(R-1)
where, a= first term; R= common ratio; n= number of terms. Here, a=1; R=1+i; n=
number of terms.
Therefore,S=(1((1+i)n-1))/(1+i-1)=[(1+i)n-1]/i
Therefore, amount of ` 1 per annum = [(1+i)n-1]/i

Problem
Mr. Chopra, a builder, took 3 years to construct a six-storey building. He incurred an
expenditure of ` 2,000,000 during the first and second years each. During the third year he
expended an amount of ` 2,500,000. What is the total cost at the end of the third year if he has
borrowed capital at 14% interest rate? Assume all costs to have been incurred at the end of the
respective year.

Solution
For all the three years he has a common expenditure of ` 2,000,000. In addition, he
expended another ` 500,000 during the third year.
So, amount of ` 2,000,000 per annum for three years = `2,000,000 x [(1+i)n–1]/i where,
i=0⋅14 and n=3
Therefore, amount of ` 2,000,000 per annum for 3 years
= ` 2,000,000 x [(1+0⋅14)3–1]/(0⋅14)
= (` 2,000,000×0⋅482)/(0⋅14)
=` 2,000,000×3⋅443
=` 6,886,000
Add another ` 500,000 for the third year
=`6,886,000 + `500,000
=`7,386,000
Therefore, total cost of the builder at the end of the third year = `7,386,000

ANNUAL SINKING FUND WORKING

This table indicates the amount which is to be deposited annually at the end of each year for a
number of years to be accumulated to ` 1 at a rate of compound interest.
Let A be the amount which is to be deposited at the end of each year for n years. Let i be
the interest rate on ` 1 in one year.
Then the series of payments will accumulate to ` 1 in the following manner as shown
below.
1= A(1+i)(n-1) + A(1+i)(n-2) + A(1+i)(n-3) +⋯ A(1+i)(n-r) + ...+ A(1+i)3 + A(1+i)2+
A(1+i) + A) or,
1= A [1 + (1+i) + (1+i)2+ (1+i)3 + ⋯ + (1+i)(n-3)+ ……+ (1+i)(n-2) +(1+i)(n-1)]
The R.H.S. is a G.P. series of n terms.
The summation of a G.P. series is obtained from the formula S=[a(R n-1)]/(R-1)
where, S denotes the summation of the series, a the first term, R the common ratio and n
the number of terms.
or, `1 = [A(1(1+i)n-1]/(1+i-1)= A (1+i)n-1]/i
or, A= i/(1+i)n-1
Therefore, the amount of annual sinking fund = i/[(1+i) n-1],which is just the inverse of the
amount of ` 1 per annum table.

Problem

Mrs. Singh took on the lease of a house for 20 years. She paid a premium of ` 923,230 at the
beginning of the lease period and paid a reserved rent of ` 10,000 per annum. The full rental
value for the lease is ` 120,000 per annum (net). What is the amount of sinking fund she should
set aside from her income from the leasehold so as to accumulate to the amount of premium
paid at 2½% compound rate of interest after 20years?

Solution

Let S be the amount of sinking fund per annum to be set aside from Mrs. Singh's income on
subletting the property that accumulates to ` 1 at 2½% compound rate of interest after 20years.
Therefore, S=i/[(1+i)n-1]where i=0⋅025 and n=20 or, S=(0⋅025)/[1⋅02520-1]=0⋅039148
Therefore, to redeem ` 923,230 the total amount to be set aside
= ` 923,230 × 0⋅039148
= `36,143 per annum.

PRESENT VALUE OF FUTURE INCOME OF RS. 1/ YEAR

(Single rate basis)


This table indicates the present capital value of a series of income of ` 1 to be received at the
end of each year at a compound rate of interest for a given number of years.
Let I be the interest on ` 1 for 1 year and n be the number of years. Then, present value of
` 1 to be received after 1 year =1/(1+i)
Similarly, for any number of years n the present value of ` 1 to be received after n years is
given by1/(1+i)n
So, the summation of a series of present values of ` 1 to be received at the end of each year for
n years is given by,

1 1 1 1
𝑆= + 2
+ 3
+. . . + +⋯
1+𝑖 (1 + 𝑖)
(1 + 𝑖) (1 + 𝑖)𝑟
1 1 1
…+ + +
(1 + 𝑖)(𝑛−2) (1 + 𝑖)(𝑛−1) (1 + 𝑖)𝑛
Where, S= summation
But this is a G.P. series with 1/(1+i)as the first term and the common ratio is also
1/(1+i).The number of terms is n.
The summation is given by the formula S= a(Rn-1)/(R-1) where, a=first term;
R=common ratio; and n = number of terms
(1 )×( 1 −1) 1 −1 1− 1
Therefore, 𝑆= 1+𝑖
1
(1+𝑖)
−1
= (1+𝑖)𝑛
−𝑖
= (1+𝑖)𝑛
𝑖
1+𝑖

1 −1
Therefore, present value of ` 1 per annum (single rate) = (1+𝑖)

The expression is also called years' purchase (single rate).


Problem

What is the present value of a net income of ` 100,000 per annum for 15 years at 8%
interestrate?
Solution

Present value of ` 1 per annum


1 1
1− 1− 15 2⋅17217
(1+𝑖) 𝑛 (1+0⋅08)
= = =
𝑖 0⋅08 0⋅25377

Therefore, present value of ` 1 per annum (years' purchase) =8⋅5596 Therefore, present
value of `100,000 per annum = `100,000 × 8⋅5596
= ` 855,960

PRESENT VALUE OF FUTURE INCOME OF RS. 1/ YEAR


(Duel rate basis)

In valuing a property by the income capitalisation technique the valuer is often asked to value a
terminable income such as a leasehold interest. Obviously, the difference between a perpetual
income flow and the terminable income is that the former lasts in perpetuity whereas the latter
comes to an end after a number of years. Therefore, a freehold income is not directly
comparable with a leasehold income which lasts for a given number of years
only. The rate of capitalisation of freehold income cannot therefore be comparable as an
investment return with that of a leasehold income. In order to make a leasehold income
comparable to a freehold income, an amount is set aside from the leasehold income annually
so as to accumulate at a given rate of interest (rather low) to a capital amount of value being
equal to the value of the leasehold interest at the end of the lease period. In this way the
terminable income flow from a leasehold interest can be perpetuated and thus the rate of
capitalisation from the leasehold may now be compared with that of a freehold interest. The
amount thus set aside so as to be accumulated at a low rate of compound
interesttoacapitalsumiscalledthe―sinkingfund‖.Andthelowrateiscalledthe
―accumulative rate of interest‖. The return on the capital or the ―remunerative rate of
interest‖ is that rate of interest that the spendable income works out on the capital value of the
lease. All leasehold interests are valued by the dual rate principle. Obviously, all incomes from
leaseholds have two parts—one part is spendable income which caters to the remunerative rate
of interest and the other part is accumulated—this part caters to the accumulative rate
ofinterest.

Let, 𝑉= present value of ` 1 per annum for 𝑛 years (dual rate)

Let, 𝑖= the interest rate on ` 1 per annum or remunerative rate of


interest

Let, 𝑠= the annual sinking fund for redemption of𝑉


𝑖1
=
(1 + 𝑖1)𝑛 − 1

where, i1= accumulative rate of interest and n = number of years. Then each unit of
` 1 income shall have two components as follows.

1= 𝑉 × 𝑖 + 𝑉 × 𝑠=𝑉(𝑖 + 𝑠)
1
Therefore, 𝑉=
𝑖+𝑠

But V = present value of `1 per annum for n years (dual rate) = years' purchase
(dual rate).
1
Therefore, years' purchase (dual rate) =
𝑖+𝑠
For capitalisation of income from leasehold interest in all cases, the net income
of the lessee should be multiplied by 1
𝑖+𝑠
Problem

A lessee took on lease of a house for 30 years on an exclusive rent of ` 10,000 per annum 15
years back. The property is worth ` 35,000 per annum exclusive of all outgoings today. What is
the value of the lessee'sinterest?
Solution

Gross rental value = ` 35,000 p.a.

Less rent paid by lessee = ` 10,000 p.a.

Net profit rent = ` 25,000 p.a.

Y.P. @ 7½% & 2½% for 15 years = 7.647


Value = ` 191,175

Therefore, value of lessee's interest = ` 191,175

9. URBAN INFRASTRUCTURE AND ITS INFLUENCE ON VALUE OF


REALESTATE
Infrastructure is the fundamental facilities and systems serving a country, city, or other area,
including the services and facilities necessary for its economy to function. Infrastructure is
composed of public and private physical improvements such as roads, bridges, tunnels, water
supply, sewers, electrical grids, and telecommunications (including Internet connectivity and
broadband speeds).The infrastructure is important for faster economic growth and alleviation of
poverty in the country. The adequate infrastructure in the form of road and railway transport
system, ports, power, airports and their efficient working is also needed for urban economy. A
distinguishing feature of infrastructure is that while the demand-supply gap in case of other
factors can be met by importing some of them, the deficiency of infrastructure cannot be made
up through imports. Because location-based the need for relevant infrastructure facility can be
met through development of its capacity in the domestic economy. Economic Survey of India
for the Year 2013 -14 declared, ―Rural economic growth in recent years has put enormous
pressure on existing infrastructure particularly on transport, energy and communication. Unless
it is significantly improved infrastructure will continue to be a bottleneck for growth and
obstacle to poverty reduction‖.

Influence of Power Infrastructure Power or energy is a crucial input into all economic activities
and therefore rapid economic growth is possible only if adequate power is made available
everywhere. It is essential for growth of industry, commercial business as well as for household-
lighting. Uninterrupted power supply attracts investors to set industries in those areas which on
the other hand generates employment giving rise to demand for real estate. Therefore, urban
areas having adequate power facility show higher rate of value of all types of real estates.
Housing areas having no threat of power failure is always more lucrative than areas having poor
power supply records. Therefore, with adequate power supply, the market value of all real
estate increases.

Influence of Road infrastructure


Road is another important infrastructure which is essential for movement of goods, raw
materials and fuel. The cities having better road network show higher degree of
development. Besides movement of goods, inadequate road facility results in congestion of
traffic during pick hours of the day as well as night. This not only results in loss in working
hours but one of the main root causes environmental pollution in urban areas. Therefore,
different parts of a city having different network of road shows distinct difference in values of
real estates. The areas having adequate road facility shows more demand for space than areas
having inadequate road network.

Influence of Railway Infrastructure


Railway transportation system is the most effective environment- friendly and economically-
efficient transport system in India. In the last few decades, railway in the form of ‗Metro
Railway‘ becomes the most efficient and convenient transport system for the city dwellers of
Indian cities. The sub-urban railway system in most of the metro cities protects the urban local
government from excessive pressure of population. Because of this railway system, cities are
expanding every day its city limit. The smooth and hassle free communication through metro
railway increases the demand for spaces on and around the metro railway channels in every
cities. Therefore, same type of units on and around a metro channel always carries better value
than their counterparts situated in other areas.

Influence of Water Infrastructure


Water is going to be the number one determinant for home buyers in this decade. Due to
alarming fall of ground water level, most of city government ban on extracting water from
ground water resources. Therefore, the only resource of potable water in cities becomes the
rivers flowing through or around the city. Rain water harvesting, that is why, becomes a
mandatory regulation for the new building owners of most of the urban areas in India. As a
result all the housing units having the facility of uninterrupted water supply for 24 hours a day
and 365 days in a year are asking for premier price. Thus, good water infrastructure facility
increases the value of a real estate in India to a goodextent.

Influence of Social Infrastructure


Social infrastructure can be broadly defined as the construction and maintenance of facilities
that support social services. Social infrastructures are created to increase social comfort and act
on economic activity. These are schools, parks and playgrounds, structures for public safety,
waste disposal plants, hospitals, sports area, etc. Social infrastructure has a positive impact on
the value of urban real estate. People love to stay as close as possible to these social
infrastructures. Therefore, closeness to social infrastructure increases the value of real estates in
urban areas.

Indian Government Policies for urban infrastructure development


1. HousingPolicy

The National Urban Housing & Habitat Policy 2007 (NUHHP-2007) has been formulated
keeping in view the changing socio-economic parameters of the urban areas and growing
requirement of shelter and related infrastructure. The Policy seeks to promote various
types of public-private partnerships for realizing the goal of
―AffordableHousingforAll‖withspecialemphasisontheurbanpoor.Giventhe
magnitude of the housing shortage and budgetary constraints of both the Central and
State Governments, the NUHHP-2007 focuses the spotlight on multiple stake-holders
namely, the Private Sector, the Cooperative Sector, the Industrial Sector for labour housing
and the Services/ Institutional Sector for employee housing.

2. Jawaharlal Nehru National Urban RenewalMission


The JNNURM was launched in 2005 as the first flagship scheme of the Ministry of Housing
and Urban Affairs. JnNURM implemented by MoHUPA has two components
e.g. Basic Services for Urban poor (BSUP) and Integrated Housing and Slum Development
Programme (IHSDP) which aimed at integrated development of slums through projects for
providing shelter, basic services and other related civic amenities with a view to providing
utilities to the urban poor.

The Mission was initially for a seven year period i.e. up to March 2012 which was extended
upto March 2014 for completion of the already approved projects. During March 2013, the
Mission period was extended by one more year i.e. upto March 2015 to complete ongoing
works. 65 Mission Cities identified based on urban population (Census 2001), cultural and
tourist importance was covered under BSUP and the remaining cities were covered under
IHSDP (887).
It supported 63 cities (7 mega cities, 28 metro cities and 28 capital cities and towns of
historical/religious importance) across the country through perspective building City
Development Plans (CDPs) for specifying infrastructure gaps relating to water, sanitation,
sewerage, drainage and roads on the one hand and deficiencies in housing and basic
services on the other hand.
3. Swachh BharatAbhiyan

Swachh Bharat Abhiyan is a movement across India spearheaded by Government of India


launched on 2ndOct,2014. The earlier schemes like Total Sanitation Campaign, Nirmal
Bharat Abhiyan have been integrated into it. The Swachh Bharat Abhiyan (Urban) is an
initiative of the Ministry of Urban Development. It's focused on elimination of
opendefecation, eradication of Manual Scavenging. It advocates modern and scientific
Municipal Solid Waste Management. It seeks to change behaviors around sanitation and its
linkages with public health. Necessary capacity augmentation of ULB's and creating an
enabling environment for private sector participation in Capex (capital expenditure) and
Opex (operation and maintenance) was alsoincorporated.

4. Atal Mission for Rejuvenation and UrbanTransformation

Atal Mission for Rejuvenation and Urban Transformation (AM RUT) was launched on June
25, 2015. It is built on the idea that Infrastructure creation should have a direct impact on
the real needs of people, such as providing taps and toilet connections to all household - a
learning from the earlier JNNURM Mission.

5. SMART CitiesMission

SMART Cities Mission was launched on June 25, 2015 with 75 percent share of centre and
balance to be financed by the respective state. It targets 98 cities. Smart City advocates
multiple points: Mixed land use in area based developments and increase
land use efficiency in 'unplanned areas' through a range of compatible activities and land
uses including flexibility in land use and building byelaws to adapt to change; Also seeks to
expand housing opportunities for all. An interesting thrust is on creating "walkable
localities" where road network is to be assessed for different types of uses - vehicles/ public
transport, cyclists and pedestrians. It will focus on promoting a variety of transport options
in a Transit Oriented Development (TOD). Reduce congestion, air pollution and resource
depletion, and ensure security. Also seeks to preserve and develop open spaces - parks,
playgrounds, and recreational spaces to improve the quality of life of citizens, reduce the
urban heat. It will offer online services especially using mobiles to bring about
accountability and transparency in service monitoring and seek feedback ofpeople.

6. National Heritage City Development and AugmentationYojana

National Heritage City Development and Augmentation Yojana (HRIDAY) was launched on
January 2015 to preserve the heritage character of 12 cities - Ajmer, Arnravati, Amritsar,
Badami, Dwarka, Gaya, Kanchipuram, Mathura, Puri, Varanasi, Velnakanni and Warrangal -
facilitating economic growth and urban planning in an inclusive manner. The interventions
include development of sanitation facilities, roads, public transportation, parking and
citizen services. The project will end in March 2017 and has a total outlay of Rupees five
hundred crores.

10. REAL ESTATE MARKET AND ITS CHARACTERISTICS, INVESTMENT IN


REAL ESTATE, FACTORS INFLUENCING DEMAND AND SUPPLY
SCHEDULE IN REAL ESTATE

REAL ESTATE MARKET AND ITS CHARACTERISTICS


In economics, a market is defined, not as a fixed place having a demarcated area where goods
and commodities are exchanged. A market is a device by which buyers and sellers come in
contact with one another to form a price. A market may not be located in a single location. E.g.
Indian diamonds may have a worldwide market. Similarly, software produced in India may have
a market in the USA and other western countries. A price is formed in a market by the
interaction of demand and supply of a commodity. This is equally true in the case of real estate
market. However, the real estate market is divided in three tiers, viz. local market, national
market and international market. Each tier of the market again has its own segment of real
estate such as local market of residential real estate, local market of commercial real estate,
local market of retail real estate. Again, national real estate market
has national market of residential real estate, national market of commercial real estate and
national market of retail real estate etc. Similarly, international real estate market has each
segment as international market of residential real estate, international market of commercial
real estate and international market of retail real estate etc. Other types of real estate may have
similar segregation in the three tiers of market.

The efficiency of a market depends on accurate price information of any commodity at different
parts of the market and free movement of commodities from one part of the market to
another. In the case of real estate however, these conditions are not fulfilled. Information on
price of real estate is difficult to obtain as the transactions in real estate are very much
restricted in information, being only available to the buyers and sellers. The lack of transparency
in transactions is a character of real estate anywhere in the world. Again, real estate cannot be
geographically transferred from one place to another. Therefore, if there is an augmentation of
demand in a location, since supply cannot be increased in response immediately, the price rises
in that location. That is why real estate market is said to be sluggish in response. Furthermore,
the dealing cost of real estate is high because of the presence of a considerable stamp duty
which is required for investment in real estate. Also, the information on price of a real estate
may require an analysis by a valuer thus making it costly for an investor to invest in real estate.
Again, investment / purchase of real estate require large funds unlike investment in securities in
the capital market. So, if interest rate for borrowed funds is high or there is a capital crunch in
the economy, investment in real estate suffers causing a shortage in supply. All these factors
make hindrance to demand and supply and make real estate an imperfect market for the
reasons as already stated.

INVESTMENT IN REAL ESTATE


In the investment market there are always a number of alternative opportunities for investment.
The investor who would prefer capital gain from the investment would risk investing his capital
in the equity shares of first-class companies whereas the investor who would prefer safe and
secure income from the investment would like to investment in long- term government
securities. Between the two extremes of highly risky equity share investment and safe and
secure government securities however there are a number of alternative opportunities with
different degrees of risk and security. Real property is one such vehicle of investment in the
totality of the investment market scenario. The role played by real property is dependent on the
various aspects of real property as a channel of investment. We discuss them as follows.
MANAGEMENT ASPECT
Real property investment is always associated with trouble and cost of management. Thus for
management of real property the services of a lawyer, an architect, a Valuer or a real estate
agent are necessary from time to time. Further, there may be involvement in litigations day to
day involvement in enforcing lease covenants, collection of rents etc, all parts of hazards of
management. Comparatively, investment in equity share is less troublesome and costly to
manage. Therefore, in order to induce an investor in the investment in real property there must
be an incentive of higher rate of return than the yield from equity share investment.

Again, the investor can invest in long-term government securities to obtain a risk-free long-
term income. Against this, there is the prospect of rental growth and appreciation of capital
value as the incentive for investment in real property in spite of the hazards of management.

Legal aspect

Law interferes with real estate investment in a number of ways. Thus, rent control acts may
severely affect the prospect of rental growth of the property. Further, taxation laws may affect
real property investment. Examples are property tax, capital gains tax, betterment levy etc.
Again, town planning acts may adversely affect real property investment through restrictions
imposed on land uses, density etc. As a result the investors should expect higher yield from real
property investment to compensate againstthis.

Liquidity

Real property is an illiquid asset. The time taken for real estate disposal is a lengthyprocess.
Further, for transfer of property one may have to incur cost on lawyers' fees, stamp duty etc.
Comparatively, the time taken for disposal of shares or cost of transfer of share are much less
than that of real property. Hence, in order to induce an investor to invest in real property one
must expect to receive a higheryield.

Size of investment

Real property is indivisible into very small components. Generally, a huge amount of capital is
associated with an investment in real property. Comparatively, a much smaller amount of
capital is required in order to invest in shares. To compensate against this the real property
investment should have higher yield.

Lack of central market

There is no central market for real property for the stock exchange market. Furthermore, the
transactions in real property are often of closed type and one is required to incur ahigh
amount of expenditure to have full knowledge of the transaction. These add to the market
imperfection of real property market and have the effect of increasing the yield from real
property investment.

It is clear from item number (1) to (5) above that cost and hazards of management and market
imperfections cause return from real estate to be higher than risk-free securities. The
benchmark of interest rate in the investment market is the yield from long-term government
securities (also called the gilt-edged securities or the risk-free securities). So, all investments
should have ideally produced higher return than the yield from government securities. But in
case of real estate, the prospect of capital appreciation and income appreciation act to set off
against the higher return required than that of the government securities. That is why, real
estate in first-class location tends to indicate lower yields than the risk-free government
securities. This phenomenon is known as the reverse yieldgap.

FACTORS INFLUENCING DEMAND IN REAL ESTATE


The price of any commodity in a market is fixed by the interaction of demand and supply. So
also in the property market, demand and supply interact to fix the prices. Any study of factors
affecting value or prices in the property market therefore amounts to the examination of the
factors affecting demand and supply of landed property.

In the demand side the following factors are relevant:

1. Population changes
2. Economic growth
3. Accessibility
4. Inflation
5. Change in taste and choice ofpeople

Population changes

As more and more people migrate in an area, there is an increase in demand for land of all
types such as residential purposes, commercial uses, industrial uses, open space and
recreational needs etc. But since total area of land is fixed by nature, the per capita allocation of
space for any use reduces. Supply of land being low and demand remaining high the value of
land sharply rises.

Economic Growth

As an economy grows there is more investment in infrastructure, industries, commercial sectors


and services. The living standard of people rises and per capita income grows. As a result there
is more demand for residential, educational, shopping and other commercial
facilities. So the overall demand of land increases. Existing land resources are developed and
redeveloped and value of land rises.

Accessibility

As a nation undertakes great road-building programs, opening up of new railway


communications, establishing aerodromes etc, there is an overall change of accessibility. The
remote parts of a country become connected with urban centers through improved
accessibility. As a result demand for land for all uses in the remote districts increases. Again re-
planning and re-development of existing road networks within an urban area improves
accessibility and increases demand for land for various uses.

Inflation

During inflationary times investment in long-term government securities or other fixed interest
securities is not secured in real terms as the real value of money associated with such
investments falls with time causing a loss of capital. But value of landed property continues to
grow with time at a faster rate than the rate of inflation. As a result, the real value of money in
landed property investment is maintained in real terms. Hence, during inflationary times there is
an increase in demand for investment in landed property from a section of investors to
maintain the real value of money. Landed property therefore is said to offer a hedge against
inflation.

Change in taste and choice of people

With increased standards of living, the large joint families came to be fragmented as people
chose to live in small family units. As a result, demand for small flats rose phenomenally.

There was large scale demand for flats in high-rises that gave rise to intensive development or
redevelopment of urban sites.

FACTORS INFLUENCING SUPPLY SCHEDULE IN REAL ESTATE


1. Property development is a time-consuming process. As a result, supply of landed property
cannot be increased suddenly. In the short run, therefore, when demand of land in an
area increases, say by population growth, supply of property cannot be increased. As a
result, the demand being high and supply remaining unchanged, price rises sharply.

2. When the demand for landed property increases in an area in the long run, re-
development of under-developed sites and development of virgin sites take place, thus
contributing to the fresh supply of goods in the property market. But since total land in an
area is fixed by nature, there is a limit up to which supply can be augmented. So, in the
long run also, demand continues to overtake supply and prices
of landed property continue to rise.

3. Landed property being a durable good, its supply cannot be reduced once constructed if
the demand suddenly falls. As a result, property prices fall and in order to avoid further
capital loss, the suppliers may be compelled to sell property at a reduced price.

4. Flow of funds to the property market from the capital investment market is largely
responsible for property development. If therefore the flow of fund in the property market
is restrained due to imperfections in the capital investment market, the supply of property
will fall and prices will rise due to a growing demand.

5. Enactments may be a hindrance to the supply of property in the market. Thus, a high
stamp-duty may restrain the investor from investment in property for development if
price incentive is not high enough to cover the cost. Again, rent restriction acts may stand
in the way of investment in property sector for letting causing a shortage of supply of
rental accommodation.

6. The decision of planning authority may control the supply of land in the land market. The
planning authorities may allocate land for some use by use zoning through the outline
development plan of an urban area. Again, they may control the intensity of development
of land by density zoning. If the authority restricts the use of land by use zoning, the
supply of such land use is reduced in the area and prices may rise due to a growth of
demand.

11. CONCEPTS OF GREENBUILDING

INTRODUCTION

Green building is one of the fastest-growing movements in the housing industry today. More
than just a passing trend, the green building movement is a response to the pressing
environmental problems prevalent in our society and provides tangible environmental,
community, and economic benefits, improving health and public spaces and lowering building
and energy costs.

Buildings have major environmental impacts during their life. Resources such as ground cover,
forests, water, and energy are dwindling to give way to buildings. Resource-intensive materials
provide structure to a building and landscaping adds beauty to it, in turn using up water and
pesticides to maintain it. Energy-consuming systems for lighting, air conditioning, and water
heating provide comfort to its occupants. Hi-tech controls add intelligence to ‗inanimate‘
buildings so that they can respond to varying conditions, and intelligently monitor and control
resource use, security, and usage of fire fighting systems and other such systems in the
building. Water, another vital resource for the occupants, gets consumed continuously during
building construction and operation. Several building processes and occupant functions
generate large amounts of waste, which can be recycled for use or can be reused directly.
Buildings are thus one of the major pollutants that affect urban air quality and contribute to
climate change. Hence, the need to design a green building is to address all these issues in an
integrated and scientific manner. It is a known fact that it costs more to design and construct a
green building compared to other buildings. However, it is also a proven fact that it costs less
to maintain a green building that has tremendous environmental benefits and provides a better
place for the occupants to live and work in. Thus, the challenge of a green building is to achieve
all its benefits at an affordable cost.

A green building depletes the natural resources to a minimum during its construction and
operation. The aim of a green building design is to minimize the demand on non- renewable
resources, maximize the utilization efficiency of these resources when in use, and maximize the
reuse, recycling, and utilization of renewable resources. It maximizes the use of efficient
building materials and construction practices; optimizes the use of on-site sources and sinks by
bioclimatic architectural practices; uses minimum energy to power itself; uses efficient
equipment to meet its lighting, air conditioning, and other needs; maximizes the use of
renewable sources of energy; uses efficient waste and water management practices; and
provides comfortable and hygienic indoor working conditions. It is evolved through a design
process that requires input from all concerned – the architect; landscape designer; and the air
conditioning, electrical, plumbing, and energy consultants – to work as a team to address all
aspects of building and system planning, designing, construction, and operation. They critically
evaluate the impacts of each design decision and arrive at viable design solutions to minimize
the negative impacts and enhance the positive impacts on the environment.

Green building encompasses a wide range of renewable construction materials, energy and
resource-efficient building techniques, and an overriding philosophy of sustainable
development. The most significant green building practices, commonly referred to as the
―six elements of green building,‖ relate to site, water, energy efficiency, indoor air quality,
materials, and operations and maintenance.

CHECK LIST OF GREEN BUILDING EVALUATION

The following are the check list of green building evaluation:

Site planning For sustainability of land, two ideas behind green site planning
and development are: (i) to protect and restore habitat and (ii)
to maximize open space, providing social and environmental
benefits.
Water efficiency Water efficiency (e.g., water use reduction, landscaping) considers
the potable water as well as storm water and waste water
management.

Energy and By energy and atmosphere, we mean renewable energy sources as


atmosphere well as ozone depletion. Green building considers the conservation
of energy in the building‘s design by closing the building envelope,
integrating energy-efficient mechanicals, electricals and fixtures,
landscaping to assist in shade or solar access, or using renewable
energy sources such as solar, wind, or geothermal alternatives.

Indoor air quality Indoor environmental quality means air quality, emissions and
passive heating. Green building design focuses on mitigating the
negative effects of combustion-based appliances and moisture.
Green buildings use low or no VOC paint to provide better indoor
air quality.

Material and Materials and resources mean reuse, recycling of materials as well as
resources use of renewable materials. The materials used have a significant
effect on indoor air quality. Green buildings use materials that are
less toxic than their conventional counterparts. Additional
considerations include what the material is made of, how it is
manufactured, and where the material originates. Green buildings
use floor coverings that have low volatile organic compound (VOC)
concentrations and are durable and recyclable.

Operation and Innovation and sustainable design includes measures to control


maintenance water and energy consumption along with the use of durable
materials and designs that are meant to lower maintenance costs
while lengthening the lives of building components.

WHOLE BUILDING APPROACH

Green building has popularized a new style of design called the ― whole building approach.
The whole building approach is elementary to green building. This approach views all of a
building‘s parts as an integrated system. The goal of the whole building approach is to ensure
that the different parts of a building work with, rather than against, one another. Whole-
systems thinking include:

1. The use of native landscaping


i. to reduce required maintenance (mowing and irrigation)
ii. to preserve and enhance groundwater and wildlife habitat

2. Day lighting
i. to reduce the need for artificial lighting and energy consumption
ii. to improve worker comfort and productivity

3. Convenient access to public transportation


i. to reduce amount of space allocated for parking
ii. to foster reductions in vehicular miles traveled (VMTs) and carbon usage

GREEN BUILDING VALUATION


Revenue, occupancy, operating expenses, and risk are the four determinants of higher value of
a green building.

1. Revenue
In today‘s real estate market, especially office property market, rental premiums are emerging
in green buildings as many of today‘s best tenants are increasingly willing to pay a premium for
green spaces. For these tenants, leasing green space is an opportunity to demonstrate a
commitment to sustainability, attract the best employees, and improve productivity. For owners
not pursuing certifications, investments in green features in conventional buildings can also
establish position in the marketplace and keep rents up. In those circumstances where you have
an underperforming property, investments in green features may result in better positioning of
the asset within the spectrum of other properties in the marketplace. A skilled valuation
professional can assist in understanding whether investments are truly a green ―premium‖ or
necessary retrofit to keep up with the market.

2. Occupancy

Occupancy premiums can lead the case for green investments. If it can be determined that the
green features will result in higher occupancy than an otherwise similar building, it can be an
acceptable feature for increases in value.

3. Operative expenses

The most effective determinant for higher value of green buildings is their very low utility bills.
In premier markets with high rents, energy savings may appear relatively small as a percent of
operating income. By contrast, in markets with lower rents, energy costs represent a much
larger percentage of income; improving cash flow. Operational savings on maintenance and
reserves also add value
4. Risk

Green building value also shows up in the risk-mitigating protections these assets offer to
owners. In the valuation and underwriting process, high-performance buildings can offer
hedges against changing consumer preferences as well as new laws and increasing energy
prices. Recognizing the relative safety of green assets, major insurers may offer discounts for
green buildings.

GREEN BUILDING RATING SYSTEM


A green building rating system is an evaluation tool that measures environmental performance
of a building through its life cycle. It usually comprises of a set of criteria covering various
parameters related to design, construction and operation of a green building. Each criterion has
pre-assigned points and sets performance benchmarks and goals that are largely quantifiable.
A project is awarded points once it fulfils the rating criteria. The points are added up and the
final rating of a project is decided. Rating systems call for independent third party evaluation of
a project and different processes are put in place to ensure a fair evaluation. Globally, green
building rating systems are largely voluntary in nature and have been instrumental in raising
awareness and popularizing green building designs. Building Research Establishment‘s
Environmental Assessment Method (BREEAM) of United Kingdom, Comprehensive Assessment
System for Building Environmental Efficiency (CASBEE) of Japan, Leadership in Energy and
Environmental Design (LEED) of United States and Green Rating for Integrated Habitat
Assessment (GRIHA) of India are the internationally acclaimed green building rating systems.
INCOME APPROACH TO VALUE
1. RELATION BETWEEN INCOME &VALUE

Consider a person seeks an income of ` 1,000 per annum from an investment. He wishes
this to be an 8% return on his investment. What is the amount he has to invest? Obviously,
(100/8) x 1000= ` 12,500 is the amount from the investment for an annual income flow of `
1,000.

From the above example, we may conclude, capital value= (100/Yield) x net income per
annum or simply value = (100/Yield) x net income per annum.

If instead of considering return on ` 100 we consider return on ` 1 then it can be shown that
value = (1/Yield on `1) x net income per annum.

It is usual to express return on ` 100 by whereas return on `1 is expressed byi.e.

So, value = (100/i) x net income per annum

Or, value= (1/i) x net income per annum

The expression 100/i or 1/i is called ―years‘ purchase or simply Y.P. for a perpetual income
flow. So, value= Y.P. x net income

The model of valuation, value = Y. P. x net income accepts that the income from the
property is a perpetual flow. Correctness of this assumption may be called into question
because a property, particularly the building does not last in perpetuity. The building
gradually deteriorates and falls to zero value. Hence no income from it could be perpetual.
However, from merely practical point of view it is arguable that the income from the
building may fall at the remote end of its life but the value and hence the income from the
land gradually rises thus maintaining the perpetual income flow of the property.

Again, from the valuation tables it can be shown that V.P. or present value of `1 per annum
at a given rate percent say 6% for 30 years is 13.756, for 50 years is 15.762 while for 100
years is 16.618 and in perpetuity on 16.667.
So, instead of lasting in perpetuity, if the building lasted for 30 years only, it accounted for
82.6% of its total value. If the building lasted for 50 years only, it accounted for about 94.6%
of its value.

Therefore, instead of lasting in perpetuity, if the building lasted for 50 years only, it
could account for about 95% of its value. So, only the remaining 5% would come from the
remaining part of the building's life considering if it lasted in perpetuity.

The model of valuation V= (100/Yield) x net income has two variables.

(a) The income.

(b) The yield.


2. VALUATION OF PROPERTY AFFECTEDBY:

A. THE RENT CONTROL ACT

1. EXISTING RENT CONTROL ACTS

Generally we do valuation by the rental method taking a rather simplistic view that this
could be applied to all rented properties where either the rent could not be enhanced or
the tenant could not be evicted. However, in reality we have situations which are far
removed from this and in certain cases there is a possibility of ejecting the tenant legally
and in some cases there is a possibility of increasing rents. We have to examine provisions
of the rent act of the particular state in order to determine value of theproperty.

Some of the states have amended rent acts under which frozen rents can be increased.
Protection against eject rent to certain group of tenants is withdrawn in some amendments.
Maharashtra Rent Control Act, 1999, proposed Delhi Rent Control Act of 1995, West Bengal
Premises Tenancy Act, 1997 and Gujarat State (2nd Amendment) Rent Control Act, 2001 are
few examples.

All these facts put together have had and will continue to have a profound effect on
valuation of properties.

2. ORIGIN AND FEATURES OF THE RENT CONTROLACTS

The Rent Control Act was started sometimes during the Second World War as
accommodation was required by the government during the war years Also due to the
partition of India in 1947, there was an influx of population in urban areas of India. This
created housing shortage in large cities and small towns. Subsequently, the rural
population in search of better employment opportunity had a tendency to migrate to the
urban areas. This put pressure on urban properties and the government realizing the
shortage of accommodation in these areas decided to protect the tenant from paying an
exorbitant rent and being evicted by enacting Rent Control Acts. Ordinarily the relationship
between the landlord and the tenant is governed by a lease agreement which takes into
account market conditions. However, the rent act was introduced to protect the needs of
the tenant rather than that of the landlord. The basic feature of the rent control act in most
of the states is the same although each state has its own specific provisions. Naturally, there
would be slight difference in the rent control act of the various states. However, two
features of the rent act are common to all the states which have had great bearing on
valuation. The more important of these two common features relates to the fact that rents
have been frozen at old levels which were fixed as per the original cost of land and
buildings. The second feature is reflected in the provision that a tenant can only be evicted
under very special conditions. The restrictive nature of the rent control act of various states
have had major effect on income to local bodies as collections of municipal taxes are
normally related to rents. Also the fair market value of the property, for whatever purpose it
is made, is dependent on the rent received as well as the provisions of the rent control act
prevailing in the area. This Rent Control Act being a special legislation, its provisions,
supersedes provisions of Transfer of Property Act under which landlord and tenants rights
and obligations are specified

3. EFFECT OF RENT CONTROL ACTS ON HOUSING STOCK

It is not necessary to go into the details of the various rent acts of the different states here.
However, one must realize that due to the Rent Control Act, the relation between a landlord
and tenant has been completely altered. This has had a profound effect on the value of
properties, vis-a-vis the yield factor. It is the tenant in these cases who enjoys security of
tenure at a low rent into perpetuity while the landlord, for all practical purposes, has to be
content with the fact that he may never get possession of his property.

As a result of these developments over several years, various properties have been removed
from the rental market and accommodation has become scarce. In fact, there is a
contradictory phenomenon that many flats or other properties in the larger cities remain
locked and unused whereas accommodation in these very areas is not available. In short,
there has been a loss of our valuable national assets, i.e., the building or housing stock.

The Rent Control Acts have caused housing shortage. Land owners stopped building
houses for letting. Many buildings have deteriorated for want of proper repairs and
maintenance. There is pressure on government and other agencies for public housing.

Sitting tenants are unwilling to move out from their existing tenancies even when these are
no longer suitable. In short, the tenant gets a lower quality product for lower price. House-
owners tend to be more selective in choice of new tenants whenever newly created
accommodation becomes available. Also, a system of ‗salami‘ or ‗pugri‘ generating
unaccounted money was started for transfer of tenancies. This has damaged our economy
and perhaps our moral and ethical standards as well.

In fact, in the long run, the very people whom the rent control was meant to help have been
hurt the most as eventually, these statutory tenants someday have to buy freeholds.

4. ILL EFFECTS OF THE OLD RENTACTS

The Central Government as well as the various State Governments have now, after almost
sixty years, realized that the Rent Control Acts have in fact had an adverse effect. It is the
case like in Vietnam, where once their leader Ho Chi-Minh said, ―Damage caused to our
nation‘s housing stock by our own Rent Control Legislation was many times more than the
damage caused by America‘s military actions in Vietnam.

We shall initially discuss some of the ill-effects of the old rent acts before studying the
provisions of the amendments in Rent Control Act.

First, it caused irreparable loss to our limited housing stock. Due to the meager income
from housing properties, landlords stopped maintaining or repairing buildings. This resulted
in severe deterioration and even collapse of several buildings. Loss of human lives as well as
loss of valuable assets was the ultimate result.

Secondly, a system of salami or pagdi or premium came into existence to compensate for
pegged rents. This practice resulted in the generation and exchange of black (unaccounted)
money, giving rise to the evil of a parallel economy. The system also damaged our ethical
and moral standards. Even honest persons were not ashamed to give or take large sums of
unaccounted money at the time of creation of new tenancies and also at time of transfer of
tenancies. This became to be accepted as a necessary evil for the survival in urban areas of
our country.

Another important effect was that value of all such tenanted properties fell in the market
substantially and it remained at artificially low levels over the period of almost 60 years.

Persons desiring to invest funds in sound securities with a fair expected return, stopped
constructing houses for rental income. Rented immovable property was not an option
considered a sound investment. In fact it was treated as a liability or a diminishing asset.

Even after the expiry of physical life of the building, it could not be demolished. If the
building collapsed on its own, even then tenancy rights of tenants did not extinguish. Vide
Tribhovandas vs. Chimanlal, Gujarat High Court GLR/1971/556. A landlord proposing a new
building on such cleared land was bound by law to give premises to tenants of collapsed
building on rental basis. Similarly, tenants of structures washed away due to flood or
collapsed under earthquake had first right as tenants in a new building built on the
particular plot. Vide Krishna Laxman vs. Narsingh Rao, Bombay High Court BLR/1973/29.
From the valuation point of view, those two provisions totally extinguished value of land to
owner in all rent act affected areas provided the property was fully developed. As there was
no scope of reversion of land back to the landlord, there was no reversionary value of land.
Land value was artificially brought to zero due to tenancy and the Rent Act. This fact was
duly endorsed by Calcutta High Court in two judgements. Vide C.I.T. vs. Ashima Sinha, 116
ITR 26 and C.I.T. vs. Anup Kumar Kapoor & O`, 125 ITR 684 of 1980. What remained was
only frozen rent income from the property and hence under the Rental Method, only the
structure is valued on the basis of rental method. It is needless to say that the land value is
taken at NIL in such cases.

There is therefore, a widespread misconception about rental method and reversionary value
of land. Several valuers believe that under the rental method, reversionary value of land can
never be taken. However, the fact is that the reversionary value of land is taken as nil
because there is practically no scope for the landlord to get the land back inreversion.
This misconception is perhaps due to the views expressed by the Calcutta High Court in the
case C.I.T. vs. Anup Kumar Kapoor & O` (125 ITR 684 of 1980). In this case, the buildings
which were more than 100 years old were let out to tenants. The court very rightly rejected
addition of reversionary value of land because the tenants were protected against
ejectment under the rent control act. However, the reasons given by the court for rejection
were not appropriate. It was stated ― Mr.Palwas unable to place before us any authority in
support of the reversionary method of valuation. We are not able to appreciate and
understand either the principle or logic behind this reversionary method applied by the
valuation officer. He has taken value of land twice. Once in arriving at figure by yield
method and again in applying reversionary method. This in our view was wholly wrong. It is
therefore, necessary that the rental approach be understood in its proper perspective.
Merely because the counsel failed to show the court in a particular case, the logic of the
reversionary value of land does not mean that such a method or approach is non-existent.
Before rejecting the approach of reversionary value of land for all purposes, it is necessary
to examine situations under different Acts.

Under the Transfer of Property Act, rent is not frozen for ever. It is governed by a tenancy
contract between landlord and tenant. If it provides for review after three years, then rent is
frozen for three years. It is increased or decreased as per any new agreement. There is no
life time tenancy under the Transfer of Property Act. If agreed rent is not paid, the tenant
can be ejected. If the building has outlived its physical life, the tenant could be ejected for
reconstruction of the building. There is no liability to the owner to re-house the old tenants
in a new building. Even otherwise where there is no rent control act, alternative premises
are readily available at market rent and no one would like to risk his life by staying in an old
dilapidated building.

We are not able to visualize these circumstances because provisions and effect of Rent Act
has brainwashed our mind and thinking process. If we see and visualize the situations
prevalent in foreign countries, it becomes very easy to understand this aspect. There,
tenants are easily ejected. Sometimes tenants themselves move out for better
accommodation with more amenities and lower rents elsewhere. No one would take up
premises in very old building even at low rents. Land with vacant old building will thus
automatically vest back to the landlord. It is therefore, proper and fair to assume
reversionary value of land under such circumstances.

On the other hand provisions of Rent Act prohibits ejectment and rent increase. It is a case
of life time tenancies and even thereafter. Land never reverts back free to the landlord. It is
therefore, proper to take reversionary value of land at NIL value in Rent Act affected
premises.

To understand this important concept with more clarity and also to study impact on
valuation, let us study the following two cases.

The same property, let us call it Property A, will in one case be valued as if it is protected by
provisions of the Rent Act. Again the same property will be valued assuming that the rent
act is not applicable.

Problem

A landlord owns a building which is fully rented to 12 tenants. Plot area is 600 sq.m. Land
is fully utilized in 3 floors of the building. Rent is ` 1800 per month. Property taxes are `
5400 per year. N.A. tax ` 300 per year. Insurance ` 600 per year. Building is 60 years old
having a future expectation of 50 years Value the property if–

(i) the tenants are protected by provisions of the RentAct.

(ii) Rent Act is not applicable.

Solution
Study of the above data clearly indicates that even though building is 60 years old, net
yield will have to be capitalized in perpetuity. We have to assume that tenants will not
vacate the premises for 100 more years and that they may keep the building duly
repaired for the said period. We have also to assume that land will not revert back to the
landlord for 100 years

Value of above Property A‘ would workout as under:

Gross Annual Receivable Yield : ` 1800×12 = ` 21,600

Less: Outgoings:

Propertytaxes ` 5400

N.A.Tax ` 300

Insurance `600

Repairs 10%GrossRent ` 2160

Collection andmanagement4% `864 = `9,324

---------------------------------------------------------------------

Net Receivable Yield = ` 12,276

Capitalize yield in perpetuity at 10.5% = ` 12,276 x (100/10.5)

= ` 1,16,914

(i) It will be seen that Rent Act is not applicable in this case and hence provisions of
Transfer of Property Act will be applicable. Ejectment will be possible and even increase in
contractual rent will be possible. We have therefore, to assume that land will revert back to
the landlord.

Net receivable yield as before = ` 12,276

This yield need not be capitalized in perpetuity but only for the years of future life of the
building. It is the expected period for future flow of income from the property. Capitalizing
net yield @ 10.5% and allowing for redemption of capital @ 4.5% for 50 years,
1
YP(dual rate) =
r
R+
(1 + r) n− 1

1
=
.105 + .045
(1 + .045)50− 1

= 9.0414

Structural Value = 9.0414 ×12,276

= 1,10,992
In addition to the structure value, we have also to provide for reversionary value of land.
Let us assume a land rate of ` 1,000 persq.m.

Total value ofopenland = 600 × 1,000 = `6,00,000

As this land will revert back to the land owner only after 50 years, we defer the value @ 9%
interest for 50 years

1 1
PV = =
(1 +R) n (1 + .09)50
Reversionary valueofland = 0.01345 × 6,00,000

= `8,070

Thus, total value of property where rent act is not applicable.

= Structural value on yield basis + Reversionary value ofland

= ` 1,10,992 +8,070

= `1,19,062

Ans: ` 1,19,062

It will be seen from the above that the impact of the Rent Act on valuation of property is
considerable. From an economic point of view, the Rent Act converts property asset into a
liability. It ignores physical life of the building. Even very old buildings are not permitted to
be demolished and the burden of repairs involving prohibitive costs is on the landlord.
Thus the Rent Act makes rented property a diminishing asset.

A point of caution is necessary while adopting the Rental Method for properties not
affected by the Rent Act. The valuer will have to be very careful in adopting the rate of
capitalization for net annual yield and also rate of reversion for assessing differed value of
land. The valuer will have to be up-to-date in knowledge of expected yield in market
analysis in different forms of investment. Little variation or error in rate or capitalization is
likely to give incorrect results.

To further elucidate the impact of Rent Control Legislation on valuation of property, we


shall consider further the following two salient features.

(i) Rent Act prohibits ejection of tenants from the premises. The provision has resulted
in unforeseen benefits to tenants of under developed properties.

(ii) Another provision is freezing of rent for ever. The provision of freezing of rent in
Rent Act has benefited tenants tremendously. Due to inflation, prices of all commodities
have increased including salary income and business income. However, rent remained the
same resulting in gain to tenants at the cost of landlords. This provision has also had a
severe impact on valuation of property.
Due to amendments in FERA and change in government policy to permit freely ownership
of immovable properties to foreign nationals, the foreign investors are very much interested
to invest funds in residential as well as commercial properties. They are more than satisfied
with 10% to 15% return on their investments in properties because in their respective
countries such returns are as low as 3% to 5%. Again prices of properties do not appreciate
that fast in their countries as in India. However, they hesitate to invest their funds because
of age old Rent Acts and slow process of judiciary to deliver justice.
5. AN ANALYSIS AND CRITIQUE OF THE NEW PROVISIONS OF
RENT ACTS ONVALUATION

The new provisions will drastically change the relationship between the landlord and the
tenant. One of the main effects will be that in many cases the provisions of the Rent Control
Act will not apply, but provisions of Transfer of Property Act, 1882, will be applicable.

(i) One of the major impact of the provisions would be that landlord will be permitted
to increase contractual rent after three to five years period on the basis of market rents
prevalent at that time. Since the rents will not be expected to continue for a long time, or
the tenancy will not be of a perpetual nature, the future rental income will have to be
estimated with greater accuracy in most cases. For this, lease agreements and also the
expected market conditions will have to be studied in greater detail. Rack rents will have to
be kept a tab on. In fact the concept of a property able to command a rent for years to
come would definitely play an important role. For this, perhaps the future life of the
building will also have to be evaluated on a scientific basis. Where rent has to be increased
in stages, other techniques of calculating future incomes will have to be pursued.

(ii) Another advantage for the landlord would be that the landlord will be able to eject
all tenants in the building when physical life of the building is over and it is uneconomic to
retain the structure for any longer period. The value of such properties will involve two
components viz.:

(a) Capitalized value of net yield on the basis of future flow of rental income i.e. based
on future life of the building.

(b) Reversionary value of land also based on future life of the building and present rate
of land prevalent in the locality.

In case where valuation is done by the rental method with reversionary value of land, it
would be necessary to cross check the valuation by means of adopting land and building
technique.

In the recent past, the courts have rejected the concept of adding reversionary value of land
in case of tenanted properties on the assumption that tenancy is to continue for a long
period of time into the future (refer to Sec. 24.12 of this book). If the rent acts are changed,
and rents not expected to continue into perpetuity but only for a restricted period of time,
the concept of reversionary value will also come into play. Capitalization rate will also have
to be investigated properly as per market conditions for similar properties or from other
investment properties and compared with investments in other securities, gilt-edged or
others as the case may be.

(iii) Another welcome provision in the new Rent Acts is the provision to increase rents of
old existing structures which were frozen for the last four to five decades. These provisions
will result in increased rental every year in future. This will pose some problem in computing
future yield receivable from the property and capitalized value of such income. It will be
necessary to capitalize the additional flow of future income, now permitted in the Rent Act,
in addition to the capitalized value of old rent in order to arrive at the correct value of the
property under the rental method.

B. LICENSED PROPERTY UNDER THE EASEMENT ACT, 1882

1. WHAT IS LICENSE

Where one person grants to another, or to a definite number of other persons, a right to
do, or continue to do, in or upon the immovable property of the grantor, something which
would, in the absence of such right, be unlawful, and such right does not amount to an
easement or an interest in the property, the right is called a license.

2. WHO MAY GRANT LICENSE

A license may be granted by anyone in the circumstances and to the extent in and to which
he may transfer his interests in the property affected by the license.

3. LICENSE WHEN TRANSFERABLE

Unless a different intention is expressed or necessarily implied, a license to attend a place of


public entertainment may be transferred by the licensee; but, save as aforesaid, a license
cannot be transferred by the licensee or exercised by his servants or agents.

4. GRANTOR'S TRANSFEREE NOT BOUND BYLICENSE

When the grantor of the license transfers the property affected thereby, the transferee is
not as such bound by the license.

5. LICENSEE'S RIGHTS ONREVOCATION

Where a license is revoked, the licensee is entitled to a reasonable time to leave the
property affected thereby and to remove any goods which he has been allowed to place on
such property.

6. LICENSEE'S RIGHTS ONEVICTION

Where a license has been granted for a consideration, and the licensee, without any fault of
his own, is evicted by the grantor before he has fully enjoyed, under the license, the right
for which he contracted, he is entitled to recover compensation from the grantor.

7. VALUATION OF LICENSEE’SRIGHT

Licensee does not have any valuable right. Therefore, valuation of licensee‘s interest is not
possible. The valuation of Licensor‘s right shall be done following the same methodology as
for valuation of freeholder‘s right of a let out or lease out property.
C. LEASEHOLD PROPERTIES UNDER THE TRANSFER OF PROPERTY ACT,1882

Leasehold interests are not so lucrative investments as the freeholds. Generally, the lessee is
restricted in his way of dealing with the property by restrictive covenants in a lease deed in
a number of ways. Some examples of such restrictions are not to sublet, not to assign, not
to redevelop, not to make any change of use etc. Further, in case the property is sublet and
if the subtenant fails to carry out any lease obligation the lessee remains liable for it to the
freeholder. Also, it is very difficult to dispose of the fag end of leases. Again, due to inflation
the provision for sinking fund made under a leasehold interest may fall short of the target
amount required for redemption of capital. For all these reasons, leasehold interests are
considered less attractive than the freehold interests and the investment return of leasehold
may be taken at 1% higher than its freehold counterpart.

The leasehold interest is of limited duration and is not perpetual like the freehold interest.
As has already been discussed, provision for a sinking fund is made for valuation of the
leasehold so that the capital value of the lease can be redeemed at the end of the lease
period. In this way, the leasehold income may be perpetuated and the investment return
can be compared with that of the freeholds. Lease hold interest is there for evalued at dual
rate of interest. One is a return of capital (called the remunerative rate of interest). Another
is a rate of capitalisation of the sinking fund (called accumulative rate of interest). The latter
is much lower than the former rate varying between say about 1% to 4% only.

The difference between the rent at which the leaseholder sublets the property and the rent
which he pays to the freeholder is called the profit rent. The value of a leasehold is the
capitalised amount of the net profit rent. The value of the leasehold is positive if the net
profit rent is positive.

Problem

Mr. Adukia has granted a lease of a shop premises to Mr. Bhalla at a rent of `150,000 per
annum all burden of outgoings being shouldered by Mr. Bhalla. The lease has today 5 years
to run. The current full rental value of the property is `300,000 per annum the tenant
undertaking all burden of outgoings. Considering the freehold yield at 4% on shop property
in this location what is the value of the lessee's (Mr. Bhalla's) interest?

Solution

Valuation of lessee's interest

Full rental value = `300,000 per annum (net)

Less, rent paid by the lessee= `150,000 per annum

Profit rent = `150,000 per annum(net)

Y.P. 5 years at 5% and2½%= 4.162


Value = `624,300

Therefore, valuation of lessee's interest = `624,300

NB: The leasehold yield has been taken at 1% higher than the freehold yield.

3. DERIVATION OF YIELD RATE FROM MARKET DERIVEDDATA


Prof. Andrew Baum and Prof. Dr. Neil Cosby in their book titled property Investment
Appraisal (1995) have shown that when riskless securities are producing an yield of 10.64%
a very optimistic approach to construct the yield securities as follows:
Yield from risk less securities = 10.64%
Additions (+):
Expected hazards from management requiring additional yield for investment in prime
shops

1. Risk = + 1.5%

2. Operating expenses = +0.5%

3. Poor liquidity = +2.5%


Total addition in yield required = + 4.5%
Deductions(-)
Yield foregone for incentives available for investment in prime shops
1. Income and capital growth = - 11%

2. Psychic income = - 0.14%


Total yield foregone = -11.14%
Therefore, resultant yield from prime shop property investment
= 10.64% + 4.5% + 11.14%
= 4%
But, it is very much difficult to construct the yield in practice for valuation of an investment
property in this way as data is not available in property market. Thus, a change in approach
is required to analyse the yield with the help of the known formula

Yield % = (Net Income / Capital Value) x 100

Therefore, when the net full rental value in the open market is Rs. 2,88,000 per annum and
market price in transaction of a prime vanilla shop is Rs. 48,00,000 the yield works out to
[(2,88000 / 48,00,000) x 100] i.e. 6%.

4. REMUNERATIVE RATE OF INTEREST AND ACCUMULATIVE


RATE OF INTEREST

REMUNERATIVE RATE OF INTEREST


The rate of interest adopted for return on capital investment is called 'Remunerative rate of
interest'. Remunerative rate of interest (for perpetual income) is always higher than
accumulative rate of interest. Remunerative rate of interest is decided by comparing market
yield rates from similar other sound securities like government security available in the
market.

ACCUMULATIVE RATE OF INTEREST


The rate of interest adopted for working out Sinking Fund is known as 'Accumulative Rate
of Interest'. Accumulative rate of interest is decided by comparing it with lowest possible
maintainable interest rate. The rate of accumulation should be so selected that at that rate
of interest income is guaranteed and said income will not fall even in a very long period of
time for the investment (say 40 to 80 years time period). Hence this rate is generally low
and in comparison with interest rates offered by Banks on deposits.

5. TYPES OF RENT: OUTGOINGS, INCOME, YIELD,


YEARS’PURCHASE
TYPES OF RENT
Actual rent is a guide for valuation purpose. Hence, actual rents must be analyzed to
determine the services they include. Some rents are inclusive of all taxes, and some are
exclusive of all taxes. Moreover, rents vary according to different factors such as location,
type of premises and size. They may be of various types. Some of these are identified
below.

1. Market Rent

Market Rent is the estimated amount for which an interest in real property should be leased
on the valuation date between a willing lessor and a willing lessee on appropriate lease
terms in an arm‘s length transaction, after proper marketing and where the parties had each
acted knowledgeably, prudently and without compulsion. Market Rent may be used as a
basis of value when valuing a lease or an interest created by a lease.
The conceptual framework of IVS 2017 supporting the definition of Market Value shown
above can be applied to assist in the interpretation of Market Rent. In particular, the
estimated amount excludes a rent inflated or deflated by special terms, considerations or
concessions. The ―appropriate lease terms‖ are terms that would typically be agreed in the
market for the type of property on the valuation date between market participants. An
indication of Market Rent should only be provided in conjunction with an indication of the
principal lease terms that have been assumed.
2. Contract Rent

Contract Rent is the rent payable under the terms of an actual lease. It may be fixed for the
duration of the lease, or variable. The frequency and basis of calculating variations in the
rent will be set out in the lease and must be identified and understood in order to establish
the total benefits accruing to the lessor and the liability of the lessee.
3. Gross Rent

This refers to the total amount payable periodically (usually per month or per annum) by a
tenant to the landlord – including all outgoings on whatever account.

4. Virtual Rent

It is the virtual gross rental value to the lessor receivable from the lessee for the leased out
property. In some cases, lessor receives some fixed lump sum amount called premium from
the lessee in advance, in the beginning of the lease itself. This premium amount is nothing
but an advance rental for the property. Thus returns to the lessor are divided in two ormore
parts. Gross Rent actually received from lessee during the lease period together with
notional rental value that is receivable on a lump sum amount (premium) received from the
lessee, constitutes total rental or the virtual rent of the property
Sometimes improvements are done on the property by lessee; this would ultimately belong
to the lessor as per lease agreement. In such a case, notional rental value on such capital
improvement works also becomes a part of the virtual rent. Thus virtual rent is the sum of
the total of actual rent received from lessee during lease period, notional rent receivable on
premium amount and notional rent (annual equivalent) receivable on capital sum invested
by lessee in the property.

PROBLEM
A lessee holds certain premises on a 21 years lease granted five years ago at a rent of ` 400
per month. He paid a premium of ` 5,000 at the time of entry. Two years ago he had spent
` 4,000 towards improvement of the property. Calculate lessee‘s virtual or sitting rent.

SOLUTION
Rent paid = 400 × 12 = ` 4,800 per annum
Interest@8% = 0.08
Annual sinking fund to produce Re. 1 at the end of 21 years @ 4%
r
= = 0.04
= 0.03128
(1 + r) −1 (1+0.04)21−1
n
Equivalent 0.04
= 0.038993
rents (1+0.04)18−1
inceentry = 5,000 × (0.08 + 0.0312801) = ` 556.40 per annum Cost of improv
Interest on
capital@8% = 0.08
Annual sinking
fund to produce
Re. 1 at the end of
18 years @ 4%
r
= =
(1+ r) −1
n
Equivalent rent= 4,000 × (0.08 + 0.0389933) = ` 476 per annum
Virtual Rent = 4,800 + 556.40 + 476 = ` 5,832.40 per annum

5. Head Rent

Many times the main Lessee called Head Lessee sub-leases the property to another person
called sub-lessee. To distinguish between lease rent paid by sub-lessee to head lessee and
rent paid by head Lessee to freeholder, the term Head Rent is used for lease rent paid by
the Head Lessee to the freeholder.

6. Profit Rent

When Head Lessee subleases a property, he normally charges higher rent than 'head rent'.
This increased rental is called Improved Rent. The difference between head rent and
improved rent is called Profit Rent.

7. Rack Rent

It is the actual full rental value (Gross Rent) receivable from the property. It may be rent for
land or for both land and buildings.

8. Acknowledgement Rent

It is a token rent paid by lessee or tenant to exercise his rights under lease or tenancy
agreements.

9. Nominal Rent

This rent, too, is just a token rent but normally a little higher than the acknowledgement
rent.

10. Annual Rent

As per the Explanation 1 to Section 23(i) of the Income Tax Act, 1961, ‗Annual Rent‘ means:

―(a) In a case where the property is let throughout the previous year, the actual rent
received or receivable by the owner in respect of such year; and

(b) in any other case, the amount which bears the same proportion to the amount of the
actual rent received or receivable by the owner for the period for which the property is let,
as the period of 12 months bears to such period.
11. Standard Rent

This is the rent which can be legally charged by a landlord from a tenant. If the Rent
Control Act is applicable in the area then standard rent will be determinable under the
provisions of this Act. The assessing authority must arrive at its own estimate of standard
rent in accordance with the Act.

The standard rent can be determined either by using the method of comparison or on the
basis of the investment theory.
In the method of comparison, the rent is compared with other similar properties. This
method may or may not yield proper results depending on circumstances of the
comparable properties. Comparables, in fact, may be difficult to come by and unless a
detailed objective analysis is done, considering situation, amenities, area, nature of
construction, and the like, it would be difficult to arrive at the correctresults.

In determining standard rent by the investment theory or contractor‘s test the fundamental
concept is that the investor would like to get a reasonable return on his investment and
also recover the outgoings required to be made by him. The method is summarizedbelow:
1. Present value of the land isestimated.
2. Depreciated cost of construction is estimated taking account of depreciation,
obsolescence,etc.
3. Capital value of property is obtained by adding the above twoitems.
4. Rate of return, as provided in the appropriate Premises Tenancy Act (or other
Acts), on the value of land and cost of construction should be applied. This is
the net yearlyreturn.

Add to this return, all outgoings like repairs, insurance, collection charges, taxes, sinking
fund, etc. This is the gross rent that can be realized by the owner of the property.

12. Feasibility rent

The rent necessary to justify new construction is called feasibility. This concept helps the
analyst determine the timing of development, as well as the difference between the
required rent and mar rentket rent based on known costs and expected returns to the
investor. Feasibility rent may also be used to estimate the depreciation of an improved
property. The capitalized difference between feasibility rent and market rent represents
total depreciation of the existing improved property if market rent is less than feasibility
rent. It is also known as feasiblerent.
PROBLEM
The ideal improvement for a small industrial facility would cost `6,00,00,000 to construct,
based on an estimate of the sale prices of comparable sites, the cost of preparing the site,
and the estimated building cost of the 50,000 sq ft facility. Market research supports an
overall capitalization rate of 5.5%. If operating expense is `50 per sq ft and stabilized
vacancy and collection loss is 4%, calculate the feasibility rent per sq ft of this industrial
facility.

SOLUTION
Net operating income (60000000 x 5.5%) = `33,00,000
Operating expenses (50x50000) = `25,00,000
Effective GrossIncome = `58,00,000
Stabilize vacancy and collection loss @ 4%= `2,32,000
Potential GrossIncome = `60,32,000
Feasibility rent per sq ft (6032000 / 50000) = `120.64

13. Effectiverent

The rental rate net of financial concessions such as periods of free rent during the lease
term and above or below-market tenant improvements (TIs) is known as effectiverent.

14. Excess rent

The amount by which contract rent exceeds market rent at the time of the valuation;
created by a lease favorable to the landlord (lessor) and may reflect unusual management,
unknowledgeable or unusually motivated parties, a lease execution in an earlier, stronger
rental market, or an agreement of the parties is known as excess rent. Due to the higher risk
inherent in the receipt of excess rent, it may be calculated separately and capitalized or
discounted at a higher rate in the income capitalizationapproach.

15. Deficit rent

The amount by which market rent exceeds contract rent at the time of the valuation;
created by a lease favorable to the tenant, resulting in a positive leasehold, and may reflect
uninformed or unusually motivated parties, special relationships, inferior management, a
lease executed in a weaker rental market, or concessions agreed to by the parties is known
as deficitrent.

16. Percentage Rent

Percentage rent is rental income received in accordance with the terms of a percentage
clause in a lease. Percentage rent is typically derived from retail tenants and is based on a
certain percentage of their sales revenue. It is usually paid at the end of each year and may
be more difficult to collect than other forms of rent paid on a more frequent basis.
Depending on the tenant, percentage rent may involve more risk than other forms of rent
and may be capitalized or discounted separately and at a different rate.

17. Overage Rent

Overage rent is percentage rent paid over and above the guaranteed minimum rent or base
rent. The level of sales at which a percentage clause is activated is specified in a lease and
called a breakpoint. The natural breakpoint is the level of sales at which the percentage rent
exactly equals the base rent. Overage rent may be market rent, part market and part excess
rent, or excess rent only.

18. Base rent

The minimum rent stipulated in a lease is called base rent.

OUTGOINGS
In the rental method of valuation, outgoings are an important aspect which needs to be
considered as they comprise the expenses incurred to maintain the property in a state so as
to command the rent. All properties are subject to outgoings in various forms. Normally,
local taxes are to be paid as well as immediate and/or annual repairs have to be carried out.
If the property in question be insured then premium is to be paid. Maintenance of a lift or
park in a housing estate may be necessary. Large estates may employ the services of
managers who are to be paid on commission basis. The most common of these outgoings
are discussed below.

1. Municipal Taxes

Usually municipal taxes are assessed as a certain percentage of the rateable value which, in
turn, is obtained from the rent realizable from the property.
The actual rates paid on a property must not be accepted unless the valuer has checked the
assessment and ascertained that the annual value assessed by the local municipal authority
is fair. The taxes to be deducted should be actual taxes paid for the relevant period.

2. Other Taxes

Other taxes include those levied by local bodies such as ground rent or land revenue.
3. Repairs and Maintenance of the Premises
There are various methods of estimating the allowance to be made for repairs and
maintenance. Most owners spend as little as possible on repairs with the result that many
dilapidated buildings can be seen throughout every city in India.
Most municipal corporations, for the purpose of rating, consider a 10 per cent deduction
from gross rent to be a fair amount for repairs and maintenance. Where the liability of any
portion of this work is on the tenant, appropriate adjustment should be made. This
recommended percentage is, however, the maximum limit. If actual expenses are less,
these should beconsidered.
Apart from the recurring allowance for repairs, a property may be in a very poor state of
repairs requiring immediate capital expenditure for renovating the property. In such case a
capital amount should immediately be deducted from the market value of the property
arrived at.

4. Collection Charges and Management

Normally, an agent collects rents. Charges vary according to the class of property and the
number of tenants. If the rents are small, an agent will charge 5 per cent to 6 per cent for
collection. This includes investigation of petty complaints and supervising petty repairs. In
many cases, the owner collects the rent himself. A valuer, must include cost of collection in
the deductions to be made from the gross rent. Generally, a maximum of 6 per cent
collection charges have been acceptable. The references cited above for maintenance have
also allowed collection charges at 6 per cent of annual letting value.
Management The owner of the property is to undertake a lot of hazards and costs for
managing the property. Thus, he has to collect the rents and enforce the tenants' liabilities
on account of the various covenants of the tenancy agreement. He may also be involved
into litigations. For a property with a single tenant the cost of management may be as low
as 5% of the gross rent. Whereas for a property with multiple tenancies the management
cost may be as high as10%.

5. Insurance

Insurance of a property is done for keeping a provision against any accidental damage, say
loss by accidental fire. Generally, for new buildings the provision is 1% of the gross rent per
annum. However, for older buildings, whose reconstruction cost is higher in case of fire
damage, the insurance premium may be as high as 3%.
6. Service Charges
Service charges are levied when a block of flats is let out to a number of tenants. In such
cases some services for common benefit of the tenants is to be rendered by the landlord.
Cost of the services provided by the landlord is charged by the landlord from the tenants.
Thus costs of maintenance of lifts, lighting of common parts of the building, payment of
wages of caretakers etc. are incurred by the landlord and charged from tenants over and
above the outgoings.

7. Voids

Where there is a possibility of the property remaining unoccupied for a period an allowance
may be made on account of voids by deducting a reasonable percentage from gross rent.
However, alternatively a higher yield may be taken for capitalisation of the income expected
from the property.

8. Bad Debts

In a block of flats or a premise in a business area let to many tenants, there are bound to be
a considerable number of bad debts. An average of the past five years is the best guide. If
these figures are not available, then the average of the past three years should be taken.

INCOME
The income from a property is the rent which is paid by the tenant when the property has
actually been let out. If the property is vacant then its potential income would mean the
rent at which it can be let out in themarket.
Gross rent is the gross amount which is paid by the tenant to the landlord for occupying
the property. The landlord is to incur some expenditure towards maintenance of the
property and continuation of the income from the property. To name some of these
expenditures are repairs and maintenance, property tax, management expenses, insurance
etc. These expenditures are collectively called outgoings. When outgoings are deducted
from the gross rent, the result is called the net rent. Rental value of a property at any point
of time would mean the rent at which the vacant property could be let out in the open
market. A property has been let out at exclusive rent means under the tenancy agreement,
the tenant will bear the burden of all outgoings. Inclusive rent under a tenancy agreement
means the tenant will pay the gross rent only and the landlord will bear the burden of all
outgoings. Fully repairing and insuring lease means a lease where the tenant bear the
burden of alloutgoings.
YIELD
It is the return on an investment. It is usually expressed annually as a percentage based on
an investment‘s cost, its current market value or its face (par) value. Often used with a
qualifying word or phrase.

Yield from an investment has the following characteristics.


1) Greater the security of the capital, lower is the yieldacceptable.
2) Greater the security and regularity of the income, lower is the yieldacceptable.
3) Greater the liquidity of the investment, lower is the yieldacceptable.
4) Lower the cost of transfer, lower is the yieldacceptable.
The long-term government securities are also fixed interest securities with the promise of a
nominal rate of interest on the face value. The government securities are safest securities
for investment as the government will never fail to pay the interest or repay the capital on
maturity. They are called gilt-edged securities and are nearest to the ideal form of
investment. The yield from the government securities is called the benchmark of interest
rate because any other investment will be riskier than the government securities and will
require a higher amount of return to compensate for therisk.

Although all investment other than a government security is riskier than the government
security, yet investment in real estate in first class locations fetches a yield lower than the
yield of long-term government securities. This is because of ‗Reverse Yield Gap‖. This is
because the rent of real estate in a first-class location grows very fast with time and is not
fixed like long-term government securities. Further, real estate also appreciates in capital
value with the passage of time.

For properties occupied by statutory tenants protected by rent control acts the yields are
much higher than the lower rate as the rent cannot be revised to market rental value and
indicate a very low rate of growth according to provisions of the statute.

In practice however, yield from any type of property should be ascertained from an analysis
of sale of similar type of property in the property market.

There are different types of yield used in valuation of properties. Those are the following:

1) All Risks Yield(ARY)


The reciprocal of the Capitalisation Factor, usually expressed as a percentage is defined as
all risk yield. Where rent is the rack rent in a freehold, the ARY is also the initial yield. This is
the traditional yield in conventional valuation. In the case of rack rent, it is assumed that
rental income remains the same throughout the life of the asset and other factors such as
inflation, growth and risk areignored.
2) Yield to Maturity
The annual rate of return anticipated on a bond if it is held until the maturity date taking
into account the current market price, the par value, coupon interest rate and the time to
maturity.
3) Reversionary Yield
The anticipated yield from an Investment Property once the Reversionary Value is attained
is known as reversionaryyield.
4) Equivalent Yield
The equivalent yield is the internal rate of return on the term and reversion.
5) Equated Yield
The yield which includes the effect of both inflation and growth is called equated yield.

YEARSPURCHASE
Please see Section2.7.4

6. DETERMINATION OF MARKET RENT AND STANDARDRENT

MARKET RENT DETERMINATION


In some circumstances the Market Rent may have to be assessed based on terms of an
existing lease (e.g., for rental determination purposes where the lease terms are existing
and therefore not to be assumed as part of a notional lease). In calculating Market Rent, the
valuer must consider thefollowing:
(a) in regard to a Market Rent subject to a lease, the terms and conditions of that lease
are the appropriate lease terms unless those terms and conditions are illegal or
contrary to overarching legislation, and
(b) in regard to a Market Rent that is not subject to a lease, the assumed terms and
conditions are the terms of a notional lease that would typically be agreed in a market
for the type of property on the valuation date between marketparticipants.

STANDARD RENT DETERMINATION


The standard rent can be determined either by using the method of comparison or on the
basis of the investment theory.
In the method of comparison, the rent is compared with other similar properties. This
method may or may not yield proper results depending on circumstances of the
comparable properties. Comparables, in fact, may be difficult to come by and unless a
detailed objective analysis is done, considering situation, amenities, area, nature of
construction, and the like, it would be difficult to arrive at the correctresults.
In determining standard rent by the investment theory or contractor‘s test the fundamental
concept is that the investor would like to get a reasonable return on his investment and
also recover the outgoings required to be made by him. The method is summarizedbelow:
1. Present value of the land isestimated.
2. Depreciated cost of construction is estimated taking account ofdepreciation,
obsolescence,etc.
3. Capital value of property is obtained by adding the above twoitems.
4. Rate of return, as provided in the appropriate Premises Tenancy Act (or other Acts), on
the value of land and cost of construction should be applied. This is the net yearly
return.
5. Add to this return, all outgoings like repairs, insurance, collection charges, taxes,
sinking fund, etc. This is the gross rent that can be realized by the owner of the
property.
6. amount paid by the tenant for municipal taxes, etc. must be deducted from the gross
rent.

PROBLEM

A residential flat in a four storied building in Calcutta has a plinth area of 1200 sq.ft. The
consolidated property tax has been assessed at Rs. 800 per quarter for the flat. Similar flats
in size, age and general construction in the area are selling at Rs. 700 per sq.ft. Calculate
the reasonable monthly rent assuming the entire floor area ratio (FAR) for the building has
been utilized. Annual repairs and maintenance are to be done by the owner. The flat is not
insured.

SOLUTION

Value of the flat including depreciation = 1,200×700 = Rs. 8,40,000


Annual return expected from the flat @ 10% perannum= Rs. 84,000
Add property tax @ Rs. 800perquarter = Rs. 3,200
Add repair & maintenance charges @ 10% of expected annualreturn
= Rs. 8,400
Add Sinking Fund @ 3.5% on the value of the flat which would be recouped in 60 years =
0.035
ofRs.8,40,000 = Rs. 4,274
(1 + 0.035)60−1
-----------------------------
Expected annual return = Rs. 99,874
Reasonable monthly rent = Rs. 8,322.83

6.1 Lease: lessor and lessee: Types of Lease, Lease provisions andCovenants

6.1.1 Lease:
A lease of immovable property is a transfer of a right to enjoy such property,
made for a certain time, expressed or implied, or in perpetuity, in consideration of a price
paid or any other thing of value, to be rendered periodically or on specified occasions to
the transferor by the transferee, who accepts the transfer on suchterms.
6.1.2 Lessor:
In a lease agreement the transferor is called the lessor.
6.1.3 Lessee:
In a lease agreement the transferee is called the lessee.
6.1.4 Types oflease:

6.1.4.1 GroundLease
This lease is for land only in return for a payment of ground rent and
sometimes for a premium. Usually there is an obligation to erect and maintain building by
the lessee on the land and to deliver this to the lessor in good condition at the end of the
lease period. This lease is also called Building Lease.
6.1.4.2 OccupationalLease
The lease of premises consists of both land and building for occupation for a
certain number of years is called as occupational lease.
6.1.4.3 Sublease
This is a lease made by the leaseholder to another person (the sub-lessee) for a
part of the premises or full, generally at a profit rent (which is a rent in excess of the rent
paid to a freeholder by the leaseholder on that part).
6.1.4.4 Lease forlife
This lease is granted for the duration of the life of the lessee. The lease
terminates upon the death of lessee.
6.1.4.5 Flat RentalLease
A flat rental lease specifies a level of rent that continues throughout the duration of the
lease. In a stable market, this type of lease may be typical and acceptable. Flat rental leases
may also be prevalent in net rent situations where changes in expenses are the
responsibility of the tenant (ortenants).
6.1.4.6 Indexlease
A lease, usually for a long term, that provides for periodic rent adjustments based on the
change in an economic index is known as index lease.
6.1.4.7 Step-up (step-down) lease
A lease that provides for a certain rent for an initial period, followed by an increase (or
decrease) in rent over stated periods is called step-up (down) lease.
6.1.4.8 Revaluationlease
A lease that provides for periodic rent adjustments to contract rent based on the prevailing
market conditions is called revaluation lease.

6.1.5 Lease provisions:


There are a number of provisions which can be parts of a lease document.
Some of those are the following:
6.1.5.1 Lease commencementdate
The lease commencement date may be the date of execution of the lease or
any other date which is explicitly expressed in the lease document as ‗the commencement
date‘.
6.1.5.2 Lease terminationdate
The lease termination date shall be the date explicitly expressed in the lease
document as ‗the termination date‘.
6.1.5.3 Leaseterm
The duration of lease between the date of commencement and the date of
termination is called the lease term.
6.1.5.4 Renewal of leaseclause
By this clause, the parties in a lease deed agree to renew the lease for a further
period of time, upon the written request by the lessee, before the termination of the lease if
the lessee does not contravene any of the clauses of the lease during the lease period.
6.1.5.5 Full repairing and insuring leaseclause
Fully repairing and insuring lease means in that lease deed the lessee agrees to
bear the burden of all outgoings.
6.1.5.6 Unearned increaseclause
When a lessee transfers property, then the lessor claims, by the application of
this clause, a share in the profits. The deduction of the lessor‘s share in the increase of the
leasehold property is known as ‗unearned increase‘.

6.1.6 Lease Covenants:


Leasehold covenants are the undertakings made in contracts concerning the
leasing of property. Lessor as well as lessee agrees to a number of covenants in a lease
deed. By a negative covenant, the affected party relinquishes some of its rights voluntarily.
The following are the examples of negative covenants:
6.1.6.1 The‗rightofuse‘ofthepropertybythelesseemaybecurtailedbyimposing
‗not to use other than as a residential building‘ clause in the lease deed.

6.1.6.2 The ‗right to income‘ from the property by the lessee may be curtailed by
imposing ‗not to sublet‘clause.

6.1.6.3 The‗righttotransfer‘oftheleaseholdinterestmayberestrictedbyimposing
‗not to assign the lease to any other person without prior consent of the lessor‘.

6.2 Valuation of Lessor’s Interest, Lessee’s Interest including Sub-Lease in Leased


Property. Premature Termination of Lease or Surrender ofLease

6.2.1 Valuation of lessor’s (freeholder’s)interest:


Valuation of freehold properties (lessor‘s interest) can be classified under
different heads depending upon duration of income, rental review periods and prevailing
market rentals.
Case 1: Long-term Fixed Rental Income
In this case the rent is fixed over a long unexpired term of lease.
PROBLEM
A freehold site is rented out for, say, 90 years to a developer at a ground rent
of Rs. 10,000 per annum net of outgoings. The lessee developer has constructed a building
fetching an annual rent of Rs. 50,000. Value the freeholder‘sinterest.
SOLUTION
Net income = Rs. 10,000 per annum
100
YP in perpetuity @ 12% =
12
100
Value of freehold interest = 10,000 × = Rs. 83,333
12
In the above example the following are to be noted:
1. The discount rate of 12% should be established from comparable similar
investments. In general, rate of return should be fixed considering the followingfacts:
(i) Property is located in first/second class area(location).
(ii) The rent reserved is either fixed for a long time, or is flexible or there is a
clause forreview.
(iii) Liability of outgoings, etc. has been borne bylandlord/lessee.
(iv) Certainty/uncertainty of futureincome.
(v) Nature, use and type of tenancy (excellent, poor,normal).
(vi) Annual income is guaranteed ornot.
(vii) Yield from gilt-edgedsecurity.
Considering the above stated reasons YP @ 12% is reasonable, compared to
other alternative investment opportunities.
2. Theinterestrateishighasthereisnogrowthcharacteristicinthe
investment.
3. The freeholder will obtain vacant possession of the property after a verylong
time but this is too remote in the present context and may be treated as perpetuity.
4. Due to inflation, the tenant is most unlikely to default. Moreover, the rent is
secured by the building that has been erected. Therefore, the ground lessee is unlikely to
default.
Case 2: Freehold Income at Rack Rent Reviewable Regularly
In this case, a property located in some good locality which has recently been
let out at full rack rent is reviewable at regular intervals.
The meaning of rack rent is the Full Rental Value (FRV) of the vacant property,
or existing property including land and building (taken together), when let out to potential
tenants in the open market operation.
PROBLEM
The rent currently fixed is Rs. 10,000 per annum which represents rack rent net
of outgoings with provision for rent review every three years. The property is located in a
good commercial area. Calculate the freeholder‘s interest.
SOLUTION
Net income = Rs. 10,000 per annum
100
YP in perpetuity @ 7%= =14.286
7
Value = 10,000 × 14.286 = Rs. 1,42,860
The following points should be noted:
1. The discount rate should be established from similarinvestments.
2. The interest rate is lower, i.e. YP is higher than in Case 1 because the
growth rate istremendous.
3. The lease period isshort.
4. The present income is at full value (rack rent) and every three years it
is likely to be brought to thisstatus.

Case 3: Freehold Property Let at below Market Value with Provision for
Reversion.
In this case rental income is less than the rack rent but review is possible early.
PROBLEM
Rental income of Rs.10,000 per annum net is being received by a freeholder
whereas the actual worth of the property is Rs.15,000 per annum at the current market rate.
The present lease is reviewable in three years. Calculate the freeholder‘s interest.
SOLUTION
This problem will be solved in two parts:
1. The term for threeyears.
2. The reversion when rent will be reviewed intoperpetuity.
1. Term
Present rent = Rs. 10,000 per annum
YP in 3 years at 6% single rate = Rs. 2.673
Term value = 10,000 × 2.673 = Rs. 26,730
2. Reversion
Full rental value (FRV) on reversion = Rs. 15,000 per annum
YP in perpetuity @ 7% deferred 3 years = 14.285 – 2.624 =11.661
Reversionvalue = 15,000 ×11.661
= Rs.1,74,915
Valuation = Term value + Reversionvalue
= 26,730 + 1,74,915
= Rs. 2,01,645
Notes: The discount rate during the term has been taken at 6 percent as
compared to 7 percent in Case 2 because the rent during this period is less than the rack
rent and the tenant is unlikely to default, being in an advantageous position.
However during the period of reversion again, the 7 per cent rate derived
from analysis is adopted. During this period there is scope for an increase.
6.2.2 Valuation of lessor’s (freeholder’s) interest:
Leasehold interests may arise because of a lessee giving a sublease to a third
party or where a leaseholder takes ground lease for a long term and develops the property
and the lessee recoups his capital on the construction or investment. In the first case, the
sublease should be for a term shorter than the main lease whereas in the second case
leases are normally for a long period, i.e. 99 years or more with an option for renewal for a
further period.
The important fact to note is that the leaseholder makes a certain amount of
profit on the rent, i.e., the full rental value is greater than the rent payable by the
leaseholder. This profit rent may continue for a few years and then the lease might
terminate, in which case it is a short-term lease, or there might be a clause for renewal of
lease and profit might be variable for the remaining term.
The leaseholder‘s interest normally diminishes with the passage of time and his
interest in the property will ultimately extinguish with the expiry of the lease. Therefore, the
investor (i.e., the leaseholder or lessee in this case) knows that one day he will have no
capital asset remaining. It is, therefore, normally necessary for him to start some sort of a
fund, so that the income produced from this fund would eventually become his total asset
at the expiry of the lease. This is why, generally, a leasehold interest is considered to be a
wasting asset. The fund that the investor might start for replenishing his capital is known as
a ‗Sinking Fund‘, i.e., every year, in order to make up for the eventual loss of the capital
asset (in this case the leasehold interest), he puts away a certain amount into a sinking fund
from his currentearnings.
Case 1: Short-term Profit Rent with Sinking Fund
In such cases, the lessee is interested in making a quick profit. At the same
time, he would like to ensure against the ultimate loss of the leasehold asset by starting a
sinking fund. An example will illustrate thissituation.

PROBLEM
A property fetches a leaseholder a rent of Rs. 30,000 per annum. The fixed rent
payable to the superior landlord by the leaseholder is Rs. 16,000 per annum which includes
all outgoings on the property and as per terms of the agreement are payable by the
superior landlord. The lease has only eight years more to run. Calculate the capital value of
the leaseholder‘s interest assuming appropriate yields as per market rates.
SOLUTION
Profitrent = Rs. 14,000 perannum
YP = 1
r
R+
(1+r) n−1
1
=
0.09 + 0.03
(1.03)8− 1
1
= =4.939
0.09+0.11246
Capitalvalue = Profit rent ×YP
= 14,000 ×4.939
= Rs69,149.46

Case 2: Long-term Lease with RegularReviews


Where the lease is long, the lessee would still start a sinking fund to ensure
against ultimate loss of the fixed asset. Normally, in a long-term lease, there would be a
clause for regular reviews. Income during such periods can be calculated and brought to
present value deferred in time.
PROBLEM
Mr. Anand has given a ground lease to Mr. Barua for 50 years which has still 44
years to run at a fixed ground rent of Rs. 10,000 per annum for the first 25 years and at Rs.
12,000 per annum for the next 25 years. The lessee, as per the terms of the agreement, was
to construct a building worth Rs. 4,00,000 which he has already done. Present value of the
land is estimated at Rs. 1,80,000. On expiry of the lease, the land and building revert to the
head lessor, i.e. Mr. Anand. Estimate the lessee‘s interest in the property.
SOLUTION
Valueofland = Rs. 1,80,000
Prime costofstructure = Rs.4,00,000
Depreciation at5%(assumed) = Rs. 20,000
Depreciated value of structure = Rs. 3,80,000
Value of land and building = Rs. 5,60,000
Assume security @ 9%
Netrackrent = 0.09×5,60,000 = Rs. 50,400 per annum
For the first 19 years
Profit renttolessee = 50,400–10,000 = Rs. 40,400
YP dual rate of Re. 1 @ 9% and 3% for 19years
1
=
R+ r
(1+r)n−1
1
=
0.09 + 0.03
(1.03)19−1
= 7.703
Firstvalue = 40,400×7.703 = Rs.3,11,201.20
For the next 25 years
Profit renttolessee = 50,400–12,000 = Rs. 38,400
YP of Re. 1 @ 9% and 3% for25 years = 8.516
Present value of Re. 1 @ 9% deferred 19 years =0.1945
YP of Re. 1 @ 9% and 3% for 25 years deferred 19years
= 8.516 × 0.1945 =1.656
SecondValue = 38,400 × 1.656 = Rs.63,590.40
Totalvalue = First value + Second value = Rs.3,74,791.60

6.2.3 Premature Termination of Lease or Surrender ofLease


Sometimes a lessee, approaches the lessor a little before the expiry of an
existing lease, to cancel the existing lease and renew the same on new terms. This may be
because the lessee may want to refurbish the premises or to reorganize the business or
sometimes to simply continue the existing business. The landlord is also benefited as it will
give him an increase in rent immediately and sometimes even a salami or pugri (premium
lump sum payment).
The basic principle involved is that the present interest must equal proposed
interest. The valuer should look at both the freeholder‘s and the leaseholder‘s point of
view. These views, willdiffer.
PROBLEM
Mr. Apte the lessee of an office building plans to reorganize his export
business and is planning at least 30 years for the future. His existing office lease which
expires in seven more years out of the 30 years in the original lease was taken out at Rs.
1,00,000 per annum on a full repairing and insuring lease. The true rental value of the
building is Rs. 2,50,000 per annum. The lessee will surrender the existing lease and sign a
new lease for 30 years at a rent of Rs. 1,00,000 per annum. What premium should Mr. Apte
pay to thefreeholder?
SOLUTION
1. Freeholder‘s viewpoint
(a) Presentinterest
(i) Rent receivable during the next 7 years @ Rs. 1,00,000 perannum.
YP for 7 years @ 8% = 5.206
Amount = 1,00,000 × 5.206 = Rs. 5,20,600
(ii) Reversion to a rent of Rs. 2,50,000 per annum in perpetuity.
100
YPperpetuitydeferred7years@8%= −5.206=7.294
8
Amount = 2,50,000 × 7.294 = Rs.18,23,500
Totalpresentinterest = 5,20,600 + 18,23,500 = Rs.23,44,100
(b) Proposedinterest
(i) Rent receivable during the next 30 years @ Rs. 1,00,000 per annum.
YP for 30 years@8% = 11.258
Amount = 1,00,000 × 11.258 = Rs.11,25,800
(ii) Reversion to a rent of Rs. 2,50,000 per annum inperpetuity
YP perpetuity deferred 30 years @8%
100
= −11.258=1.242
8
Amount = 2,50,000 × 1.242 = Rs.3,10,500
Totalproposedinterest = 11,25,800 + 3,10,500 = Rs. 14,36,300
Therefore, the freeholder should expect a premium of23,44,100 – 14,36,300
= Rs.9,07,800

6.3 Real Estate as an Investment, Yield from Real Estate vis-à-vis other forms of
Investments- Sound InvestmentComparison

6.3.1 Real Estate as anInvestment


In the investment market there are always a number of alternative
opportunities for investment. The investor who would prefer capital gain from the
investment would risk investing his capital in the equity shares of first-class companies
whereas the investor who would prefer safe and secure income from the investment would
like to investment in long-term government securities. Between the two extremes of highly
risky equity share investment and safe and secure government securities however there are
a number of alternative opportunities with different degrees of risk and security. Real
property is one such vehicle of investment in the totality of the investment marketscenario.
6.3.2 Yield from Real Estate vis-à-vis other forms of Investments- Sound
Investment Comparison
The role played by real property is dependent on the various aspects of real
property as a channel of investment.
1) MANAGEMENT ASPECT
Real property investment is always associated with trouble and cost of
management. Thus for management of real property the services of a lawyer, an architect, a
valuer or a real estate agent are necessary from time to time. Further, there may be
involvement in litigations day to day involvement in enforcing lease covenants, collection of
rents etc, all parts of hazards of management. Comparatively, investment in equity share is
less troublesome and costly to manage. Therefore, in order to induce an investor in the
investment in real property there must be an incentive of higher rate of return than the
yield from equity share investment.
Again, the investor can invest in long-term government securities to obtain a
risk-free long-term income. Against this, there is the prospect of rental growth and
appreciation of capital value as the incentive for investment in real property in spite of the
hazards of management.
2) LEGAL ASPECT
Law interferes with real estate investment in a number of ways. Thus, rent
control acts may severely affect the prospect of rental growth of the property. Further,
taxation laws may affect real property investment. Examples are property tax, capital gains
tax, betterment levy etc. Again, town planning acts may adversely affect real property
investment through restrictions imposed on land uses, density etc. As a result the investors
should expect higher yield from real property investment to compensate against this.
3) LIQUIDITY
Real property is an illiquid asset. The time taken for real estate disposal is a
lengthy process. Further, for transfer of property one may have to incur cost on lawyers'
fees, stamp duty etc. Comparatively, the time taken for disposal of shares or cost of transfer
of share are much less than that of real property. Hence, in order to induce an investor to
invest in real property one must expect to receive a higher yield.
4) SIZE OF INVESTMENT
Real property is indivisible into very small components. Generally, a huge
amount of capital is associated with an investment in real property. Comparatively, a much
smaller amount of capital is required in order to invest in shares. To compensate against
this the real property investment should have higheryield.
5) LACK OF CENTRAL MARKET
There is no central market for real property for the stock exchange market.
Furthermore, the transactions in real property are often of closed type and one is required
to incur a high amount of expenditure to have full knowledge of the transaction. These add
to the market imperfection of real property market and has the effect of increasing the yield
from real property investment.
It is clear from item number (1) to (5) above that cost and hazards of
management and market imperfections cause return from real estate to be higher than risk-
free securities. The benchmark of interest rate in the investment market is the yield from
long-term government securities (also called the gilt-edged securities or the risk-free
securities). So, all investments should have ideally produced higher return than the yield
from government securities. But in case of real estate, the prospect of capital appreciation
and income appreciation act to set off against the higher return required than that of the
government securities. That is why, real estate in first-class location tends to indicate lower
yields than the risk-free government securities. This phenomenon is known as the reverse
yield gap.
6.4 Investment Decisions: Discounted Cash Flow Techniques, Internal Rate of
Return (IRR) and Net Present Value(NPV)

6.4.1 Discounted Cash FlowTechnique


Valuers are required to estimate two types of values in the investment market – Market
Value and Investment Value (Worth). When worth exceeds the market value of the asset,
the investor decides to buy while for opposite phenomenon, i.e. when market value isfound
to be more than the worth, investor sells theproperty.
Discounted Cash Flow technique is a procedure in which a discount rate is applied to a set
of projected income streams and a reversion. The analyst specifies the quantity, variability,
timing, and duration of the income streams and the quantity and timing of the reversion,
and discounts each to its present value at a specified yield rate. In discounted cash flow
analysis, a yield rate is usually an output, while a discount rate is usually an input.
The discounted cash flow technique is used by the valuers to estimate the market value of
the income producing properties where market transactions of such properties are not
available. As a result all risk yield (ARY), in such cases, is also not available. In these cases
DCF is the best option subject to rational estimates of investment risks and other aspects of
market regarding demand and supply and appropriate income growth.
The DCF model incorporates a projected cash flow for a period for which the present is held
or proposed to be held called the holding period of the investment. The projected cashflow
are then discounted back at the target rate of return to present day value called the
presentvalue.
After the holding period is over, the reversionary value (if any) occurs which is called the
exit value. It is estimated by applying an ARY for capitalisation of income at that stage
depending upon the anticipatory future growth or otherwise of the income at that stage
and then discounting back the exit value at the target rate of return to present day value,
that is, the present value. After this the discounted value for the holding period and the
present value of the reversionary value are added to find the full value derived by the DCF
model.
6.4.2 NET PRESENTVALUE
In the evaluation of investment projects the discounted cash flow method can provide a
useful means of testing financial feasibility. The discount rate will generally be the
minimum rate of return required by the investor over the period of the cash flow. If the
total of the discounted cash flow over the period is zero or a positive figure, then the
proposal will be acceptable. If a negative discount flow value results, the proposal will be
unacceptable.
Using the net present value approach all cash flows are discounted to present value using
the required rate of return which is the minimum investors require on their investment.
To calculate the present value of a cash flow the following information is necessary:
1. Net cash flow during each period, i.e., estimated cash inflow oroutflow.
2. Discountfactorsforeachperiod–thiscanbecalculatedbyusingtheexpression 1
( 1 + R)
n

Where, R = Rate of return per period (expressed as a decimal),and


n = Number ofperiods.
Multiplication of the anticipated future cash flow by the appropriate discount factor gives
the present value.
3. Rate of returnrequired.
4. Number ofperiods
PROBLEM
In the following investment proposal, the investor would like to know the minimum sale
price X. You are requested to advise him.
Cost ofready-madehouse Rs.2,00,000
Improvements Rs.1,50,000
Rent per year for consecutive 5 years (payable at the end of each year)
Rs. 30,000
Sale price at the end of 5 years Rs. X
Capital gains tax to be paid at the time of sale Rs. 60,000
Target rate Rs. 12%
SOLUTION

Cash R in % Present
Time n D = 1/(1 +
Details Flow per Value P = C
(Years) years R)n
C annum x D (Rs.)

-
0 Cost of house 0.12 0 1 -200,000
200000

-
0 Improvements 0.12 0 1 -150,000
150000

1 Rent year 1 30000 0.12 1 0.8929 26,786

2 Rent year 2 30000 0.12 2 0.7972 23,916

3 Rent year 3 30000 0.12 3 0.7118 21,353

4 Rent year 4 30000 0.12 4 0.6355 19,066


5 Rent year 5 30000 0.12 5 0.5674 17,023

5 Sale Value X 0.12 5 0.5674 0.5674X

Capital Gain
5 -60000 0.12 5 0.5674 -34,046
Tax

0.5674X +
Net Present Value
2,75,902

The minimum sale price X of the house will be obtained when the total present value P as
calculated above is zero so that a 12% target rate is maintained.
– 2,75,902 + 0.56743X = 0
or X = Rs. 4,86,231
Two characteristics of the NPV method are:
1. The NPV method is based on the assumption that the intermediate cash inflows of the
project are at the same rate of return as the project‘s cost ofcapital.
2. The NPV of a project generally decreases as the discount rate increases. The decrease in
the NPV, however, is at a decreasingrate.
The net present value criterion has considerable merits over the previous two methods
discussed so far.
1. It takes into consideration the time value ofmoney.
2. The cash flow stream is taken into account in itsentirety.
3. The net present value represents the wealth of investors (in present day money terms)
after adjusting for the return on the project.
4. The net present value of different projects, evaluated in terms of today‘s rupees, can be
added. For example, the net present value of a group of three projects, A, B and C will
simply be the sum of the net present value of these projects takenindividually.
NPV (A+B+C) = NPV (A) + NPV (B) + NPV (C)
This property of net present value ensures that a poor project (one with a negative net
present value) may not be acceptable just because it is combined with a good project
(being a positive net presentvalue).
These advantages make the net present value appraisal technique a powerful appraisal tool
in investment analysis. However, conceptually the net present value being expressed as an
absolute number may appear intelligible to the evaluator who may be more inclined to
think in other relative terms like rate of return or profitabilityindex.
6.4.3 INTERNAL RATE OF RETURN(IRR)
The internal rate of return is the actual return obtained from an investment. Here the
internal rate of return i.e. R, is to be calculated so that all future discounted receipts and
discounted payments are equal. At this point the net present value will be zero. In the
previous method, at a target rate of 12 per cent NPV of Investments A, B, etc. were found
to be all positive which indicates that the actual rate of return, the IRR is higher than 12
percent in allcases.
Where there are several investment opportunities, those investments with an IRR below the
target rate will be rejected. Of those with an IRR greater than the required rate of return,
the investment with the highest IRR will be preferred. The IRR may be calculated by trial
and error and by interpolation. This is best demonstrated by aproblem.

PROBLEM

The following data is given regarding purchase and sale of a flat.

Cash out Cash in


Date Particular
(Rs.) (Rs.)

31.12.1987 Purchase of one flat 1,50,000

31.12.1987 Salami and rental income 18,000


received in advance for two
years, i.e. for 1988 and 1989

31.12.1989 Repairs of flat 10,000

31.12.1989 Rental income for one year, 5,000


i.e. 1990 received in advance

31.12.1990 Rental income for two years, 10,000


i.e. 1991 and 1992 received
inadvance

31.12.1990 Renovation of flat 9,000

31.12.1992 Rental income for one year 6,000


i.e., 1993

31.12.1993 Sale of flat 1,90,000

Calculate the average earning rate on the above flat per annum.
SOLUTION
Try a rate of 15%

R in % Present Value
Time in years Cash Flow D = 1/(1 +
Date per (P) P = C x D (in
(n) C R)n
annum Rs.)

31.12.1987 0 -132000 0.15 1 -132,000

31.12.1989 2 -5000 0.15 0.7561 -3,781

31.12.1990 3 1000 0.15 0.6575 658

31.12.1992 5 6000 0.15 0.4972 2,983

31.12.1993 6 190000 0.15 0.4323 82,142

Net Present Value -49,998

A negative sign indicates that a rate of 15% will not be maintained.


The benefit/cost ratio is found by dividing the income by theoutgoing.
85,777
In this case benefit/cost ratio= = 0.6317 which is less than 1 indicating higher
1,35,780
expenses than income. A close look at the figures indicates that the contribution at year six
should be much more. Try a rate of 10%.

R in % Present Value
Time in years Cash Flow D = 1/(1 +
Date per (P) P = C x D (in
(n) C R)n
annum Rs.)

31.12.1987 0 -132000 0.1 1 -132,000

31.12.1989 2 -5000 0.1 0.8264 -4,132

31.12.1990 3 1000 0.1 0.7513 751

31.12.1992 5 6000 0.1 0.6209 3,726

31.12.1993 6 190000 0.1 0.5645 107,250

Net Present Value -24,405

This again indicates that even a 10% rate will not be maintained. Try a 7% rate.
R in % Present Value
Time in years Cash Flow D = 1/(1 +
Date per (P) P = C x D (in
(n) C R)n
annum Rs.)

31.12.1987 0 -132000 0.07 1 -132,000

31.12.1989 2 -5000 0.07 0.8734 -4,367

31.12.1990 3 1000 0.07 0.8163 816

31.12.1992 5 6000 0.07 0.7130 4,278

31.12.1993 6 190000 0.07 0.6663 126,605

Net Present Value -4,668

Try a 6% rate.

R in % Present Value
Time in years Cash Flow D = 1/(1 +
Date per (P) P = C x D (in
(n) C R)n
annum Rs.)

31.12.1987 0 -132000 0.06 1 -132,000

31.12.1989 2 -5000 0.06 0.8900 -4,450

31.12.1990 3 1000 0.06 0.8396 840

31.12.1992 5 6000 0.06 0.7473 4,484

31.12.1993 6 190000 0.06 0.7050 133,943

Net Present Value 2,816

It is now evident that the rate of return lies somewhere between 6% and 7% and closer to
6%. An estimate can be made by interpolation assuming a uniform rate of change although
this assumption is not correct as the discount rate varies geometrically rather than linearly.
However a sufficient accurate rate of return can be determined by using the linear
interpolation formula:
 N1 
R =R1 + (R2 −R 1 ) 
 (N 1 + N 2)

Where, R = Actual rate of return


R1 = Lower rate ofreturn
R2 = Higher rate of return
N1 = Lower net present value
N2 = Higher net present value (without the sign)
In the above,

R = 6+ − 6) 2824 
 (7
 
(2824 +4676 ) 
= 6.3765%
6.4.4 CAPM Technique
In finance, the capital asset pricing model (CAPM) is a model used to
determine a theoretically appropriate required rate of return of an asset, to make decisions
about adding assets to a well-diversifiedportfolio.
The model takes into account the asset's sensitivity to non-diversifiable risk (also known as
systematic risk or market risk), often represented by the quantity beta (β) in the financial
industry, as well as the expected return of the market and the expected return of a
theoretical risk-free asset. CAPM assumes a particular form of utility functions (in which only
first and second moments matter, that is risk is measured by variance, for example a
quadratic utility) or alternatively asset returns whose probability distributions are
completely described by the first two moments (for example, the normal distribution) and
zero transaction costs (necessary for diversification to get rid of all idiosyncratic risk). Under
these conditions, CAPM shows that the cost of equity capital is determined only by beta.
The CAPM is a model for pricing an individual security or portfolio. For individual securities,
we make use of the security market line (SML) and its relation to expected return and
systematic risk (beta) to show how the market must price individual securities in relation to
their security risk class. The SML enables us to calculate the reward-to-risk ratio for any
security in relation to that of the overall market. Therefore, when the expected rate of return
for any security is deflated by its beta coefficient, the reward-to-risk ratio for any individual
security in the market is equal to the market reward-to-risk ratio, thus:
ERi = Rf + βi x (ERm−Rf)
Where, ERi = expected return of investment
Rf = risk-free rate
βi = beta of the investment (Systematic risk) (sensivity)
(ERm − Rf) = market risk premium

6.5 Profit Method: Valuation of Special Properties: Hotels, Cinema, Mall, Petrol
Pump, Hillresorts

6.5.3 Profit Method


The ‗profit method‘ of valuation, as the name suggests, consists of working out the profit of
a business enterprise and capitalizing at an appropriate year‘s purchase. The investments
made in such business enterprises are with the sole motive of earning regular profit. The
state government, or in some cases the central government, issues licenses before such a
business can be started. These include cinemas, hospitals, hotels and restaurants.
The licence is not only attached to the business but also specifies as to where the business
is to be carried out. For example, in case of a hotel the business embraces land, buildings
and ancillary improvements. There will also be furniture, fittings and fixtures, and goodwill
associated with the business premises. The value of the property, therefore, depends upon
the earning capacity of the business as a whole and not on the cost of construction of the
building or the value of land in the vicinity. Consequently, valuation of the property,
whether to be valued on behalf of the owner or the lessee, would include business goodwill
besides, of course, the physicalassets.
The tangible assets would consist of land, buildings and structures, plant and equipment
and other physical assets. Tangible assets are those which can be physically touched.
Intangible assets, on the other hand, would include goodwill of the business, licence
associated with the premises, quality of management, and so on.
Very often, the licenses for such businesses, by themselves, possess considerable value as
may be the case of a hotel or restaurant serving liquor. Therefore, in order to make realistic
valuation of such licensed premises, the valuer must possess an in-depth knowledge of the
particular industry and make a clear investigation and analysis of the market.
Basically, the procedure consists of determining the annual profit, apportioning it into
tangible and intangible components and capitalizing each by an appropriate year‘s
purchase.
6.5.2 Determination ofprofit
Profit is determined bytherelationship: Profit = Income –Outgoings
Income includes all receipts from sale such as food or liquor in a hotel or restaurant, or
from sale of cinema tickets in the case of a cinema, from advertisements, parking fees,etc.
The outgoings can be classified as follows:
• Preliminary expenses

• Working expenses

• Depreciation and maintenance

• Owner‘sprofit

1) Preliminary expenses include entertainment tax, licence fees, film hire charges
in case of a cinema and/or basic items necessary for running thebusiness.

2) Working expenses would include salaries and benefits to staff, operating and
running charges of the business including municipal taxes, purchase of food and/or other
consumables for thehotel.

3) Depreciation and maintenance would include depreciation on building,


furniture, fixtures and other items used regularly. Also, an amount for a sinking fund can be
provided for, besides making an allowance for maintenance andrepair.
4) Owner‘s profit could be anywhere between 15 and 20% or more and normally
takes into account interest on blocked capital, interest on working capital, trade profit, risk
factor and management skill, and thelike.
However, interest on borrowed capital or income or wealth tax paid should not be included
in the expenses.
6.5.2 Rate of capitalization
After arriving at a figure for the profit, it is necessary to determine two factors:
• The ratio of tangible profit to intangibleprofit.

• The capitalization rate for eachprofit.


The tangible portion of the net profit is due to the capital assets or assets which can be
touched. The intangible portion of the profit is due to goodwill, management, licence, etc.
Normally this would be in the ratio of the profit of 75 to 25 per cent or 70 to 30 per cent,
depending on the type of business being analyzed. This ratio will have to be determined by
the valuer considering facts and circumstances of the business and other relevant data.
The capitalization rate will have to be fixed taking into consideration the return on other
investments and prevailing market rates of interest.
6.5.3 Valuation of Hotel
In the case of hotels or restaurants which depend primarily upon the quality of customers,
the fundamental criterion to be realized is that it is a trading operation. The main object of
conducting the business is profit. In a restaurant certain goods are processed, reprocessed,
and served in a manner which depends upon management practices, location of the
premises, etc. For their profits hotels too depend, mainly for their clientele, on location,
management practices, facilities, services and conveniences to clients, type of construction
and related operations. Therefore, a hotel building probably represents the highest and
best use of the site itself. The quality of its management services also goes a long way in
maximizing its profits. Some of the salient points to be noted for valuing a hotel business
are asfollows:
1) Location of thepremises.

2) Characteristics of furniture, fittings and miscellaneousequipment.


3) Type of management and itsefficiency.

4) Type of servicesprovided.

5) Quality of rooms, as also the general construction of thepremises.

6) Quality of food that is served.

7) General standard ofcleanliness.


Thus, hotels and restaurants can best be valued on the basis of their trading potential
although the valuer must always make it a point to study occupancy rates, check for a
comparable property in the area and, in general, verify all aspects of the property. The
following example will probably serve as a good example on valuation of a hotel business.
6.5.3 Valuation of Cinema
A movie theatre in any town or city of the country is generally a landmark site. It tends to
become the centre of social activity and a meeting place. However, with the advent of T.V.
and other similar sources of entertainment, the importance of cinema may have decreased
but the film theatres are still important locational sites in a town.
The market value of a cinema depends mainly on the earning capacity, irrespective of other
considerations such as the break-up price of land, structures, fittings and fixtures, etc. Of
course, these components do have an effect on the value. However, the profit method
would take in all these including the effect of goodwill and management.
Again, the gross income is from the exhibition of movies which depends on the movie
being exhibited, quality of the hall, seats, parking arrangements, aesthetics, environment,
sound system, general comfort and other such factors. Income will also be from
advertisements, parking stands, rent from show cases, rent from food stalls, and thelike.
The outgoings would include entertainment tax, preliminary expenses for hiring of films,
other expenses pertinent to the business, depreciation, etc. Owner‘s profit should also be
considered.
6.5.3 Valuation of Shopping Mall
Shopping malls are defined as a group of commercial establishments planned, developed,
owned and managed as a unit related in location, size and type of shops to the trade area it
serves. It provides onsite parking relating to the types and sizes of its stores. Therefore a
shopping mall is more than a collection of retail uses.
Valuation of shopping mall deserves understanding of the terminology, conditions and
criteria those are currently used by the retail industry. Shopping mall valuation also requires
clear understanding of the economic rationale for the existence and viability of shopping
mall development.
Analysis of a shopping mall needs the following six types of criteria to be described:
• Shopping centresize
• Anchor tenant

• Type of productssold

• Sitesize

• Distance and traveltime

• Customer base
For a shopping mall to be financially successful, it must possess appropriate site and
building characteristics. Those are the following:
Site characteristics:
1) Site size including the building footprint, customer parking, parking lot
circulation, parking lot lighting and security, parking in relation to building entrance,
employee parking and other siterequirements

2) Siteshape,

3) Access to the tradearea

4) The site‘s relation to the streetpattern,

5) Ease ofaccess,

6) Visibility

7) Signage,

8) Landscaping

9) Topography and drainage and

10) Utilities, zoning, development regulations and construction codes

Exterior Building characteristics:


1) Building area

2) Building configuration andshape

3) Building elevation andmaterials

4) Building entrances

5) Canopies

6) Signage

7) Lighting

Interior Building characteristics:


1) Storefronts

2) Storesize

3) Building flexibility

4) Multiplelevels

5) Foodcourts

6) Interiorsignage
In conducting a physical inspection of a shopping mall property and its comparables, the
valuer should be sensitive to the above characteristics those affect the property‘s physical
and functional adequacies.
The shopping malls are almost always viewed by owners and potential owners as an
investment from which a certain minimum return ought to be realised. Hence, the income
approach is the one with which the valuer should be most concerned. Replacement and
original costs appear to have little or no effect on the marketability of such a project if it has
had several years of successful operation; and comparables are usually difficult to identify
due to difference in demographic areas, shopping themes, tenant mixes etc. found in the
different malls.

6.5.3 Valuation of Petrol Pump


Petrol pump is considered as special property. To estimate the market value of a petrol
filling station the following have to be considered by a valuer:
1) Abutting road: Passengers Car Units (PCU) passing per hour plays the
mainrolein determining the importance of abutting road of a petrol pump. Higher the PCU,
higher will be the value of the petrolpump.

2) Amenities and facilities: Modern day petrol pumps demand for motels / rest
room facility in nearby area if not in the pump site itself. The value goes on increasing in
inclusion of major spare parts shop, tyre repair service, free air and water supply as well as
adequate toilet facility within the pumpcomplex.

3) Adequate greenery and in sight washing facility add value to a petrolpump

4) Not only parking of cars during petrol filling but also parking facility of other
cars also increases the value of petrolpump.
Petrol outlets are run following different nature of agreements and those are the following:
a. Oil Company owns the land and dealer is running thebusiness.

b. Oil Company is the lessee of the land and dealer is running thebusiness.

c. Dealer owns and land and also runs the business

d. Oil company owns the land and also runs thebusiness


Valuers have to estimate either the lease rent for the land leased out to the company or to
estimate the market value of the petrol pump business run by the dealer.
In case of lease rent calculation, valuer has to find out the market value of the freehold right
of the land and then estimate the lease rent considering the yield following the company‘s
policy.
For valuation of petrol pump business, the valuer has to follow the profit method.

6.5.3 Valuation of Hill Resorts


Hill resorts valuation is done following the same principle as that of Hotel Valuation with
some differences.
MARKET APPROACH TO VALUATION
4.1.1 Types of Market
Like the commodity market there are 3 types of market conditions in Real
Estate Market namely;

• Buyers‘Market.
• Sellers‘Market.
• StableMarket.
4.1.1.1 Buyers’ Market: When number of units available to the purchasers
is far in excess of demand it is Buyers‘ Market. Prices tend to
decrease because of greater bargaining power of purchasers and
availability of plenty of alternative and substitute properties.
Sellers compete with each other to sale their units at lesser price.
We can say buyer is the king in Buyer's market.
4.1.1.2 Sellers’ Market: When supply of units is less than demand for such
units, prices tend to go up due to competition amongst buyers to
purchase these units even at higher price. Bargaining power of
purchaser reduces. Moreover, Real Estate Units are such that it
cannot be brought into market in short span of few days or few
months like consumer products. It takes pretty long time to
increase the supply to meet with market demand. Hence in seller's
market seller is king and he will dictate terms.

4.1.1.3 Stable Market: Sometimes in market demand as well as equal


supply also exists but due to various reasons, purchasers stop
purchasing or defer the purchase of premises. Artificial stoppage
of demand in the market prevails. The reasons could be likely
changes in Government policies or lack of paying capacity of
people at large in the locality, to pay high or prohibitive price of
properties.
Expectations of people about likely fall in price of real estate in
near future also results in Stable Market conditions. Prices do not
fall nor rise but remain stable year after year. No material
transaction takes place in the market. Stray sale transactions in
market do not change stable market situations.

Considering the approachability of the buyers, real estate market


again can be divided into the following two distinct categories:

• Closed Marketand
• Open Market
4.1.1.4 Closed Market: All types of buyers are not permitted to buy real
estate in such closed market. Only special buyers are allowed to
buy. Sale and purchase of property in Parsi Colony is permitted
only to Zoroastrians. Similarly, Catholic society permits deal
between Catholics only. In some societies sale is permitted only to
vegetarians. Even for commercial premises such closed market is
possible. An office premises in Diamond Bourse at B. K. Complex,
Mumbai can be sold only to Diamond merchant and to no one
else. There are many other types of closed markets. In family
partition disputes and partition cases of jointly owned properties
many a time court orders to call open bids from rival parties (Joint
owners) to purchase the property. It is a private auction. This is
nothing but type of sale under a closed market condition.
Outsiders are not permitted to bid. If such private bids fails then
the court orders for sale by public auction i.e. in open market. It is
a common experience that price of property in closed market is
less than its real worth in an open market.
4.1.1.5 Open market: It is like any other commodity market. All buyers
and all sellers are welcome to transact deal. There is no common
market place for sale or purchase of real estate like common
market place for sale and purchase of other commodities and
consumer goods. Open market for Real Estate could be therefore
defined as a notional forum or platform, where group of buyers
and group of sellers are present each one competing with other
to buy or to sell and where ample substitute properties are also
available for sale and purchase. Monopoly aspect is not present in
such open market.
4.1.2 Market Survey & Data Collection

4.1.2.1 Market Survey & Data Collection


Market Survey is an analysis of the market conditions of supply, demand,
and pricing for a specific property type in a specific area.

4.1.2.2 Data Collection


Data collection is the process of gathering and measuring information on
targeted variables in an established system, which then enables one to
answer relevant questions and evaluate outcomes. Five different types viz.
Qualitative, Quantitative, Attribute, Discrete, and Continuous Data are
collected during market survey.

4.1.3 Sources of Sale Transactions


There are several sources from which sale instances of a particulars locality
can be collected and those are the following:

✓ Sales recorded at the Registrar's office of the concerneddistrict


✓ Information from local brokers/residents (LocalEnquiry)
✓ Advertisements in Newspapers
✓ Land Acquisition cases with awarddetails
✓ Auction sale information from differentauthorities
✓ Various websites on Internet, giving details about sale offers of real
estate (flat/Shopetc)
✓ Valuers own Data Bank
4.1.3.1 Sales recorded at the Registrar's office
Each State Government at their District Registrar / Sub-registrar‘s office
maintains sales registers which are called Index II Registers. In these
registers, Registrar records all sale transactions those have taken place
during the year under his command (Jurisdiction). Each of these sales
entries in the register shows name of seller (transferor) name of purchaser
(transferee), plot number, survey number or city survey number of sale
property, name of village in which said sale property falls, date of
execution of sale document and date of registration of sale document in
Registrar's office. This register also shows sale price or transfer
consideration, area of property sold, serial number of registration of the
sale in sale register. Register also shows type of transfer document viz.
conveyance sale transaction or lease assignment or mortgage deed or
release deed etc. These sales are recorded in village wise list which are
compiled in alphabetical order. All sales in a particular village in a
particular year are recorded in the register in chronological order of dates
and months of said year. Registrar maintains separate register for each
year. Old records of past 100 years sales in entire district are preserved and
maintained by the Registrar. These are made available to the public on
payment of fixed inspection and search fees. Hence, valuer can obtain land
or property transaction details of all sales in any locality, for any previous
year (even 60 to 80 years old) from Registrar's office of concerned State
and District.

4.1.3.2 Local Enquiry


Information from local brokers dealing in such real estate transaction is
very useful. Information from local residents and property owners will also
be helpful. Particularly owners who purchased or sold property in locality
can give sale information. Local practicing architects and engineers
involved in planning and construction of buildings for their clients can also
furnish required data of ruling prices of land and buildings in the locality.

4.1.3.3 Advertisements in newspaper


Advertisements in newspaper and business papers also give plenty of
information on sale prices offered and prevalent rates of various types of
properties in the locality. Every valuer should preserve at least monthly
data of such newspaper advertisements in his own data bank for present
as well as future reference. But, these asking prices have to be discounted
properly while using these asevidences.

4.1.3.4 Land acquisition cases


If information is available about land acquisition cases and award details
for property acquired in the locality, then, such details are also valid data
for prevalent rate.

4.1.3.5 AuctionSale
Auction sale rates by Income Tax Department by Govt. or Semi Govt.
undertakings by Banks‘ custodian, Improvement Trusts, Court Receiver or
Court Registrar also give sale rate information. These are valid comparable
sale evidences.
4.1.3.6 Various websites onInternet
There are a number of property web sites available in internet where
hundreds of thousands property asking prices are posted every day. Again
these data are to be used very carefully. A discount factors ranging
between 5% to 20% on asking rates may be used. A few such websites are
www.makan.com, www.magicbricks.com, www.99acres.com etc.

4.1.3.7 Valuer’s own databank


Past experience of valuer himself, his own data bank, his local valuer
friend's opinion can also be a source of prevalent trend in real estate
market.

2. Comparison of Sale Instances – Factors


of comparison and weightage for
adjustment invalue
4.2.1 Hierarchy of comparable sales instances
Under the sales comparison method of valuation or market approach the
valuer will often use a variety of comparable evidence and has to come to
a decision as regards the relative weight of all the comparable evidence
available in the market. According to the hierarchy of comparative
evidence the valuer should adopt the following order in terms of relative
weight of the variety of comparableevidence.

(1) The sale of the subject property itself is the best possible evidence in
the market. If this is not available, a very similar property to it that has
been marketed and an offer has been made but as yet binding
contract has not been formed shall be equally good. In such case, if
the valuer has full and correct information as regards the offer
received then that may be taken as a goodevidence.
(2) Next in the order of hierarchy comes a recent completed transaction
of an identical property provided full and correct information is
available.
(3) The next rank in the order of hierarchy can be allotted to evidence on
sale transactions of similar properties provided there is full and correct
information available.
(4) Next in the order of hierarchy ranks transactions of similar propertyfor
which full data are not yet available but sufficient reliable data can be
procured.
(5) The next in the order comes the evidence from published sources or
commercial databases depending on the authority of the source or its
verifiability.
(6) Next in the order of hierarchy comes such evidences as may be
identified as historic evidences of the subject property or similar
properties.
(7) Next in the order comes indirect evidences such asindices.
(8) If none of the above data are available then the valuer may depend
upon other type of land use and different locations from which
evidence may becollected.
(9) Last of all asking prices for properties where transparency and active
market are the key factors forreliability.
(10) It has already been stated that availability of suitable evidence is very
difficult to meet in the real property market unlike goods and
commodities in other economic markets. Even in many cases, no
comparable evidence is available to enable the valuer to proceed with
his exercise. In selecting a proper evidence the valuer should follow
the above-noted hierarchy and collect evidence from amongstthem.
4.2.2 Factors of comparison and weightage for adjustment invalue
In any market (be it landed property or other goods and commodities)
there is a set of principles involved in selecting the comparable evidence.
The principles are as follows:

▪ There should be a number of comparables for which transactions


have taken place that may be regarded as evidence in the market.
The comparable should be very similar with the subject under
valuation. Ideally they should beidentical.
▪ The dates of the sale transactions should be current and not much
remote from the date of valuation so that they may reflect current
marketcondition.
▪ The sale transactions should be result of an arm's length
transaction.
▪ The transactions should be verifiable as best aspossible.
▪ They should be reflective of local marketpractice.
The main aspect of dissimilarities between landed properties can be
codified as follows:

▪ Internal or physicalcharacteristics
✓ Land
• Size of land, frontage of site, depth of site, shape of site, directional
facing, level of land, presence of water bodiesetc.
✓ Building
• Layout of building, design of building, internal planning,
construction method, specifications, area of floor, provision of
garages, additional bathrooms, age and obsolescenceetc.
▪ Externalcharacteristics
✓ Location of theproperty
This gives accessibility to roads and other transportation corridors,
services, public open spaces, proximity to amenities or shopping
complexes or undesirable featuresetc.

✓ Neighbourhoodcharacteristics
Use zoning or other restrictions by planning authorities, presence of slums,
social aspects, historic aspects amenities, law and order problems,
environmental issues etc.

✓ Transactionalcharacteristics
One-time payment, stage payment, time aspects etc.

The total number of factors affecting different aspects of real estate and
causing dissimilarities between them may be much more than this. In most
cases however, valuers generally consider four principal aspects of real
estate and make an ad hoc comparison between them to arrive at a
valuation conclusion. These four aspects are location, size, time aspect &
age and physical state of the property.

3. Hedonic Model and Adjustment Grid


Model under Sales comparison Method

4.3.1 Hedonic Model


This method is known as Hedonic Pricing Model. Under this model, price
of the property is expressed by theformula.

P = f (STLA)

P = Price of the property in the market.

f = Stands for function of

S = Size or covered area of the premises.

T = Time factor at which asset is traded in the market.


L = Location of the property.

A = Age or physical conditions of the property.

It will be seen that out of thousands of factors which affects value of the
property in real estate market, only four factors are considered in this
model. Under this Hedonic model (adhoc comparison method), the valuer
compares these four factors of both properties viz. sale instance property
and subject property. Valuer arrives at the rate to be adopted after doing
this adhoc comparison. There is no doubt that price of the property is the
function of area, location and time period. Location in respect with
proximity of civic amenities is a major factor affecting value. Similarly time
factor indicates demand and supply position and environment of market at
relevant period of time which determines price of an asset. However age
factor is only for buildings and that too in a small way. It does not apply in
case of land value comparison where characteristic of land decides its
value in the open market. Similarly size or area factor also is not a major or
a very important factor for building values but it does constitute an
important attribute for land values. Demand for more area is comparatively
less than demand for smaller areas. Under this Hedonic Model the valuer
normally follows steps as detailedbelow.

Valuer first collects data of sale instances in the locality for the relevant
period of time. Non genuine instances of sales are excluded. (ii) Rates and
attributes of genuine sales are compared with the attribute of subject
property. All relevant factors and weightages applicable are recorded. (iii)
Valuer then arrives at a concluding overall effect of these comparisons. It
may be positive weightage over sale instance rate or negative weightage
over said rate. (iv) Valuer then finally estimates the final rate for the
subject property based not only on his experience and expertise but also
on the basis of overall impression. Hence it is called adhocanalysis.

4.1.4 Adjustment Grid Model


Through the introduction of the evaluation grid model an attempt has
been made to make the approach of sale comparison less ad hoc. Under
this model, first of all a number of comparables are chosen that are sale
transactions as nearly as similar to the subject property is chosen as
comparable. About Four or five principal characteristics of the subject
property are then selected for comparison with the comparables and each
characteristic is given a rank in accordance with apre-determined
gradation of rank of different property characteristics. The gradation of
each property characteristic is made as follows.

• Rank (5) for a characteristic of highestquality


• Rank (3) for a characteristic of mediumquality
• Rank (1) for a characteristic of poorquality
Let us consider the valuation of a flat under this model.

Considering four major characteristics influencing values viz, location, size,


age and specification we may first make a gradation of each characteristic
of different weightages by giving them a suitable rank as follows:

Location aspect:

• Rank (5) if civic amenities and services are within a shortdistance


• Rank (3) if the said amenities are at a moderatedistance
• Rank (1) if the said amenities are far
awaySizeaspect:

• Rank (5) for flat area below 70 sq.m


• Rank (3) for flat area between 70 and 120sq.m
• Rank (1) for flat area beyond 120
sq.mAgeaspect

• Rank (5) for buildings less than 7 years ofage


• Rank (3) for buildings between 7 and 25 years ofage
• Rank (1) for buildings of age more than 25 years
Specification aspect

• Rank (5) for superior-most specification


• Rank (3) for general standardspecification
• Rank (1) for poor qualityspecification
In real life situations the number of property characteristics may be more
than this. It is a task of the valuer to choose the appropriate number of
characteristics in accordance to their relevance.

The next step is to place all the comparables and their different
characteristics with rank of each one and the same of the subject property
in the evaluation grid as indicated in table.

Next, rank of each characteristic of the comparable is compared with the


rank of the same characteristic of the subject property and positive and
negative weightages are given and entered in the table.
Next step is to sum up all the weightages of all characteristics of each
comparable to arrive at the resultant figure for each comparable that will
have to be applied on sale figure of each comparable for adjustment. The
finally adjusted sale figures of the comparables are then considered for
arriving at the value of the subject property.

Problem

A residential flat is situated on the first floor of a building 5 years old. The
building abuts on a major road and is situated in close proximity of a
shopping, recreation and cultural centre. The flat is finished with best
specification with marble floor and good internal planning. Area of the flat
is 75 sq.m. Find the market value of the flat today on the basis of the
comparables given below.

Sale X: A flat measuring 130 sq.m. at first floor sold @INR40,000 per sq.m.
about 6 months back. The building is 18 years old and is situated on a
narrow lane away from civic amenities. This flat is of standard specification
of cast in situ mosaic flooring.

Sale Y: A flat measuring 100 sq.m. at first floor sold @INR35,000 per sq.m.
about a year back. This building is 20 years old and situated in a middle
class locality on a narrow lane but close to shopping complex, cultural and
recreational centre. The flat is of poor specification.

Sale Z: A flat measuring 80 sq.m. at first floor sold @INR45,000 per sq.m.
about 9 months back. The building is 12 years old and situated on the
major road surrounded by a first class income-group occupier but away
from the shopping complex, recreation and cultural centre. The flat is of
superior-most specification with marble flooring.

The evaluation grid is given below showing the adjustments

Evaluation Grid
Sl. Property Subject Property Sale X Sale Y Sale Z Remarks
No. Characteristics
1 Sale Data To be evaluated `40000/m2 `35000/m2 `45000/m2
2 Time aspect Today + 6 months +12 months +9 months
3 Adjustment for Time +6% +12% +9% Allowance 1%
+ `2400/m2 + `4200/m2 + `4050/m2 per month
4 Adjusted Rate for `42400/m2 `39200/m2 `49050/m2
time
5 Location aspect 1 3 3
Rank 5 weightage +20% +10% +10%
6 Size aspect 1 3 3
Rank 3 weightage +5% Nil Nil
7 Age aspect 3 3 3 18 – 5 =13
Rank 5 -6.5% -7.5% -3.5% 20 – 5 =15
Allowance 0.5% per 12 – 5 =07
year
8 Specification aspect 3 1 5
Rank 5 weightage 10% 20% Nil
9 Overall weightage +28.5% +22.5% +6.5%
over adjustment + `12084/m2 + `8820/m2 + `3188/m2
rate for time at item
4
10 Finally adjusted rate `52000/m2 `54484/m2 `48020/m2 `52238/m2
INR/sq m
Therefore, value per sq m. of subject flat = `52,000/m2

Therefore, value ofsubjectflat = 75 x`52,000/m2

= `39,00,000

4. Land characteristics and its effect on


LandValues
Earlier Economists Adam Smith and David Recardo talked about basic land
characteristics and theory of rent. Land was considered then as fixed supply
commodity having mainly agricultural use. These theories are now outdated. Land
can now be generated in vertical form by use of F.S.I. and T.D.R. concepts. More over
Land use is not restricted to only agricultural use. According to eastern cultures,
there are five basic elements of nature. These are land (earth) water, air, fire and
space. All civilization and cultures are developed along river banks having plenty of
fertile land and enough water. All empires were land oriented in the past due to
agricultural activities. Even after industrial revolution, demand for land never ceased,
on the contrary demand for land went on increasing. More and more control and
legislations on land in each country is the proof of this high demand. In new
millennium and even thereafter, mother land will continue to exert prime influence
on economic and cultural activities of mankind because with ever increasing
population, land will remain to be scarce even with vertical usage of land .The fact
remains that land cannot be created or produced easily like buildings and other
commodities.Land possesses innumerable characteristics. Some attributes of land
are good and it enhances value of land in the open market. Some attributes of land
are bad innature and it reduces the value of the land in the property market. Due to
these innumerable characteristics of land, estimation of land value has become very
difficult task even for an expert valuer. Out of these many characteristics of land, we
shall study only some main attributes of land and shall study how it tends to
increase or decrease value of land in real estate market. Some of these attributes are
inherent qualities of land itself where as some other attributes of land are artificial or
implanted attributes like land use laws, building bye-laws, F.S.I. norms, socialbeliefs,
tenure laws etc. In present study, we shall study only the following attributes of
land. The study is restricted mainly to non-agricultural use of land. The effects of
these attributes could be better understood, if each attribute is considered in
isolation of other attributes of land. The following are the main characteristics of
land which have effects onvaluation:

- InternalCharacteristics:
o Size: Due to more restricted market in rural areas, land having large area
should have less value than that of the small plots. The concept is
completely opposite in case of urban areas, where larger sized plots will
fetch more value than smaller sizedplots.
o Shape: Plots with a regular shape should have more value than that having
irregularones.
o Frontage: Plots with lesser frontage allotment per sq. m are considered less
advantageous than plots with higher frontage per sq.mt. This is due to
visibility aspect of the built-upspaces.
o Depth: If two plots have same area then the plot having less depth shall be
preferred than the one having more depth abutting on the sameroad.
o Directional Facing: Directional advantage adds positive value to a land
parcel.
o Soil condition: The plot having more bearing strength shall fetch more
value than a plot having poor bearingstrength.
o Topography: Flat plots shall be preferred than an undulated plot and thus
will fetch morevalue.
o Abutting road width: The plot abutting on wider roads shall be having
more value than one butting on narrowroad.
- ExternalCharacteristics
o Locational Characteristics: The proximity to a road junction, service centers,
public open space etc. will increase the value of a landparcel.
o Neighbourhood Characteristics: Planning restrictions use to decrease the
value of a land parcel. Environment considerations viz. greenery, parks
nearness of natural water bodies increases the value of a landed property.
Social and historical aspects also played a big role in determination of
value of plot of land. If it is located near to a historical monument, its value
increases.
o Transactional Characteristics: The mode of payments in case of transfer of
property plays a significant role in determination of its final value. An one
time payment by the buyer to the seller fetches discount on value and
installment payment increases the final value by adding interest.
5. Hypothetical Plotting Scheme for value
of large sizeland
This is more scientific and rational method of valuing very large size plots in
one ownership. There are certain preconditions for applicability of this scheme.

❖ If sales of large size plots in the locality are not available then and
then only this scheme can beapplied.
❖ Plot should be in developing area of town where demand for
housing site exists. It should not be in fully developed C.B.D. area of
town nor should it be in area where there is no demand for housing
plots.
❖ The plot should be of sufficiently large size area so that it could be
divided into several small size plots, similar to plots in sale
instances. Small plots made will be with access from internal layout
road of thescheme.
❖ Depth of the original plot should be considerably more as
compared to road frontage (Width). It will be seen that large size
plots outside town area have value as agricultural land rather than
potential N.A. use. Similarly large size plots in C.B.D. area will be
developed into tower structures rather than development of land
by dividing plots into small sizeplots.
In case of Sp. Land Acquisition Officer Elura, Supreme Court held –
―Valuefetchedbysaleofsmallextentlandcannotbeadoptedforlarge extent
land. Loss of land for road and park, expenses for development
should be deducted‖.

Thus plotting scheme involve planning housing sites in plots, laying


out of internal roads and other utility services in the plot and
providing requisite amenity area (Garden or Recreation Space) in
theplot.

The manner of working out values of large plot, by Hypothetical


Plotting Scheme, is given below.

❖ In a large size plot, a valuer should first prepare a hypothetical


layout, subdividing the large land into small size plots as house
sites. Such hypothetical layout (selected from many alternative
layouts) should be best possible layout with maximum area in
saleable plots after observing Development ControlRules.
❖ The internal road network and amenity space should also be
planned in the layout as per provisions of development control
rules applicable to thetown.
❖ Wasteful planning by several internal cross roads should be
avoided. Normally 15% to 20% of total plot area for internal layout
roads is considered normal but planning should be so economical
which would consume minimum area of land in theroad.
❖ Similarly amenity space (Garden) should be centrally provided so as
to benefit maximum number of plots in thelayout.
❖ Generally as per Development Control rules, 10% or 15% of the
total area is required to be provided for Garden space. Area of the
small plots (House sites) in the layout should be as per general
demand in the area. In small size towns, 1000 sq.mts. plots may be
in demand and in large size city 500 sq.mts. plot may be in demand.
It may be vice versaalso.
❖ If plots for societies are in greater demand, larger size plots will be
required. Plots for personal bungalows could be as small as 200
sq.mts. for Row houses and 400 sq.mts. for independent bungalow
sites. It all depends on local conditions and need of persons in the
concerned locality and markettrend.
❖ In this hypothetical layout, valuer may also plan a combination of
small size plots and medium size plots. Say medium size plots along
main road and small size plots ininterior.
❖ Valuer should also see that all these plots also confirm to the
standards prescribed, under D. C. rules of the local authority, for
layout and subdivisions. Having prepared a layout plan, a Valuer
should find out and consider prevalent rates of small size plots in
thelocality.
❖ Valuer should then estimate values of each of the subdivided plots
in the layout scheme, depending upon advantages and
disadvantages of each of these subdivided plots in the layout by
comparing them with lands/plots involved in instances ofsale.
❖ Plot along main road may have full value. Plots abutting on internal
layout roads may be 15% less in value than the rate of land
estimated for plots along main road.
❖ Some of the internal plots, in some cases, may even fetch same rate
as main road plots. The plots overlooking large central gardenarea
of the colony, may fetch such higher price.
❖ Generally corner plots inside the colony layout, abutting on the
colony layout roads, may or may not fetch higher rate, in spite of
return frontage.
All these rates have to be determined keeping in view general demand
and preferences given by prospective buyers of the locality, to various
internal plots in the proposed scheme. Plots over-looking garden may
or may not fetch higher rate, if demand in locality is not much or
garden is not on wind ward side but on leeward side. If topography of
large plot is uneven, internal plots in low lying plots may sale at greater
discount than plots having level ground or higher ground. All these
land characteristics have to be considered while estimating land rates
for plots in hypothetical layout. All these plots in the planned scheme
may not sale immediately. Depending upon number of plots in the
colony and general demand and competition by other developers, sale
of all plots may take 2 to 4 yearsperiod.

Investor and developer undertaking such a scheme is required to


discount his offer for land, to account for this delay in getting back his
capital investment in the land. Some plots will be sold initially and
some in last year. Hence total receivable sum is generally deferred for
half the period of anticipated total sale period. This deferment of value
is for locking in period of the capital investment in land, during which
return on investment is not likely to be fully available. This deferment
would give present worth of full receivable sum (Total realisation) by
sale of all theplots.

The Developer/investor undertaking such layout development will be


required to incur expenses for providing infrastructure amenities in
such alayout.

All these expenses must be deducted from total receivable amount


from sale of subdivided plots. Such development works can be
completed in stages slightly ahead of sale plots. The period for
completing development works can be considered, say, one year less
than the period for sale of all plots and the cost of development works
be deferred for the average period for completing the development
works.
Most of local authority insists for providing following amenities and the
valuer must assess its cost and deduct the same from total receivable
sum.

▪ To construct internal layout road of required width as pernorms.


▪ To provide road side storm water gutters to drain off rain water in
monsoon and avoid flooding in the plots and the layoutarea.
▪ To provide street lights on internal roads as per prescribednorms.
▪ To lay water mains in layout roads up to each of subdivided plots
for supply of water. 5. To lay underground sewerage mains in road
to drain waste water from each building on the plot up to municipal
sewer main laid under main road. In case of answered area, this
need not be provided as each plot will be having its own septic tank
and soak pit. If central septic tank is planned, its cost should be
considered.
▪ To lay electric cable in roads and provide sub-station for electric
supply to each plot. 7. To construct wall for garden plot and to
develop garden for the use of the residents. 8. In addition to cost
of providing above amenities, the valuer should also deduct for the
cost of Architects and Consultants fees for planning and getting
approval of layout as well as for survey and demarcation of plots on
site and execution of above stated amenities under hissupervision.
From the total amount receivable by sale of all plots, valuer should also
deduct for following expenses.

I. Developer‘s profit at 10% to 15% of total saleproceeds.

II. Interest on borrowed capital, ifany.

III. Developers own remuneration for spending his time and energy in
project. (In addition to developer‘sprofit)

IV. Advertisements and brokerage charges which may be 1% to 2% of the


sale price ofplots.

V. Sale documents charges and solicitors fees for conveyances of plots.


These fees are usually adopted at about 1% to 2% of sale price. If cost
is to be borne 50% by the purchaser, as per the prevalent trend in the
locality, lesser amount can bededucted.

VI. Registration and stamp duty charges. Normally this is borne by the
purchasersbutinsomeareas50%ofthischargeisborneby
Seller/Developer. Following example will explain how the value of
large size plot is estimated with the help of Hypothetical plotting
scheme. Plan of hypothetical plotting scheme is also shown with the
solution.

Problem

Aninvestordesirestopurchaseaplotadmeasuring150mX225msizein
out-skirts area of town. Prevalent
rate of small size plots (400 sqm
to 600 sqm plot area), in the
locality, abutting main road, is Rs.
1000/sqm. Garden area required
to be provided is 10% of total plot
area. Adopt road width at 14
meter and 10 meters. Sale of all
plots is likely to take 2 years.
Adopt road cost at Rs. 200 per
sqm, cost of laying services at Rs.
20/sqm. Architect‘s fee is @ 6%of
cost of amenities. Assume Developer‘s profit at 12% and expected rate of
return 9%. Ignore legal and stamp charges and adopt brokerage and
advertisement charges at 2% of sale price. Advise on fair purchase price of
the plot by adopting plotting scheme method of valuation.

Solution

A hypothetical layout is first prepared for the plot (vide fig.). From the
figure relevant details works out as under:

Area in m Area in m
Total Area of Plot 150 x 225 = 33750 sqm
10% Garden Area = 97 x 35 = 3395 sqm
Internal Road area as per fig = 6196 sqm
Group 'A' Plots: Plot Nos. (1) to (8) = (8 Nos)
Group 'B' Plots: Plot Nos. (9) to (26) = (18 Nos)
Group 'C' Plots: Plot Nos. (27) to (42) = (16 Nos)

In the above layout, Central garden of 97m X 35m with 3,395 sqm area is
proposed. 14m and 10m wide internal roads are also proposed. Total 42
plots are proposed which could be divided into 3 groups for value
purpose.
Group ‘A’ Plots : Along main road

8 nos. Plots each with size of = 17 m x 32 m x 8 4352 sqm

Group ‘B’ Plots: (Overlooking Garden):

12 nos. =17 m x 32 m x 12 = 6528 sqm


4 nos. = 17 m x 42 m x 4 = 2856 sqm
2 nos. = 17 m x 35 m x 2 = 1155 sqm
Total = 10539 sqm

Group ‘C’ Plots : (Last zone )

6nos. = 17 m x 32 m x 6 = 3264 sqm


2 nos.= 24 m x 32 m x 2 = 1536 sqm
4 plots= 531 sqm x 4 = 2124 sqm
4 plots= 586 sqm x 4 = 2344 sqm
Total = 9268 sqm
Total area of all 42 plots = 24159 sqm

Area of internal roads:

2nos. = 14 m x 64 mx 2 = 1792 sqm


2 nos.= 10 m x 117 mx 2 = 2340 sqm
2 nos.= 10 m x 35 mx 2 = 700 sqm
1 no.= 10 m x 101 mx 1 = 1010 sqm
2 nos.= 0.79 m x 225 mx 2 = 354 sqm
Total = 6196 sqm

(18.6% of total plots)

As plots with main road frontage have value of Rs. 1000 per sqm, rate for
‘A‘ group plots is estimated at Rs. 1000 per sqm. Rate of ‗B‘ group plots is
estimated at Rs. 900 per sqm and rate of ‗C‘ group plots is estimated at Rs.
800 per sqm. This discount of 10% and 20% is not a fixed percentage and
it could vary depending upon town, locality, demand and market trend,
evidenced by instances of sale of similar comparableplots.

(A) Income from sale ofplots:


Group Area in sqm Rate per sqm Value Rs.
A 4,352 1,000 4,352,000
B 10,539 900 9,485,100
C 9,268 800 7,414,400
Total realisation 21,251,500
As sale of plots will take about 2 years, value is differed or 1 year at 9%
yield.

P.V. = Rs. 2, 12,51,500 x .917 = Rs. 1,94,87,625/-

= SayRs.1,94,88,000/- .......................................... (A) Income fromproperty

(B) Expenditure :
Area in sqm Rate per sqm Amount Rs.
Road construction 6,196 200 1,239,200
Water supply andelectrificati 24,159 20 483,180
Garden Development 3,395 10 33,950
1,756,330
Architects fee 6% 1,756,330 105,380
Developers profit 12% 19,488,000 2,338,560
Brokarage 2% 19,488,000 389,760
Total Rs. 4,590,030

(C) Net realisable value = 19,488,000 less 4,590,030


14,897,970 say 14,898,000

Belting Method

For the purpose of valuation of sites Valuers sometimes take resort to what
isknownas―Belting‖.ThisispracticedbyValuerswhenthetaskistovalue a very
big sized plot but the comparables available in this matter are plots of
smallsizes.

Depending upon the sale transactions and nature and demand on


development around, a big sized plot may be artificially sub-divided into
several belts such as fist belt, second belt, third belt etc. – greater the
distance from the road higher being the belting position.

Generally we take 50 ft depth as a reasonable depth of first belt and so on.


Fluctuation in value between first belt and second belt may reasonably be
taken as 20%. This fluctuation depends upon many factors and requires
judgment and of the valuer. But this must not go beyond 33.5%. It must be
remembered that ―Belting‖ is not a method of valuation. It is a method of
comparison.
6. Residue Technique and other
development methods
4.6.1 The Concept of Residual Technique
The residual value can be defined as the assessment of a surplus. The
surplus is that amount which is available to pay for the land for an
economic activity after making allowances for expenses and profits. The
residual method is normally adopted in valuation for properties which
have scope for development. It may be a vacant land site for development
or land with existing structures ready for demolishing and rebuilding or for
refurbishment. Mathematically, the relationship can be expressed as
follows:

S – D – P =L

where, S = Saleproceeds

D = Cost of development

P =Profit

L = Surplus for land

Notes:

1) The technique primarily consists of looking at the property as value


to the owner rather than as a markettransaction.
2) The technique of residual valuation is applied when there is no
established market for the property and it is the valuer‘s job to
assess the value of land as a developmentsite.
3) The method is popularly used where the developer is in a position
to assess sales and costs.

4.6.2 Factors to be considered in ResidualValuation


In reality, the problem associated with the valuation procedures is not as
simple as has been made out in the example above. There are several
variables which need to be considered. These variables are discussed in
thefollowing.

1.6.2.1 Cost ofSales


When the developed property is disposed, an anticipated unit price must
be determined. This can only be done considering an anticipated sale and
an anticipated income. It may be necessary to determine this by the
investment method already discussed.
In calculating sale proceeds, very often, commission and brokerage has to
be paid out and this must be taken into consideration. Sometimes it might
be necessary to calculate the estimated sale value of the property from the
rent. This can be done by considering an appropriate year‘s purchase.

1.6.2.2 Cost ofDevelopment


The major item under this head will be the cost of building. The cost of
development, which consists of the basic major costs of building and some
furnishings, will have to be first estimated on a preliminary basis. If other
developments are required these will also have to be worked out. The
services of engineers or architects may be necessary if detailed designs are
to be developed but in a simple case the valuer can perhaps estimate the
cost if the proposal is not complicated.

There may be other expenses in case the site is already an existing


structure having tenants. Negotiating the cost of demolition with the
tenants or providing alternative accommodation to them are problems
that will have to be overcome and accounted for in thecalculations.

1.6.2.3 FinanceCharges
Arrangements for finance will have to be made regardless of the type of
development. Borrowing money from banks or other sources means
paying interest and this must be taken into consideration as anexpense.

1.6.2.4 Profits
Any developer or promoter must be adequately compensated for the risk
of operation. Profits depend upon nature of the scheme, type of
development being undertaken, risk undertaken in the development, etc.
If the period of development is a long and prolonged affair, naturally the
developer would expect a higher percentage of profit in the project. The
developer also has to pay tax which he mustconsider.

1.6.2.5 Surplus for Land


The surplus for land is calculated by considering all the factors mentioned
above. The surplus for land also is a combination of (i) land value, (ii) legal
charges including stamp duty, (iii) fiancé charge on land value and (iv) land
developer‘s profit.

1.6.2.6 Legal charges including stampduty


The cost of registration, legal fees etc. are to be considered under this
head forland.
1.6.2.7 Finance charge forland
There shall be adequate finance charges for the land to be kept idle during
development period.

1.6.2.8 Land developer‘s profit


The owner / developer of the land shall charge his profit on investment
made on purchasing the land and keeping it idle during the development
period.

Problem

A vacant site situated at class I residential area in Kolkata will be developed


as a block of residential flats in a standalone building. Estimate the residual
value of the plot from the following data:

Area of land = 4000 sq m

Sanctioned residential area to be generated = 8000 sq m

Number of Car parking to be generated = 70

Total area to be generated = 9400 sq m

Period of development = 2 years

Interest on borrowed capital = 12% per annum

Market rate of sale of residential area = `85000 per sq m

Market rate of sale of car parking space = `800000 per each

Rate of cost of construction = `21500 per sq m

Agent‘s commission = @ 2%

Professional and statutory fees = @ 6%

Legal charges including stamp duty = @ 8%

Solution

Total Sale

Residentialspace = 8000 x 85000 = `68,00,00,000

Carparkingarea = 800000x70 = `5,60,00,000

Totalsale =`73,60,00,000

Less (-)

Agent‘scommission= @ 2% = (-)`1,47,20,000
Grossdevelopmentvalue = `72,12,80,000

Less(-)

(1) Cost of construction 9400 @ `21500 = `20,21,00,000


(2) Professional and statutory fees@6% = `1,21,26,000
(3) Finance cost for 1 year @ 12% p.a. on (1) and (2)above
= `2,57,07,120

(4) Developer‘s profit @ 20% on (1), (2) and (3)above


= `4,79,86,624

Totalexpenses =`28,79,19,744

Therefore,landsurplus =`43,33,60,256

Let, theland value = `R

Legal charges including stamp duty@8% = 0.08R

Finance cost @ 12% fortwoyears = 1.08R x [(1.12)2 –1]

= 0.274R

Developer‘s profit@20% = (1.08 + 0.274)R x20%

= 0.271R

Therefore,1.625R =`43,33,60,256

Thus,R =`26,66,83,234

Therefore, residual value of the site is `66,670 per sq m.

7. Valuation for Joint Venture


Development of property
A real estate joint venture (JV) is a deal between multiple parties to work
together and compile resources to develop a real estate project. Most
large projects are financed and developed as a result of real estate joint
ventures. JVs allow real estate operators (individuals with extensive
experience managing real estate projects) to work with real estate capital
providers (entities that can supply capital for a real estate project). It also
can be between the owner of a land and an investor on the real estate
project to be developed on that land. The contribution of the land owner
shall be the land and that of the developer shall be the capital investment
to develop the project and the expertise to complete and sell out the
entireproject.
There shall not be any hard and fast rule regarding sharing of profit of a
joint venture real estate project. Therefore, the role of valuers in successful
development of a joint venture real estate is very crucial. Both the owner of
the land right as well as the investor as developer may be the potential
clients for avaluer.

Generally, the valuer has to determine the market value of the land
considering Highest and best use premise of value when there is no
approved or sanctioned plan on the questioned land. If any building plan
is already sanctioned by the local authority, the applicable premise of
value shall be existinguse.

To determine the ratio of profit share, the valuer also has to determine
total cost involvement to complete the project in all respect without
considering the land value.

Finally the valuer also has to consider the risk factor to be taken by the
developer as well as the land owner to enter into this joint venture
agreement before concluding on the profit sharing ratio.

In joint venture real estate project valuation, a valuer is supposed to


provide a range of figures and not a specific figure because the final
conclusion depends upon the negotiation power of the parties of the JV
development.
COST APPROACH TO VALUE
1. Methods of Cost Estimates forBuildings
Generally three methods are used to estimate the cost of building in India.

5.1.1 Accounts Method

This method is available only after the construction is over. This method is accurate and
possibly the best. Under the accounts method, the cost of construction of the building
is ascertained from the book of accounts maintained by the builder or contractor. This
method is helpful provided the detailed expenditures are available from the accounts
book. However, in most circumstances these expenditures are not well maintained. So,
we are to take resort to other methods of estimation ofcost.

5.1.2 Quantity Survey Method

This method is also known as ‗Detail Estimate Method‘. This gives a more accurate and
precise figure for cost involved in the construction of a building and other structures.
Under this method, the detailed quantities of all the components/civil work items are
measured and worked out. Different components may require different units for
working out the quantities. E.g. the foundation trenches for excavation are measured in
cubic metres. RCC columns, brick works in plinth, RCC slabs, RCC beams, RCC
sunshades etc. are worked out in cubic metres whereas brick soling in plinth, tiled
flooring, plastering, steel-glazed windows etc. are worked out in square metres. The
quantities under different items of work are multiplied by their respective rate found by
analysis of rates. This gives us the amount against each and every item of works.
Adding all the amounts against their respective item of works gives rise to the total cost
of the structure under consideration. This cost also involves the cost of tools and
tackles, water charges as well as miscellaneous expenses on either percentage or lump
sumbasis.

5.1.3 Estimate by Plinth Area or Covered Area

This method involves calculation of the plinth area or covered area (or as per modern
trends, the ‗super-built area‘). An appropriate unit rate is then applied. The unit rate
depends on factors such as the type of construction and fittings installed and other
factors.

5.1.4 Estimate by Considering Cubical Contents

This method, instead of considering areas as in the previous case, takes the volume into
account.

5.1.3 Building cost indices method


A certain weightage is given to each building component. For example, steel, bricks
and mason‘s wages. A construction index is then constructed for a particular place for
a specific date by comparing with another or same place, evaluated on a differentdate.

Normally, government organizations, like the CPWD publish indices for Delhi and other
places with Delhi as base for January 1, 1970 or October 1, 1976 or January 1, 1992 for
each year. The index so obtained is then multiplied by the plinth area constructed.
CPWD in the past years have published plinth-area rates for different base years each
times with base index as 100. These years are 1955, 1970, 1976, 1992, 2017 and 2012.

There is sufficient scope in the method to increase or decrease the cost if there is
deviation from specifications. Further details on this method are given in the Chapter
titled ‗Building Costs and Index Numbers‘.

2. Life of Building: Economic/Physical/Legal; Factors


affecting life of thebuilding

5.1.5 Life of Building:Economic/Physical/Legal


5.1.5.1 Economic Life
The economic life is how long it is anticipated that the asset could generate financial
returns or provide a non-financial benefit in its current use. Or it can be expressed as
how long the asset can be used before repairing it becomes more expensive than
replacing it. It will be influenced by the degree of functional or economic obsolescence
to which the asset is exposed.

The Remaining Economic Life of an improvement is the total economic life minus its
effective age.

5.1.5.2 Effective Age


The Effective Age is the estimate of the age of a structure based on its utility and
physical wear and tear. The effective age is the difference between the Economic Life of
the structure and the Remaining Economic Life of the structure. The effective age may
be the same as, more than, or less than the actual age.

5.1.5.3 Actual Age


The Actual Age is the number of years that have passed since the structure was built.
Actual age is also called the Chronological Age.

5.1.5.4 Physical Life


The physical life is how long the asset could be used before it would be worn out or
beyond economic repair, assuming routine maintenance but disregarding any potential
for refurbishment or reconstruction.

5.1.5.5 Legal Life


Duration of an intangible asset (such as a patent or copyright) as allowed in law. In
cases where the economic life is shorter than the legal life of the asset, the former is
used for computingamortization.

5.1.6 Factors affecting life of the building


The following factors affect the life of buildings:

• The quality of the originalconstruction


• The environment in which the building islocated
• The quality and degree of maintenance carriedout
• Effects of Atmosphericconditions
o Moisture
o Heat
o Settlement
o Chemicals
o Biological

5.1.6.1 The quality of the original construction


✓ Condition of foundation - If the foundation has settled then the future life
is considerablyreduced.
✓ Type of construction material used - If the construction material is of
superior quality the life of the building may beprolonged.
✓ If cracks have developed, the nature of the cracks should be studied and
the possible effect on the future life should beascertained.
✓ The condition of wooden parts, where used as structural members, if
decaying, has an adverse effect on the life of thebuilding.
5.1.6.2 The environment in which the building is located
✓ The environment a building is subjected to will vary from external elements
to internal elements and also from seaward side to landward side (if it is
near the coast). The life of the buildings located at the seaward side will be
less than the buildings located at the landwardside.
✓ Whether the building is subject to any external forces such as passing of
the railway lines close by or effect of any vibration in the vicinity of the
building, etc. may also affect the life of a building. If the design of the
building is conceived where there already exists a railway line or road, the
designer must have considered the effect of vibration caused due to
passing by vehicles or trains. Therefore, in these cases, there shall be no
effect of railway track or roads. But, if the railway track has been laid after
the construction of the building, it may have an adverse effect on the life
of thebuilding.

5.1.6.3 The quality and degree of maintenance carried out


✓ The life of a building mostly depends on the condition of repairs and
whether maintenance is periodically done or not. A proper and periodic
maintenance of building increases the life of a building many fold.
Similarly, poor maintenance can drastically reduce the life of a building.
The general condition of the building also should be seen from all major
aspects to estimate the effect of maintenance and repair. Different building
elements may receive different degrees of maintenance, depending on
theirinspectability

5.1.6.4 Effects of Atmospheric conditions


✓ Moisture: This is the most common cause for deterioration. Buildings can
experience moisture from external sources (e.g. rainwater) as well as
internal sources (e.g. toilet areas, leaks from pipes and condensation in air
conditioning systems). Moisture in buildings can also impair electrical
systems, thus compromisingserviceability.
✓ Heat: Heat will accelerate all deterioration processes. In addition, heat can
cause expansion (and subsequent contraction when the heat source is
absent). Such thermal movements can weaken materials with low tensile
strengths such as masonry, and cause cracking. Heat (especially in
combination with direct solar radiation) can also weakensome
waterproofing materials, and cause them to lose their flexibility or even to
crack.
✓ Settlement: The settlement of building will also affect mainly masonry
walls. In addition, if pipes are damaged during settlement, leakage of
water will ensue, with the consequent potential fordeterioration.
✓ Chemicals: Atmospheric carbon dioxide reduces the alkalinity of concrete
and will lead to depassivation of steel reinforcement. Chlorides (the main
source of which is from sea spray near the coastline) will also lead to such
reduction in alkalinity, and also promote electrolytic corrosion processes in
both reinforced concrete and steel. Sulphates (which are found in some
ground waters) can attack the concrete itself, causing cracking and
weakening in foundations. Sulphates and chlorides can also get into
concrete through impure mixingwater.
✓ Biological: Deterioration of timber is mainly a biological process. In
particular, termite attack can be very damaging. If mosses are allowed to
grow on damp building elements, they will trap further moisture, thus
accelerating the deterioration processes associated with moisture (see
above). Apart from this, if plants are allowed to take root in buildings, they
can cause severe cracks, not only in masonry, but also inconcrete.

3. Total Life, Age, Estimating Future Life


5.1.7 TotalLife
Total life of a structure in valuation parlance is also known as Economic Life. There is no
fixed rule for estimating the lives of various types of structures which depend upon
many factors such as quality of construction, maintenance, etc. The following Table will
give an approximate life span (Economic Life) of various types of structures commonly
in use in India for solution of different valuation problems by the registeredvaluers.

Typesofstructures Lifespan

dstructure 100years

edstructure 75years

gconstruction 55years

nentstructures 30years

orarystructures 5years
5.1.8 Age
Age of structures has been elaborately dealt in 2.2.2 and 2.2.3.

5.1.9 Estimating Future Life


For estimating the future life of a building a thorough inspection of the property by a
structural engineer is necessary, since no mathematical equation or mechanical device
is available for determining future life. Structural engineer may conduct a non-
destructive tests e.g. Hammer Test, to have an idea about the stability of the building
and then issues a future life certificate of the building. If a particular structure has been
constructed in two parts and the condition of the structures is different, then different
future lives may be estimated for each portion. Again, if a different foundation is used
for upper floors when constructing on an old existing building, then too, different
future lives, i.e., depreciation, may be adopted. The structural engineer will estimate and
certify the future economic life of the structure and the valuer shall determine the
effective age of the structure on the basis of that structural engineer‘s certificate
following ‗Total Life Table‘ as provided in2.4.1.

4. Various methods of Computation of Depreciation,


Functional, Technological and Economic
Obsolescence

Depreciation is called as a measure of wearing out, consumption or other loss of value


of a depreciable asset arising from use, efflux of time or obsolescence through
technology and market changes

5.1.10 Causes of Depreciation


There are in general four causes of accrued depreciation viz. physical
deterioration, functional obsolescence, economic obsolescence and technical
obsolescence.

5.1.10.1 PhysicalDeterioration
Physical deterioration is the loss in value due to wear and tear and natural
forces. Practically every structure suffers natural physical decay. This may be
due to tension, compression, friction and chemical changes in the materials
used in construction. Some causes of physical deterioration are normal use,
breakage, neglect, infestation of insects, moisture and the elements.
Maintenance can slow physical deterioration but not completely arrestit.

Physical deterioration may be classified as curable or incurable. Curable


physical deterioration occurs when the value added by a repair equals or
exceeds the cost of repair. A prudent property owner would make such
repairs, which correct conditions caused by delayed maintenance.Examples
include leaky plumbing, broken windows, cracked paint or plaster, a leaking
roof, etc.

Incurable physical deterioration is that which, as of the date of valuation is not


economical to repair or replace; that is, the cost of repair exceeds the gain in
value. Physical components that are not easily seen, such as the structural
framework, foundation, subflooring and ceiling structures, suffer from incurable
physical deterioration.

The classification of depreciation as curable or incurable will vary with the age
and location of properties. Extensive renovations may be advisable in an
improving neighbourhood but not in a declining one.

5.1.10.2 Functional Obsolescence


Functional obsolescence is loss in value due to inability of the structure to
perform adequately for the function for which it is used, as of the valuation
date. Functional obsolescence can result from changes in demand, design and
technology. It can take the form of deficiency like having only one bathroom
when two are required or need for modernization like an outmoded kitchen,
etc. In any case, potential buyers perceive a loss in utility. Therefore, the price
offered may be lower due to reduced demand.

Functional obsolescence can also be classified as either curable or incurable,


depending on whether the cost to cure is economically justifiable or not as of
the valuation date. Example of curable functional obsolescence include old-
fashioned bathroom and kitchen fixtures, an outdated hot water heater, fewer
electrical outlets than required per room, low-hanging pipes in commercial or
industrial buildings, etc. In all these examples, the increase in value from
correcting the problem usually exceeds the cost.

Incurable functional obsolescence occurs when the cost of correcting the


condition exceeds the increase in value. Examples include outmoded design,
poor room arrangement, no garage (and no space to build one), inadequate
column spacing in a warehouse and inadequate frontage in a commercial
structure.

Sometimes an entire structure can be functional obsolete because of its


location; for example, a large, custom-built house in a remotely located
neighbourhood or a small, poor-quality house in a high-priced
neighbourhood.
5.1.10.3 Economic Obsolescence
Economic obsolescence is loss in value resulting from impairment in utility and
desirability caused by external factors i.e. outside the property‘s boundaries.
Economic obsolescence often arises from changes in the highest and best use
of the property due to market shifts or rezoning government policies. It may
be the result of inadequate public services, lack of adequate facilities for trade
in a business district, narrow streets and heavy traffic in a residential
neighbourhood, or proximity to inharmonious industrial or commercial land
use.

Economic obsolescence is seldom, if ever, curable. An undesirable location can


affect land as well as improvement values. The effects should therefore be
separated.

5.1.10.4 Technological Obsolescence


Old load bearing structures with thick walls are not preferred now in the city
areas. Every one now desires to stay in high rise R.C.C. framed structures having
thin partition and external walls. This is now possible due to technological
advancements. Timber structures are also now replaced by R.C.C. framed or
steel framed constructions. Wooden windows are replaced by aluminium
windows. Now modern technologies and planning concepts have made it
possible to design and erect even an intelligent building.

5.1.11 Methods of Calculating Depreciation


There are several methods of a calculating depreciation. Some of these are
arbitrary whereas others are based on theory. The methods have been divided
into two broad heads:

• Non interest methods


• Methods based on interesttheories
5.1.11.1 Non interest methods
✓ Good-as-new assumption
The good-as-new assumption may be stated as follows:

Property units which are so well maintained that their service efficiency and
output capacity are practically undiminished are assumed as good as new.

Obviously, this assumption is entirely erroneous and has no basis.

✓ Direct appraisal method


The direct appraisal depreciation method, which is also commonly used is also
unwarranted. It may be stated as follows:
It is sometimes assumed that an expert valuer‘s intuition can decide arbitrarily,
by merely inspecting a property, how much it has depreciated in value without
applying any particular process of reasoning or analysis or without reference to
any cost data applying to the property.

This method, too, is arbitrary having no basis and perhaps will not stand to
close scrutiny in a court of law.

✓ Arbitrary lump sum method


Although now practically abandoned, many enterprises make arbitrary lump
sum allocations as expense for depreciation. This method also has no basis.

Depreciation as a percentage of revenue Estimating cost depreciation as a


percentage of revenue involves the same motive as found in the arbitrary lump
sum method. The percentage of gross revenue method seems not to have
been used much.

✓ Sum of digits method


This method is also known as the sum of the years‘ digits method. In this
method, the sum of the arithmetic series of numbers from 1 to n is used, n
represents the probable life of a unit, as the denominator of a series of
fractions. The numerator of the fraction for any specific period is the
expectancy of life as of that period. If six years is the estimated probable
service life of a unit, the denominator of the fractions is 1 + 2 + 3 + 4 + 5 + 6
or 21. The depreciation allocations for the first to the sixth years are found by
multiplying the depreciable cost by the series of fractions; 6/21, 5/21, 4/21,
3/21, 2/21, and 1/21. Total depreciation over a life span of six years =
6/21+5/21+4/21+3/21+2/21+1/21 = 21/21 or100%.

The sum of the years‘ digits method is an arbitrary method of allocation in


which the depreciation base is held constant and the yearly rate decreased. As
with the declining balance method, the result is to allocate the larger amount
of depreciation to the first year and to decrease the amount each succeeding
year. The method has not been used to any great extent, because it is difficult
to apply to groups of units, and because the declining balance method
achieves similar results with greater ease andflexibility.

✓ Declining balance method


This method is also known as ‗constant or equal percentage‘, or ‗written down
value‘ method of depreciation.
The method is a variation of the fixed percentage depreciation idea, in which
the fixed percentage is applied to the unallocated balance of the base instead
of to the cost of the new base. In results, the declining balance method closely
parallels the sum of the digits method.

When the declining balance method is applied to a single property unit, the
annual depreciation is always a fixed percentage of the unit‘s unallocated base
at the beginning of the year.

Let, n = Expected life of the structure in years.

P = Constant percent by which the structure loses its value each year. C

= Value at the beginning(new).

F = Value at the end of life (final or scrap value), i.e., at the end of n
years.

m = Number of years after which value is to be found. V m

= Value after myears.

Value of the property at the end of the first year or beginning of the second
year

= C – CP = C(1 – P)

Value of the property at the end of the second year or beginning of the third
year

= C(1 – P) – C(1 – P)P

= (1 – P) (C –CP)

= (1 – P) C(1 –P)

= C(1 –P)2

Continuing in this way, value of the property at the end of nth year or
beginning of (n + 1)thyear

= C(1 –P)n

F= C(1 –P)n
F n1
P = 1–
 
C

If value after, say, m years is required, it can be obtained by using the


expression:
Vm = C(1 –P)m
Vmm 1
P = 1−
 
C

From above two Equations


1 1
F n Vmm
 =  
C C

If the life of the property and the scrap and new values are known then value at
any date can be calculated from the above expression. However, this
expression will fail if final value F is zero after nyears.

✓ Straight line depreciation method


The declining balance and sum of the years‘ digits methods distribute the
depreciation more heavily in the early life of the unit than in the latter periods.
The straight line depreciation method allocates the depreciable base of a
property unit uniformly throughout its service life except when the estimate of
service life is changed.

This fixed percentage depreciation method is more widely used in depreciation


calculations than any other. It is the one method most generally used for
determining depreciation for tax purposes and for profit and loss financial
statements. It is the method prescribed by most agencies.

Let n = Life of the property, inyears

C = Original cost or value at the beginning

F = Final value at the end of usefullife

D = Depreciation

C−F
Then, D =
n

5.1.11.2 Methods based on interest theory


✓ Sinking fund method
It will be recalled that the sinking fund is an amount set aside each year and
invested at compound interest so that after a specified period one gets
predetermined value of asset less salvage. The sinking fund amount goes on
accumulating at compound interest and is meant to meet the future liabilities
of replacement. The difference in sinking fund and depreciation should be
noted. Sinking fund allows for recoupment of capital but in depreciation there
is no such provision.
R
The sinking fundequation S=
(1+R) n−1

where, R = Sinking fund rate

n = Total life of the structure and the amount per


annum,

(1+R) m−1
A=
R

5. Reproduction Cost/Replacement cost

5.1.12 The Replacement Cost Method


The idea under the replacement cost method is to compare the subject property with a
modern equivalent asset offering similar utility and function as the subject property but
which is of current design using current cost effective materials and specification.

So, the first step is to select a modern equivalent building with attributes indicated in
the above paragraph. The cost of construction of such an asset offering equivalent
utility as the subject asset is worked out. In selecting the modern equivalent building
the valuer must see that it is the utility of the subject asset that is to be replicated but
not the physical features of the asset to be valued.

The second step is to find out the physical deterioration, functional and economic
obsolescence that the subject property has suffered. The total deduction on all these
aspects is to be quantified.

Finally, total amount of deduction thus quantified on account of physical deterioration


and obsolescence is deducted from the cost of construction of the modern equivalent
building. The result is the depreciated replacement cost of the building.

5.1.13 The Reproduction Cost Method


The reproduction cost is that cost which is involved in constructing the replica of the
subject property. The yardstick building in this case has the same physical feature as
the building to be valued. The instances where the reproduction cost is to be used as
the yardstick are as follows as prescribed by theIVSC.

When the cost of constructing a modern equivalent building (that is a building with
equivalent utility) is higher than the reproduction cost of the subject building, then the
reproduction cost should be the yardstick building for ascertaining the basic cost.
When the utility presented by the property to be valued can only be achieved by a
replica only and cost of a modern equivalent building is not required.

Most instances in the latter case (that is case number (ii) in the preceding paragraph)
are likely to be buildings of recent construction where modern design, current building
materials and specifications have beenmaintained.

So, the steps to be followed in the case of reproduction cost approach should be as
follows:

First of all, the cost related to the construction of an exact replica of the building to be
valued is worked out. Next, the extent of physical deterioration and obsolescence (that
is functional and economic obsolescence) are to be determined and quantified.

Finally, the total depreciation on account of the obsolescence indicated in the


preceding paragraph are to be deducted from the basic cost of the building arrived at
as above. The result is the depreciated replacement cost (DRC) of the subjectbuilding.

6. Depreciated Replacement Cost (DRC) working,


adopting DRC as Value subject to Demand and
Supply aspect
Example - 1

Calculate the depreciated replacement cost of a building having a replacement cost of


Rs. 2,50,000 and economic life of 100 years. The present age of the structure is 40
years. The salvage value may be taken as 10% of the value at the end of its economic
life.

Solution

C−F 2,50,000−0.1(2,50,000)
D = Depreciationperannum = =
n 100

= Rs. 2,250

Totaldepreciation = 2,250 ×40

= Rs.90,000

Depreciated value after40years = 2,50,000 –90,000

= Rs. 1,60,000
7. LAND VALUE BY MARKET APPROACH AND
BUILDING VALUE BY COST ESTIMATION METHOD FOR
OWNER OCCUPIED BUNGALOWS, FACTORIES,
PUBLICBUILDINGS

5.1.14 Land Value by Market Approach


Land Value by market approach is elaborately discussed in Chapter III ‗Market
Approach‘.

5.1.15 Building Value by Cost Estimation Method


Building value by cost estimation method is elaborately discussed in 2.1 of this
chapter. Moreover, the cost elements may differ depending on the type of the
asset and should include the direct and indirect costs that would be required to
replace/ recreate the asset as of the valuation date. Some common items to
consider include:

✓ Direct costs:
o materials,and
o labour.
✓ Indirect costs:
o transportcosts,
o installationcosts,
o professional fees (design, permit, architectural, legal,etc),
o other fees (commissions,etc),
o overheads,
o taxes,
o finance costs (eg, interest on debt financing),and
o profit margin/entrepreneurial profit to the creator of the asset (e.g.
return toinvestors).
An asset acquired from a third party would presumably reflect their costs
associated with creating the asset as well as some form of profit margin to
provide a return on their investment. As such, under bases of value that
assume a hypothetical transaction, it may be appropriate to include an
assumed profit margin on certain costs which can be expressed as a target
profit, either a lump sum or a percentage return on cost or value. However,
financing costs, if included, may already reflect participants‘ required return on
capital deployed, so valuers should be cautious when including both financing
costs and profit margins.
When costs are derived from actual, quoted or estimated prices by third party
suppliers or contractors, these costs will already include a third parties‘ desired
level of profit.
The actual costs incurred in creating the subject asset (or a comparable
reference asset) may be available and provide a relevant indicator of the cost
of the asset. However, adjustments may need to be made to reflect the
following:

(i) cost fluctuations between the date on which this cost was incurred and
the valuation date,and
(ii) any atypical or exceptional costs, or savings, that are reflected in the
cost data but that would not arise in creating anequivalent.
VARIOUS PURPOSE OF VALUATION
1. Valuation of properties for purposes such
as: Bank Finance, Auction Reserve,
Building Insurance, Sale, Purchase,
Valuation Disputes in Court,
Probate,Partition,

6.1.1 Bank Finance


Banks and financial institutions can provide finance to general public, or to
other borrowers in need of money, in the form of loans, secured by
mortgages on immovable property. This purpose is termed by International
Valuation Standard (IVS) as ‗Secured Lending‘.

6.1.1.1 Nature of Mortgage


Where immovable property is held as freehold or on a long-term lease,
a mortgage can be executed whereby one party known as the
mortgagor (the borrower) grants an interest in his property to the other
party, i.e., the mortgagee (the lender). The mortgagor normally agrees
to pay interest on the loan and also agrees to pay the principal amount
besides conferring several powers on the mortgagee to sell the
immovable property in case of default. There may also be several other
specific covenants regarding repairs, insurance, etc. The mortgagor, on
the other hand, retains his right to clear the amount due and free the
charge. This is known as equity ofredemption.

A legal mortgage may take the form of a conveyance of the property by


the owner (mortgagor) to the person or institution (mortgagee) making
the loan with the specific provision in the conveyance for redemption
upon repayment of all amounts returnable. The deed of conveyance
usually provides for repayment of the principal sum secured and confers
on the mortgagee various powers–the most important being a power to
sell the land in case of default. A ‗legal mortgage‘ is, therefore, a
transfer of property or a conveyance executed by the borrower in order
to obtain a loan.

On the other hand, an ‗equitable mortgage‘ is effected where a


mortgagor executes a mortgage in a prescribed form or by a written
agreement, duly registered with the Registrar acknowledging the loan.
This type of mortgage gives security to the mortgagee but does not
operate as a transfer of the property charged. However, this makes
little difference because the mortgagee has powers and remedies for
recovering his dues. This form of mortgage involves less expense by
way of registration,etc.

A mortgage is generally discharged by a memorandum endorsed


thereon and registered. In all dealings by way of mortgage or discharge
a ‗certificate of title‘ must be produced before the Registrar in order
that the proper notations may bemade.

6.1.1.2 Suitability of Property as Security


The mortgagee, whether a bank or a private lending institution, is
primarily interested in knowing the capacity of the mortgagor to repay
the loan, i.e., the investment potential of the property mortgaged or if
the property mortgaged is ultimately to be sold, then the probable
price it would fetch at the time of sale. Normally, only an advance of
about two-thirds the estimated fair market value of the property is
made to the mortgagor by the mortgagee. However, if the investmentis
thought to be risky by the mortgagee then he may advance only 50 to
60 per cent of the fair marketvalue.

6.1.1.3 The Valuation Process


In making out the valuation, generally the ordinary principles of
valuation will apply. However, for mortgage purposes, a valuer should
keep in mind an essential criterion that if the mortgagor fails in his
commitment, the mortgagee may decide to sell the property.
Therefore, adequate care should be taken to ensure that the market
value obtained by the valuer is fair and reasonable and the mortgagee
has sufficient facts and information about the property.

A valuation report should include adequate description of the property


in question, its condition of repairs, maintenance and other relevant

details. Depending on the nature of the property, details may include


permitted use, present and potential use, rents receivable, taxes, etc.

Valuation is primarily done of the basis of market transactions of


comparable properties in accordance with established principles. There
is no precise method of doing this and sometimes data collected may
be unreliable or perplexing. A valuer, therefore, must fully utilize his
ability and judgement when determining value for mortgage purposes.
Again, market value as determined by the valuer would normally have a
short-term duration only because of fluctuations in the property market.
A lender might, therefore, be particularly interested in knowing whether
the value is likely to be maintained in the future and if the amount of his
outstandings would be realizable if the property was to be disposed on
a forced salebasis.

A lender would seek the valuer‘s opinion on the following:

• The future outlook of the property mortgaged. In case the


borrower is an industrial concern, by the outlook of the industry
in general and the nature of security being provided by the
borrower.
• Is the present fair market value, as determined today, likely to be
maintained or are there circumstances which may effect its fall or
rise?
• During the period of loan, can the market value be expected to
hold? In case of a forced sale, would the property be easily
saleable?
• What would be a reasonable percentage of advance he could
safely make considering all thecircumstances?

6.1.2 Auction reserve

6.1.2.1 Reserve Auction


This is an auction where the final bid for an item can be rejected by the
seller if it is not high enough to satisfy them. They may set a particular
fixed reserve, or they may alter the reserve over the course of the
auction in response to the bids placed. Bidders are often unaware of the
reserve price.

6.1.2.2 No reserve auction/absolute auction


This is an auction where the seller sets no reserve price. This is usually
done in the hope of attracting large numbers of bidders hoping for a
bargain. If this works, the item may end up selling for a higher price
than if it had had areserve.

6.1.2.3 Reserve Price


Lowest fixed price at which an item is offered at an auction sale and (1)
at which it will be sold if no higher price is bid, or (2) below which the
seller is not obligated to accept the winning bid is called as Reserve
Price.

Valuation for fixing reserve price is asked by:

1) Customs and ExciseDepartment

2) Liquidation Officers ofCourt

3) Port / HarbourAuthorityetc.

6.1.2.4 Valuation process to fix reserve price


For the purpose of fixing reserve price in an auction, valuer has to
estimate the market value of the property to be auctioned as if it is an
ordinary sale. But, that estimated market value shall be the market value
which could be achieved satisfying all the conditions of market value
definition of International Valuation Standard. Generally the bidders in
an auction are investors who wish to buy the auctioned property for
selling in the secondary property market. Therefore, they always bid
keeping a margin of a minimum 28% which includes profit, legal cost as
well as cost of stamp duty and registration in their own names. Thus, if
the reserve sale price is kept anything more than 70% of the market
value estimated for the purpose of sale, there is every possibility of
failure of that auction. So, the reserve sale price in an auction should be
at least 28% less than its market value for the purpose of sale.

6.1.3 Building Insurance

6.1.3.1 Introduction
Insurance policies enable property owners to reduce or eliminate certain
measurable risks of financial loss through a system of equitable
contribution called a premium. The insurance distributes the risk of loss
over a large number of other insured parties. This, in effect, replaces
uncertainty with certainty in respect of losses affecting the property or
the liability of the insured persons.

In the earlier days insurance was confined to three types only, i.e.,
marine, life and fire. Nowadays, however, one can insure against many
of the perils that are likely to affect human activity. For example, an
insurance company will even insure someone‘s voice or hair. A valuer is
normally concerned with fire insurance of property. Insurance against
additional perils such as flood, earthquakes, etc. may also be endorsed
on the insurance policy.

6.1.3.2 Fire Insurance


Fire insurance is a contract of indemnity. This principle of indemnity
simply means that in case of damage or loss, the insured property is
again restored to the same or as nearly the same practicable condition
as if the loss had not occurred at all. However, this is not always
practical. In some cases the property is rebuilt completely with a new
design or renovated or simply repaired with a few alterations.Therefore,
what exactly is to be assessed is the exact replacement cost required to
theoretically put the property back in exactly the same condition as it
was before the loss.

In an ordinary policy of fire insurance the insurer is not normally


required to actually reinstate the property. Under the contract, the
insurer‘s obligations are over once payment for loss suffered by the
insured is discharged.

If a building or other structure is in new condition and adequately


insured, the amount payable in the event of total loss should enable the
insured to re-erect the structure. If not in new condition, the amount
payable under the policy would be based on the cost of construction at
the time of the loss from which a reasonable allowance for accrued
physical depreciation would be deducted.

It is not practicable for an insurance company to make a valuation of


every property which it insures. The responsibility, therefore, rests on
the insured to state the sum to be insured and to adjust the sum
insured periodically to cover fluctuations in the cost ofconstruction.

Correct insurance cover is important. Over insurance may prove costly


in wasted premium; underinsurance is risky and might result in heavy
financial loss to the insured. Generally, the insurance should be based
on replacement cost less depreciation. The depreciation would, of
course, depend on the type of structure, condition of repairs and other
such factors. Insurable value, although not so described in the policy,
may be regarded as the amount of the insurance based on the value
which could be ascribed to destructible property in order to indemnify
the insured in the event of loss.

6.1.3.3 Method of Assessment


In the case of loss, total or partial, one of the best ways to estimate the
replacement cost is by making a detailed estimate as per current rates
and then deducting for depreciation after considering age, state of
repairs, obsolescence, future life, etc.

An assessment for salvage value should also be made. After the insured
party has been paid, the insurer is entitled to the salvaged goods and
can dispose these as he likes. Sometimes it happens that the insured
party agrees to take these salvaged goods and reuse them in
reconstruction. Then the value of these salvaged goods is deductible
from the claim settled by the insurance company.

6.1.3.4 Adjustment under Average Clause


All insurance policies have what is known as an ‗average clause‘.
Adjustments by the insurance company are invariably made for
underinsurance using this clause. If after calculations it is found that the
risk was not actually fully covered as declared by the insured then the
amount payable to the insured by the insurer is reduced
proportionately. The general relationship is asfollows:

Sum insured
Amount payable = Assessed loss 
Risk inusrable
6.1.3.5 Extra Cost Reinstatement Policy
Very often after receiving payment for loss under indemnity provisions, the
insured might discover that insurance money is insufficient to replace the
property destroyed. This may be because conditions or rules under local
laws have changed for repair or erection of new buildings. Under normal
conditions it would not be possible to recover the additional cost because
reinstatement means replacement of the building by a similar building in a
condition equal to but not better than the actual which existed. Sometimes
the insurance companies also provide such extra cost replacement policies

6.1.4 Sale,Purchase

A sale is a transaction between two parties where the buyer receives


goods (tangible or intangible), services, and/or assets in exchange for
money. It can also refer to an agreement between a buyer and seller on

the price of a security. A sale functions as a contract between the buyer


and seller of the selected good or service.

A purchase means to take possession of a given asset, property, item or


right by paying a predetermined amount of money for the transaction
to be completed successfully. In other words, it is an exchange of
money for a particular good or service.

The Valuation for the purpose of sale / purchase is called as ‗supporting


transaction decision‘ by IVS

The Value for the purpose of sale / purchase can be estimated by the
application of all the three approaches i.e.

1) CostApproach

2) MarketApproach

3) IncomeApproach

6.1.5 Valuation Disputes in Court

Thispurposecanbetermedas―ValuationReview‖.

When there is a major difference between the values of a property


submitted by two different valuers, the Court uses to engage a third
valuer to review the reports and to give an expert opinion.

According to IVS, a valuation review report shall contain a minimum of


the following:

(a) the scope of the review performed, including the elements noted in
para 20.3 of IVS 101 Scope of Work to the extent each is applicable to
theassignment,
(b) the valuation report being reviewed and the inputs and assumptions
upon which that valuation wasbased,

(c) the reviewer‘s conclusions about the work under review, including
supporting reasons,and

(d) the date of the report (which may differ from the valuationdate).

6.1.6 Probate

Probate is a copy of a will certified under the seal of a court of


competent jurisdiction with a grant of administration to the estate of
the testator. Value of probate is to determine for the purpose of stamp
duty.A probate is issued for a will or any codicil attached, by persons of
Indian Hindu, Parsis, Buddhists, Sikhs or Jains, primarily in the cities of
Chennai, Kolkata and Mumbai.

The market value of the probate is to be determined by a surveyor-


valuer of the court of competent jurisdiction and court fee has to be
paid on that to a maximum of INR 1,00,000 (e.g. Calcutta High Court).

6.1.7 Partition

Very often a property or properties jointly held by several owners have


to be divided according to agreed predetermined shares. This is
common when family members jointly inherit an ancestral property.
Partition must not result in loss in intrinsic value, a partition unit must
have legal title more so, the subdivision by mats and bounds in partition
suit must be approvable by the competent authority. If it appears to the
court that by any reason a division of the property cannot bereasonably
or conveniently be made, direct a sale of the property and a distribution
of the proceeds. Properties, especially buildings or structures are
difficult to partition physically because of the peculiar construction that
may exist. Therefore, it sometimes becomes necessary to partition the
properties in such a manner that the natural boundaries such as walls or
some other features fall into one particular portion. Partitioning
according to the exact predetermined shares is therefore difficult. In
order to equalize the assets value according to predetermined exact
shares there has to be what is known as owelty money exchanged
between the concerned parties. When owelty money is a consideration
and the property is to be partitioned according to predetermined
shares, the basis of valuation should always be the market value.
However, sometimes the concerned parties agree to forego owelty
money and accept partition on mutually agreeable terms. In such a case
they may also agree to a valuation figure which is much lower than the
market value such as that arrived at for municipal rating or for valuation
made for otherpurposes.By measurement and boundaries, partition by
metes and bounds is the division of subject-matter of partition in to
demarcated areas to be held severality. The terms of reference to the
valuer is partition by metes and bounds which means partible estate
must be capable of being enjoyed in severality and if no severality
means the definition of metes and bounds will remain incomplete. Now
severelity means separate or held as separate and independent units
capable of being alienated.

It may also happen that a certain construction is to be made within a


partitioned lot to make it habitable and independent, such as adding a
staircase or laying a drain in a particular lot or laying new water lines or
electrical lines in another lot. These works should normally be carried
out either at the expense of the estate or if they are to be carried out by
the allottee concerned then proper adjustment should be made for
such works in the valuation of eachlot.

It has been found by experience that valuations made for partition


deeds are generally done at figures lower than the market value. This is,
perhaps, because the allottees try to avoid stamp duty or court fees. An
observation to this effect was made by the Calcutta High Court in the
case of Sushil Chandra Ghose & Another vs. Satish Chandra Ghose &
Another in A.O.D. No. 459 of 1928 (unreported). In this case the
Lordships(Mukherjee&GuhaJJ)observedthat―thevaluationwasmadenot for
ascertaining the real value but only to equalize the shares in a case of
partition between the co-owners who must have been interested in
keeping down the value to avoid payment of higher courtfees‖.

2. Study of Valuation Standards as per the


provisions of the Companies Act2013
6.1.8 International Valuation Standards Council
The International Valuation Standards Council (IVSC) is an independent,
not-for-profit organisation committed to advancing quality in the
valuation profession. The primary objective of IVSC is to build
confidence and public trust in valuation by producing standards and
securing their universal adoption and implementation for the valuation
of assets across the world. The IVSC also promotes leading practice
approaches for the conduct and competency of professional valuers.

6.1.9 International Valuation Standards


Valuations are widely used and relied upon in financial and other
markets, whether for inclusion in financial statements, for regulatory
compliance or to support secured lending and transactional activity. The
International Valuation Standards (IVS) are standards for undertaking
valuation assignments using generally recognised concepts and
principles that promote transparency and consistency in valuation
practice. The latest edition of International Valuation Standards was
published in 2017.

The IVS consist of mandatory requirements that must be followed in


order to state that a valuation was performed in compliance with the
IVS. Certain aspects of the standards do not direct or mandate any
particular course of action, but provide fundamental principles and
concepts that must be considered in undertaking a valuation.

The IVS are arranged as follows:

❖ The IVSFramework
This serves as a preamble to the IVS. The IVS Framework consists of
general principles for valuers following the IVS regarding objectivity,
judgement, competence and acceptable departures from the IVS.

❖ IVS GeneralStandards
These set forth requirements for the conduct of all valuation
assignments including establishing the terms of a valuation
engagement, bases of value, valuation approaches and methods, and
reporting. They are designed to be applicable to valuations of all types
of assets and for any valuation purpose.

❖ IVS AssetStandards
The Asset Standards include requirements related to specific types of
assets. These requirements must be followed in conjunction with the
General Standards when performing a valuation of a specific asset type.
The Asset Standards include certain background information on the
characteristics of each asset type that influence value and additional
asset-specific requirements on common valuation approaches and
methods used.

6.1.10 IVS General Standards


6.1.10.1 IVS 101 Scope of Work
❖ A scope of work (sometimes referred to as terms of engagement)
describes the fundamental terms of a valuation engagement, such as
the asset(s) being valued, the purpose of the valuation and the
responsibilities of parties involved in thevaluation.
❖ GeneralRequirements
▪ All valuation advice and the work undertaken in its preparation
must be appropriate for the intended purpose.
▪ A valuer must ensure that the intended recipient(s) of the
valuation advice understand(s) what is to be provided and any
limitations on its use before it is finalisedandreported.
▪ A valuer must communicate the scope of work to its client prior
to completion of the assignment, including thefollowing:
a. Identity of the valuer: The valuer may be an individual, group of
individuals or a firm. If the valuer has any material connection or
involvement with the subject asset or the other parties to the
valuation assignment, or if there are any other factors that could
limit the valuer‘s ability to provide an unbiased and objective
valuation, such factors must be disclosed at the outset. If such
disclosure does not take place, the valuation assignment is not
in compliance with IVS. If the valuer needs to seek material
assistance from others in relation to any aspect of the
assignment, the nature of such assistance and the extent of
reliance must be madeclear.
b. Identity of the client(s) (if any): Confirmation of those for whom
the valuation assignment is being produced is important when
determining the form and content of the report to ensure that it
contains information relevant to theirneeds.
c. Identity of other intended users (if any): It is important to
understand whether there are any other intended users of the
valuation report, their identity and their needs, to ensure that
the report content and format meets those users‘needs.
d. Asset(s) being valued: The subject asset in the valuation
assignment must be clearlyidentified.
e. The valuation currency: The currency for the valuation and the
final valuation report or conclusion must be established. For
example, a valuation might be prepared in Euros or US dollars.
This requirement is particularly important for valuation
assignments involving assets in multiple countries and/or cash
flows in multiplecurrencies.
f. Purpose of the valuation: The purpose for which the valuation
assignment is being prepared must be clearly identified as it is
important that valuation advice is not used out of context or for
purposes for which it is not intended. The purpose of the
valuation will also typically influence or determine the
basis/bases of value to beused.
g. Basis/bases of value used: As required by IVS 104 Bases of
Value, the valuation basis must be appropriate for the purpose
of the valuation. The source of the definition of any basis of
value used must be cited or the basis explained. This
requirement is not applicable to a valuation review where no
opinion of value is to be provided and the reviewer is not
required to comment on the basis of valueused.
h. Valuation date: The valuation date must be stated. If the
valuation date is different from the date on which the valuation
report is issued or the date on which investigations are to be
undertaken or completed then where appropriate, these dates
should be clearlydistinguished.
i. The nature and extent of the valuer‘s work and any limitations
thereon:
j. Any limitations or restrictions on the inspection, enquiry and/or
analysis in the valuation assignment must be identified (see IVS
Framework, paras 60.1-60.4) If relevant information is not
available because the conditions of the assignment restrict the
investigation, these restrictions and any necessary assumptions
or special assumptions (see IVS 104 Bases of Value, paras 200.1-
200.5) made as a result of the restriction must beIdentified.
k. The nature and sources of information upon which the valuer
relies: The nature and source of any relevant information that is
to be relied upon and the extent of any verification to be
undertaken during the valuation process must beidentified.
l. Significant assumptions and/or special assumptions: All
significant assumptions and special assumptions that are to be
made in the conduct and reporting of the valuation assignment
must beidentified.
m. The type of report being prepared: The format of the report,
that is, how the valuation will be communicated, must be
described.
n. Restrictions on use, distribution and publication of the report:
Where it is necessary or desirable to restrict the use of the
valuation or those relying on it, the intended users and
restrictions must be clearlycommunicated.
o. That the valuation will be prepared in compliance with IVS and
thatthevaluerwillassesstheappropriatenessofallsignificant
inputs: The nature of any departures must be explained, for
example, identifying that the valuation was performed in
accordance with IVS and local tax regulations. See IVS
Framework paras 60.1-60.4 relating to departures.
▪ Wherever possible, the scope of work should be established and
agreed between parties to a valuation assignment prior to the
valuer beginning work. However, in certain circumstances, the
scope of a valuation engagement may not be clear at the start
of that engagement. In such cases, as the scope becomes clear,
valuers must communicate and agree the scope of work to their
client.
▪ A written scope of work may not be necessary. However, since
valuers are responsible for communicating the scope of work to
their client, a written scope of work should beprepared.
▪ Some aspects of the scope of work may be addressed in
documents such as standing engagement instructions, master
services agreements or a company‘s internal policies and
procedures.
❖ Changes to Scope ofWork
▪ Some of the items in above paragraphs may not be
determinable until the valuation assignment is in progress, or
changes to the scope may become necessary during the course
of the assignment due to additional information becoming
available or matters emerging that require further investigation.
As such, whilst the scope of work may be established at the
outset, it may also be established over time throughout the
course of theassignment.
▪ In valuation assignments where the scope of work changes over
time, the items in above paragraphs and any changes made
over time must be communicated to the client before the
assignment is completed and the valuation report isissued.
6.1.10.2 IVS 102 Investigations and Compliance
❖ To be compliant with IVS, valuation assignments, including
valuation reviews, must be conducted in accordance with all of
the principles set out in IVS that are appropriate for the purpose
and the terms and conditions set out in the scope ofwork.
❖ Investigations
▪ Investigations made during the course of avaluation
assignment must be appropriate for the purpose of the
valuation assignment and the basis(es) of value. References to a
valuation or valuation assignment in this standard include a
valuation review.
▪ Sufficient evidence must be assembled by means such as
inspection, inquiry, computation and analysis to ensure that the
valuation is properly supported. When determining the extent
of evidence necessary, professional judgement is required to
ensure the information to be obtained is adequate for the
purpose of thevaluation.
▪ Limits may be agreed on the extent of the valuer‘s
investigations. Any such limits must be noted in the scope of
work. However, IVS 105 Valuation Approaches and Methods,
requires valuers to perform sufficient analysis to evaluate all
inputs and assumptions and their appropriateness for the
valuation purpose. If limitations on investigations are so
substantial that the valuer cannot sufficiently evaluate the
inputs and assumptions, the valuation engagement must not
state that it has been performed in compliance withIVS.
▪ When a valuation assignment involves reliance on information
supplied by a party other than the valuer, consideration should
be given as to whether the information is credible or that the
information may otherwise be relied upon without adversely
affecting the credibility of the valuation opinion. Significant
inputs provided to the valuer (eg, by management/owners), may
require consideration, investigation and/or corroboration. In
cases where credibility or reliability of information supplied
cannot be supported, such information should not beused.
▪ In considering the credibility and reliability of information
provided, valuers should consider matters suchas:
a. the purpose of thevaluation,
b. the significance of the information to the valuation
conclusion,
c. the expertise of the source in relation to the subject
matter,and
d. whether the source is independent of either the subject
asset and/or the recipient of thevaluation.
▪ Thepurposeofthevaluation,thebasisofvalue,theextentand
limits on the investigations and any sources of information that
may be relied upon are part of the valuation assignment‘s scope
of work that must be communicated to all parties to the
valuation assignment (see IVS 101 Scope of Work).
▪ If, during the course of an assignment, it becomes clear that the
investigations included in the scope of work will not result in a
credible valuation, or information to be provided by third
parties are either unavailable or inadequate, the valuation
assignment will not comply withIVS.
❖ Valuation Record
▪ A record must be kept of the work performed during the
valuation process and the basis for the work on which the
conclusions were reached for a reasonable period after
completion of the assignment, having regard to any relevant
statutory, legal or regulatory requirements. Subject to any such
requirements, this record should include the key inputs, all
calculations, investigations and analyses relevant to the final
conclusion, and a copy of any draft or final report(s) provided to
theclient.
❖ Compliance with Other Standards
▪ As noted in the IVS Framework, when statutory, legal, regulatory
or other authoritative requirements must be followed that differ
from some of the requirements within IVS, a valuer must follow
the statutory, legal, regulatory or other authoritative
requirements (called a ―departure‖). Such a valuation has still
been performed in overall compliance withIVS.
▪ Most other sets of requirements, such as those written by
Valuation Professional Organisations, other professional bodies,
or firms‘ internal policies and procedures, will not contradict IVS
and, instead, typically impose additional requirements on
valuers. Such standards may be followed in addition to IVS
without being seen as departures as long as all of the
requirements in IVS arefulfilled.
6.1.10.3 IVS 104 Bases of value
❖ Compliance with this mandatory standard requires a valuer to
select the appropriate basis (or bases) of value and follow all
applicable requirements associated with that basis of value,
whether those requirements are included as part of thisstandard
(for IVS-defined bases of value) or not (for non-IVS-defined
bases ofvalue).
❖ In addition to the IVS-defined bases of value listed below, the
IVS have also provided a non - exhaustive list of other non-IVS-
defined bases of value prescribed by individual jurisdictional law
or those recognised and adopted by internationalagreement:
▪ IVS-defined bases ofvalue:
a. MarketValue,
b. MarketRent,
c. EquitableValue,
d. InvestmentValue/Worth,
e. Synergistic Value,and
f. Liquidation Value.
▪ Other bases of value (non-exhaustivelist):
1. Fair Value (International Financial ReportingStandards),

2. Fair Market Value (Organisation for Economic Co-operation and


Development),

3. Fair Market Value (United States Internal Revenue Service),and

4. Fair Value(Legal/Statutory):

a. the Model Business Corporation Act,and

b. Canadian case law (Manning v Harris Steel GroupInc).

❖ Premise of Value/Assumed Use : A Premise of Value or Assumed


Use describes the circumstances of how an asset or liability is used.
Different bases of value may require a particular Premise of Value or
allow the consideration of multiple Premises of Value. Some common
Premises of Valueare:
a. highest and bestuse,
b. current use/existinguse,
c. orderly liquidation,and
d. forcedsale.
❖ Assumptions and Special Assumptions : In addition to stating the
basis of value, it is often necessary to make an assumption or multiple
assumptions to clarify either the state of the asset in the hypothetical
exchange or the circumstances under which the asset is assumed to be
exchanged. Such assumptions can have a significant impact on value.
These types of assumptions generally fall into one of twocategories:
a. assumed facts that are consistent with, or could be consistent
with, those existing at the date of valuation,and
b. assumed facts that differ from those existing at the date of
valuation.
Assumptions related to facts that are consistent with, or could be
consistent with, those existing at the date of valuation may be the result
of a limitation on the extent of the investigations or enquiries
undertaken by the valuer. Examples of such assumptions include,
without limitation:

a. an assumption that a business is transferred as a complete


operationalentity,
b. an assumption that assets employed in a business aretransferred
without the business, either individually or as agroup,
c. an assumption that an individually valued asset is transferred
together withother
d. complementary assets,and
e. an assumption that a holding of shares is transferred either as a
block orindividually.
Where assumed facts differ from those existing at the date of valuation,
itisreferredtoasa―specialassumption‖.Specialassumptionsareoftenused to
illustrate the effect of possible changes on the value of an asset. They
are designated as ―special‖ so as to highlight to a valuation user that
the valuation conclusion is contingent upon a change in the current
circumstances or that it reflects a view that would not be taken by
participants generally on the valuation date. Examples of such
assumptions include, without limitation:

a. an assumption that a property is freehold with vacant


possession,
b. an assumption that a proposed building had actually been
completed on the valuationdate,
c. an assumption that a specific contract was in existence on the
valuation date which had not actually been completed,and
d. an assumption that a financial instrument is valued using a yield
curve that is different from that which would be used by a
participant.
All assumptions and special assumptions must be reasonable under the
circumstances, be supported by evidence, and be relevant having
regard to the purpose for which the valuation isrequired.

❖ Transaction Costs : Most bases of value represent the estimated


exchange price of an asset without regard to the seller‘s costs of sale or
the buyer‘s costs of purchase and without adjustment for any taxes
payable by either party as a direct result of thetransaction.

6.1.10.4 IVS 105 Valuation Approaches andMethods


It is discussed in detail in Chapters 2, 3 and 4.

3. Study of Indian Accounting Standards


(Ind AS 16) as applicable to Valuation,
Ind AS 36, Ind AS 40, Ind AS113

5.3.4 Ind AS 16
6.3. 2.1 Objective
The objective of this Standard is to prescribe the accounting treatment
for property, plant and equipment so that users of the financial
statements can discern information about an entity‘s investment in its
property, plant and equipment and the changes in such investment. The
principal issues in accounting for property, plant and equipment are the
recognition of the assets, the determination of their carrying amounts
and the depreciation charges and impairment losses to be recognised in
relation to them.

6.3. 2.2Scope
This Standard shall be applied in accounting for property, plant and
equipment except when another Standard requires or permits a
different accounting treatment.

This Standard does not apply to:

a. property, plant and equipment classified as held for sale in


accordance with Ind AS 105, Non-current Assets Held for Sale
and Discontinued Operations.
b. biological assets related to agricultural activity other than bearer
plants (See Ind AS 41, Agriculture). This Standard applies to
bearer plants but it does not apply to the produce on bearer
plants.
c. the recognition and measurement of exploration and evaluation
assets (see Ind AS 106, Exploration for and Evaluation of Mineral
Resources).
d. mineral rights and mineral reserves such as oil, natural gas and
similar non-regenerativeresources.
However, this Standard applies to property, plant and equipment used
to develop or maintain the assets described in (b)–(d).

Other Indian Accounting Standards may require recognition of an item


of property, plant and equipment based on an approach different from
that in this Standard. For example, Ind AS 17, Leases, requires an entity
to evaluate its recognition of an item of leased property, plant and
equipment on the basis of the transfer of risks and rewards. However, in
such cases other aspects of the accounting treatment for these assets,
including depreciation, are prescribed by this Standard.

An entity accounting for investment property in accordance with Ind AS


40, Investment Property shall use the cost model in this Standard.

6.3. 2.3 Definition


(i) A bearer plant is a living plantthat:

a. is used in the production or supply of agriculturalproduce;


b. is expected to bear produce for more than one period;and
c. has a remote likelihood of being sold as agricultural produce,
except for incidental scrapsales.
(ii) Carrying amount is the amount at which an asset is recognised
after deducting any accumulated depreciation and accumulated
impairmentlosses.

(iii) Cost is the amount of cash or cash equivalents paid or the fair value
of the other consideration given to acquire an asset at the time of its
acquisition or construction or, where applicable, the amount attributed
to that asset when initially recognised in accordance with the specific
requirements of other Indian Accounting Standards, eg Ind AS 102,
Share-basedPayment.

(iv) Depreciable amount is the cost of an asset, or other amount


substituted for cost, less its residualvalue.

(v) Depreciation is the systematic allocation of the depreciable


amount of an asset over its usefullife.
(vi) Entity-specific value is the present value of the cash flows an
entity expects to arise from the continuing use of an asset and from its
disposal at the end of its useful life or expects to incur when settling a
liability.

(vii) Fair value is the price that would be received to sell an asset or
paid to transfer a liability in an orderly transaction between market
participants at the measurement date. (See Ind AS 113, Fair Value
Measurement.)

(viii) An impairment loss is the amount by which the carrying amount


of an asset exceeds its recoverableamount.

(ix) Property, plant and equipment are tangible items that:

a. are held for use in the production or supply of goods or


services, for rental to others, or for administrative purposes;
and
b. are expected to be used during more than oneperiod.
(x) Recoverable amount is the higher of an asset‘s fair value less costs
to sell and its value inuse.

(xi) The residual value of an asset is the estimated amount that an


entity would currently obtain from disposal of the asset, after deducting
the estimated costs of disposal, if the asset were already of the age and
in the condition expected at the end of its usefullife.

(xii) Useful lifeis:

a. the period over which an asset is expected to be available for


use by an entity;or
b. the number of production or similar units expected to be
obtained from the asset by anentity
6.3. 2.4 Recognition
The cost of an item of property, plant and equipment shall be
recognised as an asset if, and only if:

a. it is probable that future economic benefits associated with the


item will flow to the entity;and
b. the cost of the item can be measuredreliably.
Items such as spare parts, stand-by equipment and servicing equipment
are recognised in accordance with this Ind AS when they meet the
definition of property, plant and equipment. Otherwise, such items are
classified as inventory.

This Standard does not prescribe the unit of measure for recognition, ie
what constitutes an item of property, plant and equipment. Thus,
judgement is required in applying the recognition criteria to an entity‘s
specific circumstances. It may be appropriate to aggregate individually
insignificant items, such as moulds, tools and dies, and to apply the
criteria to the aggregate value.

An entity evaluates under this recognition principle all its property, plant
and equipment costs at the time they are incurred. These costs include
costs incurred initially to acquire or construct an item of property, plant
and equipment and costs incurred subsequently to add to, replace part
of, or service it.

Initial costs

Items of property, plant and equipment may be acquired for safety or


environmental reasons. The acquisition of such property, plant and
equipment, although not directly increasing the future economic
benefits of any particular existing item of property, plant and
equipment, may be necessary for an entity to obtain the future
economic benefits from its other assets. Such items of property, plant
and equipment qualify for recognition as assets because they enable an
entity to derive future economic benefits from related assets in excess
of what could be derived had those items not been acquired. For
example, a chemical manufacturer may install new chemical handling
processes to comply with environmental requirements for the
production and storage of dangerous chemicals; related plant
enhancements are recognised as an asset because without them the
entity is unable to manufacture and sell chemicals. However, the
resulting carrying amount of such an asset and related assets is
reviewed for impairment in accordance with Ind AS 36, Impairment of
Assets.

Subsequent costs

Under the recognition principle in paragraph 7, an entity does not


recognise in the carrying amount of an item of property, plant and
equipment the costs of the dayto-day servicing of the item. Rather,
thesecostsarerecognisedinprofitorlossasincurred.Costsofday-to-
day servicing are primarily the costs of labour and consumables, and
may include the cost of small parts. The purpose of these expenditures
is often described as for the ‗repairs and maintenance‘ of the item of
property, plant andequipment.

Parts of some items of property, plant and equipment may require


replacement at regular intervals. For example, a furnace may require
relining after a specified number of hours of use, or aircraft interiors
such as seats and galleys may require replacement several times during
the life of the airframe. Items of property, plant and equipment may
also be acquired to make a less frequently recurring replacement, such
as replacing the interior walls of a building, or to make a nonrecurring
replacement. Under the recognition principle in paragraph 7, an entity
recognises in the carrying amount of an item of property, plant and
equipment the cost of replacing part of such an item when that cost is
incurred if the recognition criteria are met. The carrying amount of
those parts that are replaced is de-recognised in accordance with the
de-recognition provisions of thisStandard.

A condition of continuing to operate an item of property, plant and


equipment (for example, an aircraft) may be performing regular major
inspections for faults regardless of whether parts of the item are
replaced. When each major inspection is performed, its cost is
recognised in the carrying amount of the item of property, plant and
equipment as a replacement if the recognition criteria are satisfied. Any
remaining carrying amount of the cost of the previous inspection (as
distinct from physical parts) is de-recognised. This occurs regardless of
whether the cost of the previous inspection was identified in the
transaction in which the item was acquired or constructed. If necessary,
the estimated cost of a future similar inspection may be used as an
indication of what the cost of the existing inspection component was
when the item was acquired or constructed.

6.3. 2.5 Measurement atrecognition


An item of property, plant and equipment that qualifies for recognition
as an asset shall be measured at its cost.

Elements of cost

The cost of an item of property, plant and equipment comprises:

a) itspurchaseprice,includingimportdutiesandnon-refundable
purchase taxes, after deducting trade discounts and rebates.
b) any costs directly attributable to bringing the asset to the
location and condition necessary for it to be capable of
operating in the manner intended bymanagement.
c) the initial estimate of the costs of dismantling and removing the
item and restoring the site on which it is located, the obligation
for which an entity incurs either when the item is acquired or as a
consequence of having used the item during a particular period
for purposes other than to produce inventories during that
period.
Examples of directly attributable costs are:

a) costs of employee benefits (as defined in Ind AS 19, Employee


Benefits) arising directly from the construction or acquisition of
the item of property, plant andequipment;
b) costs of sitepreparation;
c) initial delivery and handling costs;
d) installation and assemblycosts;
e) costs of testing whether the asset is functioning properly, after
deducting the net proceeds from selling any items produced
while bringing the asset to that location and condition (such as
samples produced when testing equipment);and
f) professionalfees.
An entity applies Ind AS 2, Inventories, to the costs of obligations for
dismantling, removing and restoring the site on which an item is
located that are incurred during a particular period as a consequence of
having used the item to produce inventories during that period. The
obligations for costs accounted for in accordance with Ind AS 2 or Ind
AS 16 are recognised and measured in accordance with Ind AS 37,
Provisions, Contingent Liabilities and ContingentAssets.

Examples of costs that are not costs of an item of property, plant and
equipment are:

a. costs of opening a newfacility;


b. costs of introducing a new product or service (including
costs of advertising and promotionalactivities);
c. costs of conducting business in a new location or with a
new class of customer (including costs of staff training);
and
d. administration and other general overhead costs.
Recognition of costs in the carrying amount of an item of property,
plant and equipment ceases when the item is in the location and
condition necessary for it to be capable of operating in the manner
intended by management. Therefore, costs incurred in using or
redeploying an item are not included in the carrying amount of that
item. For example, the following costs are not included in the carrying
amount of an item of property, plant andequipment:

a. costs incurred while an item capable of operating in the


manner intended by management has yet to be brought
into use or is operated at less than fullcapacity;
b. initial operating losses, such as those incurred while
demand for the item‘s output builds up;and
c. costs of relocating or reorganising part or all of an entity‘s
operations.
Some operations occur in connection with the construction or
development of an item of property, plant and equipment, but are not
necessary to bring the item to the location and condition necessary for
it to be capable of operating in the manner intended by management.
These incidental operations may occur before or during the
construction or development activities. For example, income may be
earned through using a building site as a car park until construction
starts. Because incidental operations are not necessary to bring an item
to the location and condition necessary for it to be capable of operating
in the manner intended by management, the income and related
expenses of incidental operations are recognised in profit or loss and
included in their respective classifications of income andexpense.

The cost of a self-constructed asset is determined using the same


principles as for an acquired asset. If an entity makes similar assets for
sale in the normal course of business, the cost of the asset is usually the
same as the cost of constructing an asset for sale (see Ind AS 2).
Therefore, any internal profits are eliminated in arriving at such costs.
Similarly, the cost of abnormal amounts of wasted material, labour, or
other resources incurred in self-constructing an asset is not included in
the cost of the asset. Ind AS 23, Borrowing Costs, establishes criteria for
the recognition of interest as a component of the carrying amount of a
self-constructed item of property, plant and equipment.
Bearer plants are accounted for in the same way as self-constructed
items of property, plant and equipment before they are in the location
and condition necessary to be capable of operating in the manner
intended by management. Consequently, references to ‗construction‘ in
this Standard should be read as covering activities that are necessary to
cultivate the bearer plants before they are in the location and condition
necessary to be capable of operating in the manner intended by
management.

6.3. 2.6 Measurement ofcost


The cost of an item of property, plant and equipment is the cash price
equivalent at the recognition date. If payment is deferred beyond
normal credit terms, the difference between the cash price equivalent
and the total payment is recognised as interest over the period of credit
unless such interest is capitalised in accordance with Ind AS 23.

One or more items of property, plant and equipment may be acquired


in exchange for a non-monetary asset or assets, or a combination of
monetary and nonmonetary assets. The following discussion refers
simply to an exchange of one non-monetary asset for another, but it
also applies to all exchanges described in the preceding sentence. The
cost of such an item of property, plant and equipment is measured at
fair value unless (a) the exchange transaction lacks commercial
substance or (b) the fair value of neither the asset received nor the asset
given up is reliably measurable. The acquired item is measured in this
way even if an entity cannot immediately de-recognise the asset given
up. If the acquired item is not measured at fair value, its cost is
measured at the carrying amount of the asset givenup.

An entity determines whether an exchange transaction has commercial


substance by considering the extent to which its future cash flows are
expected to change as a result of the transaction. An exchange
transaction has commercial substanceif:

a. the configuration (risk, timing and amount) of the cash


flows of the asset received differs from the configuration
of the cash flows of the asset transferred;or
b. the entity-specific value of the portion of the entity‘s
operations affected by the transaction changes as a result
of the exchange;and
c. the difference in (a) or (b) is significant relative to the fair
value of the assetsexchanged.
For the purpose of determining whether an exchange transaction has
commercial substance, the entity-specific value of the portion of the
entity‘s operations affected by the transaction shall reflect post-tax cash
flows. The result of these analyses may be clear without an entity having
to perform detailed calculations.

The fair value of an asset is reliably measurable if (a) the variability in


the range of reasonable fair value measurements is not significant for
that asset or (b) the probabilities of the various estimates within the
range can be reasonably assessed and used when measuring fair value.
If an entity is able to measure reliably the fair value of either the asset
received or the asset given up, then the fair value of the asset given up
is used to measure the cost of the asset received unless the fair value of
the asset received is more clearlyevident.

The cost of an item of property, plant and equipment held by a lessee


under a finance lease is determined in accordance with Ind AS 17.

6.3. 2.7 Measurement afterrecognition


An entity shall choose either the cost model in paragraph 30 or the
revaluation model in paragraph 31 as its accounting policy and shall
apply that policy to an entire class of property, plant and equipment.

Cost model

After recognition as an asset, an item of property, plant and equipment


shall be carried at its cost less any accumulated depreciation and any
accumulated impairment losses.

Revaluation model

After recognition as an asset, an item of property, plant and equipment


whose fair value can be measured reliably shall be carried at a revalued
amount, being its fair value at the date of the revaluation less any
subsequent accumulated depreciation and subsequent accumulated
impairment losses. Revaluations shall be made with sufficient regularity
to ensure that the carrying amount does not differ materially from that
which would be determined using fair value at the end of the reporting
period.
The frequency of revaluations depends upon the changes in fair values
of the items of property, plant and equipment being revalued. When
the fair value of a revalued asset differs materially from its carrying
amount, a further revaluation is required. Some items of property, plant
and equipment experience significant and volatile changes in fair value,
thus necessitating annual revaluation. Such frequent revaluations are
unnecessary for items of property, plant and equipment with only
insignificant changes in fair value. Instead, it may be necessary to
revalue the item only every three or fiveyears.

When an item of property, plant and equipment is revalued, the


carrying amount of that asset is adjusted to the revalued amount. At the
date of the revaluation, the asset is treated in one of the followingways:

a. the gross carrying amount is adjusted in a manner that is


consistent with the revaluation of the carrying amount of
the asset. For example, the gross carrying amount may be
restated by reference to observable market data or it may
be restated proportionately to the change in the carrying
amount. The accumulated depreciation at the date of the
revaluation is adjusted to equal the difference between
the gross carrying amount and the carrying amount of
the asset after taking into account accumulated
impairment losses;or
b. the accumulated depreciation is eliminated against the
gross carrying amount of theasset.
The amount of the adjustment of accumulated depreciation forms part
of the increase or decrease in carrying amount that is accounted for in
accordance with paragraphs 39 and 40.

If an item of property, plant and equipment is revalued, the entire class


of property, plant and equipment to which that asset belongs shall be
revalued.

A class of property, plant and equipment is a grouping of assets of a


similar nature and use in an entity‘s operations. The following are
examples of separate classes:

a. land;
b. land andbuildings;
c. machinery;
d. ships;
e. aircraft;
f. motor vehicles;
g. furniture andfixtures;
h. office equipment;and
i. bearer plants.
The items within a class of property, plant and equipment are revalued
simultaneously to avoid selective revaluation of assets and the reporting
of amounts in the financial statements that are a mixture of costs and
values as at different dates. However, a class of assets may be revalued
on a rolling basis provided revaluation of the class of assets is
completed within a short period and provided the revaluations are kept
up todate.

If an asset’s carrying amount is increased as a result of a revaluation, the


increase shall be recognised in other comprehensive income and
accumulated in equity under the heading of revaluation surplus. However,
the increase shall be recognised in profit or loss to the extent that it
reverses a revaluation decrease of the same asset previously recognised in
profit or loss.

If an asset’s carrying amount is decreased as a result of a revaluation, the


decrease shall be recognised in profit or loss. However, the decrease shall
be recognised in other comprehensive income to the extent of any credit
balance existing in the revaluation surplus in respect of that asset. The
decrease recognised in other comprehensive income reduces the amount
accumulated in equity under the heading of revaluation surplus.

6.3. 2.8 Depreciation


Each part of an item of property, plant and equipment with a cost that is
significant in relation to the total cost of the item shall be depreciated
separately.

The depreciation charge for each period shall be recognised in profit or


loss unless it is included in the carrying amount of another asset.

6.3. 2.9 Depreciable amount and depreciation period


The residual value and the useful life of an asset shall be reviewed at
least at each financial year-end.

The useful life of an asset is defined in terms of the asset‘s expected


utility to the entity. The asset management policy of the entity may
involve the disposal of assets after a specified time or after
consumption of a specified proportion of the future economic benefits
embodied in the asset. Therefore, the useful life of an asset may be
shorter than its economic life. The estimation of the useful life of the
asset is a matter of judgement based on the experience of the entity
with similarassets.

Land and buildings are separable assets and are accounted for
separately, even when they are acquired together. With some
exceptions, such as quarries and sites used for landfill, land has an
unlimited useful life and therefore is not depreciated. Buildings have a
limited useful life and therefore are depreciable assets.

6.3. 2.10 Disclosure


If items of property, plant and equipment are stated at revalued
amounts, the following shall be disclosed in addition to the disclosures
required by Ind AS 113:

a. the effective date of therevaluation;


b. whether an independent valuer wasinvolved
5.3.5 Ind AS 36
6.3. 3.1 Objective
The objective of this Standard is to prescribe the procedures that an
entity applies to ensure that its assets are carried at no more than their
recoverable amount. An asset is carried at more than its recoverable
amount if its carrying amount exceeds the amount to be recovered
through use or sale of the asset. If this is the case, the asset is described
as impaired and the Standard requires the entity to recognise an
impairment loss. The Standard also specifies when an entity should
reverse an impairment loss and prescribes disclosures.

6.3. 3.2 Scope


This Standard shall be applied in accounting for the impairment of all
assets, other than:

(a) inventories (see Ind AS 2,Inventories);

(b) contract assets and assets arising from costs to obtain or fulfill a
contract that are recognised in accordance with Ind AS 115, Revenue
from Contracts with Customers;

(c) deferred tax assets (see Ind AS 12, IncomeTaxes);


(d) assets arising from employee benefits (see Ind AS 19, Employee
Benefits);

(e) financial assets that are within the scope of Ind AS 109, Financial
Instruments;

(f) [Refer Appendix1];

(g) biological assets related to agricultural activity within the scope of


Ind AS 41 Agriculture that are measured at fair value less costs to sell;

(h) deferred acquisition costs, and intangible assets, arising from an


insurer‘s contractual rights under insurance contracts within the scope
of Ind AS 104, Insurance Contracts; and1035

(i) non-current assets (or disposal groups) classified as held for sale in
accordance with Ind AS 105, Non-current Assets Held for Sale and
DiscontinuedOperations.

3 This Standard does not apply to inventories, assets arising from


construction contracts, deferred tax assets, assets arising from
employee benefits, or assets classified as held for sale (or included in a
disposal group that is classified as held for sale) because Indian
Accounting Standards applicable to these assets contain requirements
for recognising and measuring these assets.

4 This Standard applies to financial assets classifiedas:

(a) subsidiaries, as defined in Ind AS 110, Consolidated Financial


Statements;

(b) associates, as defined in Ind AS 28, Investments in Associates and


Joint Ventures;and

(c) joint ventures, as defined in Ind AS 111, Joint Arrangements. For


impairment of other financial assets, refer to Ind AS109.

5 This Standard does not apply to financial assets within the scope of
Ind AS 109, or biological assets related to agricultural activity measured
at fair value less costs to sell within the scope of Ind AS 41. However,
this Standard applies to assets that are carried at revalued amount (ie
fair value at the date of the revaluation less any subsequent
accumulated depreciation and subsequent accumulated impairment
losses) in accordance with other Ind ASs, such as the revaluation model
inIndAS16,Property,PlantandEquipmentandIndAS38,Intangible
Assets. The only difference between an asset's fair value and its fair
value less costs of disposal is the direct incremental costs attributable to
the disposal of theasset.

(a) If the disposal costs are negligible, the recoverable amount of the
revalued asset is necessarily close to, or greater than, its revalued
amount. In this case, after the revaluation requirements have been
applied, it is unlikely that the revalued asset is impaired and recoverable
amount need not beestimated.

(b) [Refer Appendix1]

(c) If the disposal costs are not negligible, the fair value less costs of
disposal of the revalued asset is necessarily less than its fair value.
Therefore, the revalued asset will be impaired if its value in use is less
than its revalued amount. In this case, after the revaluation
requirements have been applied, an entity applies this Standard to
determine whether the asset may beimpaired.

6.3. 3.3 Definitions


The following terms are used in this Standard with the meanings
specified :

Carrying amount is the amount at which an asset is recognised after


deducting any accumulated depreciation (amortisation) and
accumulated impairment losses thereon.

A cash-generating unit is the smallest identifiable group of assets that


generates cash inflows that are largely independent of the cash inflows
from other assets or groups of assets.

Corporate assets are assets other than goodwill that contribute to the
future cash flows of both the cash-generating unit under review and
other cash generating units.

Costs of disposal are incremental costs directly attributable to the


disposal of an asset or cash-generating unit, excluding finance costs
and income taxexpense.

Depreciable amount is the cost of an asset, or other amount


substituted for cost in the financial statements, less its residual value.
Depreciation (Amortisation) is the systematic allocation of the
depreciable amount of an asset over its useful life.1

Fair value is the price that would be received to sell an asset or paid to
transfer a liability in an orderly transaction between market participants
at the measurement date. (See Ind AS 113, Fair Value Measurement.)

An impairment loss is the amount by which the carrying amount of an


asset or a cash-generating unit exceeds its recoverableamount.

The recoverable amount of an asset or a cash-generating unit is the


higher of its fair value less costs of disposal and its value inuse.

Useful life is either:

(a) the period of time over which an asset is expected to be used by the
entity;or

(b) the number of production or similar units expected to be obtained


from the asset by theentity.

Value in use is the present value of the future cash flows expected to
be derived from an asset or cash-generatingunit.

6.3. 3.4 An asset is impaired when its carrying amount exceeds its
recoverable amount.
6.3. 3.5 An entity shall assess at the end of each reporting periodwhether
there is any indication that an asset may be impaired. If any
such indication exists, the entity shall estimate the recoverable
amount of theasset.

5.3.6 Ind AS 40
6.3.4.1 Objective
The objective of this Standard is to prescribe the accounting treatment
for investment property and related disclosure requirements.

6.3.4.2 Scope
This Standard shall be applied in the recognition, measurement and
disclosure of investment property. Among other things, this Standard
applies to the measurement in a lessee‘s financial statements of
investment property interests held under a lease accounted for as a
finance lease and to the measurement in a lessor‘s financial statements
of investment property provided to a lessee under an operating lease.
This Standard does not deal with matters covered in Ind AS 17, Leases,
including:

(a) classification of leases as finance leases or operatingleases;

(b) 1recognition of lease income from investment property (see also Ind
AS 115, Revenue from Contracts withCustomers);

(c) measurement in a lessee‘s financial statements of property interests


held under a lease accounted for as an operatinglease;

(d) measurement in a lessor‘s financial statements of its net investment


in a financelease;

(e) accounting for sale and leaseback transactions;and

(f) disclosure about finance leases and operating leases.

This Standard does not applyto:

(a) biologicalassetsrelatedtoagriculturalactivity(seeIndAS41,
Agriculture and Ind AS 16 Property, Plant and Equipment); and

(b) mineral rights and mineral reserves such as oil, natural gas and
similar non-regenerativeresources.

6.3. 4.3 Definitions


The following terms are used in this Standard with the meanings
specified:

Carrying amount is the amount at which an asset is recognised in the


balance sheet.

Cost is the amount of cash or cash equivalents paid or the fair value of
other consideration given to acquire an asset at the time of its
acquisition or construction or, where applicable, the amount attributed
to that asset when initially recognised in accordance with the specific
requirements of other Ind ASs, eg Ind AS 102, Share-basedPayment.

Fair value is the price that would be received to sell an asset or paid to
transfer a liability in an orderly transaction between market participants
at the measurement date. (See Ind AS 113, Fair ValueMeasurement).

Investment property is property (land or a building—or part of a


building—or both) held (by the owner or by the lessee under a finance
lease) to earn rentals or for capital appreciation or both, rather than for:
(a) use in the production or supply of goods or services or for
administrative purposes;or

(b) sale in the ordinary course ofbusiness.

Owner-occupied property is property held (by the owner or by the


lessee under a finance lease) for use in the production or supply of
goods or services or for administrative purposes.

6.3. 4.4 Classification of property as investment property or owner-


occupied property
Investment property is held to earn rentals or for capital appreciation or
both. Therefore, an investment property generates cash flows largely
independently of the other assets held by an entity. This distinguishes
investment property from owner-occupied property. The production or
supply of goods or services (or the use of property for administrative
purposes) generates cash flows that are attributable not only to
property, but also to other assets used in the production or supply
process. Ind AS 16 applies to owner-occupied property.

6.3. 4.5 Recognition


Investment property shall be recognised as an asset when, and only
when:

(a) it is probable that the future economic benefits that are associated
with the investment property will flow to the entity;and

(b) the cost of the investment property can be measuredreliably.

6.3. 4.6 Measurement at recognition


An investment property shall be measured initially at its cost.
Transaction costs shall be included in the initialmeasurement.

The initial cost of a property interest held under a lease and classified as
an investment property shall be as prescribed for a finance lease by
paragraph 20 of Ind AS 17, ie the asset shall be recognised at the lower
of the fair value of the property and the present value of the minimum
lease payments. An equivalent amount shall be recognised as a liability
in accordance with that same paragraph.

6.3. 4.7 Measurement after recognition


An entity shall adopt as its accounting policy the cost model prescribed
in paragraph 56 to all of its investment property.
There is a rebuttable presumption that an entity can reliably measure
the fair value of an investment property on a continuing basis. However,
in exceptional cases, there is clear evidence when an entity first acquires
an investment property (or when an existing property first becomes
investment property after a change in use) that the fair value of the
investment property is not reliably measurable on a continuing basis.
This arises when, and only when, the market for comparable properties
is inactive (eg there are few recent transactions, price quotations are not
current or observed transaction prices indicate that the seller was forced
to sell) and alternative reliable measurements of fair value (for example,
based on discounted cash flow projections) are not available. If an entity
determines that the fair value of an investment property under
construction is not reliably measurable but expects the fair value of the
property to be reliably measurable when construction is complete, it
shall measure the fair value of that investment property either when its
fair value becomes reliably measurable or construction is completed
(whichever is earlier). If an entity determines that the fair value of an
investment property (other than an investment property under
construction) is not reliably measurable on a continuing basis, the entity
shall make the disclosures required by paragraphs 79(e)(i), (ii) and (iii) of
thisstandard.

If an entity has previously measured the fair value of an investment


property, it shall continue to measure the fair value of that property
until disposal (or until the property becomes owneroccupied property
or the entity begins to develop the property for subsequent sale in the
ordinary course of business) even if comparable market transactions
become less frequent or market prices become less readilyavailable.

After initial recognition, an entity shall measure all of its investment


properties in accordance with Ind AS 16‘s requirements for cost model,
other than those that meet the criteria to be classified as held for sale
(or are included in a disposal group that is classified as held for sale) in
accordance with Ind AS 105, Non-current Assets Held for Sale and
Discontinued Operations. Investment properties that meet the criteria
to be classified as held for sale (or are included in a disposal group that
is classified as held for sale) shall be measured in accordance with Ind
AS105.

6.3.4.8 Transfers
An entity shall transfer a property to, or from, investment property
when, and only when, there is a change in use. A change in use occurs
when the property meets, or ceases to meet, the definition of
investment property and there is evidence of the change in use. In
isolation, a change in management‘s intentions for the use of a property
does not provide evidence of a change in use. Examples of evidence of
a change in useinclude:

(a) commencement of owner-occupation, or of development with a


view to owner occupation, for a transfer from investment property to
owner-occupiedproperty;

(b) commencement of development with a view to sale, for a transfer


from investment property to inventories;

(c) end of owner-occupation, for a transfer from owner-occupied


property to investment property;and

(d) inception of an operating lease to another party, for a transfer from


inventoriesto

investmentproperty.

6.3.4.9 Disposals
An investment property shall be derecognised (eliminated from the
balance sheet) on

disposal or when the investment property is permanently withdrawn


from use and no

future economic benefits are expected from its disposal.

Gains or losses arising from the retirement or disposal of investment


property shall be

determined as the difference between the net disposal proceeds and


the carrying amount of the asset and shall be recognised in profit or
loss (unless Ind AS 17 requires otherwise on a sale and leaseback) in the
period of the retirement ordisposal.

Compensation from third parties for investment property that was


impaired, lost or given up shall be recognised in profit or loss when the
compensation becomes receivable.
5.3.7 Ind AS 113
6.3.4.1 Objective
This Ind AS:

(a) defines fair value;

(b) sets out in a single Ind AS a framework for measuring fair value;and

(c) requires disclosures about fair valuemeasurements.

Fair value is a market-based measurement, not an entity-specific


measurement. For some assets and liabilities, observable market
transactions or market information might be available. For other assets
and liabilities, observable market transactions and market information
might not be available. However, the objective of a fair value
measurement in both cases is the same—to estimate the price at which
an orderly transaction to sell the asset or to transfer the liability would
take place between market participants at the measurement date under
current market conditions (ie an exit price at the measurement date
from the perspective of a market participant that holds the asset or
owes theliability).

When a price for an identical asset or liability is not observable, anentity


measures fair value using another valuation technique that maximises
the use of relevant observable inputs and minimises the use of
unobservable inputs. Because fair value is a market-based
measurement, it is measured using the assumptions that market
participants would use when pricing the asset or liability, including
assumptions about risk. As a result, an entity's intention to hold an asset
or to settle or otherwise fulfil a liability is not relevant when measuring
fairvalue.

The definition of fair value focuses on assets and liabilities because they
are a primary subject of accounting measurement. In addition, this Ind
AS shall be applied to an entity's own equity instruments measured at
fairvalue.

6.3. 4.2 Scope


This Ind AS applies when another Ind AS requires or permits fair value
measurements or disclosures about fair value measurements (and
measurements, such as fair value less costs to sell, based on fair value or
disclosures about those measurements), except as specified in
paragraphs 6 and7.

The measurement and disclosure requirements of this Ind AS do not


apply to the following:

(a) share-based payment transactions within the scope of Ind AS 102,


Share basedPayment;

(b) leasing transactions within the scope of Ind AS 17, Leases;and

(c) measurements that have some similarities to fair value but are not
fair value, such as net realisable value in Ind AS 2, Inventories, or value
in use in Ind AS 36, Impairment ofAssets.

The disclosures required by this Ind AS are not required for the
following:

(a) plan assets measured at fair value in accordance with Ind AS 19,
EmployeeBenefits;

(b) (Refer Appendix 1);and

(c) assets for which recoverable amount is fair value less costs of
disposal in accordance with Ind AS36.

The fair value measurement framework described in this Ind AS applies


to both initial and subsequent measurement if fair value is required or
permitted by other Ind ASs.

6.3. 4.3 Definition of fair value


This Ind AS defines fair value as the price that would be received to sell
an asset or paid to transfer a liability in an orderly transaction between
market participants at the measurement date.

Paragraph B2 describes the overall fair value measurement approach.

6.3. 4.4 The asset or liability


A fair value measurement is for a particular asset or liability. Therefore,
when measuring fair value an entity shall take into account the
characteristics of the asset or liability if market participants would take
those characteristics into account when pricing the asset or liability at
the measurement date. Such characteristics include, for example, the
following:

(a) the condition and location of the asset;and


(b) restrictions, if any, on the sale or use of theasset.

The effect on the measurement arising from a particular characteristic


will differ depending on how that characteristic would be taken into
account by market participants.

The asset or liability measured at fair value might be either of the


following:

(a) a stand-alone asset or liability (eg a financial instrument or a non-


financial asset);or

(b) a group of assets, a group of liabilities or a group of assets and


liabilities (eg a cash-generating unit or abusiness).

Whether the asset or liability is a stand-alone asset or liability, a group


of assets, a group of liabilities or a group of assets and liabilities for
recognition or disclosure purposes depends on its unit of account. The
unit of account for the asset or liability shall be determined in
accordance with the Ind AS that requires or permits the fair value
measurement, except as provided in this IndAS.

6.3. 4.5 The transaction


A fair value measurement assumes that the asset or liability is
exchanged in an orderly transaction between market participants to sell
the asset or transfer the liability at the measurement date under current
marketconditions.

A fair value measurement assumes that the transaction to sell the asset
or transfer the liability takes place either:

(a) in the principal market for the asset or liability;or

(b) in the absence of a principal market, in the most advantageous


market for the asset orliability.

An entity need not undertake an exhaustive search of all possible


markets to identify the principal market or, in the absence of a principal
market, the most advantageous market, but it shall take into account all
information that is reasonably available. In the absence of evidence to
the contrary, the market in which the entity would normally enter into a
transaction to sell the asset or to transfer the liability is presumed to be
the principal market or, in the absence of a principal market, the most
advantageous market.
If there is a principal market for the asset or liability, the fair value
measurement shall represent the price in that market (whether that
price is directly observable or estimated using another valuation
technique), even if the price in a different market is potentially more
advantageous at the measurementdate.

The entity must have access to the principal (or most advantageous)
market at the measurement date. Because different entities (and
businesses within those entities) with different activities may have
access to different markets, the principal (or most advantageous)
market for the same asset or liability might be different for different
entities (and businesses within those entities). Therefore, the principal
(or most advantageous) market (and thus, market participants) shall be
considered from the perspective of the entity, thereby allowing for
differences between and among entities with differentactivities.

Although an entity must be able to access the market, the entity does
not need to be able to sell the particular asset or transfer the particular
liability on the measurement date to be able to measure fair value on
the basis of the price in that market.

Even when there is no observable market to provide pricing information


about the sale of an asset or the transfer of a liability at the
measurement date, a fair value measurement shall assume that a
transaction takes place at that date, considered from the perspective of
a market participant that holds the asset or owes the liability. That
assumed transaction establishes a basis for estimating the price to sell
the asset or to transfer theliability.

6.3. 4.6 Market participants


An entity shall measure the fair value of an asset or a liability using the
assumptions that market participants would use when pricing the asset
or liability, assuming that market participants act in their economic best
interest.

In developing those assumptions, an entity need not identify specific


market participants. Rather, the entity shall identify characteristics that
distinguish market participants generally, considering factors specific to
all the following:

(a) the asset orliability;


(b) the principal (or most advantageous) market for the asset or liability;
and

(c) market participants with whom the entity would enter into a
transaction in thatmarket.

6.3.4.7 Theprice
Fair value is the price that would be received to sell an asset or paid to
transfer a liability in an orderly transaction in the principal (or most
advantageous) market at the measurement date under current market
conditions (ie an exit price) regardless of whether that price is directly
observable or estimated using another valuation technique.

The price in the principal (or most advantageous) market used to


measure the fair value of the asset or liability shall not be adjusted for
transaction costs. Transaction costs shall be accounted for in
accordance with other Ind ASs. Transaction costs are not a characteristic
of an asset or a liability; rather, they are specific to a transaction and will
differ depending on how an entity enters into a transaction for the asset
or liability.

Transaction costs do not include transport costs. If location is a


characteristic of the asset (as might be the case, for example, for a
commodity), the price in the principal (or most advantageous) market
shall be adjusted for the costs, if any, that would be incurred to
transport the asset from its current location to thatmarket.

6.3. 4.8 Application to non-financial assets


Highest and best use for non-financial assets

A fair value measurement of a non-financial asset takes into account a


market participant's ability to generate economic benefits by using the
asset in its highest and best use or by selling it to another market
participant that would use the asset in its highest and best use.

The highest and best use of a non-financial asset takes into account the
use of the asset that is physically possible, legally permissible and
financially feasible, as follows:

(a) A use that is physically possible takes into account the physical
characteristics of the asset that market participants would take into
account when pricing the asset (eg the location or size of aproperty).
(b) A use that is legally permissible takes into account any legal
restrictions on the use of the asset that market participants would take
into account when pricing the asset (eg the zoning regulations
applicable to aproperty).

(c) A use that is financially feasible takes into account whether a use of
the asset that is physically possible and legally permissible generates
adequate income or cash flows (taking into account the costs of
converting the asset to that use) to produce an investment return that
market participants would require from an investment in that asset put
to thatuse.

Highest and best use is determined from the perspective of market


participants, even if the entity intends a different use. However, an
entity's current use of a non-financial asset is presumed to be its
highest and best use unless market or other factors suggest that a
different use by market participants would maximise the value of the
asset.

To protect its competitive position, or for other reasons, an entity may


intend not to use an acquired non-financial asset actively or it may
intend not to use the asset according to its highest and best use. For
example, that might be the case for an acquired intangible asset that
the entity plans to use defensively by preventing others from using it.
Nevertheless, the entity shall measure the fair value of a non-financial
asset assuming its highest and best use by marketparticipants.

6.3. 4.9 Valuation premise for non-financial assets


The highest and best use of a non-financial asset establishes the
valuation premise used to measure the fair value of the asset, as follows:

(a) The highest and best use of a non-financial asset might provide
maximum value to market participants through its use in combination
with other assets as a group (as installed or otherwise configured for
use) or in combination with other assets and liabilities (egabusiness).

(i) If the highest and best use of the asset is to use the asset in
combination with other assets or with other assets and liabilities, the fair
value of the asset is the price that would be received in a current
transaction to sell the asset assuming that the asset would be used with
other assets or with other assets and liabilities and that those assetsand
liabilities (ie its complementary assets and the associated liabilities)
would be available to market participants.

(ii) Liabilities associated with the asset and with the complementary
assets include liabilities that fund working capital, but do not include
liabilities used to fund assets other than those within the group of
assets.

(iii) Assumptions about the highest and best use of a non-financial asset
shall be consistent for all the assets (for which highest and best use is
relevant) of the group of assets or the group of assets and liabilities
within which the asset would beused.

(b) The highest and best use of a non-financial asset might provide
maximum value to market participants on a stand-alone basis. If the
highest and best use of the asset is to use it on a stand-alone basis, the
fair value of the asset is the price that would be received in a current
transaction to sell the asset to market participants that would use the
asset on a stand-alonebasis.

The fair value measurement of a non-financial asset assumes that the


asset is sold consistently with the unit of account specified in other Ind
ASs (which may be an individual asset). That is the case even when that
fair value measurement assumes that the highest and best use of the
asset is to use it in combination with other assets or with other assets
and liabilities because a fair value measurement assumes that the
market participant already holds the complementary assets and the
associatedliabilities.

Paragraph B3 describes the application of the valuation premise


concept for non-financialassets.

6.3. 4.10 Fair value at initial recognition


When an asset is acquired or a liability is assumed in an exchange
transaction for that asset or liability, the transaction price is the price
paid to acquire the asset or received to assume the liability (an entry
price). In contrast, the fair value of the asset or liability is the price that
would be received to sell the asset or paid to transfer the liability (an
exit price). Entities do not necessarily sell assets at the prices paid to
acquire them. Similarly, entities do not necessarily transfer liabilities at
the prices received to assumethem.
In many cases the transaction price will equal the fair value (eg that
might be the case when on the transaction date the transaction to buy
an asset takes place in the market in which the asset would be sold).

When determining whether fair value at initial recognition equals the


transaction price, an entity shall take into account factors specific to the
transaction and to the asset or liability. Paragraph B4 describes
situations in which the transaction price might not represent the fair
value of an asset or a liability at initial recognition.

If another Ind AS requires or permits an entity to measure an asset or a


liability initially at fair value and the transaction price differs from fair
value, the entity shall recognise the resulting gain or loss in profit or
loss unless that Ind AS specifiesotherwise.

6.3. 4.11 Valuation techniques


An entity shall use valuation techniques that are appropriate in the
circumstances and for which sufficient data are available to measure fair
value, maximising the use of relevant observable inputs and minimising
the use of unobservable inputs.

The objective of using a valuation technique is to estimate the price at


which an orderly transaction to sell the asset or to transfer the liability
would take place between market participants at the measurement date
under current market conditions. Three widely used valuation
techniques are the market approach, the cost approach and the income
approach. The main aspects of those approaches are summarized in
paragraphs B5-B11. An entity shall use valuation techniques consistent
with one or more of those approaches to measure fairvalue.

In some cases a single valuation technique will be appropriate (eg when


valuing an asset or a liability using quoted prices in an active market for
identical assets or liabilities). In other cases, multiple valuation
techniques will be appropriate (eg that might be the case when valuing
a cash-generating unit). If multiple valuation techniques are used to
measure fair value, the results (ie respective indications of fair value)
shall be evaluated considering the reasonableness of the range of
values indicated by those results. A fair value measurement is the point
within that range that is most representative of fair value in the
circumstances.
If the transaction price is fair value at initial recognition and a valuation
technique that uses unobservable inputs will be used to measure fair
value in subsequent periods, the valuation technique shall be calibrated
so that at initial recognition the result of the valuation technique equals
the transaction price. Calibration ensures that the valuation technique
reflects current market conditions, and it helps an entity to determine
whether an adjustment to the valuation technique is necessary (eg there
might be a characteristic of the asset or liability that is not captured by
the valuation technique). After initial recognition, when measuring fair
value using a valuation technique or techniques that use unobservable
inputs, an entity shall ensure that those valuation techniques reflect
observable market data (eg the price for a similar asset or liability) at
the measurementdate.

Valuation techniques used to measure fair value shall be applied


consistently. However, a change in a valuation technique or its
application (eg a change in its weightage when multiple valuation
techniques are used or a change in an adjustment applied to a valuation
technique) is appropriate if the change results in a measurement that is
equally or more representative of fair value in the circumstances. That
might be the case if, for example, any of the following events take place:

(a) new markets develop;

(b) new information becomesavailable;

(c) information previously used is no longer available;

(d) valuation techniques improve;or

(e) market conditionschange.

Revisions resulting from a change in the valuation technique or its


application shall be accounted for as a change in accounting estimate in
accordance with Ind AS 8. However, the disclosures in Ind AS 8 for a
change in accounting estimate are not required for revisions resulting
from a change in a valuation technique or its application.

6.3. 4.12 Inputs to valuation techniques


General principles

Valuation techniques used to measure fair value shall maximise the use
of relevant observable inputs and minimise the use of unobservable
inputs.
Examples of markets in which inputs might be observable for some
assets and liabilities (eg financial instruments) include exchange
markets, dealer markets, brokered markets and principal-to-principal
markets (see paragraphB34).

An entity shall select inputs that are consistent with the characteristics
of the asset or liability that market participants would take into account
in a transaction for the asset or liability (see paragraphs 11 and 12). In
some cases those characteristics result in the application of an
adjustment, such as a premium or discount (eg a control premium or
non-controlling interest discount). However, a fair value measurement
shall not incorporate a premium or discount that is inconsistent with the
unit of account in the Ind AS that requires or permits the fair value
measurement (see paragraphs 13 and 14). Premiums or discounts that
reflect size as a characteristic of the entity's holding (specifically, a
blockage factor that adjusts the quoted price of an asset or a liability
because the market's normal daily trading volume is not sufficient to
absorb the quantity held by the entity, as described in paragraph 80)
rather than as a characteristic of the asset or liability (eg a control
premium when measuring the fair value of a controlling interest) are not
permitted in a fair value measurement. In all cases, if there is a quoted
price in an active market (ie a Level 1 input) for an asset or a liability, an
entity shall use that price without adjustment when measuring fair
value, except as specified in paragraph 79. Inputs based on bid and ask
prices

70 If an asset or a liability measured at fair value has a bid price and an


ask price (eg an input from a dealer market), the price within the bid-ask
spread that is most representative of fair value in the circumstances
shall be used to measure fair value regardless of where the input is
categorised within the fair value hierarchy (ie Level 1, 2 or 3; see
paragraphs 72-90). The use of bid prices for asset positions and ask
prices for liability positions is permitted, but is notrequired.

This Ind AS does not preclude the use of mid-market pricing or other
pricing conventions that are used by market participants as a practical
expedient for fair value measurements within a bid-ask spread.

6.3. 4.13 Fair value hierarchy


To increase consistency and comparability in fair value measurements
and related disclosures, this Ind AS establishes a fair value hierarchy that
categorises into three levels (see paragraphs 76-90), the inputs to
valuation techniques used to measure fair value. The fair value hierarchy
gives the highest priority to quoted prices (unadjusted) in active
markets for identical assets or liabilities (Level 1 inputs) and the lowest
priority to unobservable inputs (Level 3inputs).

In some cases, the inputs used to measure the fair value of an asset or a
liability might be categorised within different levels of the fair value
hierarchy. In those cases, the fair value measurement is categorised in
its entirety in the same level of the fair value hierarchy as the lowest
level input that is significant to the entire measurement. Assessing the
significance of a particular input to the entire measurement requires
judgement, taking into account factors specific to the asset or liability.
Adjustments to arrive at measurements based on fair value, such as
costs to sell when measuring fair value less costs to sell, shall not be
taken into account when determining the level of the fair value
hierarchy within which a fair value measurement iscategorised.

The availability of relevant inputs and their relative subjectivity might


affect the selection of appropriate valuation techniques (see paragraph
61). However, the fair value hierarchy prioritises the inputs to valuation
techniques, not the valuation techniques used to measure fair value. For
example, a fair value measurement developed using a present value
technique might be categorised within Level 2 or Level 3, depending on
the inputs that are significant to the entire measurement and the level
of the fair value hierarchy within which those inputs arecategorised.

If an observable input requires an adjustment using an unobservable


input and that adjustment results in a significantly higher or lower fair
value measurement, the resulting measurement would be categorised
within Level 3 of the fair value hierarchy. For example, if a market
participant would take into account the effect of a

restriction on the sale of an asset when estimating the price for the
asset, an entity would adjust the quoted price to reflect the effect of
that restriction. If that quoted price is a Level 2 input and the
adjustment is an unobservable input that issignificant
to the entire measurement, the measurement would be categorised
within Level 3 of the fair value hierarchy.

6.3. 4.14 Level 1 inputs


Level 1 inputs are quoted prices (unadjusted) in active markets for
identical assets or liabilities that the entity can access at the
measurement date.

A quoted price in an active market provides the most reliable evidence


of fair value and shall be used without adjustment to measure fair value
whenever available, except as specified in paragraph 79.

A Level 1 input will be available for many financial assets and financial
liabilities, some of which might be exchanged in multiple active markets
(eg on different exchanges). Therefore, the emphasis within Level 1 is on
determining both of the following:

(a) the principal market for the asset or liability or, in the absence of a
principal market, the most advantageous market for the asset or
liability;and

(b) whether the entity can enter into a transaction for the asset or
liability at the price in that market at the measurementdate.

An entity shall not make an adjustment to a Level 1 input except in the


following circumstances:

(a) when an entity holds a large number of similar (but not identical)
assets or liabilities (eg debt securities) that are measured at fair value
and a quoted price in an active market is available but not readily
accessible for each of those assets or liabilities individually (ie given the
large number of similar assets or liabilities held by the entity, it would
be difficult to obtain pricing information for each individual asset or
liability at the measurement date). In that case, as a practical expedient,
an entity may measure fair value using an alternative pricing method
that does not rely exclusively on quoted prices (eg matrix pricing).
However, the use of an alternative pricing method results in a fair value
measurement categorised within a lower level of the fair value
hierarchy.

(b) when a quoted price in an active market does not represent fair
value at the measurement date. That might be the case if, for example,
significant events (such as transactions in aprincipal-to-principal
market, trades in a brokered market or announcements) take place after
the close of a market but before the measurement date. An entity shall
establish and consistently apply a policy for identifying those events
that might affect fair value measurements. However, if the quoted price
is adjusted for new information, the adjustment results in a fair value
measurement categorised within a lower level of the fair value
hierarchy.

(c) when measuring the fair value of a liability or an entity's own equity
instrument using the quoted price for the identical item traded as an
asset in an active market and that price needs to be adjusted for factors
specific to the item or the asset (see paragraph 39). If no adjustment to
the quoted price of the asset is required, the result is a fair value
measurement categorised within Level 1 of the fair value hierarchy.
However, any adjustment to the quoted price of the asset results in a
fair value measurement categorised within a lower level of the fair value
hierarchy.

If an entity holds a position in a single asset or liability (including a


position comprising a large number of identical assets or liabilities, such
as a holding of financial instruments) and the asset or liability is traded
in an active market, the fair value of the asset or liability shall be
measured within Level 1 as the product of the quoted price for the
individual asset or liability and the quantity held by the entity. That is
the case even if a market's normal daily trading volume is not sufficient
to absorb the quantity held and placing orders to sell the position in a
single transaction might affect the quotedprice.

6.3. 4.15 Level 2 inputs


Level 2 inputs are inputs other than quoted prices included within Level
1 that are observable for the asset or liability, either directly or
indirectly.

If the asset or liability has a specified (contractual) term, a Level 2 input


must be observable for substantially the full term of the asset or liability.
Level 2 inputs include the following:

(a) quoted prices for similar assets or liabilities in activemarkets.

(b) quoted prices for identical or similar assets or liabilities in markets


that are notactive.
(c) inputs other than quoted prices that are observable for the asset or
liability, forexample:

(i) interest rates and yield curves observable at commonly quoted


intervals;

(ii) implied volatilities;and

(iii) creditspreads.

(d) market-corroborated inputs.

Adjustments to Level 2 inputs will vary depending on factors specific to


the asset or liability. Those factors include the following:

(a) the condition or location of theasset;

(b) the extent to which inputs relate to items that are comparable to the
asset or liability (including those factors described in paragraph 39);and

(c) the volume or level of activity in the markets within which the inputs
areobserved.

An adjustment to a Level 2 input that is significant to the entire


measurement might result in a fair value measurement categorised
within Level 3 of the fair value hierarchy if the adjustment uses
significant unobservable inputs.

Paragraph B35 describes the use of Level 2 inputs for particular assets
and liabilities.

6.3. 4.16 Level 3 inputs


Level 3 inputs are unobservable inputs for the asset or liability.

Unobservable inputs shall be used to measure fair value to the extent


that relevant observable inputs are not available, thereby allowing for
situations in which there is little, if any, market activity for the asset or
liability at the measurement date. However, the fair value measurement
objective remains the same, ie an exit price at the measurement date
from the perspective of a market participant that holds the asset or
owes the liability. Therefore, unobservable inputs shall reflect the
assumptions that market participants would use when pricing the asset
or liability, including assumptions aboutrisk.

Assumptions about risk include the risk inherent in a particular


valuation technique used to measure fair value (such as a pricingmodel)
and the risk inherent in the inputs to the valuation technique. A
measurement that does not include an adjustment for risk would not
represent a fair value measurement if market participants would include
one when pricing the asset or liability. For example, it might be
necessary to include a risk adjustment when there is significant
measurement uncertainty (eg when there has been a significant
decrease in the volume or level of activity when compared with normal
market activity for the asset or liability, or similar assets or liabilities, and
the entity has determined that the transaction price or quoted price
does not represent fair value, as described in paragraphs B37-B47).

An entity shall develop unobservable inputs using the best information


available in the circumstances, which might include the entity's own
data. In developing unobservable inputs, an entity may begin with its
own data, but it shall adjust those data if reasonably available
information indicates that other market participants would use different
data or there is something particular to the entity that is not available to
other market participants (eg an entity-specific synergy). An entity need
not undertake exhaustive efforts to obtain information about market
participant assumptions. However, an entity shall take into account all
information about market participant assumptions that is reasonably
available. Unobservable inputs developed in the manner described
above are considered market participant assumptions and meet the
objective of a fair value measurement.

Paragraph B36 describes the use of Level 3 inputs for particular assets
and liabilities.

6.3. 4.17 Disclosure


For further details please consult IndAS 113

6.3. 4.18 Appendix A (Defined Terms)


For further details please consult IndAS 113

6.3.4.19 Appendix B (ApplicationGuidance)


For further details please consult IndAS113

6.3.4.20 Appendix C (References to matters contained in other


Indian AccountingStandards)
For further details please consult IndAS113
4. Valuer as an Expert witness in Court
Dramatic changes in different legislations in India and increasing legal
literacy in common Indians has been opening a new professional
window to the Valuers of this country. The current trend in the attitudes
of the government officials, both technical and non-technical, is not to
take any decision by their own on any controversial issue. This negative
attitude is increasing the number of cases in the court of law as well as
outside the court of law in an exponential manner. It is thus forcing our
society to become litigation happy.

―InfrastructureDevelopment‖isthekeyobjectiveoftheGovernmentof
India to touch the peak of the Economy. Involvement of foreign
agencies, innumerable work forces, complex laws, and most advanced
technologies giving rise to more and more disputes in the working field.
The decisions of the Employer‘s experts are frequently challenged
before the Court of law. Every such suit in court is looking for the
service of a valuer who shall have the ability to establish his findings in
the trial. For all arbitration matters where a valuer‘s service is required
that gentleman has to be an expert witness. The need for expert
witnesses has never been greater than it is today. Litigants rely on
expert witness testimony. Unfortunately (or Fortunately?), very few
expert valuers receive formal training in such subjects as the expert‘s
proper role in the legal system, how to communicate opinions
effectively at deposition, at trial, and in written report, the law and the
procedure dealing with expert witnesses, ethics and how to deal with
legal counsels. So, it is evident that demand for an valuer who is ready
to act as an expert witness is huge in India. But, surprisingly very little of
us are ready to be there at the witness box to support our technical
findings. So, in the era of sharp competition, it is an opportunity for the
professional valuers to train themselves as an Expert Witness and earn
fees and fame as per their choice.

6.3.1 To become an Expert Witness the following qualities are


essential:
i. You must be properlyqualified
ii. You must have adequateexperience
iii. You must have an in depth knowledge of thesubject
iv. You must have adequate report writingskill
v. You must have proper communication power
vi. You must have adequate knowledge of different legal procedure
of expertwitnessing
vii. You must have the patience of listeningothers
viii. You must have the patience of not being disturbed by
unnecessary and out of the wayquestions
6.3.2 What advocates look for in expert witnesses are individuals
who can help them win.
Mainly they look for the following characteristics in an expert:

✓ Qualification
✓ Credibility
✓ Availability
✓ Preparation
✓ Oral communicationskills
✓ Report writing skills
✓ Testifying skills
✓ Charisma
✓ Reasonableness
✓ Assistance tocounsel
If one possesses such qualities, he/she will be an Expert Witness very
soon.

6.3.3 The following are the parties in arbitration where expert


witness has to appear:
❖ Arbitration Tribunal
❖ Advocate of theplaintiff
❖ Advocate of thedefendant
❖ Plaintiff
❖ Defendant
❖ ExpertWitness
6.3.4 Examination-in-chief:
Witnesses are introduced to a trial by their examination-in-chief,
which is when they answer questions asked by the lawyer
representing the party which called them to the stand.

6.3.5 Cross examination:


After examination-in-chief, the other party‘s lawyer can question
witness too; this is called cross-examination. In law, cross-
examination is the interrogation of a witness called by one's
opponent.
6.3.6 The 17 Golden Rules of Testifying:
✓ An expert witness should tell the truth simply anddirectly
✓ An expert witness should stay within his true area ofexpertise
✓ Ifanexpertdoesn‘tknowananswer,theappropriateansweris,―I
don‘tknow‖
✓ An expert-witness should meticulously prepare for cross-
examination
✓ An expert-witness should not argue withcounsel
✓ An expert-witness should not be arrogant, hostile, or
condescending
✓ An expert-witness should pause before responding aquestion
✓ An expert-witness should not exaggerate, speculate orguess
✓ An expert-witness should remain cool, calm andcollected
✓ An expert-witness should actively listen to thequestion
✓ Get all facts and data straight before taking thestand
✓ An expert witness should use acceptedmethodology
✓ If interrupted an expert witness must finish hisanswer
✓ If questioned about a document an expert witness should ask to
seeit
✓ An expert witness should not useslang
✓ Be prepare to strike if opposing counsel makes amistake

5. Valuers’ Functions & Responsibilities,


Error of Judgement and Professional
Negligence
6.3.7 Valuer’s Function
The following are the functions of a valuer:

(1) To determine the value of aproperty.


(2) To estimate the cost of producing, acquiring, altering, or
completing aproperty.
(3) To estimate the monetary amount of damages to aproperty.
(4) To forecast the monetary earning power of aproperty.
(5) The formulation of conclusions andrecommendations.
(6) The presentations of alternatives (and their consequences) for the
client‘sactions.
The primary function of a valuer is to determine a numerical result,
either as a range or most probable point magnitude—therupee
amount of a value, the rupee amount of an estimated cost, the rupee
amount of an estimated earning power. This numerical result is
objective and unrelated to the desires, wishes, or needs of the client
who engages the valuer to perform the work. The amount of this figure
is as independent of what someone desires it to be as a physicist‘s
measurement of the melting point of lead or an accountant‘s statement
of the amount of net profits of acorporation.

6.3.8 Valuer’s Responsibility


The following are the responsibilities of a registered valuer:

6.5.2.1 Valuer‘s Responsibility to Determine and Describe the


Apposite Kind of Value or EstimatedCost
Because there are several kinds of value and several kinds of cost
estimates, each of which has a legitimate place as the end point ofsome
class of valuation engagement, it is the valuer‘s responsibility to
ascertain which one of these is pertinent to the particular undertaking.
In meeting this responsibility, the valuer may consider his client‘s
instructions and/or may obtain legal or other professional advice, but
the selection of the apposite kind of value or estimated cost is the
valuer‘s sole responsibility. Also, it is his responsibility, in this
connection, fully to explain and describe what is meant by the particular
value or cost estimate which he has determined, in order to obviate
misunderstanding and to prevent unwitting or deliberate
misapplication. For example, an engagement which calls for the
determination of the market value of a multi-tenant office building
leasehold estate, would not be properly discharged by a determination
of the depreciated new cost of replacement of theimprovements.

6.5.2.2 Valuer‘s Responsibility to Determine Numerical Results with


Whatever Degree of Accuracy the Particular Objectives of the
Valuation Necessitate
It is the valuer‘s responsibility to determine the appropriate and applicable
numerical results with as high a degree of accuracy as the particular objectives
of the valuation necessitate.

6.5.2.3 Valuer‘s Responsibility to Avoid Giving a False Numerical


Result
Obviously, the valuer has every responsibility to avoid giving a false
figure. The numerical result of a valuation could be false for one of two
reasons: it could be false because it is a grossly inaccurate estimate of
the apposite kind of value or cost estimate, or it could be false, even
though numerically accurate, because it is an estimate of an inapposite
kind of value of cost estimate.

6.5.2.4 Valuer‘s Responsibility to Attain Competency and to Practice


Ethically
In order to meet his responsibilities, the valuer must be competent in
his field. This competency he attains by education, training, study,
practice, and experience. He must also recognize, understand, and abide
by those ethical principles that are interwoven with and are an essential
part of truly professionalpractice.

6.5.2.5 Valuers‘ Responsibility to ThirdParties


Under certain specific circumstances a valuation report may be given by
a client to a third party for their use. If the purpose of the valuation
includes a specific use by a third party, the third party has a right to rely
on the validity and objectivity of the valuer‘s findings as regards the
specific stated purpose and intended use for which the valuation was
originally made. Members of a registered valuers organisation
recognize their responsibility to those parties, other than the client, who
may be specifically entitled to make use of theirreports.

6.5.2.6 Valuers‘ Responsibility to maintain the confidential Character


of a valuationEngagement
The fact that a valuer has been employed to make a valuation is a
confidential matter. In some instances, the very fact of employment may
be information that a client, whether private or a public agency, prefers
for valid reasons to keep confidential. Knowledge by outsiders of the
fact of employment of a valuer may jeopardize a client‘s proposed
enterprise or transaction. Therefore, it is imperative that the client be
made aware that ethical standard requires full disclosure by the valuer,
both to the new client prior to the acceptance of an assignment and
within the certification of the valuation report of his/her involvement in
any capacity with the subject property within the past three years.

In the absence of an express agreement to the contrary, the identifiable


contents of a valuation report are the property of the valuer‘s client or
employer and, ethically, cannot be submitted to any professional
Valuation institute as evidence of professional qualifications, and cannot
be published in any identifiable form without the client‘s or employer‘s
consent.

6.5.2.7 Valuer‘s Responsibility to Give CompetentService


It is not proper for a valuer to accept an engagement to make a
valuation of property of a type he is not qualified to value or in a field
outside his registered valuers organisation membership classification,
unless (a) he fully acquaints the client with the limitations of his
qualifications or (b) he associates himself with another valuer or valuers
who possess the required qualifications.

6.5.2.8 Valuer‘s Responsibility Relative to GivingTestimony


When a valuer is engaged by one of the parties in a controversy, it is
unethical for the valuer to suppress any facts, data, or opinions which
are adverse to the case his client is trying to establish; or to
overemphasize any facts, data, or opinions which are favorable to his
client‘s case; or in any other particulars to become an advocate. It is the
valuer‘s responsibility to present the data, analysis, and value without
bias, regardless of the effect of such unbiased presentation on his
client‘scase.

6.5.2.9 Valuer‘s Responsibility to Document Valuation Testimony When


a member of a registered valuers organisation accepts
employment to make a valuation, or to testify as to value of property
before a court of law or other judicial or quasi-judicial forums, thevaluer
shall, before testifying, complete an adequate written valuation report,
or have complete documentation and substantiation available in hisfiles.

6.5.2.10 Valuer‘s Responsibility Relative to Serving More Than One


Client in the SameMatter
When two or more potential clients seek a valuer‘s services with respect
to the same property or with respect to the same legal action, the valuer
may not properly serve more than one, except with the consent of all
parties.

6.5.2.11 Agreements and Contracts for ValuationServices


It is good practice to have a written contract, or at least a clear oral
agreement, between valuer and client, covering objectives and scope of
work, time of delivery of report, and amount of fees. In certain
circumstances, it may be desirable to include in the valuation-service
contract a statement covering the objective character of valuation
findings and a statement that the valuer cannot act as an advocate or
negotiator.

6.5.2.12 Protection of Professional Reputation of OtherValuers


The valuer has a responsibility to protect the professional reputation of
all valuers (whether members of a registered valuers organisation or
not) who subscribe to and practice in accord with the Principles of
Valuation Practice of any registered valuers organisation. It is unethical
for a valuer to injure, or attempt to injure, by false or malicious
statements or by innuendo the professional reputation or prospects of
any valuer.

6.5.2.13 Valuer‘s Responsibility Relative to registered valuers


organisation‘s Disciplinary Actions
A member of a registered valuers organisation, having knowledge of an
act by another member which, in his opinion, is in violation of the
ethical principles incorporated in the Principles of Valuation Practice
and Code of Ethics of that registered valuers organisation, has the
responsibility to report the matter in accordance with the procedure
specified in the Constitution and Bylaws.

It is the valuer‘s responsibility to cooperate with a registered valuers


organisation and its officers in all matters, including investigation,
censure, discipline, or dismissal of members who are charged with
violation of the Principles of Valuation Practice and Code of Ethics of
the registered valuersorganisation.

6.5.2.14 Various Kinds ofValue


It is globally recognized that different kinds of property may have
different kinds of value depending on the particular attendant
circumstances and, further, that there are both basic and subordinate
kinds of value. Good professional practice requires that the valuer
describe in sufficient detail, in each case, the nature and meaning of the
specific value that he or she is determining.

6.5.2.15 Selection of ValuationMethod


The procedure and method for determining the particular value in
question is a matter for the valuer to decide—the valuer cannot be held
responsible for the result unless he or she has a free hand in selecting
the process by which that result is to be obtained. However, good
valuation practice requires that the method selected be adequate for
the purpose, embrace consideration of all the factors that have a
bearing on the value, and be presented in a clear and logicalmanner.

6.5.2.16 FractionalValuations
Certain classes of properties can be considered as made up of
components (for example, in the case of real estate: land and buildings).
If an element is considered as an integrated part of the whole property,
its value, in general, is different from the value the same element has if
considered as a fraction separated from the whole property.

A valuation of an element of a whole property considered by itself and


ignoring its relation to the rest of the whole property, is called a
"fractional valuation." There are legitimate uses for fractional valuations
(valuation of buildings for fire insurance purposes; valuation to
determine the value of land as if cleared of existing improvements;
valuation in connection with public utility rate-making, etc.) but good
practice requires that a fractional valuation be labeled as such and that
the limitations on its use by the client and/or third parties be clearly
stated.

6.5.2.17 Contingent and Limiting Conditions Affecting avaluation


In many instances the validity of the valuer‘s conclusions as to the value
of a subject property is contingent upon the validity of statements,
information, and/or data upon which he has relied, supplied to him by
members of other professions or secured by him from official sources.
Such material may be obtained, for example, from architects, engineers,
lawyers, accountants, government officials, government agencies, etc. It
is proper for the valuer to rely upon and use such material provided (1)
he states in his report that he has done so, (2) he stands ready to make
his sources and/or the material itself available for any required
verification, and (3) he does not pass to others the responsibility for
matters that are, or should be, within the scope of his own professional
knowledge.

Good valuation practice requires that the valuer state any other
contingent or limiting conditions which affect the valuation, such as, for
example, that the value is contingent upon the completion of projected
public or private improvements, etc.

6.5.2.18 Valuations Based on HypotheticalConditions


A valuation based on a hypothetical condition shall be prepared when
such a condition is required for legal purposes, for purposes of
comparison, for purposes of reasonable analysis and when the use of
the hypothetical condition in the assignment results in a credible
analysis, opinion and/or conclusion. A hypothetical condition is one that
is based on facts that are contrary to what is known by the valuer to
exist as of the effective date of thevaluation.

Whiletheterm―hypotheticalcondition‖doesnotneedtobeincludedin the
valuation report, the valuer must clearly and prominently disclose within
the report that the condition is contrary to fact, that the condition is
being used for the purpose of analysis only, and that the condition has
an effect on the valuer‘s analysis, opinions and/or conclusion.

6.5.2.19 Valuations Based on ExtraordinaryAssumptions


A valuation based on an extraordinary assumption shall be prepared
when such a condition is required for purposes of analysis. An
extraordinary assumption is one based on facts, which if found to be
false as of the effective date, could alter the valuer‘s analysis, opinions,
and/or conclusions. An extraordinary assumption is one that presumes
as fact otherwise uncertain information about the physical, legal or
economic characteristics of the subject property, or about the external
conditions (such as market conditions or trends). While the term
―extraordinary assumption‖ does not need to be included in the
valuation report, the valuer must clearly and prominently disclose within
the report that the valuation was prepared using an assumption, and
should that assumption not be valid, this could alter the valuer‘s
analysis, opinions and/orconclusion.

6.5.2.20 Valuations in Which Access to Pertinent Data isdenied


A valuation based on an extraordinary assumption shall be prepared
when such a condition is required for purposes of analysis. An
extraordinary assumption is one based on facts, which if found to be
false as of the effective date, could alter the valuer‘s analysis, opinions,
and/or conclusions. An extraordinary assumption is one that presumes
as fact otherwise uncertain information about the physical, legal or
economic characteristics of the subject property, or about the external
conditions (such as market conditions or trends).
While the term ―extraordinary assumption‖ does not need to be
included in the valuation report, the valuer must clearly and
prominently disclose within the report that the valuation was prepared
using an assumption, and should that assumption not be valid, this
could alter the valuer‘s analysis, opinions and/orconclusion.

Situations sometimes occur in which data that the valuer considers


pertinent to the making of a valid valuation are in existence but access
to them is denied to the valuer, either by the client or some other party
(for example: the past production records of an oil field; the records of
prior revenue and expense of a motel property; etc.). In such a case, the
valuer, at his option, may properly decline to carry out the assignment.
In the event he considers such data essential to the making of a valid
valuation, he may not properly proceed with the assignment.

6.5.2.21 Ranges of Value or Estimated Cost and Reliability Estimates


Some valuation engagements call for the determination of a probable
range of value or estimated cost, either with or without a collateral
statement of the most probable figure within that range. It is entirely
within the scope of good valuation practice to give a range of value or
estimatedcost.

Inasmuch as the valuer‘s determination of the amount of a value or an


estimated cost cannot, by its very nature, be exact, it is good valuation
practice to append to such numerical results a statement as to the
degree of reliability to be accorded thereto. Such reliability estimates
are usually expressed as plus and minuspercentages.

6.5.2.22 Values or Estimated Costs under DifferentHypotheses


The objective of a valuation undertaking may be the determination of
different values or different cost estimates based on different
hypotheses. It is entirely within the scope of good valuation practice to
give such differing numerical results, provided the valuer adheres to the
principles set forth in the previousparagraphs.

6.5.2.23 Inspection, Investigation, Analysis, and Description of Subject


Property
The valuation of a property is a procedure based on an analysis of all
the characteristics of the property which contribute to or detract from
its value; good valuation practice requires that the valuer‘s inspection,
investigation, and study be thorough enough to uncover all of the pertinent
characteristics.

Good valuation practice requires that the description of the property,


tangible or intangible, which is the subject of a valuation, cover
adequately (a) identification of the property (b) statement of the legal
rights and restrictions comprised in the ownership, and (c) the
characteristics of the property which contribute to or detract from its
value.

In the case of prospective real estate improvements, identification is


particularly important in order to prevent unscrupulous persons from
representing the valuation as applying to substituted inferior property.

In general, in the case of real property, statements of zoningrestrictions,


building codes, easements, leases, etc., are essential elements of the
description. It is understood, however, that the legal rights of the
ownership of an interest in real property are matters of legal, not
valuation, opinion, and that the valuer discharges his responsibilities in
this regard by stating the sources of thesedata.

The physical condition of real property is an element contributing to or


detracting from their value; good valuation practice requires adequate
inspection and investigation to determine it.

6.5.2.24 Collaboration between Valuers and utilization of the Services


of Members of otherprofessions
Collaboration between valuers is desirable, in some situations, to
expedite the completion of work and, in other situations, to obtain the
benefits of combined judgment or combined data. Such collaboration is
entirely proper providing all the collaborators sign a joint report or, if
there be dissenting opinions, providing these dissenting opinions are
made a part of the report.

In some cases, the nature of the valuation undertaking calls for special
professional knowledge and abilities in addition to those possessed by
the valuer. In such an instance, it is both necessary and proper for the
valuer to employ other valuers and/or members of other professions to
obtain data and derive conclusions relative to specific parts of the work.
The principal valuer builds his final conclusions, in part, on these
contributions, taking responsibility for the final result but subject to the
validity of the underlying or constituent contributions.
6.5.2.25 Unethical and Unprofessional ValuationPractices
The principles of valuation practice given in previous paragraphs relate
to the primary objective of a valuation undertaking, namely the
determination of the apposite numerical result with that degree of
accuracy required by the attendant circumstances, whereas the
principles given in this section relate to the establishment and
maintenance of the confidence of clients and other interested parties in
the validity of the results of valuation undertakings. To this end, certain
practices are declared by registered valuers organisation to be unethical
and unprofessional.

6.5.2.26 Valuer‘s responsibility not to charge ContingentFees


If a valuer were to accept an engagement for which the amount of his
compensation is contingent upon the amount of an award in a property
settlement or a court action where his services are employed; or is
contingent upon the amount of a tax reduction obtained by a client
where his services are used; or is contingent upon the consummation of
the sale or financing of a property in connection with which his services
are utilized or is contingent upon his reaching any finding or conclusion
specified by his client; then, anyone considering using the results of the
valuer‘s undertaking might well suspect that these results were biased
and self-serving and therefore, invalid. Such suspicion would militate
against the establishment and maintenance of trust and confidence in
the results of valuation work, generally; therefore a registered valuers
organisation declares that the contracting for or acceptance of any such
contingent fee is unethical and unprofessional.

6.5.2.27 PercentageFees
A registered valuers organisation takes the position that it is
unprofessional and unethical for the valuer to contract to do work for a
fixed percentage of the amount of value, or of the estimated cost (as
the case may be) which he determines at the conclusion of hiswork.

6.5.2.28 DisinterestedValuations
Anyone using a valuer who has an interest or a contemplated future
interest in the property valued, might well suspect that the report was
biased and self-serving and, therefore, that the findings were invalid.
Such suspicion tends to break down trust and confidence in the results
of valuation work, generally.
Interests which a valuer may have in a property which is to be valued,
include ownership of the subject property; acting, or having some
expectation of acting, as agent in the purchase, sale, or financing of the
subject property; and managing, or have some expectation of
managing, the subject property. Such interests are particularly apt to
exist if the valuer, while engaged in professional valuation practice, is
also engaged in a related retail business (real estate, jewelry, furs,
antiques, fine arts,etc.).

A Valuation institute declares that, subject to the provision for


disclosure given in the following paragraph, it is unethical and
unprofessional for a valuer to accept an assignment to value a property
in which he has an interest or a contemplated futureinterest.

However, if a prospective client, after full disclosure by the valuer of his


present or contemplated future interest in the subject property, still
desires to have the valuer do the work, the latter may properly accept
the engagement provided he discloses the nature and extent of his
interest in his valuation report.

6.5.2.29 Responsibility Connected with Signatures to Valuation


Reports
The user of a valuation report, before placing reliance on its
conclusions, is entitled to assume that the party signing the report is
responsible for the findings, either because he did the work himself or
because the work was done under hissupervision.

In cases where two or more valuers are employed to prepare a joint


report, the user thereof is entitled to assume that, if all of them sign it,
they are jointly and severally responsible for the validity of all of the
findings therein; and, if all do not sign, he has a right to know what the
dissenting opinions are.

In cases where two or more valuers have been engaged by a single


client to make independent valuations of the same property, the client
has the right to expect that he will receive opinions which have been
reached independently and that he may use them as checks against
each other and/or have evidence of the range within which the
numerical resultslie.

To implement these principles, all registered valuers organisations


generally declare that it is unethical (a) to misrepresent who made a
valuation by appending the signature of any person who neither did the
work himself nor had the work done under his supervision, (b) in the
case of a joint report to omit any signatures or any dissenting opinions,
(c) in case two or more valuers have collaborated in a valuation
undertaking, for them, or any of them, to issue separate valuation
reports, and (d) in case two or more valuers have been engaged by a
single client to make independent valuations of the same property, for
them to collaborate or consult with one another or make use of each
other‘s findings or figures.

6.5.2.30 Description of the property Which is the subject of a


valuation report
It is the responsibility of a valuer that the property with which a
valuation report is concerned, whether tangible, intangible, real, or
personal, be fully described therein, the elements of such description
being: (a) identification, (b) legal rights and restrictions encompassed in
the ownership, where these are not obvious, (c) value characteristics,
and (d) physical condition, whereapplicable.

6.5.2.31 Statement of the objectives of the ValuationWork


It is the responsibility of a valuer that a valuation report includes a
statement of the objectives for which the work was performed: to
determine a value, to estimate a cost, to forecast an earning power, to
ascertain certain facts, to reach conclusions and make
recommendations for action in specified matters, etc. It is required that
the meaning attached by the valuer to any specific kind of value or
estimated cost which is the objective of the valuation undertaking be
described and explained in the valuation report. It is also required that a
valuation report includes a statement as to the date which the value
estimate, cost estimate or forecast of income applies. When
appropriate, an analysis of the highest and the best use of the property
should be included in the investigation andstudy.

6.5.2.32 Statement of the Contingent and Limiting Conditions to


Which the Valuation Findings areSubject
It is required that statements, information, and/or data, which were
obtained by the valuer from members of other professions, or official or
other presumably reliable sources, and the validity of which affects the
valuation findings, be summarized or stated in full in the valuation
reportandthesourcesgiven,sothatverificationdesiredbyanyuserof
the report may be accomplished. If a valuation is a hypothetical one, it
is required that it be labeled as hypothetical, that the reason a
hypothetical valuation was made, be stated, and that the assumed
hypothetical conditions be setforth.

If a valuation is a fractional valuation, it is required that it be labeled as


fractional and that the limitations on the use of the reported figure be
stated. If a preliminary valuation report is issued, namely, one in which
the figures are subject to refinement or change, it is required that the
report be labeled as preliminary and that the limitation on its use be
stated.

6.5.2.33 Description and Explanation in the Valuation Report of the


Valuation MethodUsed
It is required that the method selected by the valuer as applicable to the
subject valuation undertaking be described and explained in the
valuation report.

6.5.2.34 Statement of the Valuer‘sDisinterestedness


It is required that the valuer include a statement in his valuation report
that he has no present or contemplated future interest in the subject
property or any other interest which might tend to prevent his making a
fair and unbiased valuation or, if he does have such an interest, to set
forth fully the nature and extent of that interest.

6.5.2.35 Valuers Responsibility to Communicate Each Analysis,


Opinion and Conclusion in a Manner that is notmisleading.
The valuer should state in each report "I hereby certify that, to the best
of my knowledge and belief, the statements of fact contained in this
report are true and correct, and this report has been prepared in
conformity with the International Valuation Standards promulgated by
the International Valuation Standards Council and the Principles of
Valuation Practice and Code of Ethics of the registered valuers
organisation of which I am a member.

6.5.2.36 Signatures to Valuation Reports and the Inclusion of


Dissenting Opinions
It is required that the registered valuer who makes the valuation or who
has the valuation made under his supervision sign the valuation report.
It is required that all collaborating valuers, issuing a joint the report,
who agree with the findings, sign the report; and that anycollaborating
valuer who disagrees with any or all of the findings of the others,
prepare, sign, and include in the valuation report his dissenting opinion.

6.3.9 Error of Judgement and ProfessionalNegligence


Any violation of the functions and responsibilities of a valuer as
described in 6.5.2 and 6.5.3 shall construe Professional Negligence and
may attract punishment according to the disciplinary action procedure
of the respective registered valuers organisation. The punishment may
be determined as per Section 247(3) and 247(4) of The Companies Act,
2013. Apart from the above the following may also be considered as
professional negligence:

Unconsidered Opinions and Preliminary Reports

If a valuer gives an opinion as to the value, earning power or estimated


cost of a property without having ascertained and weighed all of the
pertinent facts, such opinion, except by an extraordinary coincidence,
will be inaccurate. The giving of such offhand opinions tends to belittle
the importance of inspection, investigation, and analysis in valuation
procedure and lessens the confidence with which the results of good
valuation practice are received, and therefore the submission of hasty
and unconsidered opinions to be construed as professional negligence.

If a valuer makes a preliminary report without including a statement to


the effect that it is preliminary and that the figures given are subject to
refinement or change when the final report is completed, there is the
possibility that some user of the report, being under the impression that
it is a final and completed report, will accord the figures a degree of
accuracy and reliability they do not possess. The results of such
misplaced confidence could be damaging to the reputation of
professional valuers, generally, as well as of the valuer concerned.
Therefore, it is declared to be professional misconduct to omit a proper
limiting and qualifying statement in a preliminary report.

The following may be accepted as Error of Judgement of a valuer‘s


report:

1) There is a difference between the estimated value expressed in


figure and words of which one is correct and the other iswrong.
2) Evidences cited are found to be wrong but the declared sources are
correct.
3) The declared assumptions do not match with theactual.
4) The estimated value is found to be wrong which is based on
declared extraordinary assumptions only and the valuer did not have
access to any realdata.
5) Valuations Based on Hypothetical Conditions is found to be wrong
while the valuer has clearly and prominently disclosed within the
report that the condition is contrary to fact, that the condition is
being used for the purpose of analysis only, and that the condition
has an effect on the valuer‘s analysis, opinions and/orconclusion.

The valuation conclusion is found to be incorrect which is contingent upon the validity of
statements, information, and/or data upon which he has relied, supplied to him by
members of other professions or secured by him from official sources which is found to
be incorrect. But, he stated in his report that he relied upon the data provided by other
professional, stated in the report that he stood ready to make his sources and/or the
material itself available for any required verification, and also declared in the report that
he did not not pass to others the responsibility for matters that were, or should be,
within the scope of his own professional knowledge.
IMPORTANT CASE LAWS ON PRINCIPLES
OF VALUATION OF REAL ESTATE
1. Rustam C Cooper vs Union of India AIR
1970 SC564
In Bank Nationalisation Case, the Supreme Court has explained the word
propertyas,―Initsnormalconnotation‗property‘meansthe―highestrighta
man can have to anything, being that right which one has to lands or
tenements, goods or chattels which does not depend on another‘s courtesy:
it includes ownership, estates and interests in corporeal things, and also
rights such as trademarks, copyrights, patents and even rights in personam
capable of transfer or transmission, such as debts, and signifies a beneficial
right to or a thing considered as having a money ‗ value, especially with
reference to transfer or succession, and to their capacity of being injured‖.
Thus the court has included all types of property viz. tangible
andintangible.

2. SorabTalati vs JoshephMichem Appeal


101 0f 1949 - Vol. - 2 of SOC- page 162
(Bombay) (Invest Theory ofRent)
In 1949, Small Causes Court of Bombay, for the first time, laid down
principles of Rent Fixation (Standard Rent) in the case of SorabTalati. In this
case, the Court approved of Investment Theory in preference to Comparable
Rent Theory to fix Standard Rent of the rent controlled premises. In this
case, the court considered return or yield from Gilt Edged Security as
thebasis for determining fair return to the landlord on his investments
inland and buildings. Considering and comparing alternative forms of
sound investments viz. Govt. Security and immovable property, the Court
upheld following returns as fair returns to the landlord on his capital
investment in land andbuildings.

1.50% return i.e. more than the average yield on long term government
security was approved as fair return on land investment and 2.50 %
return more than the average yield on long term government
securitywas approved as fair return on investment in buildings. For
leaseholdproperties, 1% extra yield on both types of investment was
consideredfair, to account for extra risk of investing capital, in
leaseholdproperties. Obviously, these norms and principles continued to
be followed for several years for all types of rented properties and even for
other purposes.
3. CWT vs P N Sikand (1977) 107 ITR
922SC
In case of P. N. Sikand, the Supreme Court held, ―It is clear on the
application of this test that in present case, 50% of the unearned increase in
the value of the land would be diverted to the lessor before it reaches the
hands of the assessee as part of the price. The assessee holds theleasehold
interest on condition that if he assigns it, 50 % of the unearned increase
in the value of the land will be payable to the lessor.‖ Court further
stated, ―it must be held that in determining the value of the leasehold
interest of the assesseein the land for the purpose of assessment to wealth
tax, the price which the leasehold interest would fetch in the open market
were it not encumbered or affected by the burden or restriction contained
in clause (13) of the lease deed, would have to bereduced by 50%. of the
unearned increase in the value of the land onthe basis of the
hypothetical sale on the valuationdate.‖

4. Wenger & Co. vs DVO (1978) 115 ITR


648 Delhi HC (Combination of Methods)
In the case of Wenger & co., Delhi the High Court, held, ―District Valuation
officer adopted two methods to value the property. For owner
occupiedportion he calculated the value on the basis of what were the
ratesprevalent for sale of commercial flats in connaught place
extensionarea. For the tenanted portion he capitalised the rental
value.It is well- known fact that giving possession of buildings, though
previously rented out, fetches better market price. It cannot be assumed
that the hypothetical purchaser would let out the self-occupied portions
which he buys from the hypothetical seller or would let out such portions in
the condition in which he buys them. The method adopted by District V.
O. and his approach isnot only acceptable but also in accordance with
the principles ofevaluation.

5. JawajiNagnathan vs REV. DIV. Officer


(1994) SCC - 4 Page 595 SC
In case of JawajeeNagnatham, the Court held, ―It is therefore, clear
that the Basic Valuation Register prepared andmaintained for
the purpose of collecting stamp duty has nostatutory base or
force. It cannot form a foundation todetermine the market value
mentioned there under in instrument brought for registration.
Evidence of bona fide sales between willing prudent vendor and
prudent vendee of the land acquired or situated near about that
land possessing same or similar advantageous features would
furnish basis to determine marketvalue.‖

6. Chimanlal Hargovinddas vs SLAO- Pune,


AIR 1988 SC1652
The following factors must be etched on the mental screen:

(1) A reference under section 18 of the Land Acquisition Act is


not an appeal against the award and the Court cannot take into
account the material relied upon by the Land Acquisition officer
in his Award unless the same material is produced and proved
before theCourt.

(2) So also the Award of the Land Acquisition officer is not to be


treated as a judgment of the trial Court open or exposed to
challenge before the Court hearing the Reference. It is merely an
offer made by the Land Acquisition officer and the material
utilised by him for making his valuation cannot be utilised by the
Court unless produced and proved before it. It is not the function
of the Court to suit in appeal against the Award, approve or
disapprove its reasoning, or correct its error or affirm, modify or
reverse the conclusion reached by the Land Acquisition officer, as
if it were an appellatecourt.

(3) The Court has to treat the reference as an original proceeding


before it and determine the market value afresh on the basis of
the material produced beforeit.

(4) The claimant is in the position of a plaintiff who has to show


that the price offered for his land in the award is inadequate on
the basis of the materials produced in the Court. Ofcourse the
materials placed and proved by the other side can also be taken
into account for this purpose.

(5) The market value of land under acquisition has to be


determined as on the crucial date of publication of the
notification under sec. 4 of the Land Acquisition Act(dates of
Notifications under secs. 6 and 9 areirrelevant).

(6) The determination has to be made standing on the date


line of valuation (date of publication of notification under
sec. 4) as if the valuer is a hypothetical purchaser willing to
purchase land from the open market and is prepared to pay a
reasonable price as on that day. It has also to be assumed
that the vendor is willing to sell the land at a reasonable
price.

(7) In doing so by the instances method, the Court has to


correlate the market value reflected in the most comparable
instance which provides the index of market value.

(8) only genuine instances have to be taken into account.


(Sometimes instances are rigged up in anticipation of Acquisition
ofland).

(9) Even post notification instances can be taken into account (1)
if they are very proximate, (2) genuine and (3) the acquisition
itself has not motivated the purchaser to pay a higher price on
account of the resultant improvement in development prospects.

(10) The most comparable instances out of the genuine instances


have to be identified on the following considerations:

(i) proximity from time angle,

(ii) proximity from situation angle.


(11) Having identified the instances which provide the index of
market value the price reflected therein may be taken as the
norm and the market value of the land under acquisition may be
deduced by making suitable adjustments for the plus and minus
factors vis-a-vis land under acquisition by placing the two in juxt
a position.
(12) A balance-sheet of plus and minus factors may be drawn for
this purpose and the relevant factors may be evaluated in terms
of price variation as a prudent purchaser would do.

(13) The market value of the land under acquisition has thereafter
to be deduced by loading the price reflected in the instance taken as
norm for plus factors and unloading it for minus factors

(14) The exercise indicated in clauses (11) to (13) has to be undertaken


in a common sense manner as a prudent man of the world of business
would do. We may illustrate some such illustrative (not exhaustive)
factors:Plus factors Minus factors

1. smallness of size. 1. largeness of area.

2. proximity to a road. 2. situation in the interior at a distances from the


Road.

3. frontage on a road. 3. narrow strip of land with very small frontage


compared to death.

4. nearness to developed area. 4. lower level requiring the depressed


portion to be filled up.

5. regular shape. 5. remoteness from developed locality.

6. level vis-a-vis land under 6. some special disadvantageous acquistion.


factor which would deter a purchaser.

7.special value for an owner of an adjoining property to whom it may


have some very specialadvantage.

(15) The evaluation of these factors of course depends on the facts of


each case. There cannot be any hard and fast or rigid rule. Common
sense is the best and most reliable guide. For instance, take the factor
regarding the size. A building plot of land say 500 to 1000 sq. yds
cannot be compared with a large tract or block of land of say l000 sq.
yds or more. Firstly while a smaller plot is within the reach of many, a
large block of land will have to be developed by preparing a lay out,
carving out roads, leaving open space, plotting out smaller plots,
waiting for purchasers (meanwhile the invested money will be blocked
up) and the hazards of an entrepreneur. The factor can be discounted
by making a deduction by way of an allowance at an appropriate rate
ranging approx. between 20% to 50% to account for land required to be
set apart for carving out lands and plotting out small plots. The
discounting will to some extent also depend on whether it is a rural area
or urban area, whether building activity is picking up, and whether
waiting period during which the capital of the entrepreneur would be
looked up, will be longer or shorter and the attendant hazards.

(16) Every case must be dealt with on its own facts pattern bearing in
mind all these factors as a prudent purchaser of land in which position
the Judge must placehimself.

(17) These are general guidelines to be applied with understanding


informed with common sense

7. CED Vs. Radhadevi Jalan (1968) 67 ITR


761(Cal)
The valuation of the property would be the amount which a person
would pay for it on the date of valuation, if he were to purchase it
subject to the tenancy of the managing director of the said company.
The intending buyer must take into consideration the fact that the
tenancy is an old one and the prospect of enhancing the rent by all
legal means was remote and that his eviction was remoter still. On the
basis of the realities, a fair method of estimating valuation of the
property would be the rental basis. Taking the actual rent of Rs. 1,600
per mensem as the basism 33 1/3% as the outgoing and 17 as the years
purchased, which gives a rent security of nearly 6 per cent on the outlay,
the value of the house is estimated at Rs. 2,20,000 in round figures for
the purpose of payment of estateduty.

A fair method of estimating valuation of a property subject to an


old tenancy where the prospect of enhancing the rent by all legal
means was remote and that his eviction was remoter still ,would be
the rental basis.

8. CIT Vs. Ashima Sinha (1979) 116 ITR


26(Cal)
Smt. Ashima Sinha, the respondent herein, sold the said property in
equal undivided shares to Mst. Sayeeda Khatun and Mst. Fatima Khatun
by executing two separate conveyances both dt. 10th Dec., 1973, for Rs.
40,000 each. Statements under s.269P(1) of the IT Act, 1961, in the
prescribed form and verified in the prescribed manner were duly filed
by the transferees along with a copy of the instrument of transfer
disclosing the following particulars : (a) The said property consisted ofa

partly two and partly three-storeyed buildings together with one-


storeyed out-houses in a total area of 6 cottahs, 5 chittacks and 25 sft..
(b) The buildings were approximately over 50 years old. (c) The entire
premises were let out to tenants. (d) The fair market value of the
undivided half part of the said property was approximately Rs. 35,000.

The Valuation Officer of Income Tax Authority inspected the said


property on the 26th June, 1974, and on the 9th July, 1974, submitted
his valuation report. The valuation of the said property was determined
to be Rs. 1,16,000, which according to the Valuation Officer was the fair
market price of the said property, the half share thereof being valued at
Rs. 58,000. He applied rental method to calculate the fair market value
of the property during term period. Further, assuming the future life of
the building as 20 years, applied land and building method to estimate
the fair market value of the property on reversion. Citing Park‘s
Valuation, Hon‘ble Justice Banerjee opined that when a property was
valued on rental basis the result was the value of the land and building
taken together which could not afterwards be apportioned. In the
method adopted by the Valuation Officer the value of the land was
taken twice, being included in the amount arrived at by the "yield or
rental" method and again under "reversionary" method. This was an
entirely novel approach but in their viewerroneous.

When a property was valued on rental basis the result was the
value of the land and building taken together which could not
afterwards be apportioned.

9. CIT Vs. Anupkumar Kapoor & others


(1980) 125 ITR 684(Cal)
The competent authority in coming to his conclusions did not rely on
any relevant evidence except the report of the said Valuation Officer. No
comparable units of other sales were brought on record by him. The
vacant land at No. 47, Shakespeare Sarani, sold on 29th August, 1964, at
the rate of Rs. 27,468 per cottah or the instances of sales of. two other
vacant plots of land in 1974, one near the crossing of Royd Street and
Acharyya Jagadish Bose Road at more than Rs. 24,000 per cottah, and
the other at a distance of about 400 yards to the south of the said
property at Rs. 55,000 per cottah; could hardly be said to becomparable
units with the said property having an old building and out-houses
wholly tenanted and subject to the rent controllegislation, while all the
said alleged comparable units were vacant lands with prospects of
immediate development.

The vacant land with prospects of immediate development could


hardly be said to be comparable units with a property having an
old building wholly tenanted and subject to the rent control
legislation.

10. CIT Vs. Smt. Vimlaben Bhagwandas


Patel (1979)118 ITR134(Guj)
Smt Vimlaben Bhagwandas Patel and another have paid each a
consideration of Rs. 29,521 to the transferor, M/s. Sigil (India) Services
Pvt. Ltd. to purchase two sheds in the industrial estate of BIDCO at
Baroda. They also had purchased shares of BIDCO of the face value of
rs. 18,600, which the transferor-company was holding under the original
scheme of allotment. Inn pursuance of they were holding as allottees,
as well as under the ownership scheme in pursuance of which they
purchased. The Income Tax authority did not accept the consideration
mentioned in the conveyance deed as proper and allowed their
valuation officer to estimate a fair market value of these properties.
Being convinced by the departmental valuation, the IAC of Income Tax
issued one acquisition notice. The notice was challenged before the
Appellate tribunal and the tribunal upheld the challenge. Against the
order of the tribunal the CIT appeal the High Court. The Hon‘ble High
Court of Ahmedabad decided over eight points, out of which the
followingarerelatedtovaluation.Firstly―Whethertheprinciplespertaining to
fixation of fair market value enunciated by courts in the matter of
acquisition of land and building for public purpose should be wholly
imported in determining the fair market value in the context of
acquisition proceedings under Chapter XX-A of the I.T. Act, 1961?” and
secondly “What should be the best method of valuation that may be
adopted by the competent authority for finding out the fair market value
of a property in dispute? Can he prefer any known or recognized method
of valuation of property which may be beneficial solely to the revenue or
is he required to adopt the least of those valuations arrived at after
applying all the known and recognized methods?‖. Hon‘ble High Court
deal with both these questions simultaneously as they are inter-
connected. They declared ― It is axiomatic to say that "all valuations
involve an element of guesswork and intuition. If it is not actually put to
the test, who can be sure how much a building is worth? If it is put to the
test, is it certain that the property was let at the best rent which could be
obtained? Is the building ideal, either in size or location, for one particular
occupant, who would be prepared to pay more than any other tenant ? If
so, ought this to be taken into account? ……………….. It would be too
hazardous to prefer one of the recognized methods of valuation which
may be advantageous to the cause of the revenue and arrive at an
estimation of fair market value of a property on the basis thereof.
………………………… They further opined that "The competent authority shall,
therefore, as far as possible, collect all the relevant materials and evidence
by applying the well-known and recognized methods, viz., land and
building method, contractor's method, rental or yield basis method and
comparable sales method, and work out each estimation of the market
value on application of all the four methods, and for purposes of
countercheck compare it with the municipal valuation for retable
purposes wherever it is possible, since it cannot be said with certainty that
preference of one method to another should be a real aid in arriving at
the real estimation as pointed out by Russel L.J. in Garton's case [1969] 1
All ER 451." In the subsidiary contention, what should be the basis in the
rental method of valuation for estimating the fair market value, they
opined that in the application of rental method of valuation only the
standard rent should be taken as the basis for purposes of annual yield
of a property. It was also decided that the rate of capitalisation should,
therefore, be not unreal and must have regard to the commercial rate of
return after taking into consideration the various constraints and
insecurities in the property market. Hon‘ble Court finally opined that the
just, reasonable and appropriate rate of capitalisation would be eight
and one-third times the net average annual income which would give
the yield of 12% per annum on the investment of capital in property.
The net average annual income may be worked out in the course of
application of the rental method of valuation by determining the net
average annual income after allowing the proper deductions or
outgoings from gross income on account of insurance, taxes, repairs
and management.

In the application of rental method of valuation only the standard rent


should be taken as the basis for purposes of annual yield of a property.
The rate of capitalisation should, therefore, be not unreal and must have
regard to the commercial rate of return after taking into consideration
the various constraints and insecurities in the property market.

The net average annual income may be worked out in the course of
application of the rental method of valuation by determining the net
average annual income after allowing the proper deductions or
outgoings from gross income on account of insurance, taxes, repairs
and management.
PRINCIPLES OF INSURANCE &
LOSS ASSESSMENT
1. INSURANCE
Insurance is a means of protection from financial loss. It is a form of risk management,
primarily used to hedge against the risk of a contingent or uncertain loss.An entity which
provides insurance is known as an insurer, insurance company, insurance carrier or underwriter.
A person or entity who buys insurance is known as an insured or as a policyholder. The
insurance transaction involves the insured assuming a guaranteed and known relatively small
loss in the form of payment to the insurer in exchange for the insurer's promise to compensate
the insured in the event of a covered loss. The loss may or may not be financial, but it must be
reducible to financial terms, and usually involves something in which the insured has
an insurable interestestablished by ownership, possession, or preexisting relationship.

FUNCTIONS AND BENEFITS OF INSURANCE

Insurance has many functions and benefits, some of which we may describe as primary and
others as ancillary or secondary, as follows:

1. Primary functions/benefits:
2. Insurance is essentially a risk transfer mechanism, removing, for a premium, the
potential financial loss from the individual and placing it upon the insurer. The
primary benefit is seen in the financial compensation made available to insured
victims of the various insured events. On the commercial side, this enables
businesses to survive major fires, liabilities, etc. From a personal point of view, the
money is of great help in times of tragedy (life insurance) or other times ofneed.
3. Ancillary functions/benefits:
4. Insurance contributes to society directly or indirectly in many different ways. These
willinclude:
i. Employment: the insurance industry is a significant factor in the local
workforce.
ii. Financial services: since the relative decline in manufacturing in Hong
Kong, financial services have assumed a much greater role in the local
economy, insurance being a major element in the financial services sector.
iii. Loss prevention and loss reduction (collectively referred to as ‗loss
control‘): the practice of insurance includes various surveys and
inspections related to risk management.
iv. Savings/investments: life insurance, particularly, offers a convenient and
effective way of providing for the future. With the introduction of the
Mandatory Provident Fund Schemes in 2000, the value of insurance
products in providing for the welfare of people in old age or family
tragedy is very evident.
v. Economic growth/development: it will be obvious that few people would
venture their capital on costly projects without the protection of insurance
(in most cases, bank financing will just not be available without insurance
cover). Thus, developments of every kind, from erection of bridges to
building construction and a host of other projects, are encouraged and
made possible partly because insurance is available.

INSURANCE CONTRACTS
Through a contract of insurance, the insurer agrees to make good any loss on the insured
property or loss of life (as the case may be) that may occur in course of time in
consideration for a small premium to be paid by the insured. Apart from the above
essentials of a valid contract, insurance contracts are subject to additional principles.

The principles of insurance are as follows,

1. Principle of Utmost goodfaith


2. Principle of Insurable interest

3. Principle of Indemnity

4. Principle of Subrogation
5. Principle of Contribution

6. Principle of Proximate cause

7. Principle of Loss of Minimization

The distinctive features are based on the basic principles. Though all contracts share
fundamental concepts and basic elements, insurance contracts typically possess a number
of characteristics not widely found in other types of contractual agreements. The most
common features of insurance are as follows,

1. Aleatory
2. Adhesion

3. Utmost GoodFaith
4. Executory

5. Unilateral

6. Conditional

7. Personalcontract

8. Warranties andRepresentations
9. Misrepresentations andConcealments

10. Fraud

11. Impersonation (Falsepretenses)

12. Parol (or Oral) evidencerule

INSURABLE INTEREST

An insurable interest is a stake in the value of an entity or event for which an insurance policy
is purchased to mitigate risk of loss. Insurable interest is a basic requirement for the issuance
of an insurance policy, making it legal and valid and protecting against intentionally harmful
acts. Entities not subject to financial loss from an event do not have an insurable interest and
cannot purchase an insurance policy to cover that event.

Legal liability will also create an insurable interest. Bailment commonly creates a legal liability
for the Bailee, and, thus, creates an insurable interest in the property for the Bailee. For
instance, an auto repair shop can have insurance on the vehicles on its lot for possible damage
or theft, even though the shop does not own them.

DUTIES OF INSURER
1. Pay Benefits

2. Risk Assessment

3. Reserves for Policy

4. Privacy Protection

RIGHTS OF THE INSURER


1. Right to avoid thepolicy

2. Right of entry and control over the property.


3. Right of reinstatement.

4. Right to subrogation.
5. Right of contribution.
6. Right to salvage.

DUTIES OF INSURED
1. Application honesty

2. Keep down exposure torisk

3. Pay premiums
4. Keep current information

RIGHTS OF THE INSURED


Insured persons have the right to,

1. Cancel a life insurance policy within 15 days from the date of receipt of the policy
document. If you disagree to any of the terms or conditions in the policy

2. Return the policy stating the reasons for objection


3. Insured will be entitled to a refund of the premium paid

4. A proportionate risk premium for the period on cover and the expenses incurred
by the insurer on medical examination and stamp duty charges will be deducted

5. If it is a unit linked insurance policy (ULIP) in addition, the insurer can repurchase
the units at the price on the cancellation date.

THE SPECIFIC RIGHTS OF POLICYHOLDERS/ INSURED ARE AS


FOLLOWS
1. Right to a financially sound and viable insurance company.

2. Right to access insurance companies‘ official financial information.

3. Right to be informed of the license status of insurance companies, intermediaries


and soliciting agents.

4. Right to be offered a duly approved insurance product.


5. Right to be informed of the benefits, exclusions and other provisions under the
policy.

6. Right to receive the policy.

7. Right to confidentiality of information.

8. Right to efficient service from insurance companies, intermediaries, and soliciting


agents.

9. Right to prompt and fair settlement of claims.

10. Right to seek assistance from the Insurance Commission.


2. FIRE POLICIES
There are a number of fire insurance policies to suit different interests. A number of
factors are considered before deciding about the kinds of policies to be taken. Factors
considered are as below,

1. The type of risk involved.

2. The nature of the property to be insured.

3. The contents of the property.

4. Occupancy hazards.

5. Exposure hazards.

6. The time element.

TYPES OF FIRE POLICIES


The following kinds of policies are generally issued for fire insurance,

1. Valued Policy:

These policies are generally issued for those goods or property whose value cannot be
determined after their loss or damage. These goods may include works of art, jewelry,
paintings, etc.

2. Specific Policy:

Under this policy the risk is insured for a specific sum. In case of loss of property, the
insurer will pay the loss if it is less than the specified amount. The actual value of property
is not taken into consideration.

3. Average Policy:

If the average clause is applicable to a policy, it is called Average Policy. Average clause is
added to penalize the insured for taking up a policy for a lesser sum than the value of the
property. The compensation payable is proportionately reduced if the value of the policy
is less than the value of the property.

4. Floating Policy:

A floating policy is taken up to cover the risk of goods lying at different places. The goods
should belong to the same person and one policy will cover the risk of all these goods.
This policy is useful to those businessmen who are engaged in import and export of
goods and the goods lie in warehouses at different places. The premium charged is
generally the average of the premium that would have been paid, if specific policies would
have been taken for all these goods. Average clause always applies to these policies.

5. Comprehensive Policy:

A policy may be taken up to cover up all types of risks, including fire. A policy may be
issued to cover risk like fire, explosion, lightening, burglary, riots, labour disturbances etc.
This is called a comprehensive policy or all-risk policy.

6. Consequential Loss Policy:

Fire may dislocate work in the factory. Production may go down while the fixed expenses
continue at the same rate. A policy may be taken up to cover up consequential loss or loss
of profits. The loss of profits is calculated on the basis of loss of sales. A separate policy
may be taken up for standing charges also.

7. Replacement Policy:

The underwriter provides compensation on the basis of market price of the property. The
amount of compensation is calculated after taking into account the amount of
depreciation.

REINSTATEMENT
If an insured person fails to pay the premium due to various circumstances and as a result
the insurance policy gets terminated, then the insurance coverage can be renewed. This
process of putting the insurance policy back after a lapse is known as reinstatement.

POLICIES FOR OTHER PERILS


The insurance provides protection to the properties against damages caused due to perils
like natural calamities, machinery breakdown, theft/ burglary, etc. as covered under the
insurance policy

The fire insurance policies do not cover perils mentioned below:

1. Spontaneouscombustion

2. Burning of property by order of any PublicAuthority

3. Property undergoing any heating or dryingprocess

4. Explosion of boilers (other than domestic boilers)

5. Total or partial cessation ofwork


6. Permanent or temporary dispossession by order ofGovernment
7. Normal cracking or settlement or bedding down of newstructures
8. War or warlike operations, Nuclearperils

9. Pollution or contamination

10. Overrunning, excessive pressure, short circuiting,etc.

Perils which are not covered unless add-on covers are bought for the specific risk:

1. Terrorism

2. Earthquake

3. Burglary, Housebreaking, theft,etc.

The additional premium must be paid for add-on covers.

THE BENEFICIARY & RESTRICTIVE CLAUSES IN INSURANCE


Beneficiary Clause:

This is customary to most policies and allows you to name the recipient of your death
benefit. Generally the recipient is either an individual or your estate. It is a wise decision to
make an individual the beneficiary to insure prompt payment of the death benefit.

Restrictive Clause:

The restrictive clause is important in the healthcare setting because it partially determines
the extent and limitations of coverage.

The best way to define a restrictive clause in the healthcare insurance setting is to start by
defining what is known as an insuring clause. It is the clause in an insurance policy setting
forth the kind and degree of coverage granted by theinsurer.

3. ELEMENTS OF INSURANCE
VALUE AT RISK (VAR)
It is a measure of the risk of loss for investments. It estimates how much a set of
investments might lose (with a given probability), given normal market conditions, in a set
time period such as a day. VaR is typically used by firms and regulators in the financial
industry to gauge the amount of assets needed to cover possible losses.

In case of Insurance, one can look at not only the risky business lines, but also the risks by
type of risk: interest rate risk, mortality, morbidity, lapses, credit risk, currency risk, equity
risk, event risk. He can look atthe amount of risk that each type of risk contributes to the
total risk of the portfolio, and the amount of risk that each business line contributes.

SOME IMPORTANT TERMINOLOGIES


1. Sum Insured:

The sum insured is the maximum amount the insurance company will pay in the event
of a claim.

2. Over Insurance:

Over insurance can be defined as the situation where an insured has bought so much
coverage that it exceeds the actual cash value (or the replacement cost) of the risk or
property insured.

3. Under Insurance:

It is the state of an individual having some form of insurance that does not offer
complete financial protection to health or asset insured.

4. Condition of average:
It is the insurance term used when calculating a payout against a claim where the
policy undervalues the sum insured. In the event of partial loss, the amount paid
against a claim will be in the same proportion as the value of the underinsurance. The
formula used is,

5. Payout= Claim x Sum Insured/ Currentvalue

6. Insurance InflationProtection:
Insurance inflation protection is an insurance policy feature in which the value of
benefits increases by a pre-defined percentage at specific time periods. Insurance
inflation protection is designed to allow policyholders to make sure that the benefits
they receive can keep up with the inflation rate.

7. Depreciation in Insurance:
Depreciation is calculated by evaluating an item‘s Replacement Cost Value (RCV) and
its life expectancy. RCV represents the current cost of repairing the item or replacing it
with a similar one, while life expectancy is the item‘s average expected lifespan.

8. Obsolescence in insurance:

It is significant decline in the competitiveness, usefulness, or value of an article or


property. Obsolescence occurs generally due to the availability of alternatives that
perform better or are cheaper or both, or due to changes in user preferences,
requirements, or styles. It is distinct from fall in value (depreciation) due to physical
deterioration or normal wear and tear. Obsolescence is a major factor in operating
risk. Insurance companies take obsolescence into account to reduce the amount of
claim to be paid on damaged or destroyed property.

9. Betterment Insurance:
Betterment insurance refers to insurance coverage that a tenant in a rental property
carries to cover any additions or modifications to the leased space that are made to fit
his or her, or a business's, particular needs. The property owner typically holds an
insurance policy to cover the structure; the tenant procures a policy to cover any
improvements made to the property.
4. PREPARATION OF CLAIM
GENERALLY FOLLOWED STEPS WHILE PREPARATION OF CLAIMS:

Settling insurance claims is just one aspect of the claims management process. The time it
takes to process a claim involves several stages beginning with a person filing a claim. The
stages that follow determine if a claim has merit as well as how much the insurance
company will pay. Insurance customers expect a company to settle claims quickly and to
their satisfaction. Because high customer satisfaction levels can give a company a
competitive edge, reducing the time it takes to settle insurance claims is one way to
decrease the number of customer complaints and improve service. The use of claims
management system software that speeds the process and minimizes costs offers a
practical solution.
1. Analyze the Coverage
2. Review the provisions and exclusions of the policy under claim analyze the policy
to understand its relation to the losses occurred.
3. Get Assistance for Immediate Needs
4. Request that the insurance company release an advance offunds.
5. Organize and prepare a complete list of additional living expenses in order to get
reimbursed for them. Organize and prepare a business continuity plan (for the
business owner) to facilitate continued operations in a timely manner.
6. Prepare a Damage Estimate
7. Provide a detailed review of all damaged property, documenting damage to all
areas.
8. Provide line item estimates, calculate quantities, unit cost and total costs to cover
the loss, take help of professional, in case of need.
9. Prepare and submit all of the above to the insurance company using industry-
standard forms; thereby, enhancing a prompt and accurate response to the claim.
10. Negotiate The Settlement
11. Attend all meetings with adjuster(s)
12. Manage any insurance claim problems or challenges.
13. Accept settlement offers after proper review.
14. Settle the Claim Successfully
15. Resolve claims as quickly as possible with the insurance company.
REPORT WRITING
1. Reports-Quality, Structure,Style

IVS 103 Reporting

❖ Introduction
▪ It is essential that the valuation report communicates the information
necessary for proper understanding of the valuation or valuation review. A
report must provide the intended users with a clear understanding of the
valuation.
To provide useful information, the report must set out a clear and accurate
description of the scope of the assignment, its purpose and intended use
(including any limitations on that use) and disclosure of any assumptions,
special assumptions (see Chapter 6.2.2.1.2 of this Study Material), significant
uncertainty or limiting conditions that directly affect the valuation.

▪ This standard applies to all valuation reports or reports on the outcome of a


valuation review which may range from comprehensive narrative reports to
abbreviated summary reports.
▪ For certain asset classes there may be variations from these standards or
additional requirements to be reported upon. These are found in the relevant
IVS AssetStandards.
❖ GeneralRequirements
▪ The purpose of the valuation, the complexity of the asset being valued and
the users‘ requirements will determine the level of detail appropriate to the
valuation report. The format of the report should be agreed with all parties as
part of establishing a scope of work (see Chapter 6.2.2.1.1 of this Study
Material).
▪ Compliance with this standard does not require a particular form or format of
report; however, the report must be sufficient to communicate to the
intended users the scope of the valuation assignment, the work performed
and the conclusion sreached.
▪ The report should also be sufficient for an appropriately experienced
valuation professional with no prior involvement with the valuation
engagement to review the report and understand the items in below
paragraphs, as applicable.
❖ Valuation Reports
▪ Where the report is the result of an assignment involving the valuation of an
asset or assets, the report must convey the following, at aminimum:
a. thescopeoftheworkperformed,includingtheelementsnotedinChapter
6.2.2.1.1 of this Study Material, to the extent that each is applicable to the
assignment,
b. the approach or approaches adopted,
c. the method or methods applied,
d. the key inputs used,
e. the assumptions made,
f. the conclusion(s) of value and principal reasons for any conclusions reached,
and
g. the date of the report (which may differ from the valuationdate).
▪ Some of the above requirements may be explicitly included in a report or
incorporated into a report through reference to other documents
(engagement letters, scope of work documents, internal policies and
proceduresetc).
❖ Valuation Review Reports
▪ Where the report is the result of a valuation review, the report must convey
the following, at a minimum:
a. The scope of the review performed, including the elements noted in Chapter
6.2.2.1.1 of this Study Material to the extent each is applicable to the
assignment,
b. the valuation report being reviewed and the inputs and assumptions upon
which that valuation was based,
c. the reviewer‘s conclusions about the work under review, including supporting
reasons, and
d. the date of the report (which may differ from the valuation date).
▪ Some of the above requirements may be explicitly included in a report or
incorporated into a report through reference to other documents (eg,
engagement letters, scope of work documents, internal policies and
procedures etc).
2. Report writing for various purposes of
valuation-Sale, Purchase, Mortgage, Taxation,
Insurance, Liquidation etc
9.1.1 The following minimum are to be reported in a valuation report prepared for
the purpose of Sale /Purchase:
a. Date of valuation
b. Date of report
c. Date of work order
d. Purpose of valuation
e. Scope of work
f. Name of the owner of the property
g. Description of the property with boundary description
h. Bases of valuation
i. Premise of valuation (Highest and BestUse)
j. Valuation approach
k. Valuation Method
l. Valuation terminologies
m. Valuation calculation
n. Assumption and limitation
o. Market Value (range ofvalues)
p. Departure statement
q. Compliance statement
9.1.2 The following minimum are to be reported in a valuation report prepared for
the purpose of Mortgage:
a. Date of valuation
b. Date of report
c. Date of workorder
d. Purpose of valuation
e. Scope of work
f. Name of the owner of the property
g. Description of the property with boundary description
h. Bases of valuation
i. Premise of valuation (existinguse)
j. Valuation approach
k. Valuation Method
l. Valuation terminologies
m. Valuation calculation
n. Assumption and limitation
o. Market Value (one value only)
p. Realizable Value
q. Forced / Distressed sale value
r. Departure statement
s. Compliance statement
9.1.3 The following minimum are to be reported in a valuation report prepared for
the purpose of Taxation:
a. Date of valuation (may be any past date)
b. Date of report
c. Date of work order
d. Purpose of valuation
e. Scope of work
f. Name of the owner of the property
g. Description of the property with boundary description
h. Bases of valuation (as per the statute)
i. Premise of valuation (existing use)
j. Valuation approach
k. Valuation Method
l. Valuation terminologies
m. Valuation calculation as per the statute
n. Assumption and limitation
o. Fair Market Value (in case of capital gain tax, incometax)
p. Annual Value (in case of property tax)
q. Departure statement
r. Compliance statement
9.1.4 The following minimum are to be reported in a valuation report prepared for
the purpose of Insurance:
a. Date of valuation
b. Date of report
c. Date of work order
d. Purpose of valuation
e. Scope of work
f. Name of the owner of the property
g. Description of the property with boundary description
h. Bases of valuation (market value / Reinstatement Value)
i. Premise of valuation (existing use)
j. Valuation approach (Cost approach)
k. Valuation Method
l. Valuation terminologies
m. Valuation calculation
n. Assumption and limitation
o. Market Value / Reinstatement Value
p. Departure statement
q. Compliance statement
9.1.5 The following minimum are to be reported in a valuation report prepared for
the purpose of Liquidation:
a. Date ofvaluation
b. Date ofreport
c. Date of workorder
d. Purpose ofvaluation
e. Scope ofwork
f. Name of the owner of the property
g. Description of the property with boundary description
h. Bases of valuation (Liquidation value)
i. Premise of valuation (orderly liquidation)
j. Valuation approach
k. Valuation Method
l. Valuation terminologies
m. Valuation calculation
n. Assumption and limitation
o. Realizable Value
p. Departure statement
q. Compliance statement

3. Contents of the report: Instruction of Clients,


Date as on which valuation is made, date of
Report and Site Inspection, Location,
Ownership History, Data Collection and
Analysis, Type of Construction, Valuation
Method, Value Estimation, Assumptions and
Limiting Conditions including Caveats and
Conclusion

9.1.6 Instruction of Clients


• The instruction of the clients shall be in writing as far as possible
• The copy of the instruction shall annexed with the valuation report
• It shall be mentioned at the beginning of a valuation report
• All instructions of the client shall be verified by the registered valuer in
respect of the code of conduct stipulated by the registered valuer
organisations for their members. Any violation shall be immediately brought
to the notice of the client and rectified accordingly.
9.1.7 Date as on which valuation is made
• This date may be the same as that of the date of report or any past date. In
most of the cases the date as on which valuation is made is the date of site
inspection of the property. But in no case, this date shall be any future date.
In that case it will not be termed as a valuation report but a Speculation
Report. For the purpose of sale / purchase, mortgage, insurance, liquidation
etc. the date as on which valuation is made is the date of site inspection. For
the purpose of capital gain tax calculation, the date as on which fair market
value is estimated was 01.04.2001 for the properties whose date ofacquisition
was before 01.04.2001. For any property whose date of acquisition was any
date after 01.04.2001, was the date of its fair market value estimation. In case
of land acquisition, the fair market value of the property under acquisition
was the date on which acquisition notice was given under Right to Fair
Compensation and Transparency in Land Acquisition, Rehabilitation and
Resettlement Act,2013.
9.1.8 Date of Report
• The date of report is the date on which the valuation report is signed by the
registered valuer.
9.1.9 Date of Site Inspection
• The date of site inspection may be one or more. The date(s) on which the
property under question was physically inspected by the registered valuer or
his authorised agent is known as date of site inspection. If the registered
valuer is not satisfied with the data collected from his site visit, he / she may
visit the site under question for a number of times. All such visits shall be
recorded as date of site visit and has to be reported in the valuation report.
9.1.10 Location
• Location of the property which is the subject matter of a valuation report, use
to play a great role in identification of the property. Generally location of an
immovable property is reported by its latitude and longitude (coordinates).
Sometimes nearest land mark is reported by the registered valuer in his/her
report to locate the questioned property. In rural areas, numbered electric
poles sometimes act, as a locator. Apart from coordinates, reporting of
boundary of a property to identify its location is another typical character of a
standard valuation report.
9.1.11 Ownership History
• Ownership history of an immovable property can be found in the recital
portion of the deed of conveyance of that immovable property. But,
investigation and report of ownership history is not an essential job of a
valuer.
9.1.12 Data Collection and Analysis
• IVS104Paragraph30.6states― The data available and the circumstances relating
to the market for the asset being valued must determine which valuation
method or methods are most relevant and appropriate. If based on
appropriately analysed market-derived data, each approach or method used
should provide an indication of Market Value.
• A valuer shall disclose all his sources of data under the head Key inputs use.
Transparency is the basic requirement of an ideal valuation report. The data
sources shall be placed in the valuation report in such a manner that the
client can check the authenticity and reliability of those very easily before
taking any decision.
• Analysis of collected data also shall be done in a very transparent manner and
to be reported correctly in the report. Justification is an inseparable part of
this analysis of data. Wherever necessary, detail numerical calculation shall be
performed to provide a clear understanding of the analysis of data to the
Client.
9.1.13 Type of Construction
• Generally the valuers encountered seven types of construction in India. These
are – Steel framed, R.C.C. framed, Brick masonry load bearing wall with R.C.C.
roof, Corrugated galvanized roof with sand and cement brick walls,
Corrugated asbestos roof with sand and cement plastered brick walls, Red
and country tiled building with exposed bricks and Temporary shed. The
economic life / total life of a construction varies according to the type of
construction. Therefore, depreciation as well as value of the property also
depends on the type of construction. So, a valuer is to report a clear
description of the type of construction of the property in is valuation report
especially under cost approach.
9.1.14 Valuation Method
• Each of three (market, income and cost) valuation approaches includes
different, detailed methods of application. The goal in selecting valuation
approaches and methods for an asset is to find the most appropriate method
under the particular circumstances. No one method is suitable in every
possible situation. The selection process should consider, at aminimum:
o the appropriate basis(es) of value and premise(s) of value, determined by the
terms and purpose of the valuation assignment,
o the respective strengths and weaknesses of the possible valuation
approaches and methods,
o the appropriateness of each method in view of the nature of the asset, and
the approaches or methods used by participants in the relevant market,and
o the availability of reliable information needed to apply the method(s).
• Valuers are not required to use more than one method for the valuation of an
asset, particularly when the valuer has a high degree of confidence in the
accuracy and reliability of a single method, given the facts and circumstances
of the valuation engagement. However, valuers should consider the use of
multiple methods and more than one valuation method should be considered
and may be used to arrive at an indication of value, particularly when there
are insufficient factual or observable inputs for a single method to produce a
reliable conclusion. Where more than one method is used within a single
approach, the conclusion of value based on those multiple methods should
be reasonable and the process of analysing and reconciling the differing
values into a single conclusion, without averaging, should be described by the
valuer in the report.
• When different methods result in widely divergent indications of value, a
valuer should perform procedures to understand why the value indications
differ, as it is generally not appropriate to simply weight two or more
divergent indications of value. In such cases, valuers should reconsider the
guidance in above paragraphs to determine whether one of the
approaches/methods provides a better or more reliable indication of value.
9.1.15 Value Estimation
• Estimation of value is the ultimate goal of a valuation report. The valuer shall
estimate the value of a property according to his scope of work after
collection of relevant data, analysis of such data using appropriate approach
and method and through detail calculation. The estimated value of a property
in a valuation report shall be backed by proper justification and appropriate
flawless numerical calculation. The value estimation shall be reported at the
beginning of the report under the head Valuer‘s Opinion or Executive
Summary.
9.1.16 Assumptions and Limiting Conditions including Caveats and Conclusion
9.1.16.1 Assumptions and Limiting Conditions
In addition to stating the basis of value, it is often necessary to make an
assumption or multiple assumptions to clarify either the state of the asset in
the hypothetical exchange or the circumstances under which the asset is
assumed to be exchanged. Such assumptions can have a significant impact
on value.

These types of assumptions generally fall into one of two categories:


a. assumed facts that are consistent with, or could be consistent with, those
existing at the date of valuation,and
b. assumed facts that differ from those existing at the date of valuation.
Assumptions related to facts that are consistent with, or could be consistent
with, those existing at the date of valuation may be the result of a limitation
on the extent of the investigations or enquiries undertaken by the valuer.
Examples of such assumptions include, withoutlimitation:

a. an assumption that a business is transferred as a complete operationalentity,


b. an assumption that assets employed in a business are transferred without the
business, either individually or as agroup,
c. an assumption that an individually valued asset is transferred together with
other
d. complementary assets,and
e. an assumption that a holding of shares is transferred either as a block or
individually.
Where assumed facts differ from those existing at the date of valuation, it is
referred to as a―special assumption. Special assumptions are often used to
illustrate the effect of possible changes on the value of an asset. They are
designated as ―special‖ so as to highlight to a valuation user that the
valuation conclusion is contingent upon a change in the current
circumstances or that it reflects a view that would not be taken by
participants generally on the valuation date. Examples of such assumptions
include, without limitation:

a. an assumption that a property is freehold with vacant possession,


b. an assumption that a proposed building had actually been completed on the
valuation date,
c. an assumption that a specific contract was in existence on the valuation date
which had not actually been completed,and
d. an assumption that a financial instrument is valued using a yield curve that is
different from that which would be used by aparticipant.
All assumptions and special assumptions must be reasonable under the
circumstances, be supported by evidence, and be relevant having regard to
the purpose for which the valuation is required.

The following are some model assumptions and limitations to be inserted in a


valuation report:

a. The property description supplied to the valuer is assumed to becorrect;


b. No survey of the property has been made or reviewed by the valuer, and no
responsibility is assumed in connection with such matters. Illustrative
Material, including maps and plot plans, utilized in this report are included
only to assist the reader in visualizing the property. Property dimensions and
sizes are considered to be approximate;
c. No responsibility is assumed for matters of a legal nature affecting title to the
property, nor is any opinion of title rendered. Property titles are assumed to
be good and marketable unless otherwisestated;
d. Information furnished by others is believed to be true, correct, and reliable.
However, no responsibility for its accuracy is assumed by thevaluer;
e. All mortgages, liens, encumbrances, leases, and servitudes have been
disregarded unless so specified within the report. The property is assumed to
be under responsible, financially sound ownership and competent
management;
f. It is assumed that there are no hidden or unapparent conditions of the
property, subsoil, or structures which would render the property more or less
valuable. No responsibility is assumed for such conditions or for arranging for
engineering studies which may be required to discoverthem;
g. Unless otherwise stated in this report, the existence of hazardous material,
which may or may not be present on the property, was not observed by the
valuer. However, the valuer is not qualified to detect such substances. The
presence of substances such as asbestos, urea-formaldehyde foam insulation,
or other potentially hazardous materials may affect the value of the property.
The value conclusions in this report are predicated on the assumption that
there is no such materials on or in the property that would cause a loss of
value. No responsibility is assumed for any such conditions, or for the
expertise required to discover them. The client is urged to retain an expert in
this field if desired. The analysis and value conclusions in this report are null
and void should any hazardous material bediscovered;
h. Unless otherwise stated in this report, no environmental impact studies were
either requested or made in conjunction with this report. The valuer reserves
the right to alter, amend, revise, or rescind any opinions of value based upon
any subsequent environmental impact studies, research, orinvestigation;
i. It is assumed that there is full compliance with all applicable central, state and
local environmental regulations and laws unless noncompliance is specified,
defined, and considered in thisreport;
j. It is assumed that all applicable zoning and use regulations and restrictions
have been complied with, unless non-conformity has been specified, defined
and considered in this report;

k. It is assumed that all required licenses, certificates of occupancy, consents, or


other legislative or administrative authority from any local, state, or central
governmental or private entity or organization have been or can be obtained
or renewed for any use on which the value estimate isbased;
l. The valuer will not be required to give testimony or appear in court because
of having made this report, unless arrangements have previously beenmade;
m. Possession of this report, or a copy thereof, does not carry with it the right of
publication. It may not be used for any purpose by any person other than the
client without the written consent of the valuer, and in any event, only with
properly written qualification and only in itsentirety;
n. Neither all nor any part of the contents of this report, or copy thereof, shall
be conveyed to the public through advertising, public relations, news, sales,
or any other media without written consent and approval of the valuer. Nor
shall the valuer, client, firm, or professional organization of which the valuer is
a member be identified without the written consent of thevaluer;
o. The liability of the valuer, employees, and subcontractors is limited to the
client only. There is no accountability, obligation, or liability to any third party.
If this report is placed in the hands of anyone other than the client, the client
shall make such party aware of all limiting conditions and assumptions of the
assignment and related discussions. The valuer is in no way responsible for
any costs incurred to discover or correct any deficiencies of theproperty;
p. It is assumed that the public project which is the object of this report, will be
constructed in the manner proposed and in the foreseeablefuture;
q. Acceptance and/or use of this report constitutes acceptance of the foregoing
assumptions and limitingconditions.
9.1.16.2 Caveats
Caveat is a Latin term that means "let him beware." There are many types of
caveats in law and finance, with the most common being "caveat emptor,"
meaning "let the buyer beware," and "caveat venditor," meaning "let the
seller beware."

Caveats in valuation report is acceptable to the extent they explain or elucidate the
limitations faced by Valuer, which shall not be for the purpose of limiting his
responsibility for the valuation report.

9.1.16.3 Conclusion
In conclusion, the valuer will conclude his report by giving either a specific value or
a range of values of the property following the direction of the client as
described in the scope of work. The valuation conclusion shall be in figure as
well as in words.
CASE STUDIES
10.1 Mr. Adukia has granted a lease of a shop premises to Mr. Bhalla at a rent
of `150,000 per annum on fully insuring and repairing lease basis. The lease
hastoday5yearstorun.Thecurrentfullrentalvalueofthepropertyis
`300,000 per annum which is an exclusive rent. The leasehold yield on shop
property in this location is at 5% and the rate of interest adopted for
working out Sinking Fund is 2.5%.

Q.1. What is the outgoing of Mr. Adukia per annum?


a) `15,000
b) `22,500
c) Nil
d) `30,000

Q.2. What is the Rack Rent of the property per annum?


a) `150,000
b) `300,000
c) `450,000
d) `360,000

Q.3. What is the value of Mr. Bhalla's interest?


a) `624,357
b) `651,474
c) `551,474
d) `724,357

Q.4. What is the accumulative rate of interest?


a) 2.5%
b) 5.0%
c) 4.0%
d) 6.5%

Answer
(1) c (2) b (3) a (4)a
Solution to Q.3

Valuation of lessee's interest

Fullrentalvalue = `300,000 per annum(net)

Less, rent paid bythelessee = `150,000 per annum

Profitrent = `150,000 per annum(net)

Y.P. 5 years at 5%and2½% = 4.162

Value = Y.P x ProfitRent

= `624,357

Therefore, valuation of lessee's interest = `624,357

10.2 Mr. Banerjee has leased out his freehold site for 99 years to Mr.
Bhatnagar, a developer at a ground rent of ` 10,000 per annum net of
outgoings. Mr. Bhatnagar has constructed a building fetching an
exclusive rent of `50,00,000 per annum The remunerative rate of interest
is 12% and accumulative rate of interest is2.5%.

Q.1. Which of the following is not an outgoing of thebuilding?


a) Insurancepremium
b) MunicipalTax
c) IncomeTax
d) Annual maintenancefees

Q.2. What is the Net Income per annum of Mr. Bhatnagar?


a)`50,000
b) `45,000
c) `40,000
d) `35,000

Q.3. What is the value of Mr. Banerjee's interest in the property?


a)`1,25,000
b) `83,333
c) `4,00,000
d) `1,00,000

Q.4. What is the rate of interest for calculation of annual sinking fund?
a)2.5%
b) 8.0%
c) 9.0%
d) 3.5%
Answer
(1) c (2) a (3) b (4) a
Solution to Q.3

Valuation of Banerjee's interest

Net income = `10,000 per annum

YP in perpetuity @8%
100
Value of freehold interest = 10,000 × = Rs.83,333
12

10.3 The ground floor, first floor and second floor of a residential RCC framed
building in Durgapur having areas 120 sqm, 120 sqm and 100 sqm were
built in 1996, 1997 and 1998 respectively. 10% construction of the building
was done in 1996, 70% in 1997 and final 20% construction was completed
in 1998. The basic cost of construction of RCC framed residential building
as published by CPWD at Delhi in 1992 was `2810 per sqm. The Cost Index
of Durgapur in 1996, 1997, 1998 and 2018 were 188, 202, 211 and 854
respectively. The total life of RCC Framed building is 85years.

Q.1. What was the historical cost of construction of thebuilding?


a)`9,55,400
b) `17,96,152
c) `20,15,894
d) `19,33,730

Q.2. What was the replacement cost new of the building in 2018?
a)`64,31,303
b) `62,58,522
c) `81,59,116
d) `79,32,116

Q.3. What was the age of the second floor in2018?


a) 22years
b) 21years
c) 20years
d) 23years

Q.4. Which approach is applicable to find the market value ofthe


building on the basis of the givendata?
a) CostApproach
b) MarketApproach
c) IncomeApproach
d) ProfitApproach
Answer

(1) d (2) c (3) a (4) a

Solution to Q.1

Historical cost of construction calculation


Cost of construction in 1996 = Total area x basic cost per sqm x (CI 1996/
CI1992) x %
= 340 x 2810 x (188/100) x 10%
= 1,79,615
Cost of construction in 1997 = Total area x basic cost per sqm x (CI 1997/
CI1992) x %
= 340 x 2810 x (202/100) x 70%
= 13,50,936
Cost of construction in 1998 = Total area x basic cost per sqm x (CI 1998/
CI1992) x %
= 340 x 2810 x (211/100) x 20%
= 4,03,179
Therefore, historical cost of construction of the building
= 1,79,615 + 13,50,936 + 4,03,179
= 19,33,730

Solution to Q.2

Replacement cost new calculation


Cost of construction in 2018 = Total area x basic cost per sqm x(CI 2018/
CI1992) = 340 x 2810x
(854/100)
= 81,59,116

10.4 Mr. Vasoya purchased a plot of 1000 Sq.mt. in a posh locality of a city in
the year 1987 for a price of `30,00,000. In the year 1988, he constructed a
residential bungalow having 300 Sq.mt. built up floor area at ground
level and in 1995, 100 Sq.mt. built up area at first floor level at the costof
`14,00,000. Prevalent replacement cost new of similar bungalow as on
today is `30,000 per Sq.mt. Prevalent land price in the locality at present
is `60,000 per Sq.mt. The remaining economic life of building is 60 years
and the economic life of the building is 85years.

Q.1. What is the effective age of the first floor of the building?
a) 24years
b) 25years
c) 32years
d) 30years
Q.2. What is the reinstatement value of the building for the purpose of
insurance?
a) `150,00,000
b) `120,00,000
c) `100,00,000
d) `94,00,000
Q.3. What is the depreciated value of the building by decliningbalance
method?
a) `88,23,529
b) `78,23,529
c) `60,96,261
d) `70,96,261
Q.4. What will be the total market value of the bungalow property for
bank loan purpose??
a) `6,88,23,529
b) `7,78,23,529
c) `6,60,96,261
d) `7,70,96,26

Answer
(1) b (2) b (3) c (4) c

Solution to Q.1

Effectiveage = Economic Life – Remaining economiclife


= 85 – 60
= 25 years

Solution toQ.2

Reinstatementvalue = Area of the building x Replacement cost new


per sqm
= (300 + 100) sq m x `30,000 per sq m
= `120,00,000

Solution to Q.3

1 1
F n Vmm
 =  
C C
F = Scrap value = 10% of Replacement cost new = `120,00,000 x
10%
= `12,00,000
C = Replacement cost new =`120,00,000
Vm = Value after m years = Depreciatedvalue
n = Expected life of the structure in years = Economic Life =85
years
m = Number of years after which value is to be found = Effective
Age = 25yr
Therefore,
Vm =`60,96,261

Solution to Q.4

Marketvalue = Market Value of land x Depreciated market valueof


building
= (1000 sq m x `60,000) + `60,96,261
= `6,60,96,261

10.5 A warehouse property is situated close to a railway siding at Tatanagar. It


is let out on a 55 years lease to M/s. MNC Ltd. The lessee is paying to the
lessor an exclusive rent @ `2000 per annum after payment of anone time
premium of `2,50,00,000. The rack rental value on full repairing terms
amounts to `25,00,000 per annum. The yield from freehold warehouses in
similar locations is considered to be 12% and for long term lease is16%.

Q.1. What is the Y.P. during term period?


a)5.91
b) 8.32
c) 7.32
d) 9.51

Q.2. What is the value of freeholder‘s interest?


a)`12,47,318
b) `47,318
c)`1,47,318
d) `57,118

Q.3. What is the marketrent?


a) `25,00,000
b) `25,02,000
c) `2,25,000
d) `2,25,200
Q.4. What is the outgoing of the lessor?
a)`2,50,000
b) `200
c)Nil
d) `25,000

Answer
(1) b (2) b (3) a (4) c

Solution to Q.1
1
1 −(1+𝑖)𝑛
Y.P. =
𝑖
Here, i = 12%, n = 55 years

Therefore, Y.P. = 8.32

Solution to Q.2

Y.Pterm = 8.32
Y.P. reversion = 1/16% = 6.25
Freeholder‘sinterest = 2000 x 8.32 + (2500000 x 6.25)x
[1/(1+0.12)55]
= `47,318

Solution to Q.3

Valuation of lessee's interest

Fullrentalvalue = `300,000 per annum(net)

Less, rent paid bythelessee = `150,000 per annum

Profitrent = `150,000 per annum(net)

Y.P. 5 years at 5%and2½% = 4.162

Value = Y.P x ProfitRent

= `624,357

Therefore, valuation of lessee's interest = `624,357

10.6 Mr. Roy built his two storied R.C.C. framed building in 2002. The plinth
area of each floor is 2000 sq ft. He leased his entire property in 2020 @ 3%
return on the market value of his property. The unit plinth area rate of cost
of construction of such type of RCC Frame structure in 1992 was `150 per
sq ft. The cost index of construction of 1992, 2002 and 2012 was 100, 210
and 530 respectively. The market value of land of the said property is
`1,35,00,000. The life of RCC frame structure is 75 years. The rental value of
the property in 2020 is `7,20,000 per annum

Q.1. What was the historical cost of construction of the building ofMr.
Roy?
a) `12,60,000
b) `6,30,000
c) `11,30,000
d) `12,00,000

Q.2. What is the reproduction cost new of the building of Mr. Roy?
a)`35,40,000
b) `37,80,000
c) `31,80,000
d) `27,00,000

Q.3. What is the market value of the property?


a)`1,70,40000
b) `1,47,60,000
c) `1,62,75,360
d) `1,52,75,360

Q.4. What is the lease rent per month?


a)`36,620
b) `40,688
c) `36,688
d) `40,620

Q.5. What is the notional profit rent per month?


a)`18,312
b) `17,312
c) `19,312
d) `20,312

Q.6. What is the market rent per month?


a)`60,000
b) `50,000
c) `70,000
d) `55,000
Answer
(1) a (2) a (3) c (4) b (5) b, (6) (a)
10.7 Smt. Rekha purchased marshy land in 2017 at `50,00,000. She developed
land in the next two years by making a recurring expenses @ `30,00,000
every year. She leased out the land in 2019 to Ms. Amina @ 5% yield per
annum of her investment. Ms. Amina constructed a godwn and sub-leased
out it to M/s. Patel & Co. @ `12,00,000 per annum. The rate of interest
remainsas7%perannumforthelastthreeyears.Themarketrentis
`15,00,000 per annum. All the rents are exclusive rent.

Q.1. What was the total value of investment of Smt. Rekha in2019?
a)`57,24,500
b) `1,17,24,500
c) `1,19,34,500
d) `1,10,00,000

Q.2. What is the ground rent of the property?


a)`5,96,725
b) `5,98,654
c) `5,86,225
d) `5,50,000

Q.3. What is the profit rent of Ms. Amina?


a)`6,13,775
b) `6,50,000
c) `6,03,275
d) `6,01,346

Q.4. What is the rental value of the property in 2020?


a)`15,00,000
b) `12,00,000
c) `17,00,000
d) `11,00,000

Q.5. What is the notional profit rent of M/s. Patel &Co.?


a)`5,00,000
b) `0
c) `3,00,000
d) `1,00,000

Q.6. What is the outgoing of Ms. Amina?


a)`3,00,000
b) `0
c) `3,60,000
d) `1,20,000
Answer

(1)c (2) a (3) c (4) b (5) c (6) (b)

10.8 In Hotel Friday Inn there are 40 double-letting bed rooms, three
restaurants,onespaonehealthclubandtwobanquethalls.Roomrentis
`5,000 per day. Income from food and beverage sales is `3,73,00,000 per
annum. Income from spa, health club, banquet and other miscellaneous
sales is `40,00,000 per annum. Total purchase of the hotel is `1,20,00,000
per annum. The working expenses of the hotel is `3,80,00,000 per annum.
Value of content is `9,00,00,000. Block capital is `8,00,00,000. Return on
tenant capital @ 15%. Rental value is 45% of the divisible balance. Return
on land and building is 8% and that on goodwill is 45%.

Q.1. What is the maintainable income of the hotel?


a)`7,39,00,000
b) `6,39,00,000
c) `9,51,00,000
d) `8,51,00,000

Q.2. What is the gross trading profit of the hotel?


a)`5,96,00,000
b) `7,31,00,000
c) `5,35,00,000
d) `8,12,00,000

Q.3. What is the divisible balance?


a)`1,18,00,000
b) `5,42,00,000
c) `2,31,00,000
d) `3,12,00,000

Q.4. What is the capital value of land and building of the hotel?
a)`12,99,37,500
b) `12,08,85,200
c) `17,18,12,100
d) `11,12,15,300

Q.5. What is the value of good will of the hotel?


e)`5,49,00,000
f) `8,12,00,000
g) `3,48,00,000
h) `13,24,00,00
Q.6. What is the value of
the hotel?
a)`28,50,12,200
b) `27,48,37,500
c) `32,00,60,000
d) `25,92,45,800
Answer

(1)d (2) b (3) c (4) b (5) c (6) (b)

You might also like